Você está na página 1de 279

Olá Estudante, seja muito bem-vindo(a)!

Estamos iniciando o 4º bimestre de 2021. Chegar até aqui não foi fácil, não é mes-
mo? Por isso, estamos muito orgulhosos de você e de seu desempenho. Agora que
mais um bimestre vai começar é hora de preparar a mente para novos conhecimen-
tos e o coração para novas aventuras.
Para isso, preparamos o Plano de Estudos Tutorado – Volume 4. Um material cheio
de propostas de atividades instigantes e inovadoras para você. São histórias,
situações-problemas, exercícios, imagens, pesquisas, desafios, temas e textos
que irão orientá-lo (a) na aquisição de conhecimentos e habilidades importantes
para que você se torne um cidadão cada vez mais curioso, pesquisador, autônomo
e atuante em nossa sociedade.
Atenção, algumas das várias experiências de aprendizagem que você encontrará
no PET 4 irão abordar a temática da Consciência Negra, pois o mês de novembro
é dedicado a pensarmos em questões importantes como: racismo, discriminação,
igualdade social e a cultura afro-brasileira.
Seu professor(a) irá acompanhá-lo (a) nesta jornada de conhecimentos do PET 4
por meio de alguns canais de comunicação como o APP Conexão 2.0, o site https://
estudeemcasa.educacao.mg.gov.br. Ah! Acompanhe também as aulas na TV Minas,
todas as manhãs de segunda à quinta-feira. Elas irão auxiliá-lo(a) na resolução das
atividades propostas no PET.
Estamos gratos de poder contribuir com seus estudos em tempos de Pandemia
da Covid 19, mas a nossa expectativa é que no próximo ano nossas esperanças
sejam renovadas.
E você é a nossa maior motivação e esperança de um futuro melhor!

Boas aprendizagens nesta 4ª etapa escolar!

II
SUMÁRIO

LÍNGUA PORTUGUESA............................................................. pág 01


Semana 1: Gênero Romance Policial....................................... pág 01
Semana 2: Músicas narrativas................................................pág 05
Semana 3: Intertextualidade...................................................pág 09
Semana 4: Coesão e Coerência.............................................. pág 13
Semana 5: Regência Verbal e Regência Nominal......................pág 17
Semana 6: Interpretação de textos.........................................pág 20

MATEMÁTICA.......................................................................... pág 24
Semana 1: Geometria espacial e os sólidos geométricos.........pág 24
Semana 2: Poliedros...............................................................pág 29
Semana 3: Prismas.................................................................pág 34
Semana 4: Pirâmides.............................................................pág 40
Semana 5: Corpos redondos...................................................pág 46
Semana 6: Cone.....................................................................pág 52

BIOLOGIA................................................................................ pág 58
Semana 1: Os grandes grupos dos seres vivos.........................pág 58
Semana 2: Reino dos fungos..................................................pág 65
Semana 3: Reino Protoctista..................................................pág 70
Semana 4: Doenças causadas por bactérias e protozoários.... pág 75
Semanas 5 e 6: Reino Plantae.................................................pág 82

QUÍMICA.................................................................................. pág 94
Semana 1: Equilíbrio dinâmico................................................pág 94
Semana 2: Equilíbrio Químico e Constante de Equilíbrio..........pág 99
Semana 3: Equilíbrio Químico e o Princípio De Le Chatelier... pág 103
Semana 4: Acidez e Basicidade.............................................pág 107
Semana 5: Equilíbrio químico em sistemas aquosos.............. pág 110
Semana 6: Equilíbrio de Ionização da água: pH e pOH............ pág 113

FÍSICA..................................................................................... pág 117


Semana 1: Calor e Movimento I............................................... pág 117
Semana 2: Calor e Movimento II............................................ pág 122
Semana 3: Ondas - parte 1.....................................................pág 126
Semana 4: Ondas - parte 2.................................................... pág 130
Semana 5: Ondas sonoras.................................................... pág 134
Semana 6: As propriedades do som.......................................pág 138

III
GEOGRAFIA...........................................................................pág 142
Semana 1: Cenários de um mundo globalizado...................... pág 142
Semana 2: Globalização e fragmentação I............................. pág 146
Semana 3: Globalização e fragmentação II............................ pág 150
Semana 4:Globalização e fragmentação III........................... pág 154
Semana 5: A África no mundo global I....................................pág 158
Semana 6: A África no mundo global II...................................pág 162

HISTÓRIA............................................................................... pág 166


Semana 1: Embates Políticos e Culturais no Processo
      de Construção e Afirmação do Estado Nacional...pág 166
Semana 2: Confrontos: fim da monarquia no Brasil e início
     da República .......................................................pág 169
Semana 3: Principais diferenças entre as Constituições
      de 1891 e a Constituição Imperial de 1824.............pág 172
Semana 4: Movimento abolicionista e Movimento republicano
     sobre a sociedade brasileira................................pág 175
Semana 5: As camadas populares no processo político
     brasileiro.............................................................pág 179
Semana 6: Confrontos: fim da monarquia no Brasil e início
     da República.......................................................pág 182

FILOSOFIA............................................................................. pág 185


Semana 1: Ética e moral - parte 1...........................................pág 185
Semana 2: Ética e moral - parte 2..........................................pág 189
Semana 3: Ética e moral - parte 3..........................................pág 193
Semana 4: Ética e moral - parte 4..........................................pág 198
Semana 5: Ética e moral - parte 5.........................................pág 202
Semana 6: Ética e moral - parte 6......................................... pág 207

LÍNGUA INGLESA....................................................................pág 211


Semana 1: Connecting old and young..................................... pág 211
Semana 2: Paintings that changed the world.........................pág 216
Semana 3: Noun phrases and concrete poetry...................... pág 221
Semana 4: Nossas raízes africanas: conhecer, entender para,
     então, ser...........................................................pág 226
Semana 5: De onde vem a escravidão?..................................pág 229
Semana 6: Slam poetry as a tool to reflect about
     your identity....................................................... pág 232

IV
ARTE.....................................................................................pág 237
Semana 1: Teatro e identidade - parte 1................................. pág 237
Semana 2: Teatro e identidade - parte 2...............................pág 240
Semana 3: Teatro na história................................................pág 242
Semana 4: Prática audiovisual..............................................pág 244
Semana 5: O áudio e o visual................................................. pág 247
Semana 6: Tecnologia e Produção Audiovisual na Arte
     Contemporânea .................................................pág 250

EDUCAÇÃO FÍSICA................................................................ pág 253


Semana 1: Futebol................................................................ pág 253
Semana 2: Futsal.................................................................. pág 258
Semana 3: Hóquei................................................................pág 262
Semana 4: Dança Folclórica.................................................. pág 265
Semana 5: Esportes Adaptados............................................ pág 268
Semana 6: Padrões Estéticos................................................pág 271

V
SECRETARIA DE ESTADO DE EDUCAÇÃO DE MINAS GERAIS

PLANO DE ESTUDO TUTORADO


COMPONENTE CURRICULAR: LÍNGUA PORTUGUESA
ANO DE ESCOLARIDADE: 2º ANO – EM
PET VOLUME: 04/2021
NOME DA ESCOLA:
ESTUDANTE:
TURMA: TURNO:
BIMESTRE: 4º TOTAL DE SEMANAS:
NÚMERO DE AULAS POR SEMANA: NÚMERO DE AULAS POR MÊS:

SEMANA 1

EIXO TEMÁTICO:
Compreensão e Produção de Textos.

TEMA/TÓPICO:
Gêneros/ Contexto de produção, circulação e recepção de textos.

HABILIDADES:
Ler textos de diferentes gêneros, considerando o pacto de recepção desses textos/ Reconhecer o objetivo
comunicativo (finalidade ou função sociocomunicativa) de um texto ou gênero textual.

CONTEÚDOS RELACIONADOS:
Gêneros Literários.

TEMA: Gênero Romance Policial


Olá, estudante! Estamos iniciando o último Plano de Estudo Tutorado do ano de 2021! Na SEMANA 1,
iremos falar sobre um gênero literário muito divertido e interessante: o romance policial.
O romance é um texto narrativo mais longo que a novela e o conto. Ele tem um número maior de perso-
nagens, conflitos, tempo e espaço. Os romances são classificados de acordo com a sua temática e os
tipos mais conhecidos são o de amor, de aventura, policial, de ficção científica, entre outros.
O romance policial foi criado por Edgar Allan Poe, que publicou em uma revista “Os assassinatos da rua
Morgue”. No Brasil, a primeira narrativa policial foi publicada em capítulos no jornal “A Folha”, em 1920:
“O mistério”, escrito por Coelho Neto, Afrânio Peixoto, Medeiros e Albuquerque e Viriato Corrêa.
Há dois sujeitos indispensáveis no enredo de um romance policial: o sujeito-criminoso e o sujeito-de-
tetive. A história ocorre a partir de um crime cometido e o leitor acompanha todos os procedimentos de
investigação, por meio do olhar do narrador.
Um romance policial pode ser clássico, centrado na investigação do detetive sobre a identidade do cri-
minoso; ou thriller, no qual a identidade do criminoso já é conhecida, mas é preciso saber se ele será ou
não capturado pelo detetive (romance policial de perseguição).

1
A narrativa policial, quando o leitor não sabe quem é o assassino, é centrada nos suspeitos: o desenro-
lar da trama faz com que diversas personagens causem desconfianças, nos levando a pensar em muitas
possibilidades de culpados.
Um exemplo de romance policial é o livro “Atemporal”, escrito por Poliana Nogueira. Vamos ler a sinopse
da narrativa, fornecida pela própria autora:

“Helena tem 19 anos e não está em uma boa fase. Desempregada, acabou de
levar o fora de Felipe, seu namorado, e ao encontrá-lo aos beijos com uma
famosa digital influencer resolve se esconder em um dos acervos do Museu
de Artes e Ofícios, indo parar misteriosamente no ano de 1940.
Apavorada, tenta entender como aquilo foi acontecer e, principalmente,
como fará para retornar a 2019. Por sorte, ela conhece Joana, que lhe dá
abrigo e a ajuda a encontrar uma forma de voltar para casa.
Enquanto isso, Helena faz turismo em Belo Horizonte, na década de quaren-
ta, e conhece várias personalidades, como os escritores Carlos Drummond
de Andrade, Pedro Nava, Fernando Sabino e Roberto Drummond. Como se
tudo isso não bastasse, ela toma conhecimento de um caso de sérias conse-
quências que marcaram a história da cidade, e cabe a ela intervir para que
um crime não ocorra.”
NOGUEIRA, Poliana. Disponível em: https://poliananogueiraescritora.com.br/produto/livro-
atemporal/ - Acesso em: 14 ago. 2021.

ATIVIDADES
Leia um trecho do romance policial “Atemporal”, de Poliana Nogueira, e responda o que se pede.

Uma criminosa
“Na quarta-feira, dia 23 de outubro, segundo dia de prazo que dei ao Melo Scott, recebo um bilhete pedin-
do que eu vá até o consultório dele às 18h, para buscar o projeto original. A princípio, fico com medo, mas
como ele marcou no seu ambiente de trabalho, caso seja necessário pedirei socorro aos seus clientes.
O dia da minha volta já está se aproximando e tudo corre bem. Francisco vai reproduzir a música do
Pato Fu na flauta e Melo Scott devolverá o projeto original, poupando Antônio de refazer todo o trabalho.
Poderei voltar tranquila para 2019.
Chego em frente ao casarão da Rua Espírito Santo pontualmente, às seis horas da tarde. Abro o portão
e subo as escadas até a recepção. A secretária não está em sua mesa, calculo que esteja dando algum
suporte no consultório. Vou à antessala e não gosto de não encontrar nenhum cliente aguardando para
ser atendido, contava com eles para me socorrerem em caso de um possível ataque. Sento no con-
fortável sofá e aguardo. Após alguns minutos de espera, resolvo bater na porta da sala do Melo Scott.
Ao encostar na porta, ela se abre.
- Doutor Melo! - chamo. Como não obtenho resposta, termino de abrir a porta e vejo que a sala está
vazia.
- Melo Scott! - chamo novamente.

2
Próxima à mesa há uma porta à esquerda, provavelmente a do banheiro. Vou até ela para ver se en-
contro o médico, mas não é preciso bater. Eu o encontro estirado ao chão, atrás da mesa, com o corpo
ensanguentado. Grito por socorro, mas não aparece ninguém para ajudar. Pelo estado do corpo, parece
estar morto. Nervosa, encosto minha orelha próxima à boca do cadáver para ter a certeza de que não
está respirando e vejo furos em seu peito, parecidos com golpes de facão. Não restam dúvidas, ele foi
assassinado. Ouço um barulho no jardim. Da janela, vejo policiais armados subindo a escada. Por medo
de ser incriminada e por não ter documentos de identificação, resolvo fugir. Corro até a antessala e sigo
por um corredor que sai na cozinha. Há uma porta de vidro com a chave na maçaneta. Abro a porta que
dá para os fundos da casa e vou até o muro”.
NOGUEIRA, 2021.

1 - Há um acontecimento retratado neste trecho que nos faz identificar que se trata de uma parte de um
romance policial. Que acontecimento é esse? Por que podemos relacionar o texto ao gênero romance
policial? Justifique sua resposta.

2 - O que difere um romance policial de um romance comum? Quais as principais características deste
gênero?

3
3 - Retome a leitura do texto e responda: por que Helena não presta socorro ao Dr. Melo Scott e prefere
fugir do local do crime?

Bom, estudante, chegamos ao fim da SEMANA 1. Espero você na próxima.

REFERÊNCIAS:
NOGUEIRA, Poliana. Atemporal. 1. ed. Belo Horizonte: Ed. do autor, 2021. Disponível em: <https://
poliananogueiraescritora.com.br/produto/livro-atemporal/>. Acesso em: 14 ago 2021.
PEREZ, Luana Castro Alves. Romance policial. Disponível em: https://www.portugues.com.br/li-
teratura/romance-policial.html - Acesso em: 14 ago 2021. Questões de concurso. Disponível em:
<https://www.qconcursos.com/questoes-de-concursos/questoes/9d360876-74>. Acesso em: 15
de maio de 2021.

4
SEMANA 2

EIXO TEMÁTICO:
Compreensão e Produção de Textos.

TEMA/TÓPICO:
Gêneros/ Contexto de produção, circulação e recepção de textos.

HABILIDADES:
Ler textos de diferentes gêneros, considerando o pacto de recepção desses textos/ Reconhecer o objetivo
comunicativo (finalidade ou função sociocomunicativa) de um texto ou gênero textual.

CONTEÚDOS RELACIONADOS:
Narrativas cantadas.

TEMA: Músicas narrativas


Ei, estudante. Como vai? A SEMANA 2 começa agora e vem com um tema bem legal: falaremos sobre
narrativas cantadas.
Uma história pode ser contada de várias formas: livros, filmes, novelas, séries, relatos orais, cartas… há
também um outro modo de narração, que é por meio de músicas.
Talvez você nunca tenha reparado, mas muitas canções contam histórias, que apresentam os mesmos
cinco elementos exigidos por qualquer narrativa (enredo, personagens, narrador, tempo e espaço).
Enredos diversos compõem músicas que conhecemos e muitas outras que ainda não ouvimos: histó-
ria de traição, como cantada por Naiara Azevedo em “50 reais”; história de toda uma vida, representa-
da, por exemplo, por “Faroeste caboclo”, contada por Legião Urbana; ou ainda histórias reais, como a
cantada em “Sou vitorioso”, de Mc Lele JP. Além dessas que te falei, ainda existem muitas outras.
Falaremos, entretanto, de uma música brasileira famosa que brinca muito com a questão temporal,
assim como o romance policial “Atemporal”, de Poliana Nogueira, estudado na SEMANA 1. O nome da
música é “Eu nasci há dez mil anos atrás”, escrita por Raul Seixas e Paulo Coelho e interpretada por Raul
Seixas. Vamos à letra da música:

5
Eu nasci há dez mil anos atrás

Um dia, numa rua da cidade Eu vi as velas se acenderem para Eu li!


Eu vi um velhinho sentado na o papa Eu li os símbolos sagrados de
calçada Vi Babilônia ser riscada no mapa umbanda
Com uma cuia de esmola Vi conde Drácula sugando sangue Eu fui criança pra poder dançar
E uma viola na mão novo ciranda
O povo parou para ouvir E se escondendo atrás da capa E quando todos praguejavam
Ele agradeceu as moedas Eu vi! contra o frio
E cantou essa música Eu fiz a cama na varanda
Que contava uma história Eu vi a arca de Noé cruzar os
Que era mais ou menos assim mares Não! Não!
Vi Salomão cantar seus salmos Eu tava junto com os macacos na
Eu vi Cristo ser crucificado pelos ares caverna
O amor nascer e ser assassinado Eu vi Zumbi fugir com os negros Eu bebi vinho com as mulheres na
Eu vi as bruxas pegando fogo pra floresta taberna
Pra pagarem seus pecados Pro Quilombo dos Palmares E quando a pedra despencou da
Eu vi! Eu vi! ribanceira
Eu também quebrei a perna
Eu vi Moisés cruzar o mar vermelho Eu vi o sangue que corria da mon- Eu também
Vi Maomé cair na terra de joelhos tanha
Eu vi Pedro negar Cristo por três Quando Hitler chamou toda Ale- Eu fui testemunha do amor de
vezes manha Rapunzel
Diante do espelho Vi o soldado que sonhava com a Eu vi a estrela de Davi brilhar no
Eu vi! amada céu
Numa cama de campanha E para aquele que provar que eu tô
Eu nasci (eu nasci) mentindo
Há dez mil anos atrás (eu nasci há Eu tiro o meu chapéu
10 mil anos)
E não tem nada nesse mundo
Que eu não saiba demais

A música conta a história de um senhor que cantava na rua e narrava a sua passagem por diversos fatos
históricos e literários, afirmando que havia nascido há mais de dez mil anos atrás e que, por isso, teria
vivido e visto tanta coisa. A canção apresenta hipérbole (figura de linguagem que expressa exagero),
intertextualidades e ironia. Ademais, a canção desafia o tempo cronológico, assim como Helena, per-
sonagem protagonista de “Atemporal”, que viaja no tempo.
Percebemos, então, como a música, além de despertar em nós diversas emoções, pode também narrar
histórias, sejam elas reais ou não.

ATIVIDADES

1 - Leia o trecho a seguir.


“Eu nasci há dez mil anos atrás
E não tem nada nesse mundo que eu não saiba demais”
(Eu nasci há dez mil anos atrás, Paulo Coelho e Raul Seixas. LP, Há dez mil anos atrás, Philips, 1976)

6
É possível observar, no trecho sublinhado, a seguinte figura de linguagem:
a) Metonímia.
b) Hipérbole.
c) Catacrese.
d) Ironia.
e) Sinestesia.

2 - Descreva alguma referência feita na música “Eu nasci há dez mil anos atrás” que você conheça.

3 - Qual a semelhança que há entre a música “Eu nasci há dez mil anos atrás”, de Raul Seixas e Paulo
Coelho, e o romance policial “Atemporal”, de Poliana Nogueira? Justifique sua resposta.

7
4 - Justifique: por que o título da música é “Eu nasci há dez mil anos atrás”? Qual a relação do título com
o conteúdo da canção?

Mais uma semana do PET 4 se encerra, estudante. Até a SEMANA 3.

REFERÊNCIAS:
Música Raul Seixas. Disponível em: https://www.letras.mus.br/raul-seixas/48309/ - Acesso em: 14
ago 2021.

8
SEMANA 3

EIXO TEMÁTICO:
Compreensão e Produção de Textos.

TEMA/TÓPICO:
Gêneros Contexto de produção, circulação e recepção de textos.

HABILIDADES:
Situar um texto no momento histórico de sua produção a partir de escolhas linguísticas (lexicais ou morfos-
sintáticas) e/ou de referências (sociais, culturais, políticas ou econômicas) ao contexto histórico.

CONTEÚDOS RELACIONADOS:
Intertextualidade.

TEMA: Intertextualidade
Estudante, estamos começando mais uma semana de nosso PET, na qual estaremos falando sobre in-
tertextualidade.
Hoje em dia, após tantos séculos de existência do homem e tantas produções sobre milhares de assun-
tos, é muito raro nos depararmos com um texto que seja totalmente inédito. Com frequência, percebe-
mos elementos de um texto em outro, mesmo que utilizados sem a intenção do autor. Esse fenômeno é
chamado de intertextualidade.
Isso ocorre quando o autor de um texto utiliza informação, estrutura, citação, referência de um outro texto.
A intertextualidade pode ter funções diferentes que variam de humor à crítica, por exemplo. É importante
ressaltar que ela ocorre em várias formas de comunicação como em músicas, filmes, imagens, novelas etc.
Existem duas maneiras de se construir a intertextualidade: a explícita, quando um texto cita outros
deixando claras as referências e indicando o autor do texto original. Dessa forma, os elementos para a
percepção da intertextualidade são dados ao leitor. Outra maneira é a implícita, em que há uma referên-
cia a elementos do texto original, sem, no entanto, explicitar o nome do texto ou o autor. Para perceber
esse tipo de intertextualidade, é importante que o leitor possua conhecimento prévio.
Além disso, há alguns tipos de intertextualidade. Veremos as principais:

Citação O tema da participação política pode ser visto em


Menção idêntica de um trecho que foi extraído diversas expressões artísticas. O Gabriel O Pen-
de outro texto. Geralmente, ela vem entre aspas sador, na canção “Até Quando?”, canta que “quan-
e tem como função sustentar uma ideia ou ilus- do a gente muda, o mundo muda com a gente.
trar um pensamento. A gente muda o mundo na mudança da mente”.

Epígrafe “O Brasil vai alcançar o pleno desenvolvimento


Citação colocada no início de um capítulo ou em quando respeitar o passado com o mesmo ardor
uma única página antes do texto principal. Ape- com que anseia pela conquista do futuro”
sar de essa citação não se misturar com o texto, Flávio Gutierrez
por vir antes dele, seu conteúdo deve estar rela-
Epígrafe do livro “Atemporal”, de Poliana Nogueira
cionado ao que será abordado.

9
Alusão “Eu vi Pedro negar Cristo por três vezes
Referência sutil ao texto original em que pode Diante do espelho
não citar o nome da obra ou autor, mas fazer Eu vi!”
somente uma leve menção a alguns elementos Trecho da música “Eu nasci há dez mil anos atrás”, em que há
presentes nele. A alusão apenas sugere. alusão à Bíblia.

Paráfrase
O termo “paráfrase” vem do grego (paraphrasis)
e significa “reproduzir algo de uma sentença”.
Esse recurso consiste em dizer uma mensagem
de outro modo. Quando ela é feita, o conteúdo
textual permanece o mesmo, porém, a forma de
expressar é alterada.
“O Grito”, de Edward Munch parafraseado pela série
“Os Simpsons”

Paródia
Releitura de outro texto, geralmente com o ob-
jetivo cômico, irônico ou crítico. A paródia ironi-
za o texto original, inverte seu sentido.

“Mona Lisa”, de Leonardo da Vinci, parodiada.

A intertextualidade é muito marcante na obra “Atemporal”, de Poliana Nogueira, já que a personagem


Helena, em suas aventuras por Belo Horizonte da década de 1940, se encontra com diversos auto-
res da literatura brasileira, tais como Carlos Drummond de Andrade, Pedro Nava, Fernando Sabino
e Roberto Drummond. Na música de Raul Seixas e Paulo Coelho, “Eu nasci há dez mil anos atrás”,
a intertextualidade pode ser encontrada em toda canção, que faz várias referências à literatura e
fatos históricos.
Agora que já vimos sobre o tema, vamos fazer alguns exercícios.

10
ATIVIDADES
1 - Fundamentando-se na tirinha ao lado, assinale
a alternativa CORRETA:
a) A intertextualidade é um recurso utilizado nos
textos de ficção de modo muito sazonal, logo, o que
se percebe na tirinha em análise é algo extraordi-
nário.
b) Do primeiro ao quarto quadrinho, o leitor identi-
fica que as personagens estão fazendo referência
a outro texto de natureza ficcional.
c) A expressão “Reis Magos”, citada por Mafalda e
refutada com intensidade por Manolito, remete o
leitor a reflexões exclusivamente textuais.
d) Há, no último quadrinho, remate interdiscursivo
que traduz para o leitor a finalidade inventiva de Ma-
falda e de Manolito, duas personagens argentinas.
e) A expressão “Reis Magos”, utilizada pelas per-
sonagens da tirinha, remete o leitor a um episódio
retratado pelas narrativas bíblicas.

2 - Releia um trecho da música “Eu nasci há dez mil anos atrás”, escrita por Raul Seixas e Paulo Coelho,
e responda o que se pede.
Eu vi a arca de Noé cruzar os mares
Vi Salomão cantar seus salmos pelos ares
Eu vi zumbi fugir com os negros pra floresta
Pro quilombo dos palmares
Eu vi!
O trecho é marcado pela alusão, um tipo de intertextualidade. Identifique a que são feitas as referências
descritas pelas alusões no trecho.

11
3 - Leia o trecho a seguir do livro “Atemporal”, de Poliana Nogueira:

Seguimos pela trilha.


- Meu caderno! - Lembra Pedro.
- Deve estar perto da árvore - Carlos fala.
- Vou buscar.
Pedro corre até o local onde o homem misterioso me atacou.
- Ele não pode perder aquele caderno. Tem várias memórias para transformar em um livro - avisa
Carlos.
- Vocês são escritores?
- Sim, apesar de o meu poema ter sido rejeitado pela crítica.
- Por quê? - fico curiosa.
- A estrutura baseada na repetição não os agradou. Não são todos que sabem apreciar uma poesia.
“Se cometesse um crime, bastava sumir reaparecer dez anos depois, e então o crime estaria prescrito,
mas em relação ao poema No meio do caminho, pode se passar cinquenta anos e ele nunca deixará de
ser julgado”.
- Achei o caderno! - avisa Pedro, se juntando a nós. - Isso é um verdadeiro baú de ossos.
Paro estupefata e os dois me olham, preocupados.
- Está se sentindo mal? - pergunta Carlos.
- Pedro, Baú de ossos... Carlos, No meio do caminho... - balbucio.
NOGUEIRA, 2021.

Pesquise e descubra: a partir das informações dadas pelo trecho e com base em seus conhecimentos
e suas pesquisas, identifique a quais dois escritores brasileiros são feitas referências no texto acima.
Justifique sua resposta.

REFERÊNCIAS:
NOGUEIRA, Poliana. Atemporal. 1. ed. Belo Horizonte: Ed. do autor, 2021.
Imagem Monalisa. Disponível em: https://photoarts.com.br/magazine/11-parodias-da-mona-lisa-
-para-voce-se-divertir/ - Acesso em: 14 ago 2021.
Imagem O grito. Disponível em: https://www.stoodi.com.br/blog/portugues/intertextualidade/ -
Acesso em: 14 ago 2021.
Música Raul Seixas. Disponível em: https://www.letras.mus.br/raul-seixas/48309/ - Acesso em: 14
ago 2021.

12
SEMANA 4

EIXO TEMÁTICO:
Compreensão e Produção de Textos.

TEMA/TÓPICO:
Gêneros/Textualização do discurso argumentativo.

HABILIDADES:
Reconhecer e usar mecanismos de coesão verbal em um texto ou sequência argumentativa.

CONTEÚDOS RELACIONADOS:
Coesão e Coerência.

TEMA: Coesão e Coerência


Tudo bem com você, estudante? A SEMANA 4 chegou com tudo! Nela, iremos falar sobre o que é e a
importância da coesão e da coerência para a construção de um texto.
Coesão é a ligação entre as partes do texto (palavras, expressões, frases, parágrafos) por meio de de-
terminados elementos linguísticos. Com ela, fica mais fácil ler e compreender um texto.
Veja um exemplo de texto coeso:
Último Recurso
Quando fazemos tudo para que nos amem e não conseguimos, resta-nos um último recurso: não fazer
mais nada. Por isso, digo, quando não obtivermos o amor, o afeto ou a ternura que havíamos solicitado,
melhor será desistirmos e procurar mais adiante os sentimentos que nos negaram. Não fazer esforços
inúteis, pois o amor nasce, ou não, espontaneamente, mas nunca por força de imposição. Às vezes,
é inútil esforçar-se demais, nada se consegue; outras vezes, nada damos e o amor se rende aos nossos
pés. Os sentimentos são sempre uma surpresa. Nunca foram uma caridade mendigada, uma compaixão
ou um favor concedido. Quase sempre amamos a quem nos ama mal, e desprezamos quem melhor nos
quer. Assim, repito, quando tivermos feito tudo para conseguir um amor, e falhado, resta-nos um só
caminho... o de mais nada fazer.
Clarice Lispector

Já a coerência é a relação semântica que se estabelece entre as diversas partes do texto, criando uma
unidade de sentido. Está ligada ao entendimento, à possibilidade de interpretação daquilo que se ouve
ou lê. Enquanto a coesão está para os elementos conectores de ideias no texto, a coerência está para a
harmonia interna do texto, o sentido.
A coesão e a coerência são bem amigas, andam sempre juntas, mas pode haver situações em que elas
estejam separadas. Observe:
Coeso e incoerente
“Os jornalistas se comprometem a divulgar artigos políticos de maneira polida e imparcial, no entanto
eles comumente afligem a opinião daqueles que se empenham em ter um cerne ou um ponto de vista
menos fundamentalista. ”

13
Do que o texto fala mesmo? O elemento coesivo “no entanto” estabelece uma relação de oposição com o
quê? Com o fato de os artigos ou os jornalistas afligirem a opinião de quem? Dos leitores, dos jornalistas
ou dos artigos políticos? Percebe que há uma confusão, que gera uma incompreensão do texto? Logo,
podemos dizer que não houve coerência, apesar de ter havido coesão.
Incoeso e coerente
Chinelos, vaso, descarga. Pia, sabonete. Água. Escova, creme dental, água, espuma, creme de barbear,
pincel, espuma, gilete, água, cortina, sabonete, água fria, água quente, toalha. Creme para cabelo, pente.
Cueca, camisa, abotoaduras, calça, meias, sapatos, gravata, paletó. Carteira, níqueis, documentos, cane-
ta, chaves, lenço. Relógio, maço de cigarros, caixa de fósforos, jornal. Mesa, cadeiras, xícara e pires, pra-
to, bule, talheres, guardanapos. Quadros. Pasta, carro. Cigarro, fósforo. Mesa e poltrona, cadeira, cinzeiro,
papéis, telefone, agenda, copo com lápis, canetas, blocos de notas, espátula, pastas, caixas de entrada,
de saída, vaso com plantas, quadros, papéis, cigarro, fósforo. Bandeja, xícara pequena. Cigarro e fósforo.
Papéis, telefone, relatórios, cartas, notas, vales, cheques, memorandos, bilhetes, telefone, papéis. Re-
lógio. Mesa, cavalete, cinzeiros, cadeiras, esboços de anúncios, fotos, cigarro, fósforo, bloco de papel,
caneta, projetos de filmes, xícara, cartaz, lápis, cigarro, fósforo, quadro-negro, giz, papel. Mictório, pia.
Água. Táxi, mesa, toalha, cadeiras, copos, pratos, talheres, garrafa, guardanapo, xícara. Maço de cigarros,
caixa de fósforos. Escova de dentes, pasta, água. Mesa e poltrona, papéis, telefone, revista, copo de papel,
cigarro, fósforo, telefone interno, externo, papéis, prova de anúncio, caneta e papel, relógio, papel, pasta,
cigarro, fósforo, papel e caneta, telefone, caneta e papel, telefone, papéis, folheto, xícara, jornal, cigarro,
fósforo, papel e caneta. Carro. Maço de cigarros, caixa de fósforos. Paletó, gravata. Poltrona, copo, revis-
ta. Quadros. Mesa, cadeiras, pratos, talheres, copos, guardanapos. Xícaras. Cigarro e fósforo. Poltrona,
livro. Cigarro e fósforo. Televisor, poltrona. Cigarro e fósforo. Abotoaduras, camisa, sapatos, meias, calça,
cueca, pijama, espuma, água. Chinelos. Coberta, cama, travesseiro.
Ricardo Ramos
Fonte: https://revistamacondo.wordpress.com/2012/02/29/conto-circuito-fechado-ricardo-ramos/

Perceba que não houve nenhum elemento conectando as frases; houve apenas justaposição delas.
Realmente não houve coesão, mas houve total coerência, pois as frases mantêm relações de sentido.
A “incoesão”, ausência de elementos conectores ou referenciadores, não prejudicou o sentido do texto,
ou seja, a coerência.
Agora vamos treinar o uso da coesão e da coerência?

ATIVIDADES

1 - Escreva um texto opinativo em que você discuta as consequências da violência urbana no Brasil.
Os textos abaixo são para motivar a sua escrita. Atente-se para a coesão e para a coerência do seu texto
e escreva-o na modalidade formal da língua portuguesa.

14
Texto I

FONTE: Atlas da violência/Ipea.

Texto II
A violência urbana tornou-se um problema social grave em todo o país a partir dos anos 1990. Nessa
época, a falta de planejamento urbano e o tráfico de drogas fizeram eclodir “guerras” nas periferias
das cidades.
Segundo o Mapa da Violência 2012, elaborado pelo Instituto Sangari, o número de assassinatos no
país passou de 13.910 em 1980 para 49.932 em 2010, correspondendo a um aumento de 259% ou o
equivalente ao crescimento de 4,4% ao ano. O número é superior a países em conflitos, como Iraque
e Afeganistão, e comparado a nações africanas e caribenhas com governos e instituições jurídicas
precárias. Na América do Sul, somente Venezuela (45,1) e a Colômbia (33,4) possuem taxas maiores.
Entre as principais causas da violência estão fatores sociais (pobreza, falta de educação e moradia),
despreparo das polícias, lentidão na Justiça e superlotação de presídios.
FONTE: https://vestibular.uol.com.br/resumo-das-disciplinas/atualidades/violencia-urbana-homicidios-no-brasil-superam-
numerosde-paises-em-guerra.htm#Direto%20ao%20ponto

15
REFERÊNCIAS:
ANDRADE, Magno Felipe de. Preparatório Concurso Mário Campos. Contagem, 2019.

16
SEMANA 5

EIXO TEMÁTICO:
Linguagem e Língua.

TEMA/TÓPICO:
A regência verbal e nominal no português padrão (PP) e não padrão (PNP).

HABILIDADES:
Reconhecer e usar mecanismos de regência verbal e nominal, produtiva e autonomamente/ Corrigir um texto
ou sequência textual, considerando a necessidade de uso da norma padrão de regência verbal e nominal.

CONTEÚDOS RELACIONADOS:
Regência Verbal e Regência Nominal.

TEMA: Regência Verbal e Regência Nominal


A penúltima SEMANA do nosso PET chegou e trouxe a gramática junto. Vamos estudar o que é e como
funcionam as regências verbal e nominal.
A regência nominal estuda os casos em que nomes (substantivos, adjetivos e advérbios) exigem outra
palavra para completar-lhes o sentido. Em geral, a relação entre um nome e o seu complemento é esta-
Belecida por uma preposição.

“O rapaz se mostrava insensível ao sofrimento Para saber mais…


da garota.” A classe de palavras preposição é fixa, ou seja,
Perceba que o adjetivo insensível necessita de um existem algumas palavras que se classificam
complemento para ter sentido completo (sofrimen- como preposição e elas não mudam. Abaixo,
to da garota). O adjetivo se liga ao seu complemen- um quadro com as preposições que temos em
to por meio da preposição a (a+o=ao). língua portuguesa para você decorar e não er-
rar mais.
“Era uma pessoa ávida de emoções fortes”
Aqui temos mais um caso de regência nominal.
O adjetivo ávida se liga ao seu complemento (emo-
ções fortes) por meio da preposição de.
“Infelizmente, o sangue do doador não era compatí-
vel com o do paciente”
Neste caso, compatível é ligado ao complemento
por meio da preposição com.

Já a regência verbal estuda a relação que se estabelece entre o verbo (termo regente) e seu comple-
mento (termo regido).
“Não gosto de assistir a filmes de ação”

17
O verbo assistir, com o sentido de “ver, presenciar”, necessita do auxílio da preposição a para se ligar ao
seu complemento.
“O médico assistiu o doente durante toda a noite”
Porém, o verbo assistir, com o sentido de “acompanhar, prestar assistência”, não precisa de preposição.

ATIVIDADES

1 - Leia o infográfico a seguir que indica a regência de várias palavras.

2 - Relacione a primeira coluna com a segunda,


de modo que as regências nominais fiquem de
acordo com a norma-padrão.
Primeira coluna
a) A água é essencial
b) O programa não é compatível
c) Estávamos ansiosos
d) Júlia é muito querida
e) O site não estava acessível
f) O exercício era muito fácil
g) O gato era inseparável
h) Ele é rico
Segunda coluna
(  ) pelos amigos.
(  ) aos usuários.
(  ) em humor.
(  ) de fazer.
(  ) com a máquina.
(  ) para a continuidade da vida.
(  ) pelos resultados das provas
(  ) do seu dono.

3 - No enunciado “Unidos contra a exploração


sexual de crianças e adolescentes”, qual é o
sentido da preposição contra?
a) direção.
b) oposição.
c) finalidade.
d) posse.

18
4 - Complete as frases com a preposição exigida pela regência dos verbos na norma-padrão:

a) Assisti ____________ um fantástico espetáculo de mágicas.

b) São estes os alunos que desobedeceram __________ regulamento da escola.

c) Maria prefere contos _______ crônicas.

d) Os avós querem ________ seus netos tanto quanto _______ seus filhos.

e) Este é um direito que assiste ____________ o professor e também _________ o aluno.

5 - Empregue ou não nas frases a seguir uma das proposições abaixo, de acordo com a regência do
verbo recomendada pela norma-padrão.
a - com - de - por

a) A peça ____________ que assistimos estreou no mês passado.

b) A pessoa ____________ que mais amei na vida não me amou.

c) A pessoa ____________ quem mais gostei na vida não me amou.

d) A estudante ____________ quem me correspondo virá ao Brasil no próximo mês.

e) Você compreendeu ____________ que não participaremos da festa?

Muito bom, estudante. Chegamos ao fim da SEMANA 5!

REFERÊNCIAS:
ANDRADE, Magno Felipe de. Preparatório Concurso Mário Campos. Contagem, 2019.
CEREJA, Willian Roberto; MAGALHÃES, Thereza Cochar. Gramática: texto, reflexão e uso. 4.ed. São
Paulo: Atual, 2012.

19
SEMANA 6

EIXO TEMÁTICO:
Compreensão e Produção de Textos.

TEMA/TÓPICO:
Gêneros/ Contexto de produção, circulação e recepção de textos.

HABILIDADES:
Ler textos de diferentes gêneros, considerando o pacto de recepção desses textos/ Reconhecer o objetivo
comunicativo (finalidade ou função sociocomunicativa) de um texto ou gênero textual.

CONTEÚDOS RELACIONADOS:
Interpretação de textos.

TEMA: Interpretação de textos


Estamos começando a última semana do último PET. E vamos fechar este ciclo realizando algumas
atividades de interpretação de textos.
Estudante, para realizar as atividades, você deve retomar os conteúdos estudados no Plano de Estudo
Tutorado Volume 4.
Os exercícios correspondem à matéria de romance policial, músicas narrativas, intertextualidade, coe-
são, coerência e regências verbal e nominal.

ATIVIDADES

1 - Leia o trecho do romance policial “Atemporal”, de Poliana Nogueira, e responda o que se pede:

Às seis da tarde, saio em direção ao Bairro Floresta. Pego o bonde e desço próximo à casa que Antô-
nio alugou para se encontrar com a amante. As luzes estão acesas, demonstrando que Sofia já che-
gou. Passo pelo portão e toco a campainha. Ninguém atende. Coloco o ouvido na porta e não escuto
nenhum barulho vindo de dentro. Bato na porta e, sem paciência, rodo a maçaneta aberta. está
— Sofia! — Chamo. — É a Helena.
Ninguém responde, entro na casa. As luzes de todos os cômodos estão acesas. Como não tem nin-
guém na sala, olho a cozinha e sigo o corredor em direção ao quarto principal. No chão, ao lado da
cama, está o corpo de Sofia.
— Sofia! — grito.
Encosto o ouvido no peito dela e vejo que ainda respira. Noto que próxima a ela há uma taça quebra-
da. Escuto um barulho na sala e deduzo ser a polícia novamente. As janelas estão fechadas, não tem
para onde fugir. Ouço passos no corredor e me escondo embaixo da cama. Vejo um par de sapatos
masculinos próximo à porta, que se ajoelha perto do corpo estendido no chão.
NOGUEIRA, 2021.

20
Há um acontecimento retratado neste trecho que nos faz identificar que se trata de uma parte de um
romance policial. Que acontecimento é esse? Por que podemos relacionar o texto ao gênero romance
policial? Justifique sua resposta.

2 - Leia um trecho da música “Eu nasci há dez mil anos atrás” e responda:
Eu vi Cristo ser crucificado
O amor nascer e ser assassinado
Eu vi as bruxas pegando fogo
Pra pagarem seus pecados
Eu vi!
Qual o tipo de intertextualidade feita aqui? Justifique sua resposta.

3 - Retome SEMANA 1 do PET 4, releia a sinopse do romance policial de Poliana Nogueira e responda:
a partir das informações dadas pela sinopse, por que você acha que o título do livro é “Atemporal”?
Justifique sua resposta.

21
4 - Leia período seguinte, retirado do livro “Atemporal”:
“Quando o programa termina, Juliana vai até a janela e observa a rua.”
A palavra destacada pertence a qual classe de palavras? Por que essa palavra acompanha o verbo ir
(vai)? Há aqui uma relação de regência verbal? Justifique sua resposta.

5 - O texto a seguir é composto somente por verbos. Leia-o e responda o que se pede:

Como se conjuga um empresário


Acordou. Levantou-se. Aprontou-se. Lavou-se. Barbeou-se. Enxugou-se. Perfumou-se. Lanchou.
Escovou. Abraçou. Beijou. Saiu. Entrou. Cumprimentou. Orientou. Controlou. Advertiu. Chegou.
Desceu. Subiu. Entrou. Cumprimentou. Assentou-se. Preparou-se. Examinou. Leu. Convocou. Leu.
Comentou. Interrompeu. Leu. Despachou. Conferiu. Vendeu. Vendeu. Ganhou. Ganhou. Ganhou. Lu-
crou. Lucrou. Lucrou. Lesou. Explorou. Escondeu. Burlou. Safou-se. Comprou. Vendeu. Assinou.
Sacou. Depositou. Depositou. Depositou. Associou-se. Vendeu-se. Entregou. Sacou. Depositou.
Despachou. Repreendeu. Suspendeu. Demitiu. Negou. Explorou. Desconfiou. Vigiou. Ordenou. Tele-
fonou. Despachou. Esperou. Chegou. Vendeu. Lucrou. Lesou. Demitiu. Convocou. Elogiou. Bolinou.
Estimulou. Beijou. Convidou. Saiu. Chegou. Despiu-se. Abraçou. Deitou-se. Mexeu. Gemeu. Fungou.
Babou. Antecipou. Frustrou. Virou-se. Relaxou-se. Envergonhou-se. Presenteou. Saiu. Despiu-se.
Dirigiu-se. Chegou. Beijou. Negou. Lamentou. Justificou-se. Dormiu. Roncou. Sonhou. Sobressal-
tou-se. Acordou. Preocupou-se. Temeu. Suou. Ansiou. Tentou. Despertou. Insistiu. Irritou-se. Te-
meu. Levantou. Apanhou. Rasgou. Engoliu. Bebeu. Rasgou. Engoliu. Bebeu. Dormiu. Dormiu. Dor-
miu. Acordou. Levantou-se. Aprontou-se .
MIMO.Como se conjuga um empresário. In: PINILIA, Aparecida; RIGONI, Cristina;INDIANI,Maria Thereza. Coesão e coerência como
mecanismo para a construção do texto.

a) O texto lido apresenta coerência nas informações?

22
b) Há coesão no texto “Como se conjuga um empresário”? Justifique sua resposta.

c) Justifique: qual a relação entre o título e o conteúdo do texto?

Estudante, nós chegamos ao fim do Plano de Estudo Tutorado Volume 4, o último PET de 2021. Espero
que, embora diferente, você tenha tido um bom ano de estudo. Agora é focar no 3º ano do Ensino Médio
para encerrar o seu ciclo de estudos na educação básica com chave de ouro.

REFERÊNCIAS:
NOGUEIRA, Poliana. Atemporal. Belo Horizonte: Ed. do autor, 2021.
Música Raul Seixas. Disponível em: https://www.letras.mus.br/raul-seixas/48309/ - Acesso em: 14
ago 2021.
Um texto só com verbos? Disponível em: http://ivanetenunes.blogspot.com/2014/09/como-se-
-conjuga-um-empresario.html - Acesso em: 14 ago 2021.

23
SECRETARIA DE ESTADO DE EDUCAÇÃO DE MINAS GERAIS

PLANO DE ESTUDO TUTORADO


COMPONENTE CURRICULAR: MATEMÁTICA
ANO DE ESCOLARIDADE: 2º ANO – EM
PET VOLUME: 04/2021
NOME DA ESCOLA:
ESTUDANTE:
TURMA: TURNO:
BIMESTRE: 4º TOTAL DE SEMANAS:
NÚMERO DE AULAS POR SEMANA: NÚMERO DE AULAS POR MÊS:

SEMANA 1

EIXO TEMÁTICO:
Geometria e Medidas.

TEMA/TÓPICO(S):
Geometria Métrica e de Posição. / 30. Prismas e cilindros.

HABILIDADE(S):
30.1. Identificar os vértices, as arestas e as faces de um prisma.
30.2. Resolver problemas que envolvam o cálculo da diagonal de um paralelepípedo retângulo.

CONTEÚDOS RELACIONADOS:
Geometria espacial, noções primitivas de geometria espacial, sólidos geométricos, poliedros.

TEMA: Geometria espacial e os sólidos geométricos


Caro (a) estudante, nesta semana você vai reconhecer os conceitos básicos da geometria espacial e dos
sólidos geométricos.

APRESENTAÇÃO
Geometria espacial de posição
A geometria é a ciência que estuda as medidas das formas de figuras planas ou espaciais, além das suas
posições relativas no espaço e também as suas propriedades. A geometria espacial de posição estuda
as figuras geométricas espaciais, também conhecidas como sólidos geométricos, e as suas relativas
posições no espaço.
Conceitos fundamentais da geometria
Para termos um melhor entendimento sobre os assuntos que iremos trabalhar nesta semana, precisa-
mos relembrar alguns conceitos da geometria. Observe a figura geométrica espacial abaixo (paralelepí-
pedo) abaixo e vamos extrair os conceitos fundamentais da geometria. São eles:

24
Ponto: na figura temos os pontos A, B, C, D, I, J, K e L.
Os pontos desta figura representam seus vértices.
Reta: . Esta reta passa sobre uma das arestas deste só-
lido geométrico.
Semirreta: 𝐴𝐴𝐴𝐴 . A semirreta parte do vértice A do sólido pas-
sa pelo ponto B, também vértice desta figura.
Segmento de reta: AB, AD, DC, CB, IJ, IL, LK, KJ, AI, BJ, CK
e DL. Estes segmentos representam as arestas da figura
espacial geométrica.
Plano: ABCD. Uma das faces deste sólido está contida nes-
te plano.
Espaço: é o conjunto de todos os pontos.

Postulados
Postulados da reta
1º) Dois pontos distintos do espaço determinam uma única reta.
2º) Em uma reta existem infinitos pontos.
Uma reta pode ser representada por uma letra minúscula s, r, t, u, ... ou pela junção de duas letras maiús-
culas (que representam dois pontos pertencentes a ela) acrescido de uma seta dupla acima destas le-
tras conforme exemplo a seguir.

Reta ,... dentre outras diversas representações possíveis.

Postulados do plano
1º) Três pontos não colineares do espaço determinam um único plano.
2º) Em um plano existem infinitos pontos.
3º) Por uma reta passam infinitos planos.

1º) 2º)

25
3º) Imagem disponível em: https://www.somatematica.com.br/
emedio/espacial/espacial1.php. Acesso em: 12 jul. 2021.

Posições relativas entre duas retas no espaço


• Retas coplanares: são retas que estão contidas em um mesmo plano.
Exemplos: t e w; r e s; t e r; w e v; v e s.
• Retas reversas: são retas que não estão contidas em um mesmo plano.
Exemplos: t e v; v e r; w e s; w e r.
• Retas concorrentes: são retas que possuem um único ponto em comum.
Exemplos: w e v; w e t; w e u; s e v.
• Retas perpendiculares: são retas que, além de possuírem um ponto em comum, formam ângulos
de 90° entre elas.
Exemplos: w e t; w e u; w e v.
• Retas paralelas: são retas coplanares que não possuem ponto em comum.
Exemplos: r e t; u e r; r e s.
• Retas coincidentes: são aquelas que possuem todos os seus pontos em comum.
Exemplo: u e t.

26
Posições relativas entre dois planos no espaço
• Planos paralelos (fig. 1): são aqueles que não possuem ponto em comum.
• Planos secantes ou concorrentes (fig. 2): são aqueles que possuem uma única reta em comum.
• Planos perpendiculares (fig. 3): são planos concorrentes em que um dos planos possui uma reta
perpendicular ao outro.
• Planos coincidentes (fig. 4): são aqueles que possuem todos os seus pontos em comum.

Figura 1 Figura 2 Figura 3 Figura 4


Figuras 2 e 4 disponíveis em: https://iranmarkus.wordpress.com/2017/08/17/geometria-de-posicao-de-planos/.
Acesso em 19 jul. 2021.

Posições relativas entre uma reta e um plano


• Reta está contida no plano: quando uma reta possui dois pontos distintos que pertencem tam-
bém a um plano, logo ela está contida neste plano.
• Reta paralela ao plano: uma reta é paralela á um plano quando não possuírem nenhum ponto em
comum entre eles.
• Reta secante ao plano: se uma reta possui apenas um ponto em comum com um plano, dizemos
que eles são secantes.
• Reta perpendicular ao plano: uma reta r é perpendicular a um plano se, e somente se, r é perpen-
dicular a todas as retas de que passam pelo ponto de intersecção de r com esse plano.

PARA SABER MAIS:


Leia também: https://www.educamaisbrasil.com.br/enem/matematica/geometria-espacial. Acesso
em 12 ago. 2021.
Leia também: https://www.preparaenem.com/matematica/geometria-espacial.htm. Acesso em: 12
ago. 2021.

ATIVIDADES
1 - (UFMS – 2010) A seguir foram feitas afirmações sobre geometria espacial, assinale a(s) correta(s).
(  ) Toda reta paralela a dois planos, não paralelos, é paralela à interseção deles.
(  ) Toda reta que contém dois pontos de um plano pertence a esse plano.
(  ) A partir de quatro pontos não coplanares, são definidos exatamente quatro planos distintos.
(  ) Três retas concorrentes num único ponto definem um único plano.
(  ) Toda reta perpendicular a duas retas não paralelas pertence ao plano definido por essas duas retas
não paralelas.

27
2 - Sobre as posições relativas de retas no espaço, enumere a segunda coluna de acordo com a primeira.

1. Retas coplanares ( ) Não possuem pontos em comum e passam em um mesmo plano.
2. Retas reversas ( ) Estão sempre em um mesmo plano.
3. Retas concorrentes ( ) Não possuem pontos em comum e estão em planos distintos.
4. Retas perpendiculares ( ) Estão em um mesmo plano e possuem pontos em comum.
5. Retas paralelas ( ) Formam um ângulo reto entre elas.
6. Retas coincidentes ( ) Estão em um mesmo plano e formam um ângulo entre elas.

3 - Sobre os postulados da geometria espacial, é INCORRETO afirmar que:


a) Se uma reta existem infinitos pontos, logo quaisquer dois pontos definem uma outra reta qualquer.
b) Infinitos planos passam por uma reta.
c) Três pontos definem um plano, se e somente se, não estiverem alinhados.
d) Para definirmos uma reta, precisamos de 2 pontos quaisquer.
e) Existem infinitos pontos e infinitas retas em um plano.

4 - (EsPCEx – 2009) Considere duas retas r e s no espaço e quatro pontos distintos, A, B, C e D, de modo
que os pontos A e B pertencem à reta r e os pontos C e D pertencem à reta s. Dentre as afirmações abaixo
I – Se as retas e são concorrentes, então r e s são necessariamente concorrentes.
II – Os triângulos ABC e ABD serão sempre coplanares.
III – Se e forem concorrentes, então as retas r e s são coplanares.
Pode-se concluir que
a) somente a I é verdadeira.
b) somente a II é verdadeira.
c) somente a III é verdadeira.
d) as afirmações II e III são verdadeiras
e) as afirmações I e III são verdadeiras.

28
SEMANA 2

EIXO TEMÁTICO:
Geometria e Medidas.

TEMA/TÓPICO(S):
Geometria Métrica e de Posição. / 30. Prismas e cilindros.

HABILIDADE(S):
30.1. Identificar os vértices, as arestas e as faces de um prisma.

CONTEÚDOS RELACIONADOS:
Sólidos geométricos, poliedros, relação de Euler, cubo, prismas.

TEMA: Poliedros
Caro (a) estudante, nesta semana você vai reconhecer os poliedros e as suas características.
Poliedros
Os poliedros são sólidos geométricos limitados por superfícies planas poligonais. Os poliedros pos-
suem três elementos básicos, que são: faces, arestas e vértices. Observe na figura abaixo cada um dos
elementos de um poliedro.

Imagem disponível em: https://brasilescola.uol.com.br/matematica/poliedros.htm. Acesso em 19 jul. 2021.

As faces: que são os polígonos que delimitam e formam o só-


lido. Neste poliedro temos 5 faces. ABDF, ACEF, BCED, DFE e
ABC.
As arestas: são as linhas de encontro entre as faces do polí-
gono. Temos 9 arestas neste poliedro, que são os segmentos
de retas BC, BA, AC, DF, DE, FE, BD, CE e AF.
Os vértices: são os pontos de intercessão entre as arestas
do polígono. Este poliedro possui 6 vértices. A, B, C, D, E e F.
Este poliedro é chamado de pentaedro, pois possui 5 faces,
mas também pode ser chamado de prisma de base triangular.

29
Classificação dos poliedros
Podemos classificar um poliedro de acordo com o número de faces que ele possui. Observe os nomes na
tabela abaixo os principais poliedros:

Nome do poliedro Número de faces Nome do poliedro Número de faces


Tetraedro 4 Undecaedro 11
Pentaedro 5 Dodecaedro 12
Hexaedro 6 Tridecaedro 13
Heptaedro 7 Tetradecaedro 14
Octaedro 8 Pentadecaedro 15
Eneaedro 9 ... ...
Decaedro 10 Icosaedro 20

Outra classificação possível para um poliedro é se ele é convexo ou não convexo.

Um poliedro é convexo quando todos os


segmentos de reta que ligam dois pontos
quaisquer pertencentes a ele, está total-
mente dentro deste poliedro. Caso con-
trário será um poliedro não convexo.
Observe que para decidir se um poliedro
é ou não convexo, devemos considerar
segmentos de reta conectando dois de
seus pontos. Caso pelo menos um desses
segmentos de reta “passar” por pontos ex-
ternos ao poliedro, dizemos que ele é não
convexo. Caso contrário ele será dito um
poliedro convexo.
Disponível em: https://casadamatematica.com.br/poliedros-
absolutamente-tudo/. Acesso em 15 jul. 2021.

Relação de Euler
Em todo poliedro convexo com A arestas, V vértices e F faces é válida a relação de Euler, conforme a
igualdade a seguir:
V+F=A+2
Exemplo
Um poliedro convexo possui quatro faces hexagonais e quatro faces triangulares. Qual é o número de
vértices e arestas desse poliedro?
Resolução: como cada aresta compartilha duas faces devemos calcular o número de arestas em cada
um dos tipos de faces.

30
Arestas das faces hexagonais Arestas das faces triangulares
4$6 24 𝟒𝟒 $ 𝟑𝟑 𝟏𝟏𝟏𝟏
𝐴𝐴 = ⇒ 𝐴𝐴 = ⇒ 𝐴𝐴 = 12 𝐴𝐴 = ⇒ 𝐴𝐴 = ⇒ 𝐴𝐴 = 𝟔𝟔
2 2 𝟐𝟐 𝟐𝟐

Somando as arestas, temos que este polígono possui 18 arestas. Agora podemos utilizar a relação de
Euler para descobrir o número de vértices deste poliedro.
V+F=A+2
V + 8 = 18 + 2
V = 20 – 8
V = 12

Poliedros de Platão
Os poliedros convexos, chamados Poliedros de Platão, são aqueles que satisfazem simultaneamente as
três condições a seguir:
• Todas as suas faces têm o mesmo número de arestas.
• De cada vértice partem o mesmo número de arestas.
• A relação de Euler é válida.
Existem apenas cinco classes de poliedros de Platão que são: tetraedros, hexaedros, octaedros, dode-
caedros e icosaedros. Estes poliedros não precisam ser necessariamente regulares.

Poliedros regulares
Um poliedro convexo é regular quando todas as suas faces são polígonos regulares e congruentes entre
si e de cada um dos seus vértices partem o mesmo número de arestas.
Existem apenas 5 poliedros regulares convexos. São eles o tetraedro regular, o cubo, o octaedro regu-
lar, o dodecaedro regular e o icosaedro regular.

Disponível em: https://mundoeducacao.uol.com.br/matematica/solidos-de-platao.htm. Acesso em: 15 jul. 2021.

PARA SABER MAIS:


Assista o Vídeo: https://www.youtube.com/watch?v=TnbP-oYFMXI. Acesso em: 15 jul. 2021.
Assista o Vídeo: https://www.youtube.com/watch?v=UbGevL3sp6U. Acesso em: 15 jul. 2021.
Leia também: https://mundoeducacao.uol.com.br/matematica/solidos-de-platao.htm. Acesso em: 15
jul. 2021.

31
ATIVIDADES
1 - Dado os poliedros abaixo identifique o que se pede:

a) Quantos e quais são os vértices.

b) Quantos e quais são as arestas.

c) Quantos e quais são as faces.

d) Qual é o nome do poliedro.

a) Quantos e quais são os vértices.

b) Quantos e quais são as arestas.

c) Quantos e quais são as faces.

d) Qual é o nome do poliedro.

a) Quantos e quais são os vértices.

b) Quantos e quais são as arestas.

c) Quantos e quais são as faces.

d) Qual é o nome do poliedro.

2 - Um poliedro convexo é composto por 7 faces e 15 arestas, quantos vértices ele possui?

3 – Um poliedro possui 2 faces octogonais e oito faces retangulares, determine o número de arestas e
vértices desse poliedro.

32
4 – Observe os poliedros abaixo analise as afirmações.

Disponível em: https://mundoeducacao.uol.com.br/matematica/classificacao-poliedros.htm. Acesso em: 19 jul. 2021.

I. O primeiro poliedro é um poliedro de Platão, já o segundo não é.


II. Ambos poliedros são convexos.
III. A relação de Euler é válida para o primeiro poliedro, mas não é válida para o segundo.
IV. A classificação dos poliedros quanto ao número de faces é hexaedro para o primeiro e heptaedro
para o segundo.
Considerando as afirmações verdadeiras (V) ou falsas (F), a alternativa que apresenta a sequência cor-
reta para as afirmações de I a IV é:
a) V, F, F, V.
b) F, F, V, V.
c) V, V, F, V.
d) F, V, F, F.
e) V, F, V, F.

5 - (PUCAMP – SP) Sobre as sentenças:


I. Um octaedro regular tem 8 faces quadradas.
II. Um dodecaedro regular tem 12 faces pentagonais.
III. Um icosaedro regular tem 20 faces triangulares.
É correto afirmar que, apenas:
a) I é verdadeira.
b) II é verdadeira.
c) III é verdadeira.
d) I e III são verdadeiras.
e) II e III são verdadeiras.

33
SEMANA 3

EIXO TEMÁTICO:
Geometria e Medidas.

TEMA/TÓPICO(S):
Geometria Métrica e de Posição.
30. Prismas e cilindros.
33. Planificações de figuras tridimensionais.
35. Áreas laterais e totais de figuras tridimensionais.
36. Volumes de sólidos.

HABILIDADE(S):
30.1. Identificar os vértices, as arestas e as faces de um prisma.
33.1. Reconhecer a planificação de figuras tridimensionais usuais: cubo, paralelepípedo retangular, prismas
retos, pirâmide, cilindro e cone.
35.1. Resolver problemas que envolvam o cálculo da área lateral ou total de figuras tridimensionais.
36.1. Resolver problemas que envolvam o cálculo de volume de sólidos.

CONTEÚDOS RELACIONADOS:
Sólidos geométricos, cubo, paralelepípedo, prismas.

TEMA: Prismas
Caro (a) estudante, nesta semana você vai reconhecer as características de prismas e também do cubo,
irá identificar também as suas planificações e será capaz de realizar os cálculos de áreas superficiais e
dos seus respectivos volumes.
Prisma
Podemos definir um prisma, considerando dois planos distintos e paralelos, α e β, um polígono convexo
contido no plano α e uma reta concorrente aos planos α e β. A reunião de todos os segmentos de
reta paralelos a esta reta com uma de suas extremidades no polígono contido em α e outra contida
em β damos o nome de prisma.

34
Elementos de um prisma
Um prisma possui diversos elementos, que são:
Bases: são os polígonos contidos nos planos α e β, neste caso EFGH e ABCD;
Faces laterais: são os paralelogramos ABFE, BCGF, CDHG e ADHE;
Arestas: em um prisma existem arestas das bases que são AB, BC, CD, AD, EF, FG, GH e EH e as arestas
laterais que são AE, BF, CG e DH;
Altura: é a distância h entre os planos das bases.

Prismas retos e oblíquos


Os prismas retos são aqueles que possuem as suas arestas laterais perpendiculares às arestas da sua
base, já os prismas oblíquos possuem suas arestas laterais não perpendiculares à base.

Disponível em: https://brasilescola.uol.com.br/o-que-e/matematica/o-que-prisma.htm. Acesso em 30 jul. 2021.

Um prisma pode ser nomeado de acordo com o polígono de sua base. Damos o nome de prisma triangu-
lar se sua base for triangular por exemplo, ou pentagonal se sua base for um pentágono. Veja os exem-
plos abaixo.

Prisma Triangular Prisma quadrangular Prisma pentagonal Prisma hexagonal


Disponível em: https://www.educamaisbrasil.com.br/enem/matematica/prisma. Acesso em: 15 jul. 2021.

Área superficial de um prisma


A área superficial das faces de um prisma é calculada pela soma das áreas das faces laterais (Al ) com
as áreas de suas bases (Ab ), estas áreas superficiais somadas damos o nome de área superficial total
do prisma (At ).
At =Al + Ab
A planificação de um sólido são as figuras geométricas das suas superfícies externas em duas dimen-
sões, como se estivéssemos abrindo o sólido, mas lembre-se que esta representação nos mostra so-
mente as suas faces, e não o sólido como um todo. Esta área planificada é a área superficial do sólido.

35
Prisma triangular e a sua planificação

Volume de um prisma
O volume de um prisma é calculado multiplicando-se a área de sua base (Ab ), por sua altura (h).
V =Ab ⋅ h

O cubo
O cubo é um poliedro regular de seis faces (hexaedro), nele todas as suas arestas são iguais e todas as
suas faces são quadradas e congruentes. O cubo possui 12 arestas (a), 8 vértices e 6 faces.

Área superficial do cubo


Como todas as suas faces são iguais, para calcularmos a sua área
superficial, basta calcularmos a área de uma face e multiplicar
por 6. Então temos:
A = 6 ⋅ a2
Volume do cubo
Como todas as arestas possuem a mesma medida, basta multi-
plicar a área da base, que mede a2, pela altura, que mede a. Te-
mos então:
V = a3
Área superficial do paralelepípedo reto
Temos 3 pares de faces, então basta calcularmos a área de cada
uma destas faces, somarmos e multiplicarmos por 2.
A = 2(ab + bc + ac)
Volume do paralelepípedo reto
Para determinarmos o volume de um paralelepípedo reto, basta
multiplicarmos as medidas a, b e c.
V = abc

36
Exemplo

1 - Uma empresa vende cubos mágicos que possuem a medida de suas arestas (a) igual a 7 cm. Para
enviá-los para seus clientes utiliza uma caixa de papelão no formato paralelepípedo com dimensões
98 cm de largura por 49 cm de comprimento e 56 cm de altura. Utilizando estas medidas calcule o que
se pede:
a) Desconsiderando as dobraduras para o fechamento da caixa, qual é a quantidade de papelão que esta
empresa irá gastar para fazer cada caixa? (Responda em m2)
Para respondermos esta pergunta, basta calcularmos a área superficial desta caixa.
Considerando a = 98, b = 49 e c = 56, temos:
A = 2(ab + bc + ac)
A = 2 ⋅ (4802 + 2744 + 5488)
A = 2 ⋅ 13034
A = 26068 cm2
Transformando 26068 cm2 para m2 temos 2,607 m2.

b) Quantos cubos mágicos podem ser enviados dentro desta caixa de forma que se ocupe o máximo
espaço possível?
Para respondermos esta questão, precisamos comparar as dimensões da caixa com as do cubo e de-
pois calcularmos os volumes, depois dividimos para determinarmos quantos cubos irão caber na caixa.
Comparando as medidas.
Comprimento (a): como a caixa tem o comprimento de 98 cm dividindo por 7 cm do cubo, podemos
colocar exatamente 14 cubos no sentido do comprimento.
Largura (b): como a caixa tem uma largura de 49 cm dividindo por 7 cm do cubo, podemos colocar exa-
tamente 7 cubos no sentido de sua largura.
Altura (c): como a caixa tem uma altura de 56 cm dividindo por 7 cm do cubo, podemos colocar exata-
mente 8 cubos no sentido de sua altura.
Agora vamos calcular os volumes:

Cubo: Caixa:
V = a3 V = abc
V = 73 V = 984956
V = 343 cm3 V = 268912 cm3

Agora dividimos os valores 268912 : 343 = 784 cubos.


É possível colocar 784 cubos dentro desta caixa e não sobrará nenhum espaço.

PARA SABER MAIS:


Leia: https://brasilescola.uol.com.br/o-que-e/matematica/o-que-prisma.htm. Acesso em 30 jul. 2021.
Assista: https://www.youtube.com/watch?v=QjW6pSH0jVc. Acesso em 02 jul. 2021.
Assista: https://www.youtube.com/watch?v=_sKcgx590J4. Acesso em 02 jul. 2021.

37
ATIVIDADES
1 - (ENEM - 2017) Uma rede hoteleira dispõe de cabanas simples na ilha de Gotland, na Suécia, conforme
Figura 1. A estrutura de sustentação de cada uma dessas cabanas está representada na Figura 2. A ideia
é permitir ao hóspede uma estada livre de tecnologia, mas conectada com a natureza.

A forma geométrica da superfície cujas arestas estão representadas na figura 2 é:


a) tetraedro.
b) pirâmide retangular.
c) tronco de pirâmide retangular.
d) prisma quadrangular reto.
e) prisma triangular reto.

2 - (UFRGS - 2014) No cubo de aresta 10, da figura abaixo, encontra-se representado um sólido sombreado
com as alturas indicadas no desenho. O volume do sólido sombreado é

a) 300.
b) 350.
c) 500.
d) 600.
e) 700.

3 - Uma empresa de cosméticos fez um kit para vender especificamente


para o dia dos pais e utilizou uma embalagem no formato de um prisma de
base pentagonal conforme figura abaixo. Sabendo que esta embalagem
foi toda feita de um papelão especial e considerando que para fazer as
partes onde são feitas o fechamento será usado 10% a mais de papelão,
qual foi a quantidade de papelão gasto nesta embalagem?

38
4 - (VUNESP – 2021) Um recipiente na forma de um prisma reto de base quadrada, com 26 cm de altura
interna, contém 4,5 litros de água em seu interior, conforme mostra a figura. Sabendo que a altura da
água dentro desse recipiente é de 20 cm, e lembrando que 1 cm³ = 1 mL, o volume máximo de água que
ainda pode ser colocada dentro dele, sem transbordar, é

a) 1450 mL.
b) 1400 mL.
c) 1350 mL
d) 1300 mL.
e) 1250 mL.

5 - (ENEM – 2010) Uma fábrica produz barras de chocolates no formato de paralelepípedos e de cubos,
com o mesmo volume. As arestas da barra de chocolate no formato de paralelepípedo medem 3 cm
de largura, 18 cm de comprimento e 4 cm de espessura. Analisando as características das figuras
geométricas descritas, a medida das arestas dos chocolates que têm o formato de cubo é igual a
a) 5 cm.
b) 6 cm.
c) 12 cm.
d) 24 cm.
e) 25 cm.

39
SEMANA 4

EIXO TEMÁTICO:
Geometria e Medidas.

TEMA/TÓPICO(S):
Geometria Métrica e de Posição.
31. Pirâmides e cones.
33. Planificações de figuras tridimensionais.
35. Áreas laterais e totais de figuras tridimensionais.
36. Volumes de sólidos.

HABILIDADE(S):
31.1. Identificar os elementos de uma pirâmide e de um cone.
33.1. Reconhecer a planificação de figuras tridimensionais usuais: cubo, paralelepípedo retangular, prismas
retos, pirâmide, cilindro e cone.
35.1. Resolver problemas que envolvam o cálculo da área lateral ou total de figuras tridimensionais.
36.1. Resolver problemas que envolvam o cálculo de volume de sólidos.

CONTEÚDOS RELACIONADOS:
Sólidos geométricos, pirâmide.

TEMA: Pirâmides
Caro (a) estudante, nesta semana você vai reconhecer as pirâmides e as suas características, irá re-
conhecer as suas planificações e será capaz de realizar os cálculos de áreas superficiais e dos seus
respectivos volumes.
Pirâmides
Definimos uma pirâmide considerando um plano α, e um polígono convexo qualquer contido nele, e um
ponto V qualquer não pertencente a este plano α. Uma pirâmide é a reunião de todos os segmentos de
reta com uma extremidade em V e a outra em um ponto qualquer do polígono contido em α.

40
Elementos de uma pirâmide
Uma pirâmide possui diversos elementos, que são:
Base: é o polígono contido no plano α, neste caso ABCD;
Faces laterais: são os triângulos ΔABV, ΔADV, ΔCDV e ΔBCV;
Vértice da pirâmide: é o ponto V;
Arestas: em uma pirâmide existem arestas das bases que são AB, BC, CD e AD, e as arestas laterais que
são AV, BV, CV e DV;
Altura: é a distância h entre o plano da base e o vértice da pirâmide.

Classificação de pirâmides retas e oblíquas


Uma pirâmide é considerada reta quanto a projeção do vértice (V) coincide com o centro da sua base(E),
já na pirâmide oblíqua a projeção do vértice não coincide com o centro da base.

Pirâmide reta Pirâmide obliqua

Classificação de pirâmides quanto a sua base


Uma pirâmide pode ser classificada e nomeada de acordo com o polígono que forma a sua base.

Pirâmide triangular Pirâmide quadrangular Pirâmide pentagonal Pirâmide hexagonal


Disponível em: https://reforcandomatematica.blogspot.com/2017/06/formas-geometricas.html. Acesso em: 05 ago. 2021.

Uma pirâmide é chamada de regular quando ela é reta e a sua base é composta por um polígono regular.
Nas pirâmides regulares existem algumas características que são essenciais nos cálculos de área su-
perficial e volume da pirâmide.

41
Nessa pirâmide de base quadrada, temos:
A medida da altura da pirâmide: h
A medida da aresta da lateral da pirâmide: a
A medida da apótema da pirâmide: p (que é a altura
do triângulo da face lateral)
A medida da apótema da base: m
A medida da aresta da base: b
Utilizando estas medidas e o teorema de Pitágoras
podemos definir as seguintes relações:
p2 = m2 + h2
𝒃𝒃
a2 = p2 + 𝟐𝟐 2 Disponível em: https://brainly.com.br/tarefa/22872835.
Acesso em: 05 ago. 2021.

Área superficial de uma pirâmide


A área superficial das faces de uma pirâmide é calculada pela soma das áreas das faces laterais (Al) com
as área de sua base (Ab), esta área superficial somada damos o nome de área superficial total da pirâmi-
de (At). A planificação de uma pirâmide representa a área superficial dela.
At = Al + Ab

Pirâmide Pentagonal e sua planificação

Disponível em: https://brasilescola.uol.com.br/matematica/planificacao-solidos-geometricos.htm. Acesso em: 05 ago. 2021.

Volume de um prisma
O volume de uma pirâmide é igual a ⅓ do produto entre a área de sua base (Ab ) por sua altura (h).
𝑨𝑨𝒃𝒃 ⋅ 𝒉𝒉
𝑽𝑽 =
𝟑𝟑

42
Exemplo
1 - Calcule o volume e a área superficial de uma pirâmide de base quadrada
de lado 10 cm e altura h = 15 cm.
Cálculo da área superficial da pirâmide
Para calcularmos a área dos triângulos das faces laterais, precisamos do valor
da apótema a, utilizando o teorema de Pitágoras. O valor da apótema da base r
é 5 cm, pois é a metade do valor do lado do quadrado.
𝑎𝑎 ! = 𝑟𝑟 ! + ℎ!
𝑎𝑎! = 5! + 15!
𝑎𝑎 ! = 25 + 225
𝑎𝑎! = 250

𝑎𝑎 = 250

𝑎𝑎 = 5 10

Agora vamos calcular o valor da área lateral da pirâmide.


𝑏𝑏 ⋅ ℎ 10 ⋅ 5 10 50 10
𝐴𝐴 = ⇒ 𝐴𝐴 = ⇒ 𝐴𝐴 =
2 2 2

𝐴𝐴 = 25 10 𝑐𝑐𝑐𝑐!

Agora calcularemos o valor da área da base da pirâmide.


𝐴𝐴 = 𝑙𝑙! ⇒ 𝐴𝐴 = 10! ⇒ 𝐴𝐴 = 100 𝑐𝑐𝑐𝑐!

A área total será as 4 faces laterais somadas á área da base, então teremos:
𝐴𝐴! = 𝐴𝐴" + 𝐴𝐴# ⇒ 𝐴𝐴! = 4×25 10 + 100 ⇒ 𝐴𝐴! = 100 10 + 100 𝑐𝑐𝑐𝑐$ 𝑜𝑜𝑜𝑜 𝐴𝐴! ≅ 416,23 𝑐𝑐𝑐𝑐$

Cálculo do volume
Como o valor da base já foi calculado, basta aplicarmos a fórmula:
𝐴𝐴! ⋅ ℎ 100 ⋅ 15 1500
𝑉𝑉 = ⇒ 𝑉𝑉 = ⇒ 𝑉𝑉 =
3 3 3

𝑉𝑉 = 500 𝑐𝑐𝑐𝑐"

PARA SABER MAIS:


Leia mais: https://pt.wikipedia.org/wiki/Pir%C3%A2mide_do_Louvre. Acesso em: 05 ago. 2021.
Assista também: https://www.youtube.com/watch?v=N_mZq_22CvQ. Acesso em: 05 ago. 2021.

43
ATIVIDADES
1 - (Enem) Uma fábrica produz velas de parafina em forma de pirâmide quadrangular regular com 19 cm
de altura e 6 cm de aresta da base. Essas velas são formadas por 4 blocos de mesma altura — 3 troncos
de pirâmide de bases paralelas e 1 pirâmide na parte superior —, espaçados de 1 cm entre eles, sendo
que a base superior de cada bloco é igual à base inferior do bloco sobreposto, com uma haste de ferro
passando pelo centro de cada bloco, unindo-os, conforme a figura.

Se o dono da fábrica resolver diversificar o modelo, retirando a pirâmide da


parte superior, que tem 1,5 cm de aresta na base, mas mantendo o mesmo
molde, quanto ele passará a gastar com parafina para fabricar uma vela?
a) 156 cm³.
b) 189 cm³.
c) 192 cm³.
d) 216 cm³.
e) 540 cm³.

2 - (Enem 2019) As luminárias para um laboratório de matemática serão fabricadas em forma de sólidos
geométricos. Uma delas terá a forma de um tetraedro truncado. Esse sólido é gerado a partir de secções
paralelas a cada uma das faces de um tetraedro regular. Para essa luminária, as secções serão feitas de
maneira que, em cada corte, um terço das arestas seccionadas serão removidas. Uma dessas secções
está indicada na figura.

Essa luminária terá por faces:


a) 4 hexágonos regulares e 4 triângulos equiláteros.
b) 2 hexágonos regulares e 4 triângulos equiláteros.
c) 4 quadriláteros e 4 triângulos isósceles.
d) 3 quadriláteros e 4 triângulos isósceles.
e) 3 hexágonos regulares e 4 triângulos equiláteros.

44
3 - (FUVEST 2009) Os papiros mostram que os egípcios antigos possuíam diversos conhecimentos
matemáticos. Eles sabiam que o volume da pirâmide equivale a um terço do volume do prisma que a
contém. A maior pirâmide egípcia, Quéops, construída por volta de 2560 a.C., tem uma altura aproximada
de 140 metros e sua base é um quadrado com lados medindo aproximadamente 230 metros. Logo,
o volume da pirâmide de Quéops é de aproximadamente (em milhões de metros cúbicos):

a) 1,2.
b) 2,5.
c) 5.
d) 7,5.
e) 15.

4 - A Pirâmide do Louvre é uma estrutura de forma piramidal, construída em vidro e metal, rodeada por
três pirâmides menores, no pátio principal do Palácio do Louvre em Paris, França. (Fonte: Wikipédia)
Sabendo que esta pirâmide de base quadrada, tem suas bases medindo 35 m e sua altura é de 20 m,
e desconsiderando a área da base da pirâmide, quantos metros quadrados de vidro foram gastos na
construção dessa pirâmide?

45
SEMANA 5

EIXO TEMÁTICO:
Geometria e Medidas.

TEMA/TÓPICO(S):
Geometria Métrica e de Posição.
30. Prismas e cilindros.
32. Esferas e bolas.
33. Planificações de figuras tridimensionais.
35. Áreas laterais e totais de figuras tridimensionais.
36. Volumes de sólidos.

HABILIDADE(S):
32.1. Identificar os elementos de uma esfera e de uma bola.
32.2. Identificar as interseções entre planos e esferas.
30.3. Identificar as seções feitas por planos paralelos à base de um prisma ou de um cilindro.
35.1. Resolver problemas que envolvam o cálculo da área lateral ou total de figuras tridimensionais.
36.1. Resolver problemas que envolvam o cálculo de volume de sólidos.

CONTEÚDOS RELACIONADOS:
Sólidos geométricos, não poliedros, corpos redondos, esfera, cilindro.

TEMA: Corpos redondos


Caro (a) estudante, nesta semana você vai reconhecer os sólidos chamados de corpos redondos e as
suas características, irá reconhecer uma esfera e suas características, também irá reconhecer um ci-
lindro e as suas características, irá reconhecer a planificação do cilindro e será capaz de realizar os
cálculos de áreas superficiais e dos volumes desses sólidos.
Corpos redondos
Também chamados de não poliedros, os corpos redondos são sólidos geométricos que possuem pelo
menos uma das suas superfícies curvas. Iremos estudar nesta semana a esfera e o cilindro.

46
Cilindro

Para definirmos um cilindro, consideramos dois planos pa-


ralelos α e β, e um círculo de raio r e o centro O contido no
plano α, considerando uma reta r que é secante aos planos α
e β, consideramos o sólido geométrico chamado de cilindro
a união de todos os segmentos de reta paralelos a reta r que
partem do círculo contido em α e tem a sua outra extremida-
de em um ponto contido no plano β.

Disponível em: https://brasilescola.uol.com.br/


matematica/cilindro.htm. Acesso em: 09 ago. 2021.

Elementos de um cilindro
Bases: são os círculos contidos nos planos α e β.
Raio: é o raio do círculo da base do cilindro de centros O e O’,
representada pela reta r.
Altura: é a distância h entre os planos α e β onde estão con-
tidas as bases.
Eixo: é a reta que passa pelos centros dos círculos das bases.
Geratrizes (g): são os segmentos de retas paralelos ao eixo
do cilindro que possuem as suas extremidades nas circunfe- Disponível em: https://escolaeducacao.com.br/
rências das bases do cilindro. cilindro/.Adptada. Acesso em: 09 ago. 2021.

Cilindros retos e oblíquos


Um cilindro é reto quanto as suas geratrizes são perpendiculares às suas bases, já quando as suas ge-
ratrizes são oblíquas às suas bases ele será um cilindro oblíquo.

Cilindro oblíquo       Cilindro reto


Disponível em: https://canal.cecierj.edu.br/012016/9ccd303eb400e81d63c539ade90b5784.pdf. Acesso em 26 ago. 2021.

Área superficial de um cilindro reto


A área superficial de um cilindro reto é calculada pela soma das áreas da superfície lateral (Al ) com as
área de suas bases (Ab ), esta área superficial somada damos o nome de área superficial total do cilindro
(At ). A planificação de um cilindro representa a sua área superficial.
At = Al + 2 ⋅ Ab

47
A área da base do cilindro é dada por: Ab = π ⋅ r2
A área da lateral do cilindro é dada por: Al = 2 ⋅π⋅r⋅h
A área total do cilindro é dada por: At = 2 ⋅ π ⋅ r ⋅ (r+h)

Cilindro e sua planificação

Disponível em: https://matematicabasica.net/area-do-cilindro/. Acesso em: 12 ago. 2021.

Volume de cilindro
O volume de um cilindro é calculado multiplicando se a área de sua base (Ab), por sua altura (h).
V = Ab ⋅ h
Dai podemos definir o volume em termos do raio da sua base.
V = π ⋅ r2 ⋅ h

Esfera
Dado um ponto O e um número real positivo r, chamamos de esfera a união de todos os pontos no espa-
ço que estão a uma distância menor ou igual á r ao ponto O.

Elementos da esfera
Centro (C): é o ponto central da esfera.
Raio (r): distância entre o centro C e um ponto qualquer da superfície
da esfera.
Eixo ( ): é uma reta que passa pelo centro C,
Equador: é a circunferência obtida a partir de uma seção de um pla-
no perpendicular ao eixo que passa pelo centro C da esfera. Esta é a
maior circunferência que podemos obter em uma esfera.

48
Área superficial de uma esfera
A área superficial de uma esfera de raio r é dada por:
A = 4 ⋅ π ⋅ r2

Volume de uma esfera


O volume de uma esfera de raio r é dada por:
𝟒𝟒 ⋅ 𝝅𝝅 ⋅ 𝒓𝒓𝟑𝟑
𝑽𝑽 =
𝟑𝟑

Exemplos
1 - Uma lata de formato cilíndrico reto tem o raio da base r = 5 cm e altura 15 cm. Qual é a capacidade
volumétrica total e a sua área superficial? (Vamos adotar o valor de π = 3,14)
Resolução

Área superficial: Volume:


At = 2 ⋅ π ⋅ r ⋅ (r + h) V = π ⋅ r2 ⋅ h
At = 2 ⋅ π ⋅ 5 ⋅ (5 + 15) V =3,14 ⋅ 52 ⋅ 15
At = 2 ⋅ 3,14 ⋅ 5 ⋅ 20 V = 1177,5 cm3
At = 628 cm2

2 - Determine a área superficial e o volume de uma esfera de raio 0,6 m. (Vamos adotar o valor de π = 3,14)
Resolução

Área superficial: Volume:


At = 4 ⋅ π ⋅ r2 𝑽𝑽 =
4 % 𝝅𝝅 % 𝒓𝒓!
3
At = 4 ⋅ 3,14 ⋅ 0,62
4 % 𝝅𝝅 % 0,6!
At = 4,5216 cm 2 𝑽𝑽 =
3

V = 0,9 cm3

PARA SABER MAIS:


Leia também: https://brasilescola.uol.com.br/matematica/corpos-redondos.htm. Acesso em: 09
ago. 2021.
Veja também: https://www.youtube.com/watch?v=PpcCqsmdyW0. Acesso em: 09 ago. 2021.

49
ATIVIDADES
1 - Os reservatórios do Gás Natural Veicular (GNV) utilizado como combustível em carros, possuem
um formato cilíndrico e nas suas extremidades são soldados duas meias esferas em cada um deles
conforme a figura abaixo. Desprezando a espessura do reservatório e considerando as medidas do raio
de 17 cm da esfera e do cilindro e comprimento total do reservatório de gás de 120 cm, determine a área
superficial (m²) e o volume deste reservatório em m³.

Disponível em: https://www.facebook.com/pagobemcilindrosgnv/. Adaptada. Acesso em: 09 de ago. 2021.

2 - (ENEM – 2021) Uma loja de materiais de construção vende dois tipos de caixas-d’água: tipo A e tipo
B. Ambas têm formato cilíndrico e possuem o mesmo volume, e a altura da caixa-d’água do tipo B é igual
a 25% da altura da caixa-d’água do tipo A. Se R denota o raio da caixa-d’água do tipo A, então o raio da
caixa-d’água do tipo B é:

a) R/2. b) 2R. c) 4R. d) 5R. e) 16R.

3 - (ENEM – 2021) Uma construtora pretende conectar um reservatório central (Rc) em formato de um
cilindro, com raio interno igual a 2 m e altura interna igual a 3,30 m, a quatro reservatórios cilíndricos
auxiliares (R1, R2, R3 e R4), os quais possuem raios internos e alturas internas medindo 1,5 m.

As ligações entre o reservatório central e os auxiliares são feitas


por canos cilíndricos com 0,10 m de diâmetro interno e 20 m de
comprimento, conectados próximos às bases de cada reserva-
tório. Na conexão de cada um desses canos com o reservatório
central há registros que liberam ou interrompem o fluxo de água.
No momento em que o reservatório central está cheio e os auxi-
liares estão vazios, abrem-se os quatro registros e, após algum
tempo, as alturas das colunas de água nos reservatórios se igua-
lam, assim que cessa o fluxo de água entre eles, pelo princípio dos
vasos comunicantes.
A medida, em metro, das alturas das colunas de água nos reserva-
tórios auxiliares, após cessar o fluxo de água entre eles, é

a) 1,44.
b) 1,16.
c) 1,10.
d) 1,00.
e) 0,95.

50
4 - (FUVEST) Um recipiente cilíndrico cujo raio da base é 6 cm contém água até uma certa altura. Uma
esfera de aço é colocada no interior do recipiente ficando totalmente submersa. Se a altura da água
subiu 1 cm então o raio da esfera é:
a) 1 cm.
b) 2 cm.
c) 3 cm.
d) 4 cm.
e) 5 cm.

5 - (UFJF – 2007) Um reservatório de água tem a forma de um hemisfério acoplado a um cilindro circular
como mostra a figura a seguir.
A medida do raio do hemisfério é a mesma do raio da base do cilindro e igual a r = 3 m. Se a altura do
reservatório é h = 6 m, a capacidade máxima de água comportada por esse reservatório é:

a) 9π m3.
b) 18π m3.
c) 27π m3.
d) 36π m3.
e) 45π m3.

6 - (UFRGS–2006) Duas esferas de raio r foram colocadas dentro de um cilindro circular reto com altura
4r, raio da base r e espessura desprezível, como na figura a seguir: Nessas condições, a razão entre o
volume do cilindro não ocupado pelas esferas e o volume das esferas é
a) 1/5.
b) 1/4.
c) 1/3.
d) 1/2.
e) 2/3.

51
SEMANA 6

EIXO TEMÁTICO:
Geometria e Medidas.

TEMA/TÓPICO(S):
Geometria Métrica e de Posição.
31. Pirâmides e cones.
33. Planificações de figuras tridimensionais.
35. Áreas laterais e totais de figuras tridimensionais.
36. Volumes de sólidos.

HABILIDADE(S):
31.1. Identificar os elementos de uma pirâmide e de um cone.
33.1. Reconhecer a planificação de figuras tridimensionais usuais: cubo, paralelepípedo retangular, prismas
retos, pirâmide, cilindro e cone.
35.1. Resolver problemas que envolvam o cálculo da área lateral ou total de figuras tridimensionais.
36.1. Resolver problemas que envolvam o cálculo de volume de sólidos.

CONTEÚDOS RELACIONADOS:
Sólidos geométricos, corpos redondos, cone.

TEMA: Cone
Caro (a) estudante, nesta semana você vai reconhecer o sólido geométrico chamado de cone e as suas
características, irá reconhecer a sua planificação e será capaz de realizar os cálculos da sua área su-
perficial e do seu volume.
Cone
Considerando um plano α, um círculo qualquer contido neste plano e um ponto V não pertencente ao
plano α, chamamos de cone, o sólido geométrico obtido pela união de todos os segmentos de retas que
possuem uma extremidade no círculo e outra no ponto V.

52
Elementos de um cone
Centro (O): é o ponto central do círculo da base do cone.
Raio (r): distância entre o centro O e um ponto qualquer da circunferên-
cia da base.
Base: é o círculo de centro O e raio r.
Geratriz (g): são os segmentos de reta (AV, CV, PV) que possuem uma ex-
tremidade na circunferência da base e outra no vértice V.
Eixo (reta ): é uma reta que passa pelo centro O e o vértice V.
Altura (h): é a distância entre o plano da base e o plano que contém o
vértice V.
Relação entre as medidas r, h e g.
Utilizando o teorema de Pitágoras obtemos a seguinte relação:
g2 = r2 + h2
Cones retos e oblíquos
Um cone é reto quando o seu eixo forma um ângulo reto com plano de sua base, já o cone oblíquo possui
um ângulo diferente de 90° entre o seu eixo e o plano de sua base.

Cone reto Cone oblíquo


Disponível em: https://brasilescola.uol.com.br/matematica/volume-cone.htm. Acesso em 10 ago.2021.

Área superficial de um cilindro reto


A área superficial total de um cone reto (At ) é calculada pela soma das área da superfície lateral (Al ) com
a área de sua base (Ab ). A planificação de um cone representa a sua área superficial total.
At = Al + Ab

53
Cone e a sua planificação

Disponível em: https://brasilescola.uol.com.br/matematica/calculo-area-cone.htm. Acesso em 10 ago. 2021.

A área da base do cone é dada por: Ab = π ⋅ r2


A área da lateral do cone é dada por: AL = π ⋅ r ⋅ g
A área total do cone é dada por: At = π ⋅ r ⋅ (g + r)

Volume de um cone
O volume de um cone de raio r e altura h é dada por:
𝝅𝝅 ⋅ 𝒓𝒓𝟐𝟐 ⋅ 𝒉𝒉
𝑽𝑽 =
𝟑𝟑

Exemplos:
1 - Um cone reto tem as medidas r = 8 cm e altura 20 cm, qual é a capacidade volumétrica total e a sua
área superficial? (Vamos adotar o valor de π = 3,14)
Resolução
Primeiramente é necessário calcularmos o valor da geratriz (g) utilizando o teorema de Pitágoras.
g2 = r2 + h2
g2 = 82 + 202
g2 = 64 + 400
𝑔𝑔 = 464
𝑔𝑔 ≃ 21,54 𝑐𝑐𝑐𝑐

Agora calculamos as áreas superficiais e volumes.

Área superficial: Volume:


At = π ⋅ r ⋅ (r + g) 𝑉𝑉 =
𝜋𝜋 % 𝑟𝑟 ! % ℎ
3
At = 8 ⋅ π ⋅ (8 + 21,5)
𝜋𝜋 % 𝟖𝟖𝟐𝟐 % 𝟐𝟐𝟐𝟐
At = 8 ⋅ 3,14 ⋅ 29,5 𝑉𝑉 =
𝟑𝟑
At = 741,04 cm2 V = 1339,73 cm3

54
PARA SABER MAIS:
Leia também: https://brasilescola.uol.com.br/matematica/volume-cone.htm. Acesso em: 10 ago.
2021.
Veja o vídeo: https://www.youtube.com/watch?v=K7shv1b-dHI. Acesso em: 12 ago. 2021.

ATIVIDADES
1 - Determine a área superficial e o volume de um cone reto de altura h = 62 mm e raio r = 20 mm.

2 - (ENEM – 2014) Para fazer um pião, brinquedo muito apreciado pelas crianças, um artesão utilizará o
torno mecânico para trabalhar num pedaço de madeira em formato de cilindro reto, cujas medidas do
diâmetro e da altura estão ilustradas na Figura 1. A parte de cima desse pião será uma semiesfera, e a
parte de baixo, um cone com altura 4 cm, conforme Figura 2. O vértice do cone deverá coincidir com o
centro da base do cilindro.

O artesão deseja fazer um pião com a maior altura


que esse pedaço de madeira possa proporcionar
e de modo a minimizar a quantidade de madeira
a ser descartada.
Dados:
4𝜋𝜋. 𝑟𝑟 !
O volume de uma esfera de raio r é  𝑉𝑉 = ;
3
O volume do cilindro de altura h e área da base S é 
V = S.h;

O volume do cone de altura h e área da base S é 


𝑆𝑆. ℎ
𝑉𝑉 = ;
3
Por simplicidade, aproxime π para 3.
A quantidade de madeira descartada, em centímetros cúbicos, é
a) 45.
b) 48.
c) 72.
d) 90.
e) 99.

3 - (ENEM – 2016) Em regiões agrícolas, é comum a presença de silos para armazenamento e secagem
da produção de grãos, no formato de um cilindro reto, sobreposto por um cone, e dimensões indicadas
na figura. O silo fica cheio e o transporte dos grãos é feito em caminhões de carga cuja capacidade é de
20 m3. Uma região possui um silo cheio e apenas um caminhão para transportar os grãos para a usina
de beneficiamento.

55
Utilize 3 como aproximação para π.
O número mínimo de viagens que o caminhão preci-
sará fazer para transportar todo o volume de grãos
armazenados no silo é
a) 6.
b) 16.
c) 17.
d) 18.
e) 21.

4 - Festas de aniversários sempre tem chapeuzinhos de papel, estes chapéus possuem um formato
cônico porém sem a sua base. Abaixo temos alguns modelos utilizados nestas festas.

Imagem de: <a href=’https://br.freepik.com/vetores/natal’>Natal vetor criado por pch.vector - br.freepik.com</a>.


Acesso em: 10 ago. 2021.

Sabendo que estes chapéus cônicos tem um raio de 8 cm em sua base e uma altura de 25 cm, deter-
mine a sua área superficial lateral. Supondo que uma pessoa queira utilizar um destes chapéus para
colocar doces para levar para sua casa, qual é o volume máximo que ela poderia levar utilizando um
destes chapéus.

5 - (ENEM – 2020) Em um casamento, os donos da festa serviam champanhe aos seus convidados em
taças com formato de um hemisfério (Figura 1), porém um acidente na cozinha culminou na quebra de
grande parte desses recipientes. Para substituir as taças quebradas, utilizou-se um outro tipo com
formato de cone (Figura 2). No entanto, os noivos solicitaram que o volume de champanhe nos dois
tipos de taças fosse igual.

56
Sabendo que a taça com o formato de hemisfério é servida completamente cheia, a altura do volume de
champanhe que deve ser colocado na outra taça, em centímetros, é de
a) 1,33.
b) 6,00.
c) 12,00.
d) 56,52.
e) 113,04.

Chegamos ao final de mais um plano de estudo tutorado de matemática, espero que você tenha gos-
tado dos conteúdos e aprendido bastante! Se você se dedicou e fez todas as atividades e enviou tudo
para sua escola, parabéns! Se ainda não enviou continue se dedicando e entregue tudo! Deus abençoe
você e até o próximo!

REFERÊNCIAS:
SOUZA, Joamir Roberto de; GARCIA, Jaqueline da Silva Ribeiro, #Contato matemática, 2º ano – 1º ed.
São Paulo: FTD, 2016.
PAIVA, Manoel, Matemática Paiva, 2ºano – 3º ed. – São Paulo: Moderna, 2015.
SOUZA, Joamir Roberto de; Multiversos Matemática: Geometria: Ensino Médio – 1° ed. – São Paulo:
Editora FTD, 2020.
BONJORNO, José Roberto; JÚNIOR, José Ruy Giovanni; SOUSA; Paulo Roberto Câmara de; Prisma
Matemática: Geometria: Ensino Médio – 1° ed. – São Paulo: Editora FTD, 2020.
CHAVANTE, Eduardo; PRESTES, Diego; Quadrante Matemática e suas tecnologias: Geometria plana
e espacial – 1° ed. – São Paulo: Edições SM, 2020.
SMOLE, Kátia Stocco; DINIZ, Maria Ignez; Ser protagonista: Matemática e suas tecnologias: Geome-
tria plana e espacial: Ensino Médio – 1° ed. – São Paulo: Edições SM, 2020.
SILVA, Luiz Paulo Moreira. “O que é poliedro?”; Brasil Escola. Disponível em: https://brasilescola.uol.
com.br/o-que-e/matematica/o-que-e-poliedro.htm. Acesso em 19 de julho de 2021.
SILVA, Luiz Paulo Moreira. “O que é prisma?”; Brasil Escola. Disponível em: https://brasilescola.uol.
com.br/o-que-e/matematica/o-que-prisma.htm. Acesso em 30 de julho de 2021.
SILVA, Luiz Paulo Moreira. “Cilindro”; Brasil Escola. Disponível em: https://brasilescola.uol.com.br/
matematica/cilindro.htm. Acesso em 09 de agosto de 20.

57
SECRETARIA DE ESTADO DE EDUCAÇÃO DE MINAS GERAIS

PLANO DE ESTUDO TUTORADO


COMPONENTE CURRICULAR: BIOLOGIA
ANO DE ESCOLARIDADE: 2º ANO – EM
PET VOLUME: 04/2021
NOME DA ESCOLA:
ESTUDANTE:
TURMA: TURNO:
BIMESTRE: 4º TOTAL DE SEMANAS:
NÚMERO DE AULAS POR SEMANA: NÚMERO DE AULAS POR MÊS:

SEMANA 1

EIXO TEMÁTICO:
Biodiversidade.

TEMA/TÓPICO(S):
2. História da Vida na Terra.
13. Ciclo de vida dos seres vivos e suas adaptações em diferentes ambientes.
14. Características fisiológicas e adaptações dos seres vivos nos diferentes ambientes da Terra.

HABILIDADE(S):
13.1. Reconhecer a diversidade das adaptações que propiciam a vida nos diferentes ambientes.
13.1.1. Identificar em situações-problema que a diversidade das adaptações propiciam a vida em diferentes
ambientes.
14.5. Reconhecer a importância de alguns representantes do grupo Monera no ambiente e na saúde.
14.5.1. Reconhecer a importância das bactérias como organismos decompositores de matéria orgânica e seu
papel na indústria e saúde.

CONTEÚDOS RELACIONADOS:
Classificação dos Seres Vivos; Seres procariontes.

TEMA: Os grandes grupos dos seres vivos


Caro(a) estudante, nesta semana de estudo você vai conhecer e aplicar os sistemas de classificação
dos seres vivos adotados pela ciência, contextualizando criticamente as diferenças e semelhanças en-
tre os reinos dos seres vivos e conhecer sobre a diversidade dos seres procariontes.

58
OS GRANDES GRUPOS DOS SERES VIVOS
Até o fim do século XIX, os seres vivos eram classificados no reino Plantae (que englobava plantas e fun-
gos) ou no reino Animalia (dos animais), ambos propostos pelo botânico e zoólogo alemão Carlos Lineu
(1707-1778) com base no sistema de classificação aristotélico. Em 1866, o biólogo alemão Ernst Haeckel
(1834-1919) propôs a criação do reino Protista para acomodar as inúmeras espécies microscópicas, que,
até então, eram classificadas como animais ou plantas. Contudo, com o surgimento do microscópio
eletrônico em 1932, ficou claro que as bactérias eram microrganismos bastante diferentes dos demais
protistas, o que levou o biólogo estadunidense Herbert Copeland (1902-1968) a incluí-las em um reino
à parte, o reino Monera, em 1936. Em 1969, os fungos também deixaram de fazer parte do grupo das
plantas e passaram a ser incluídos em um novo reino, o reino Fungi, criado pelo biólogo estadunidense
Robert Whittaker (1920-1980). Na década de 1980, a cientista estadunidense Lynn Margulis (1938-2011)
propôs que o reino Protista incluísse não apenas os protozoários, mas também as algas, antes classifi-
cadas como plantas (veja quadro abaixo).

QUADROS COMPARATIVOS DOS CINCO REINOS DOS SERES VIVOS


Núcleo
Reino Parede celular Nutrição Número de células Tecidos
celular
Autótrofos ou heteró-
Monera Não Sim Unicelulares Não
trofos
Presente em algas e
Autótrofos ou heteró-
Protoctista Sim ausente em proto- Uni ou multicelulares Não
trofos
zoários
Fungi Sim Sim Heterótrofos Uni ou multicelulares Não
Plantae Sim Sim Autótrofos Multicelulares Sim
Animalia Sim Não Heterótrofos Multicelulares Sim*
(*) Exceção dos poríferos (esponjas)

Análises moleculares mostraram, contudo, que o reino Protista não podia ser considerado um grupo
monofilético, pois seus integrantes não compartilham um ancestral comum exclusivo. Assim, foram
propostas novas maneiras de classificar os seres vivos em grandes grupos.
Após comparar a sequência de nucleotídios do RNA ribossômico de muitos representantes de cada um
dos cinco reinos, o biólogo estadunidense Carl Woese (1928-2012) propôs, em 1990, uma nova maneira
de classificar os seres vivos conhecidos. Seu sistema de classificação, que é bem aceito atualmente,
organiza os seres vivos em três domínios: Bacteria, Eukarya e Archaea.
O domínio Bactéria, como sugere o próprio nome, agrupa todas as bactérias – microrganismos uni-
celulares e procarióticos (sem núcleo celular). Já o domínio Eukarya agrupa todos os organismos
eucarióticos (com núcleo celular), uni ou multicelulares, o que inclui os animais, as plantas, os fungos
e os protistas.
O domínio Archaea, por sua vez, compreende os procariontes que não podem ser considerados bac-
térias. Inicialmente, pensava-se que as arqueas fossem microrganismos antigos que viviam restritos a
ambientes de condições extremas (salinidade ou temperaturas elevadas, acidez excessiva, etc.), como
aquelas provavelmente dominantes na Terra primitiva. Contudo, hoje se sabe que as arqueas são muito
mais recentes e comuns do que se supunha, podendo ser encontradas em ambientes não extremos.

59
PRINCIPAIS DIFERENÇAS ENTRE OS TRÊS DOMÍNIOS BIOLÓGICOS
Características Bactéria Archaea Eukarya
Célula procarionte procarionte eucarionte
Envoltório nuclear ausente ausente presente
Número de cromossomos um um mais de um
Configuração cromossômica circular circular linear
Organelas membranosas ausente ausente presente
Citoesqueleto ausente ausente presente
Fotossíntese com uso de clorofila presente ausente presente

SERES PROCARIONTES
As bactérias têm sido a forma de vida mais abundante em nosso planeta nesses últimos 2,5 bilhões de
anos. Bactérias e arqueas provavelmente foram os primeiros organismos a habitar a Terra. Ao longo do
tempo, esses seres primitivos foram transformando ambientes inóspitos, como o entorno de vulcões
submarinos, em locais favoráveis ao aparecimento e à evolução de outras espécies de seres vivos.
Bactérias e arqueas são organismos constituídos por uma única célula procariótica, que se caracteriza
por não ter núcleo nem organelas membranosas no citoplasma. A maioria das bactérias apresenta um
envoltório externo rígido, a parede bacteriana, responsável pela forma da célula e por sua proteção.
Internamente à parede bacteriana e aderida a ela situa‑se a membrana plasmática, que delimita o ci-
toplasma, no qual se situam o cromossomo bacteriano e milhares de pequenos grânulos denominados
ribossomos, responsáveis pela produção das proteínas.

Fonte: LINHARES, Sérgio. GEWANDSZNAJDER, Fernando. PACCA, Helena. Biologia Hoje - volume 2. 3ª Edição. Editora Ática, São Paulo, 2017.

O cromossomo das bactérias é constituído por uma longa molécula de DNA cujas extremidades estão
unidas. Essa molécula “circular” de DNA contém alguns milhares de genes necessários ao crescimen-
to e à reprodução da bactéria. O cromossomo bacteriano geralmente fica na região central da célula,
formando um emaranhado chamado de nucleotídio. Não há membrana envoltória no nucleoide, como
ocorre no núcleo de células eucarióticas.
Além do cromossomo, a célula bacteriana pode conter moléculas de DNA circulares denominadas plas-
mídios. Os plasmídios são menores que o cromossomo bacteriano e sua presença não é essencial à
vida da bactéria. Possuir plasmídios, entretanto, pode ser vantajoso, pois neles geralmente há genes
responsáveis pela resistência a substâncias prejudiciais à bactéria. Na superfície de muitas bactérias,
há filamentos proteicos móveis, os flagelos bacterianos, que atuam na movimentação da célula.

60
As bactérias reproduzem‑se assexuadamente por divisão binária, processo em que a célula bacteriana
duplica o cromossomo e em seguida divide‑se ao meio, formando duas novas bactérias. Certas espécies
bacterianas, em condições ideais, dividem-se a cada 20 minutos. Assim, em poucas horas, uma única
bactéria pode originar uma população bacteriana composta de milhões de células geneticamente idên-
ticas. Esses indivíduos constituem o que os cientistas denominam clone, ou seja, um conjunto de células
ou indivíduos geneticamente idênticos que resulta da propagação assexuada de um indivíduo original.

Fonte: LINHARES, Sérgio. GEWANDSZNAJDER, Fernando. PACCA, Helena. Biologia Hoje - volume 2. 3ª Edição. Editora Ática, São Paulo, 2017.

As arqueas são seres procarióticos bastante semelhantes às bactérias em tamanho e aparência; por
isso elas só foram identificadas como um grupo distinto há poucas décadas, graças ao emprego de
técnicas de análise molecular. Uma diferença marcante entre bactérias e arqueas está na organização
e no funcionamento dos genes.
Muitas arqueas vivem em ambientes impróprios à maioria dos outros seres vivos. Um grupo de ar-
queas é o das termoacidófilas, que suportam condições extremas de acidez e temperatura, vivendo
em fontes termais ácidas, onde a temperatura oscila entre 60 °C e 80 °C, ou em fendas vulcânicas nas
profundezas oceânicas.
Estudos mostram que as arqueas são evolutivamente mais próximas dos organismos eucarióticos do que
das bactérias. Isso é interpretado pelos cientistas da seguinte maneira: nos primórdios da vida na Terra,
um grupo de organismos unicelulares primitivos separou-se em duas linhagens, uma que deu origem às
bactérias atuais e outra que diversificou‑se em duas, originando as arqueas e os seres eucarióticos.

PARA SABER MAIS:


Se você quer aprofundar os seus conhecimentos no assunto confira abaixo algumas sugestões:
Biologia - Seres Procariontes. Aula exibida para o 2º Ano do Ensino Médio no dia 06/08/2020 com o
professor Vinícius Braz no programa Se Liga na Educação. Disponível em: https://drive.google.com/
file/d/1j0LnpJ63w4Rw42Sax29LBtgFw32-ib58/view>. Acesso em: 08 ago. 2021.

61
ATIVIDADES
1 - (UCS 2019) A reciclagem é um processo que consiste na reutilização de materiais − cuja primeira
utilidade terminou − em um novo produto igual ou sem relação com o anterior. Diversos materiais
como vidro, plástico, metal, papel, entre outros, podem ser reciclados, visando à redução do consumo
de matérias-primas. Além disso, a reciclagem minimiza a quantidade de lixo acumulado em aterros
sanitários e lixões e contribui para a limpeza das cidades. Esse processo é igualmente importante do
ponto de vista socioeconômico, uma vez que gera empregos em cooperativas e colabora diretamente
para o aumento da renda de diversas pessoas.
Disponível em: http://www.blogmodainfantil.com.br/dia-mundial-da-reciclagem-ensinar-os-pequenos-desde-cedo-e-pensar-no-
futuro-do-planeta/ infografico-2/. Acesso em: 27 nov. 2018. (Parcial e adaptado.)

Nesse sentido, a questão aborda o eixo temático “Reciclagem”.


A compostagem é uma técnica que vem sendo utilizada pelo homem há séculos: trata-se de um proces-
so biológico de reciclagem da matéria orgânica, de origem urbana, doméstica, industrial, agrícola ou
florestal, mediada por fungos e bactérias. Com a popularização da técnica, as composteiras ganharam
mais espaço nas casas das pessoas e, para os mais animados, a internet conta com inúmeros tutoriais
de como fazê-las.
Disponível em: ttps://cienciainformativa.com.br/pt_BR/compostagem-uma-ideia-inteligente/;https://cienciainformativa.com.br/
en_US/compostagemuma-ideia-inteligente/;https://www.ecycle.com.br/2368-compostagem.html. Acesso em: 21 nov. 2018.
(Parcial e adaptado.)

A partir das informações apresentadas acima e de seus conhecimentos em Biologia sobre fungos e
bactérias, assinale a alternativa correta.
a) Os fungos, pertencentes ao reino Fungi, são procariontes, pluricelulares e apresentam nutrição
heterotrófica.
b) As bactérias, pertencentes ao reino Monera, distinguem-se dos demais seres vivos porque suas
células possuem carioteca envolvendo o material nuclear, isto é, são eucariontes.
c) A principal forma de reprodução das bactérias é a gemiparidade, processo de reprodução asse-
xuada simples e rápido, que consiste na divisão de uma célula, por meiose, em outras duas iguais.
d) Os fungos são organismos aclorofilados que, em associação mutualística com clorofíceas, for-
mam as chamadas micorrizas.
e) A reprodução assexuada em certas espécies de fungos filamentosos pode ocorrer por fragmen-
tação do micélio que, por mitose, dá origem a novos indivíduos idênticos ao seu precursor.

2 - (UCS 2013) As bactérias são organismos que atualmente auxiliam o desenvolvimento científico
e tecnológico. Elas são utilizadas diretamente em biotecnologia, principalmente como vetores de
manipulação do DNA, bem como fornecedores de diversas enzimas. A utilização das bactérias na
biotecnologia se deve à:
a) sua simplicidade procariótica e à manipulação dos plasmídeos.
b) sua simplicidade quimiossintetizante e aos micoplasmas.
c) complexidade da parede celular e ao seu RNA.
d) simplicidade das ficobilinas e ao seu DNA.
e) sua alta capacidade de multiplicação e por serem revestidas por um capsídeo.

62
3 - (UFRGS 2016) Observe a tira abaixo, que ilustra o processo reprodutivo das bactérias.

A alternativa que apresenta a afirmação correta sobre o processo reprodutivo, ilustrado na tira.
a) A reprodução depende da quitina, presente na parede celular.
b) Os cloroplastídios distribuem-se em igual número para as células filhas.
c) O processo denomina-se esporogênese, que ocorre também nos fungos.
d) O processo resulta em duas novas bactérias geneticamente idênticas.
e) Uma das novas células forma um endosperma, estrutura resistente ao calor.

4 - (PUC-SP 2010) Leia com atenção o trecho a seguir:


HISTÓRIA DE DUAS BACTÉRIAS
A bactéria Zi e a bactéria Wu encontram-se em um meio de cultura contendo um antibiótico A.
Zi comenta com Wu: - “Esse antibiótico me deixa muito mal. Estou com dificuldade de sintetizar molé-
culas de RNA”.
Responde Wu: - “Puxa, eu continuo produzindo normalmente proteínas e sinto-me muito bem. Zi, farei
imediatamente uma ponte citoplasmática com você e vou lhe transferir um plasmídio especial”.
Um pouco depois, Zi comenta: - “Wu, muito obrigada, meu processo de síntese de proteínas se norma-
lizou. Sou uma nova bactéria! “

Com relação ao trecho descrito, é INCORRETO afirmar que:


a) a bactéria Zi, inicialmente, teve dificuldade de sintetizar moléculas de RNA e isso interferiu na
síntese de proteínas.
b) a bactéria Wu tem constituição genética que permite sobreviver em meio contendo o antibiótico A.
c) ocorreu conjugação entre as bactérias Wu e Zi.
d) a bactéria Zi recebeu molécula de RNA mensageiro presente no plasmídio, o que lhe garantiu
resistência ao antibiótico A.
e) a bactéria Wu transferiu DNA para a bactéria Zi.

63
5 - (UEPA 2011) Artefato bélico desenvolvido para espalhar agentes vivos, capazes de infectar um
grande número de pessoas, é chamado de arma biológica, contendo vírus e bactérias modificados
geneticamente em laboratórios, para se tornarem resistentes, matando ou incapacitando humanos,
animais e plantas de uma nação adversária. Potencialmente, para este fim, destacam-se os bacilos
Bacillus anthracis e Clostridium botulinum, os vírus da varíola e o ebola.
A respeito dos micro-organismos destacados, afirma-se que:
a) pertencem ao mesmo reino por não apresentar material genético envolvido pela carioteca.
b) as bactérias pertencem ao mesmo gênero porque possuem forma esférica.
c) são autótrofos e só se reproduzem no interior de outras células.
d) formam esporos que são as estruturas de resistência no ambiente externo.
e) as bactérias possuem mesma forma e pertencem a gêneros e espécies diferentes.

64
SEMANA 2

EIXO TEMÁTICO:
Biodiversidade.

TEMA/TÓPICO(S):
2. História da Vida na Terra.
13. Ciclo de vida dos seres vivos e suas adaptações em diferentes ambientes.
14. Características fisiológicas e adaptações dos seres vivos nos diferentes ambientes da Terra.

HABILIDADE(S):
13.1. Reconhecer a diversidade das adaptações que propiciam a vida nos diferentes ambientes.
13.1.1. Identificar em situações-problema que a diversidade das adaptações propiciam a vida em diferentes
ambientes.
14.4. Reconhecer a importância de alguns representantes do grupo Fungi no ambiente e na saúde.
14.4.1. Reconhecer a importância dos fungos como organismos decompositores de matéria orgânica nos
ecossistemas e seu papel na indústria e saúde.

CONTEÚDOS RELACIONADOS:
Reino dos fungos.

TEMA: Reino dos fungos


Caro(a) estudante, nesta semana de estudo, você vai reconhecer a diversidade do Reino Fungi.
REINO FUNGI
Os fungos atualmente são classificados em um reino próprio – Fungi. Eles são organismos eucarióticos
e heterotróficos cuja parede celular contém quitina, substância também presente no esqueleto dos
artrópodes (crustáceos, insetos, aranhas, entre outros). Fungos vivem no solo, na água ou no corpo
de outros seres vivos, como parasitas. Seus principais representantes são os bolores, os cogumelos,
as orelhas‑de‑pau e as leveduras, estas últimas também chamadas levedos ou fermentos.
Com poucas exceções, os fungos são seres multicelulares constituídos por longos filamentos micros-
cópicos, ramificados e com parede quitinosa, as hifas, nas quais se encontra o conteúdo celular do
fungo. O conjunto de hifas constitui o micélio, equivalente ao “corpo” do fungo.

65
Fonte: CAMPBELL, N.A.; REECE, J.B.; URRY, L.A.; CAIN, M.L.; WASSERMAN, S.A.; MINORSKY, P.V. & Jackson, R.B. 2010. Biologia.10ª ed.
Artmed, Porto Alegre, 1488 p.

O emaranhado de hifas que formam o micélio pode crescer indefinidamente, enquanto houver alimen-
to disponível e condições favoráveis. O crescimento das hifas ocorre apenas nas extremidades; nas
regiões mais afastadas das pontas, o conteúdo citoplasmático pode desaparecer, restando apenas as
paredes das hifas.
Os fungos produzem esporos, os quais podem surgir de modo assexuado ou sexuado, após a fusão de
núcleos celulares e a formação do zigoto. Quando encontra condições apropriadas, o esporo germi-
na e origina uma hifa, que cresce e se ramifica, produzindo um novo micélio. Durante o crescimento,
as hifas do micélio liberam enzimas digestivas que atuam extracelularmente, degradando substâncias
orgânicas presentes no substrato. Em seguida, as hifas absorvem os produtos da digestão, utilizan-
do-os como fonte de energia e de matéria‑prima para viver e crescer. Esse modo de vida dos fungos é
responsável pelo apodrecimento de diversos materiais, como frutas, verduras, cereais e uma gama de
substratos orgânicos (madeira, couro etc.).
Muitos fungos têm reprodução assexuada por fragmentação do micélio. Os fragmentos crescem e ori-
ginam novos micélios. Certos fungos, como bolores do grupo dos zigomicetos, reproduzem‑se assexua-
damente por esporulação, processo em que o fungo forma hifas especiais, em cujas extremidades se di-
ferenciam esporângios, onde se formam esporos haploides (esporos assexuados). Ao cair em local com
condições adequadas, o esporo germina e origina um novo micélio. Leveduras como Saccharomyces
cerevisiae reproduzem‑se por brotamento, ou gemulação. Os brotos (gêmulas) geralmente se separam
da célula original, embora eventualmente possam permanecer unidos, formando cadeias de células.
De tempos em tempos, a maioria dos fungos passa por um estágio sexuado, em que se formam zigotos
diploides. O zigoto sofre meiose e origina células haploides, que se diferenciam em esporos haploides
(esporos sexuados).
Nos fungos, a reprodução sexuada inicia‑se pela conjugação de duas hifas sexualmente maduras e
compatíveis que se aproximam e se fundem. A hifa resultante da fusão é dicariótica, ou seja, tem dois
núcleos haploides por célula, um de cada hifa parental. Durante o crescimento e a ramificação dessa
hifa, os núcleos se multiplicam diversas vezes no citoplasma, mantendo‑se emparelhados, mas sem se
fundir. Em certas espécies, as hifas dicarióticas originam um micélio que pode formar corpos de fruti-
ficação chamados de cogumelos.

66
Nas hifas que formam as lamelas sob o “chapéu” dos cogumelos, pares de núcleos procedentes das hifas
parentais finalmente se unem, originando núcleos zigóticos diploides. Por meiose, esses núcleos produ-
zem esporos sexuados. Em cogumelos do grupo dos basidiomicetos, os esporos são denominados basi-
diósporos, e o cogumelo, basidiocarpo. Os basidiósporos são liberados do cogumelo e levados pelo ven-
to. Ao cair em local com condições adequadas, os basidiósporos germinam e originam novos micélios.

Fonte: CAMPBELL, N.A.; REECE, J.B.; URRY, L.A.; CAIN, M.L.; WASSERMAN, S.A.; MINORSKY, P.V. & Jackson, R.B. 2010. Biologia.10ª ed.
Artmed, Porto Alegre, 1488 p.

PARA SABER MAIS:


Se você quer aprofundar os seus conhecimentos no assunto confira abaixo algumas sugestões:
Biologia - Reino Fungi. Aula exibida para o 2º Ano do Ensino Médio no dia 27/08/2020 com o profes-
sor Vinícius Braz no programa Se Liga na Educação. Disponível em: https://drive.google.com/file/d/1_
iKkjHEoL_0CKZw2zBcIR1HZvVWgsimf/view>. Acesso em: 08 ago. 2021.

ATIVIDADES
1 - (UEG 2019)

67
O ser vivo apresentado como a moradia dos “smurfs” tem diferentes espécies. Popularmente, esta “ca-
sinha de cogumelo” pode se referir também às espécies conhecidas como “orelhas-de-pau”, que apre-
sentam um corpo frutífero, e, às vezes, com a forma de chapéu, apresentando hifas, que são “mofos”.
Esses seres vivos possuem sofisticado conjunto de enzimas, muitas vezes, de interesse da indústria de
biorremediação de solos contaminados e no tratamento de efluentes. Considerando-se a taxonomia
desses seres vivos, verifica-se que eles são considerados: 
a) zigomycota.
b) ascomicota.
c) ascomicetos.
d) basidiomicetos.
e) deuteromicetos.

2 - (UCS 2018) Os líquens são encontrados em locais com as mais diversas condições, inclusive em
ambientes desérticos, frios ou de calor intenso. São os seres predominantes na vegetação de regiões
polares (Tundra), servindo como alimento para renas e caribus.
Sobre líquens, é correto afirmar que:
a) são associações entre fungos e algas ou fungos e cianobactérias e reproduzem-se apenas asse-
xuadamente através de sorédios.
b) são um exemplo de associação harmônica, tipo protocooperação.
c) liberam pequenos fragmentos através dos sorédios denominados “esporos”.
d) podem associar-se às plantas parasitando-as, formando o que popularmente se denomina de
barba de velho.
e) podem desenvolver-se separadamente em certas condições, formando um píleo e abaixo dele
um substrato verde ou azulado.

3 - (MACKENZIE 2014) Os fungos constituem um grupo de organismos com características que lembram
um vegetal, mas com outras que lembram um animal. Foram, no passado, considerados como vegetais
e, atualmente, são colocados em um reino próprio, o Reino Fungi. A respeito deles é correto afirmar que
são seres:
a) procariontes, uni ou pluricelulares, sempre autótrofos.
b) procariontes, uni ou pluricelulares, autótrofos e heterótrofos.
c) eucariontes, uni ou pluricelulares, sempre heterótrofos.
d) eucariontes, pluricelulares, autótrofos e heterótrofos.
e) eucariontes, pluricelulares, sempre autótrofos.

4 - (UNESP 2018) Em uma aula de campo, os alunos encontraram, crescendo sobre um tronco caído na
mata, organismos conhecidos como orelhas-de-pau. O fato que chamou a atenção dos alunos foi que
alguns desses organismos eram de cor verde, como mostra a figura.

68
Paula afirmou que o organismo observado era um fungo fotossintetizante e, portanto, autótrofo. Gil-
berto concordou que seria um fungo fotossintetizante, mas, por estar crescendo em um tronco em
decomposição, seria heterótrofo necessariamente.
Ricardo sugeriu que o organismo observado, na verdade, eram dois organismos, um autótrofo e outro
heterótrofo.
Tiago complementou a ideia de Ricardo, afirmando tratar-se de um musgo, que é uma associação entre
um fungo e uma alga.
Fernanda discordou de Tiago, afirmando tratar-se de um líquen, no qual o fungo fornece os carboidra-
tos necessários para o crescimento da alga.
A explicação correta para o fato foi dada por:
a) Fernanda.
b) Gilberto.
c) Ricardo.
d) Paula.
e) Tiago.

5 - (PUCPR 2017) Observe o cartoon.

Publicado <http://www1.folha.uol.com.br/ilustrada/cartum/cartunsdiarios/?cmpid=menupe#25/2/2017. Acesso em: 25/02/2017.

Sobre o cartoon, analise as afirmativas a seguir.


I. Estão representados três reinos, entre citados e desenhados.
II. O termo colônia usado pelo autor está corretamente empregado para bactérias e fungos.
III. Apenas um dos grupos citados é procarionte.
IV. Apenas um dos grupos citados é decompositor.
V. Existem exemplares comestíveis e exemplares prejudiciais à saúde entre fungos e bactérias.
Estão CORRETAS apenas:
a) I, III, e V.
b) III, IV e V.
c) II, III e V.
d) I, II, III, e V.
e) II, III e IV.

69
SEMANA 3
EIXO TEMÁTICO:
Biodiversidade.
TEMA/TÓPICO(S):
2. História da Vida na Terra.
13. Ciclo de vida dos seres vivos e suas adaptações em diferentes ambientes.
14. Características fisiológicas e adaptações dos seres vivos nos diferentes ambientes da Terra.
HABILIDADE(S):
13.1. Reconhecer a diversidade das adaptações que propiciam a vida nos diferentes ambientes.
13.1.1. Identificar em situações-problema que a diversidade das adaptações propiciam a vida em diferentes
ambientes.
14.3. Reconhecer a importância de alguns representantes do grupo Protista no ambiente e na saúde.
14.3.1. Reconhecer a importância das algas como organismos produtores de matéria orgânica e oxigênio nos
ecossistemas aquáticos e da utilização das algas na indústria alimentícia e cosmética. Reconhecer a impor-
tância dos protozoários no funcionamento dos ambientes aquáticos e como indicadores de poluição e as con-
dições ambientais que favorecem as principais protozooses humanas brasileiras e formas de contaminação.
CONTEÚDOS RELACIONADOS:
Reino Protoctista.

TEMA: Reino Protoctista


Caro(a) estudante, nesta semana de estudo, você vai reconhecer a diversidade do Reino Protoctista.

REINO PROTOCTISTA
O termo protozoário (do grego protos, primitivo, primeiro, e zoon, animal) designa organismos unicelu-
lares heterotróficos, atualmente distribuídos em diversos filos.
A maioria dos protozoários vive em água doce ou salgada, em regiões lodosas e em terra úmida, alimen-
tando‑se tanto da matéria orgânica de cadáveres (hábito saprofágico) como de microrganismos vivos,
que podem ser bactérias, algas e outros protozoários (hábito predatório). Há espécies de protozoários
de hábito parasita, que vivem no interior do corpo de animais invertebrados e vertebrados, incluindo a
espécie humana; em muitos casos essas infestações causam doenças. Há também protozoários que
trocam benefícios com outros seres vivos em uma relação de mutualismo, isto é, uma relação em que
ambas as espécies participantes se beneficiam, como é o caso de espécies do gênero Trichonympha,
protozoários que vivem no intestino dos cupins.
O tamanho dos protozoários varia entre 10 μm e 50 μm, mas alguns podem atingir 1 milímetro de com-
primento. A organização celular dos protozoários é complexa e há organelas bem desenvolvidas, como
o vacúolo digestivo, no qual ocorre a digestão intracelular das partículas de alimento ingeridas por fa-
gocitose. Em espécies de água doce, há vacúolos contráteis bem desenvolvidos e ativos, que atuam na
eliminação do excesso de água que penetra na célula por osmose.

70
Fonte: CAMPBELL, N.A.; REECE, J.B.; URRY, L.A.; CAIN, M.L.; WASSERMAN, S.A.; MINORSKY, P.V. & Jackson, R.B. 2010. Biologia.10ª ed.
Artmed, Porto Alegre, 1488 p.

As amebas movimentam‑se por meio de pseudópodes, enquanto outros protozoários movimentam‑se


por ação de flagelos ou de cílios; há também espécies sem estruturas locomotoras. O tipo de estrutura
locomotora é um dos critérios utilizados para dividir os protozoários em diversos grupos.
A maioria dos protozoários de vida livre reproduz‑se assexuadamente por divisão binária. A célula cres-
ce até determinado tamanho e divide‑se ao meio, originando dois novos indivíduos.
Alguns grupos podem se reproduzir assexuadamente por meio de divisão múltipla. Nesse caso, a célula
multiplica o núcleo diversas vezes por mitose, antes de se fragmentar em inúmeras pequenas células.
A maioria dos protozoários apresenta processos sexuais. No tipo mais comum de reprodução sexuada,
dois protozoários de sexos diferentes originam um zigoto, que posteriormente passa por meiose e ori-
gina indivíduos geneticamente recombinados.

Fonte: CAMPBELL, N.A.; REECE, J.B.; URRY, L.A.; CAIN, M.L.; WASSERMAN, S.A.; MINORSKY, P.V. & Jackson, R.B. 2010. Biologia.10ª ed.
Artmed, Porto Alegre, 1488 p.

71
As algas são organismos eucarióticos e fotossintetizantes, unicelulares ou multicelulares; o corpo das
algas multicelulares é chamado de talo. O grupo das algas é bastante diversificado e atualmente com-
preende diversos filos.
As algas unicelulares podem viver no mar, em água doce e em superfícies úmidas. Elas são parte im-
portante dos ecossistemas aquáticos, produzindo gás oxigênio e substâncias orgânicas que servem de
alimento para diversos outros seres.
Dependendo da espécie de alga, suas células podem conter um ou vários cloroplastos, organelas
citoplasmáticas responsáveis pela fotossíntese. Na maioria das algas, as células são revestidas por
uma parede celular composta de celulose geralmente combinada a outras substâncias, como o ágar,
a carragenina, o carbonato de cálcio (CaCO3), entre outras. Algumas dessas substâncias têm impor-
tância econômica.
Algas microscópicas vivem na parte superficial da água de mares e grandes lagos. Nesses locais, em
conjunto com bactérias, protozoários, larvas de diversos animais, microcrustáceos etc., formam a co-
munidade aquática denominado plâncton (do grego plankton, errante). Algas e outros seres planctôni-
cos fotossintetizantes constituem o fitoplâncton.
Um dos critérios para dividir as algas em grupos é quanto aos tipos de pigmentos apresentados além da
clorofila. Assim, há o grupo das algas verdes (clorófitas), das algas marrons (feófitas), das algas verme-
lhas (rodófitas), das algas douradas (crisófitas), além de outros grupos, como o das diatomáceas, dos
euglenoides e dos dinoflagelados.
Algas unicelulares reproduzem‑se assexuadamente por divisão binária. Algas multicelulares filamento-
sas podem se reproduzir por fragmentação, processo em que o talo simplesmente se fragmenta e ori-
gina novos indivíduos. Diversas espécies se reproduzem por esporulação, processo em que se formam
assexuadamente células reprodutivas denominadas esporos.
O processo sexuado de reprodução das algas envolve fusão de dois gametas haploides com formação
de um haploides (n) e diploides (2n), fenômeno denominado alternância de gerações.
As algas verdes do gênero Ulva, por exemplo, apresentam dois tipos de talo de aparência muito seme-
lhante, mas constituídos por células diploides ou por células haploides. Indivíduos de talos diploides,
chamados esporófitos, apresentam células que, na maturidade, passam por meiose e originam espo-
ros haploides; por isso, fala‑se em meiose espórica. Os esporos libertam ‑se do talo que os formou e
germinam, originando talos haploides.
Os indivíduos de talos haploides, denominados gametófitos, apresentam células que se multiplicam
por mitose e, na maturidade, se diferenciam em gametas haploides flagelados. Estes se libertam do
gametófito e fundem‑se dois a dois, produzindo zigotos diploides. O desenvolvimento de um zigoto
origina um novo talo diploide (esporófito), que, na maturidade, repetirá o ciclo.

72
Fonte: CAMPBELL, N.A.; REECE, J.B.; URRY, L.A.; CAIN, M.L.; WASSERMAN, S.A.; MINORSKY, P.V. & Jackson, R.B. 2010. Biologia.10ª ed.
Artmed, Porto Alegre, 1488 p.

PARA SABER MAIS:


Se você quer aprofundar os seus conhecimentos no assunto confira abaixo algumas sugestões:
Biologia - Reino Protoctista. Aula exibida para o 2º Ano do Ensino Médio no dia 17/09/2020 com o pro-
fessor Vinícius Braz no programa Se Liga na Educação. Disponível em: <https://drive.google.com/file/
d/17iPhYx3R62b_I-lwMFUwlRDeYjASIvIp/view>.

ATIVIDADES
1 - O reino Protista atualmente é conhecido como Protoctista, englobando uma diversidade de seres
vivos que não apresentam ancestralidade em comum (polifiléticos). Os organismos presentes neste
reino são eucariontes, uni ou pluricelulares e podem ou não realizar fotossíntese. Os principais grupos
presentes neste reino são:
a) Moneras e Protozoários.
b) Protozoários e Fungos.
c) Algas e Moneras.
d) Fungos e Algas.
e) Protozoários e Algas.

73
2 - Os protozoários são organismos que em sua maioria habitam o ambiente aquático, entretanto,
não apresentam parede celular. Eles apresentam como mecanismo para eliminar o excesso de água
absorvido, em ambiente dulcícola, uma estrutura que permite a osmorregulação. Essa estrutura é
conhecida como:
a) Vacúolos contráteis.
b) Pseudópodes.
c) Membrana Plasmática.
d) Flagelos.
e) Cílios.

3 - (PUC-SP) O filo Protozoa é subdividido em quatro classes: Sarcodínea, Mastigophora, Sporozoa e


Ciliophora.
A característica considerada para tal classificação é:
a) o modo de reprodução.
b) a presença ou ausência de carioteca.
c) a composição química do pigmento fotossintetizante.
d) a estrutura de locomoção.
e) a composição química do citoplasma.

4 - (UDESC SC/2012) - Assinale a alternativa correta quanto aos protozoários. 


a) Os protozoários de água doce possuem vacúolos pulsáteis ou contráteis, que permitem a diges-
tão através da clasmocitose. 
b) As amebas são protozoários do filo Sarcodina e se movimentam por meio de cílios. 
c) Os protozoários são unicelulares, eucariontes e heterotróficos. Vivem na água ou no solo, livres
ou em associações com outros seres vivos. Sua digestão é intracelular. 
d) A reprodução assexuada das amebas ocorre por esquizogonia, originando duas células filhas.
e) Os paramécios são protozoários do filo Mastigophora e se locomovem por meio de cílios.

5 - (Mack- SP) A respeito dos protozoários, são feitas as afirmações a seguir:


I. Todos eles apresentam vacúolos contráteis em suas células.
II. Todos eles são heterótrofos e de respiração aeróbia.
III. Alguns podem se reproduzir sexuadamente.
Assinale:
a) se todas estiverem corretas.
b) se todas estiverem erradas.
c) se apenas I e II estiverem corretas.
d) se apenas I e III estiverem corretas.
e) se apenas III estiver correta.

74
SEMANA 4

EIXO TEMÁTICO:
Biodiversidade.

TEMA/TÓPICO:
2. História da Vida na Terra.
14. Características fisiológicas e adaptações dos seres vivos nos diferentes ambientes da Terra.

HABILIDADE(S):
14.3. Reconhecer a importância de alguns representantes do grupo Protista no ambiente e na saúde.
14.3.1. Reconhecer a importância das algas como organismos produtores de matéria orgânica e oxigênio nos
ecossistemas aquáticos e da utilização das algas na indústria alimentícia e cosmética. Reconhecer a impor-
tância dos protozoários no funcionamento dos ambientes aquáticos e como indicadores de poluição e as con-
dições ambientais que favorecem as principais protozooses humanas brasileiras e formas de contaminação.
14.5. Reconhecer a importância de alguns representantes do grupo Monera no ambiente e na saúde.
14.5.1. Reconhecer a importância das bactérias como organismos decompositores de matéria orgânica e seu
papel na indústria e saúde.

CONTEÚDOS RELACIONADOS:
Doenças causadas por bactérias e protozoários.

TEMA: Doenças causadas por bactérias e protozoários


Caro(a) estudante, nesta semana de estudo, você vai reconhecer a importância médica de algumas bac-
térias e protozoários causadores de doenças nos seres humanos.

DOENÇAS PARASITÁRIAS
Em todos os ecossistemas, há populações de diferentes espécies que vivem em relação de interde-
pendência – seja ela direta ou indireta. Tais relações podem estar baseadas na alimentação, na repro-
dução, na proteção, na ocupação do espaço, e podem implicar benefício ou prejuízo para os indivíduos
associados. Genericamente, essas associações são chamadas de simbioses e incluem o parasitismo
– que, se não é a relação mais difundida entre as espécies, pelo menos é a mais estudada. De fato, pra-
ticamente não existem seres vivos que não tenham parasitas. Até as bactérias podem ser atacadas por
vírus, os bacteriófagos.
O parasitismo é uma associação entre seres de espécies diferentes, na qual há benefício unilateral: um
dos seres, o parasita, abriga-se e alimenta-se à custa de outro, o hospedeiro. Parasitas e hospedeiros
se adaptam um ao outro, processo que pode ocorrer até em curto período, quando determinado parasi-
ta se torna resistente a uma nova defesa desenvolvida pelo hospedeiro. Esse é o caso dos vírus, como
o da gripe, que sofrem mutações genéticas relativamente rápidas e originam linhagens mutantes resis-
tentes a anticorpos específicos produzidos pelos hospedeiros.
Além dos microrganismos parasitas, pertencentes a diferentes grupos (vírus, bactérias, protozoários e
fungos), há muitos vermes e artrópodes parasitas.
O corpo dos hospedeiros sofre inúmeros efeitos prejudiciais em função da infestação e do desenvolvi-
mento do ciclo vital dos parasitas. Esses efeitos podem ser desde um simples incômodo a problemas
mais graves que podem ser até letais.

75
Os parasitas podem provocar: obstruções intestinais (lombriga) e linfáticas (filárias); perfurações na
pele e em órgãos internos (esquistossomo e ancilóstomo); ulcerações (ameba e leishmania); necrose
de tecidos por ação enzimática; irritação de mucosas; prurido na região perianal (larvas da lombriga e
do oxiúro); ação tóxica (lombriga e bactérias); espoliação, com enfraquecimento e anemia (ancilósto-
mo e plasmódio, causador da malária); febre (bactérias, vírus e plasmódio); e infecções locais ou gene-
ralizadas (bactérias e fungos).
Os parasitas são organismos adaptados anatômica, fisiológica e bioquimicamente para obter alimen-
tos e ser capaz de se reproduzir em seu respectivo hospedeiro. Não à toa, é possível observar que, em
geral, estabelece-se uma grande especificidade entre o parasita e o hospedeiro.
Muitas bactérias constituem a flora normal do nosso organismo, vivendo na pele e em órgãos internos
de vários sistemas, principalmente nas mucosas gastrointestinal, respiratória, urinária e genital. Entre-
tanto, outras espécies parasitam o ser humano, provocando doenças (bactérias patogênicas). Ao invadir
o organismo, as bactérias patogênicas causam várias manifestações locais resultantes da reação dos
tecidos, como dor, calor e edema. Multiplicando-se ativamente, elas podem migrar para outros órgãos
através da pele ou do sangue. Algumas bactérias provocam a formação de pus, o que indica a ação dos
leucócitos fagocitários. A esses microrganismos dá-se o nome de bactérias piogênicas.
Várias enzimas podem ser liberadas pelas bactérias patogênicas. Um exemplo é a colagenase, que ata-
ca o colágeno da derme, facilitando a disseminação subcutânea da infecção.
Outras importantes substâncias bacterianas responsáveis por efeitos prejudiciais ao organismo são
as toxinas. As endotoxinas são componentes das paredes celulares e normalmente provocam febre.
Já as exotoxinas são secreções da célula bacteriana. As mais importantes são a diftérica, a botulínica
e a tetânica. Esta última tem efeito mortal em doses de apenas frações de miligramas. Por outro lado,
quando a exotoxina tetânica é alterada pela ação de calor e de formaldeído, constitui o toxoide, que não
apresenta toxicidade. No entanto, o toxoide mantém a capacidade de funcionar como antígeno e, por
isso, é usado como vacina, induzindo a formação de anticorpos específicos no hospedeiro.
O organismo apresenta importantes defesas contra a ação de bactérias patogênicas. Podemos citar
como exemplos as secreções sebáceas da pele; o pH ácido de algumas mucosas, como a vaginal (ácido
lático) e a gástrica (ácido clorídrico); a lisozima das lágrimas; as substâncias produzidas pelo sistema
imunitário, como as imunoglobulinas e os interferons; e a fagocitose.
Substâncias como as sulfas e os antibióticos são importantes drogas que auxiliam no tratamento de
doenças bacterianas. Elas podem agir de duas maneiras: como bactericidas, matando as bactérias, ou
como bacteriostáticas, inibindo a síntese da parede celular, impedindo assim o crescimento e a repro-
dução desses parasitas.
A tabela a seguir fornece algumas informações das principais doenças provocadas por bactérias no ser
humano.
PRINCIPAIS DOENÇAS BACTERIANAS DO SER HUMANO
Doença Bactéria Transmissão Sintomas
Saliva e secreções da laringe e dos
Haemophilus pertussis Acessos de tosse forte e prolonga-
Coqueluche brônquios. Bactéria penetra nas mu-
(bacilo) da. Há vacina.
cosas das vias respiratórias.
Uretrite, com corrimento, que se
Neisseria gonorrhoeae Infeccção sexualmente transmissí-
Gonorreia propaga para outros órgãos do sis-
(gonococo) veis (IST).
tema genital.
Meningite menin- Neisseria meningitidis Febre alta, vômito em jato e rigidez
Secreções nasobucais.
gocócica (meningococo) da nuca. Há vacina.

76
PRINCIPAIS DOENÇAS BACTERIANAS DO SER HUMANO
Doença Bactéria Transmissão Sintomas
Diplococcus pneumo- Febre alta e fortes dores pulmona-
Peneumonia Secreções nasobucais.
niae (diplococo) res na região dorsal.
Evolução lenta. Início: lesão primá-
Treponema pallidum Infecção sexualmente transmissível ria (cancro duro) e disseminação
Sífilis
(espiroqueta) (IST) pelo sangue. Tardiamente: graves
lesões no sistema nervoso central.
Clostridium tetani Ferimentos profundos provocados Intoxicação aguda com enrijecimen-
Tétano
(bacilo) por objetos contaminados. to muscular. Há vacina.
Tosse, expectoração, inapetência,
Mycibacterium tubercu-
Tuberculose Saliva e catarro. cansaço e sudorese noturna. Há va-
losis (bacilo de Koch)
cina.
Mycobacterium leprae Secreções em contato com narinas, Lesões cutâneas, perda da sensibili-
Hanseníase
(bacilo de Hansen) boca e pele. dade e manchas na pele.
Sensação de haver um corpo estra-
Contato pessoal (secreções) ou atra-
nho (“areia”) alojado nas pálpebras.
Tracoma Chlamydia trachomatis vés de objetos contaminados (len-
A inflamação afeta a córnea e a con-
ços, toalhas).
jutiva, podendo levar à cegueira.
Picada do carrapto-estrela (Amblyo-
ma cajennense), que é também o re-
servatório natural do patógeno. Esse
carrapato tem como hospedeiros Manchas (máculas) vermelhas na
Febre maculosa Rickettsia rickettsii preferenciais os equinos, os bovi- pele do indivíduo, devido a hemorra-
nos, os cães, as capivaras e outros gias subcutâneas.
animais, e, incidentalmente ataca
também o ser humano, transmitin-
do-lhe a doença.
Contaminação fecal de água e ali- Forte diarreia, com desidratação e
Cólera Vibrio cholerae (vibrião)
mentos. prostração. Há vacina.
Ferimentos e mucosas em contato
Leptospira ssp. Febre, dores musculares e lesão he-
Leptospirose com água contaminada por urina de
(espiroqueta) pática.
rato.

As protozooses são as doenças causadas por protozoários, das quais merecem destaque a amebíase,
a leishmaniose, a doença de Chagas e a malária. As duas últimas são as maiores e mais graves ende-
mias brasileiras causadas por protozoários. A malária, em particular, representa um grave problema
de saúde pública em muitos locais do mundo. No Brasil, o maior número de casos registrados está na
região Norte.
Observe a tabela a seguir, que fornece as principais informações referentes a algumas protozooses que
atingem o ser humano no Brasil.
PRINCIPAIS PROTOZOOSES HUMANAS NO BRASIL
Espécie Grupo Doença Sintomas Transmissão
Entamoeba his- Ulcerações intestinais, diarreia Ingestão de cistos eliminados
Rizópode Amebíase
tolytica e enfraquecimento. com as fezes humanas.
Miocardite e lesões na muscu- Fezes do inseto Triatoma sp.
Trypanosoma Doença de Cha-
Flagelado latura do tubo digestório (esô- (barbeiro) por meio de lesões na
cruzi gas
fago). pele.
Leishmaniose Te- Ulcerações no rosto, nos bra- Picada do mosquito-palha ou
Leishmania brazi-
Flagelado gumentar Ameri- ços e nas pernas e necrose de brigui (Luttzomyia ou Pheboto-
liensis
cana tecidos conjuntivos. mus).

77
PRINCIPAIS PROTOZOOSES HUMANAS NO BRASIL
Espécie Grupo Doença Sintomas Transmissão
Picada do mosquito-palha ou
Leishmania L e i s h m a n i o s e Hipertrofia do braço e do fíga-
Flagelado brigui (Luttzomyia ou Pheboto-
chagasi Visceral do, febre e enfraquecimento.
mus).
Trichomonas Relação sexual ou toalhas e ob-
Flagelado Tricomoníase Vaginite, uretrite e corrimento.
vaginalis jetos úmidos contaminados.
Colite, com dores intestinais e Ingestão de cistos eliminados
Giardia lamblia Flagelado Giardíase
diarreia. com fezes humanas.
Febre, anemia e lesões no baço, Picada da fêmea do mosquito-
Plasmodium vivax Apicomplexo Malária
no fígado e na medula óssea. -prego (gênero Anopheles).
Ingestão de cistos expelidos
Toxoplasma Cegueira, aborto e problemas com as fezes de gatos, que fi-
Apicomplexo Toxoplasmose
gondii neurológicos. cam em caixas de areia e no
lixo.

PARA SABER MAIS:


Se você quer aprofundar os seus conhecimentos no assunto confira abaixo algumas sugestões:
https://drive.google.com/file/d/1lskhfg9pP3Wf2DE5VkEpzO6t7ck9CZhT/view - Biologia - Parasitoses I.
Aula exibida para os alunos do Ensino Médio (ENEM) no dia 18/09/2020 com o professor Vinícius Braz no
programa Se Liga na Educação.
https://drive.google.com/file/d/1txKO7PIdzJ8-RDAq4Lz90rsozxhacE0T/view - Biologia - Parasitoses II.
Aula exibida para os alunos do Ensino Médio (ENEM) no dia 25/09/2020 com o professor Vinícius Braz no
programa Se Liga na Educação.

ATIVIDADES
1 - (UNESP 2020) No romance O amor nos tempos do cólera, Gabriel García Márquez relata os primeiros
contatos do jovem médico Juvenal Urbino, um dos três protagonistas do romance, com o cólera.
O cólera se transformou em obsessão. Não sabia a respeito mais do que aprendera na rotina de algum
curso marginal, e lhe parecera inverossímil que há apenas trinta anos tivesse causado na França, in-
clusive em Paris, mais de cento e quarenta mil mortes. Mas depois da morte do pai aprendeu tudo que
se podia aprender sobre as diversas formas do cólera, quase como uma penitência para dar descanso
à sua memória, e foi aluno do epidemiólogo mais destacado do seu tempo […], o professor Adrien
Proust, pai do grande romancista. De modo que quando voltou à sua terra e sentiu vinda do mar a pes-
tilência do mercado, e viu os ratos nos esgotos expostos e os meninos se revolvendo nus nas poças
das ruas, não só compreendeu que a desgraça tivesse acontecido como teve a certeza de que se re-
petiria a qualquer momento.
(O amor nos tempos do cólera, 1985.)

78
A partir desse trecho, pode-se inferir que Juvenal Urbino:
a) se preocupou em combater, no século XX, o retorno da epidemia de cólera na França, principal-
mente em Paris, constatando que a doença era transmitida pela urina de ratos.
b) tivera seu pai morto pelo cólera ainda no século XIX, o que o motivou a investigar as causas des-
sa doença, no caso, microrganismos eucariotos transmitidos por ratos que se proliferam nos
esgotos.
c) viveu na Europa da Idade Média, quando ocorria a grande epidemia de cólera e quando ainda se
acreditava que a doença era transmitida por vapores pestilentos dos esgotos.
d) temia uma epidemia de cólera em sua cidade natal e, ainda no século XVIII, aprendeu com seu
professor que a falta de saneamento básico favorece os surtos dessa virose.
e) se interessou pela doença entre o final do século XIX e o início do século XX, percebendo que
as pessoas que entravam em contato com fezes contaminadas contraíam cólera, uma doença
transmitida por bactérias.

2 - (MACKENZIE 2019) O quadro abaixo apresenta três parasitoses, seus causadores, principais sintomas
e formas de contágio.

As letras X, Y e Z correspondem, respectivamente, a


a) X - Confusão mental, convulsões.
Y – Meningite.
Z - Protozoário Plasmodium vivax.
b) X - Contração involuntária da musculatura, inclusive do diafragma.
Y – Raiva.
Z - Protozoário Trypanosoma cruzi.
c) X - Lesões nos pulmões, coração e fígado.
Y – Tuberculose.
Z - Bactéria Yersinia pestis.
d) X - Espasmos musculares, convulsões.
Y - Peste bubônica.
Z - Bactéria Leptospira interrogans.
e) X - Parada cardiorrespiratória.
Y – leptospirose.
Z - Protozoário Leishmania brasiliensis.

79
3 - (ENEM 2010) A tabela apresenta dados comparados de respostas de brasileiros, norte-americanos
e europeus a perguntas relacionadas à compreensão de fatos científicos pelo público leigo. Após cada
afirmativa, entre parênteses, aparece se a afirmativa é Falsa ou Verdadeira. Nas três colunas da direita
aparecem os respectivos percentuais de acertos dos três grupos sobre essas afirmativas.

De acordo com os dados apresentados na tabela, os norte-americanos, em relação aos europeus e aos
brasileiros, demonstram melhor compreender o fato científico sobre:
a) a ação dos antibióticos.
b) a origem do ser humano.
c) os períodos da pré-história.
d) o deslocamento dos continentes.
e) o tamanho das partículas atômicas.

4 - (ENEM 2016) A sombra do cedro vem se encostar no cocho. Primo Ribeiro levantou os ombros;
começa a tremer. Com muito atraso. Mas ele tem no baço duas colmeias de bichinhos maldosos, que
não se misturam, soltando enxames no sangue em dias alternados. E assim nunca precisa de passar um
dia sem tremer.
ROSA, J. G. Sagarana. Rio de Janeiro: Nova Fronteira, 1984.

O texto de João Guimarães Rosa descreve as manifestações das crises paroxísticas da malária em seu
personagem. Essas se caracterizam por febre alta, calafrios, sudorese intensa e tremores, com inter-
valos de 48 h ou 72 h, dependendo da espécie de Plasmodium. Essas crises periódicas ocorrem em
razão da
a) lise das hemácias, liberando merozoítos e substâncias denominadas hemozoínas.
b) invasão das hemácias por merozoítos com maturação até a forma esquizonte.
c) reprodução assexuada dos esporozoítos no fígado do indivíduo infectado.
d) liberação de merozoítos dos hepatócitos para a corrente sanguínea.
e) formação de gametócitos dentro das hemácias.

80
5 - (PUC-SP 2017) As imagens a seguir mostram dois vetores de doenças que afetam milhares de pessoas
no Brasil.

Os agentes etiológicos transmitidos pelos vetores A e B acima são, respectivamente, classificados


como:
a) vírus e nematelminto.
b) bactéria e vírus.
c) vírus e protozoário.
d) bactéria e nematelminto.
e) N.D.A.

81
SEMANAS 5 E 6

EIXO TEMÁTICO:
Biodiversidade.

TEMA/TÓPICO(S):
2. História da Vida na Terra.
13. Ciclo de vida dos seres vivos e suas adaptações em diferentes ambientes.
14. Características fisiológicas e adaptações dos seres vivos nos diferentes ambientes da Terra.

HABILIDADE(S):
13.1. Reconhecer a diversidade das adaptações que propiciam a vida nos diferentes ambientes.
13.1.1. Identificar em situações-problema que a diversidade das adaptações propiciam a vida em diferentes
ambientes.
14.2. Reconhecer características adaptativas das plantas em diferentes ambientes.
14.2.1. Identificar características morfológicas e fisiológicas das plantas relacionadas a: sustentação, econo-
mia de água, reprodução, transporte e trocas gasosas, relacionando-as com o habitat.

CONTEÚDOS RELACIONADOS:
Reino Plantae.

TEMA: Reino Plantae


Caro(a) estudante, nesta semana de estudo, você vai reconhecer a diversidade do Reino Plantae.

DIVERSIDADE DAS PLANTAS


As plantas foram os primeiros seres vivos a conquistar a terra firme. Ancestrais das plantas atuais co-
meçaram a ocupar as regiões litorâneas há mais de 500 milhões de anos, abrindo caminho para a che-
gada dos animais. A Botânica, ramo da Biologia que estuda as plantas, e outras áreas biológicas têm
conseguido alterar variedades de plantas domesticadas, tornando‐as mais produtivas ou mais nutriti-
vas, entre outras características. Um dos grandes desafios atuais é compreender melhor esses orga-
nismos, para que possamos atuar em sua preservação.
Graças ao estudo das plantas e de sua fisiologia, podemos conhecer as incríveis soluções e estraté-
gias adaptativas desses seres vivos. Uma delas, talvez a principal, é o fato de elas serem autotróficas,
ou seja, capazes de produzir seu próprio alimento; elas certamente sobreviveriam sem nós, mas nós
dificilmente sobreviveríamos sem elas. Além disso, as plantas se utilizam de processos relacionados
às propriedades da água, como a evaporação, a capilaridade e a tensão superficial, entre outros, para
realizar com alta eficiência seus processos metabólicos.
Há evidências de que as primeiras plantas que conquistaram a terra firme eram semelhantes aos mus-
gos e às hepáticas de hoje. Essas plantas não apresentavam vasos condutores de seiva. Plantas sem
vasos condutores são denominadas pelos botânicos plantas avasculares (do grego a, prefixo de nega-
ção, e do latim vasculum, pequeno vaso, túbulo).

82
Fonte: CAMPBELL, N.A.; REECE, J.B.; URRY, L.A.; CAIN, M.L.; WASSERMAN, S.A.; MINORSKY, P.V. & Jackson, R.B. 2010. Biologia.10ª ed.
Artmed, Porto Alegre, 1488 p.

Com a evolução das plantas pioneiras surgiram novidades, como o desenvolvimento de estruturas es-
pecializadas na absorção de água e sais minerais do solo – raízes – e o aparecimento de estruturas tu-
bulares internas – vasos condutores – que agilizam o transporte de soluções nutritivas por todo o corpo
da planta. Plantas dotadas de tecidos condutores de seiva são denominadas plantas vasculares, ou
traqueófitas. As primeiras plantas vasculares aparecem no documentário fóssil cerca de 40 milhões de
anos após a conquista da terra firme pelas primeiras plantas.
Com exceção das briófitas (musgos, hepáticas e antóceros atuais), todos os outros grupos vegetais
são traqueófitas, assim chamadas por causa da presença de reforços nas paredes dos vasos condu-
tores de seiva que lembram as traqueias dos insetos. As plantas vasculares também desenvolveram
sistemas eficientes de proteção contra a perda de água, como epidermes impermeáveis e estômatos
de abertura regulável, características que possibilitaram sua adaptação a regiões relativamente secas,
distantes das bordas de rios e lagos onde viveram suas ancestrais. Acredita-se que tenha sido nessa
etapa do processo evolutivo que as plantas assumiram uma organização corporal típica, com raízes,
caule e folhas. As novidades evolutivas possibilitaram às plantas vasculares atingir grandes tamanhos
e constituir vastas comunidades florestais. Esses novos ambientes ofereceram condições propícias à
colonização por animais, que encontraram na vegetação abrigo e alimento.
Outra novidade importante ocorrida na evolução das plantas foi o surgimento da semente, estrutura re-
sistente que abriga um embrião envolto por tecidos nutritivos produzidos pela planta ‐mãe. A semente
possibilitou às plantas espermatófitas, grupo que reúne plantas com sementes, dispersarem ‐se pelos
mais diversos ambientes da Terra.
No sistema de classificação, que iremos utilizar, as plantas são classificadas em 10 filos e agrupadas
em 4 grupos, conforme tabela abaixo.

83
CARACTERÍSTICAS E CLASSIFICAÇÃO DOS COMPONENTES ATUAIS DO REINO PLANTAE
CARACTERÍSTICAS
TECIDOS FILOS GRUPO
SEMENTE FRUTO
CONDUTORES
Bryophyta (musgos)
Ausentes Hepatophyta (hepáticas) Briófitas
Ausente Anthocerophytas (antóceros)
Pteridophyta (samambaias)
Pteridófitas
Ausente Lycopodiophyta (licopódios)
Coniferophyta (coníferas)
Presentes Cycadophyta (cicadófita)
Gimnospermas
Presente Gnetophyta (gnetófitas)
Ginkgophyta (gincófitas)
Presente Anthophyta (angiospermas) Angiospermas

Esses grupos apresentam em comum: parede celular formada por celulose; embriões protegidos por
tecidos originados do corpo materno; cloroplastos contendo as clorofilas a e b, carotenos e xantofilas;
reserva constituída por amido e reprodução por alternância de gerações (metagênese), na qual a meio-
se é espórica (intermediária).
Pelo fato de produzirem embriões, são classificados como embriófitos.
Os embriófitos podem ou não desenvolver tecido condutor (vascular), daí a classificação:
• Embriófitos avasculares: representados pelas briófitas.
• Embriófitos vasculares ou traqueófitas: são incluídas as pteridófitas, gimnospermas e angios-
permas.
Produzem sementes apenas as gimnospermas e angiospermas, por isso recebem o nome de espermá-
fitas ou espermatófitas.
As únicas plantas que produzem flores e frutos são as angiospermas.
As gimnospermas produzem estróbilos (pinhas) e sementes. Não formam verdadeiras flores.
O ciclo de vida das plantas apresenta meiose intermediária ou espórica. Nesse ciclo de vida, surgem dois
organismos adultos: um diploide (2n), chamado esporófito e outro haploide (n), chamado gametófito.
O esporófito, através da meiose, produz células assexuadas: esporos.
O gametófito, através de mitoses, produz células sexuadas, os gametas.
Podem-se reconhecer, neste ciclo (haplodiplobionte), duas fases bem distintas: uma haploide (n), pro-
dutora de gametas, chamada geração gametofítica e outra diploide(2n), produtora de esporos, chama-
da geração esporofítica. Essas fases alternam-se entre si para completar o ciclo de vida, constituindo
a alternância de gerações ou metagênese. Quando se analisa a metagênese, nos diversos grupos ve-
getais, reconhecerem-se diferenças quanto à duração das fases esporofítica e gametofítica e quanto à
complexidade do esporófito e gametófito.
Nas briófitas, o gametófito é desenvolvido e duradouro, e o esporófito é reduzido e dependente do game-
tófito. Nas pteridófitas, o gametófito é reduzido e transitório, enquanto o esporófito passa a ser vegetal
desenvolvido, complexo e duradouro. Nestas plantas, ambos, gametófito e esporófito, são verdes e in-
dependentes. Nas fanerógamas (gimnospermas e angiospermas), o esporófito é a planta verde, com-
plexo, duradouro e visível, enquanto o gametófito é muito reduzido e dependente do esporófito.

84
Fonte: CAMPBELL, N.A.; REECE, J.B.; URRY, L.A.; CAIN, M.L.; WASSERMAN, S.A.; MINORSKY, P.V. & Jackson, R.B. 2010. Biologia.10ª ed.
Artmed, Porto Alegre, 1488 p.

As briófitas mais conhecidas são os musgos e as hepáticas. São plantas muito primitivas, vivendo, na
maioria das vezes, em ambientes terrestres úmidos e sombreados (plantas umbrófilas). Poucas espé-
cies vivem em água doce e nenhuma é marinha.
Não possuem, evidentemente, flores, frutos e sementes. O corpo primitivo é desprovido de tecidos
vasculares, e isso representa o fator responsável pelo tamanho reduzido que caracteriza essas plantas.
Nessas plantas, encontra-se uma nítida alternância de gerações (metagênese), em que o gametófito
representa a planta verde, complexo e duradouro (permanente), enquanto o esporófito é uma planta
reduzida (transitório) e dependente (parasita) do gametófito.
Ciclo de Vida de um musgo

Fonte: CAMPBELL, N.A.; REECE, J.B.; URRY, L.A.; CAIN, M.L.; WASSERMAN, S.A.; MINORSKY, P.V. & Jackson, R.B. 2010. Biologia.10ª ed.
Artmed, Porto Alegre, 1488 p.

85
Ao contrário das plantas avasculares, as primeiras plantas vasculares tinham esporófitos ramificados,
independentes dos gametófitos para a nutrição. À medida que os corpos das plantas tornaram-se pro-
gressivamente complexos, a competição por espaço e por luz solar provavelmente foi aumentando.
Como veremos, essa competição pode ter estimulado ainda mais a evolução nas plantas vasculares,
levando, por fim, à formação das primeiras florestas.
As pteridófitas são plantas pluricelulares, autótrofas, que vivem em ambientes terrestres úmidos e
sombreados, sendo a Mata Atlântica o hábitat da maioria das espécies.
São plantas vasculares (traqueófitas), isto é, desenvolvem tecidos especializados para o transporte de
seiva bruta (lenho ou xilema) e de seiva elaborada (líber ou floema). Apresentam meiose intermediária
ou espórica e uma alternância de gerações (metagênese) muito nítida, ou seja, possuem gametófito e
esporófito macroscópicos e verdes. O esporófito é o vegetal complexo, com vida duradoura (permanen-
te); é autótrofo e, na maioria das vezes, possui raízes, caules e folhas.
Não forma flor, fruto nem semente.

Ciclo de Vida de uma samambaia

Fonte: CAMPBELL, N.A.; REECE, J.B.; URRY, L.A.; CAIN, M.L.; WASSERMAN, S.A.; MINORSKY, P.V. & Jackson, R.B. 2010. Biologia.10ª ed.
Artmed, Porto Alegre, 1488 p.

As gimnospermas são plantas terrestres de grande porte (árvores e arbustos), vasculares, que vivem de
preferência em climas frios. Apresentam metagênese pouco nítida.
O esporófito é a planta verde, complexo e duradouro, organizado em raiz, caule, folha, estróbilos e se-
mentes. Os gametófitos são dioicos, reduzidos em tamanho, tempo de vida e complexidade, e depen-
dentes do esporófito.
O gametófito chama-se tubo polínico ou microprótalo. Em algumas gimnospermas primitivas, os ga-
metas ainda são os anterozoides; nas mais evoluídas, como os pinheiros, os gametas são as células
espermáticas ou núcleos espermáticos. O gametófito chama-se saco embrionário ou megaprótalo e

86
forma-se no interior do óvulo. O megaprótalo produz arquegônios bastante rudimentares, no interior
dos quais se encontram os gametas femininos chamados oosferas. Cada arquegônio produz apenas
uma oosfera.
As gimnospermas produzem estróbilos e sementes, mas nunca produzem frutos.

Ciclo de Vida de um pinheiro

Fonte: CAMPBELL, N.A.; REECE, J.B.; URRY, L.A.; CAIN, M.L.; WASSERMAN, S.A.; MINORSKY, P.V. & Jackson, R.B. 2010. Biologia.10ª ed.
Artmed, Porto Alegre, 1488 p.

As angiospermas são as plantas mais adaptadas aos ambientes terrestres. São encontradas nos mais
variados lugares: desde os muito úmidos até os desérticos. Poucas são as espécies que vivem em
água doce.
Existem raríssimas espécies marinhas. Podem ser ervas, arbustos ou árvores.
A maioria apresenta nutrição autótrofa fotossintetizante, mas existem algumas espécies holoparasi-
tas, como o cipó-chumbo, que não possuem clorofila e não realizam fotossíntese. Estes vegetais reti-
ram a seiva elaborada de outro vegetal hospedeiro.
Muitas espécies são epífitas, isto é, vivem apoiadas sobre ramos de outros vegetais, com a única finali-
dade de obter maior luminosidade. Existem muitas espécies de orquídeas e bromélias epífitas.
Quanto ao ciclo reprodutor, as angiospermas apresentam meiose intermediária ou espórica e uma al-
ternância de gerações pouco nítida. A planta que vemos crescer na natureza é o esporófito, organiza
do em raiz, caule e folhas e produtor de flores, frutos e sementes. Estes vegetais são os únicos que
formam frutos.

87
Os gametófitos são dioicos, extrema mente reduzidos e dependentes do esporófito. Na verdade, cres-
cem no interior da flor.
O gametófito feminino é o saco embrionário (megaprótalo) contido no óvulo. Não forma arquegônio e
possui uma única oosfera (gameta).
O gametófito masculino é o tubo polínico (microprótalo), no interior do qual se formam dois núcleos
espermáticos (gaméticos), que representam os gametas. Não dependem de água para a fecundação.

Ciclo de Vida de uma angiosperma

Fonte: CAMPBELL, N.A.; REECE, J.B.; URRY, L.A.; CAIN, M.L.; WASSERMAN, S.A.; MINORSKY, P.V. & Jackson, R.B. 2010. Biologia.10ª ed.
Artmed, Porto Alegre, 1488 p.

PARA SABER MAIS:


Se você quer aprofundar os seus conhecimentos no assunto confira abaixo algumas sugestões:
https://drive.google.com/file/d/1oFSK9pbRS9PxPLjkpqXzS-6XhebGxX71/view - Biologia - Origem e
Evolução das Plantas. Aula exibida para o 2º Ano do Ensino Médio no dia 01/10/2020 com o professor Viní-
cius Braz no programa Se Liga na Educação.
https://drive.google.com/file/d/1otaL0UAXtI-jzWiWmgoP9klCuI9vfpgn/view - Biologia - Plantas Avas-
culares. Aula exibida para o 2º Ano do Ensino Médio no dia 22/10/2020 com o professor Vinícius Braz no
programa Se Liga na Educação.

88
https://drive.google.com/file/d/1rsr33eNipZjeRLAEqNfw9BQlTvyuEwNt/view - Biologia - Diversidade
das Plantas I. Aula exibida para os alunos do Ensino Médio (ENEM) no dia 23/10/2020 com o professor
Vinícius Braz no programa Se Liga na Educação.
https://drive.google.com/file/d/1sMMSWk7XYk57umpVyC6br631Se8qF40N/view - Biologia - Diversida-
de das plantas II. Aula exibida para os alunos do Ensino Médio (ENEM) no dia 12/11/2020 com o professor
Vinícius Braz no programa Se Liga na Educação.
https://drive.google.com/file/d/13EpCwt-Ypu_7GsdfiW0F-N8CPa48njwB/view - Biologia - Diversidade
das Plantas III. Aula exibida para os alunos do Ensino Médio (ENEM) no dia 13/11/2020 com o professor
Vinícius Braz no programa Se Liga na Educação.

ATIVIDADES
1 - (FUVEST 2021) Considere três espécies de plantas (X, Y e Z) e suas características:
• A planta X não possui flores, mas é polinizada pelo vento. Além disso, não possui frutos, mas suas
sementes são dispersas por aves.
• A planta Y não possui flores, nem sementes, nem frutos.
• A planta Z possui flores e é polinizada por aves. Além disso, possui frutos e suas sementes são
dispersas por aves.
A que grupos pertencem as plantas X, Y e Z, respectivamente?
a) Pteridófitas, angiospermas e gimnospermas.
b) Gimnospermas, pteridófitas e angiospermas.
c) Pteridófitas, gimnospermas e angiospermas.
d) Angiospermas, gimnospermas e pteridófitas.
e) Gimnospermas, angiospermas e pteridófitas.

2 - (UCS 2019) Os grupos vegetais desenvolveram, ao longo do processo evolutivo, diferentes


características anatômicas, funcionais e reprodutivas, que garantiram às plantas a colonização dos
mais distintos ambientes dentro dos ecossistemas. Observe o quadro abaixo e assinale a alternativa
em que essas características estão corretamente relacionadas.

a) a
b) b
c) c
d) d
e) e

89
3 - (PUCPR 2019) Leia o texto a seguir.
Chá de cavalinha
De nome científico Equisetum arvense, a cavalinha é uma herbácea de rizoma horizontal de onde saem
dois caules aéreos: os férteis e os estéreis. Ambos chegam, em média, até 30 cm de altura e são de cor
amarelada na base e avermelhada na ponta, de onde sai a “espiga”. A cavalinha possui folhas pequenas
em formato de agulha, sendo uma planta que não possui flores nem sementes.
Um dos seres vivos mais antigos da Terra, a cavalinha é datada do período Paleozoico, quando havia
espécimes de até 10 metros de altura por 2 de diâmetro.
[...]
Atualmente, a planta vem sendo utilizada, principalmente, com finalidades terapêuticas, sendo o seu
consumo em forma de chá um dos mais populares. Ela pode ser plantada em pequenos vasos e guarda-
das em residências, já que vive bem em meia sombra.
Para que serve
A cavalinha é uma planta muito eficiente para o tratamento de problemas ósseos, para o tratamento
de disfunções renais, de disfunções de vias urinárias e de problemas na próstata. O chá também pode
ser utilizado para tratar hemorragias nasais, perda excessiva de sangue na menstruação, inflamação
da próstata e casos de inchaço. O chá de cavalinha estimula a cicatrização e atua como profundo hi-
dratante da pele e do organismo como um todo. Além disso, ele é muito eficiente na recuperação de
ferimentos na pele e no tratamento de frieiras, de aftas e de úlceras. Outro benefício do consumo do
chá de cavalinha é a boa aparência das unhas e da pele.
[...]
Disponível em <http://www.saudemedicina.com/cha-de-cavalinha/>. Acesso em: 13/06/17.

Com base nas descrições do texto, é CORRETO afirmar que a cavalinha pertence ao grupo das

a) briófitas. b) angiospermas. c) pteridófitas. d) gimnospermas. e) espermatófitas.

4 - (UFRGS 2018) Considere as estruturas esquematizadas abaixo, coletadas no Parque Farroupilha, em


Porto Alegre. 

Assinale a alternativa correta sobre essas estruturas.


a) 1 e 3 são estruturas reprodutivas.
b) 2 e 3 são estruturas de angiospermas.
c) 3 é uma estrutura com função de absorção de nutrientes.
d) 2 é uma estrutura que corresponde ao fruto.
e) 1, 2 e 3 são estruturas de plantas vasculares.

90
5 - (MACKENZIE 2012)
A respeito das plantas representadas acima, são feitas
as seguintes afirmações:
I. B e D representam as fases esporofíticas, formadas
por células diploides (2n).
II. A e C representam as fases gametofíticas, formadas
por células haploides (n).
III. B e C são originadas a partir do zigoto.
IV. Anterozoide e oosfera são produzidos por meiose.
Estão corretas, apenas,

a) I e II. b) I e III. c) II e III. d) I e IV. e) III e IV.

6 - (MACKENZIE 2011) Durante o processo evolutivo das plantas, ficou marcada a transição do meio
aquático para o terrestre. Algumas adaptações surgiram, tais como vasos condutores, flor, tubo
polínico, sementes e frutos. 

Considerando a sequência evolutiva representada acima, é correto afirmar que o surgimento


a) de sementes ocorreu em B.
b) de vasos condutores ocorreu em A.
c) de tubo polínico ocorreu em C.
d) de frutos ocorreu em C.
e) de flores ocorreu em D.

7 - (UNICAMP 2020) Relatório publicado em 2019 alertou que um número crescente de espécies de
animais polinizadores está ameaçado de extinção em todo o mundo em decorrência de fatores como
desmatamento, uso indiscriminado de agrotóxicos e alterações climáticas. Importantes medidas
devem ser adotadas para prevenir as consequências econômicas, a redução na produção de alimentos
e o desequilíbrio dos ecossistemas. Entre as espécies cultivadas no Brasil que dependem de polinização
animal, destacam-se o maracujá, a maçã, a acerola e a castanha-do-brasil.
(Fonte: Marina Wolowski e outros, Relatório temático sobre polinização, polinizadores e produção de alimentos no Brasil. BPBES e
REBIPP, 2019. Acessado em 23/05/2019.)

Considerando as informações fornecidas no texto e os conhecimentos sobre botânica e ecologia, é


correto afirmar que a polinização pode ser beneficiada
a) por insetos que transportam o pólen da antera para o estigma nas eudicotiledôneas mencionadas.
b) por pequenos vertebrados que transferem pólen do estigma para o estame nas monocotiledô-
neas mencionadas.
c) por insetos que transferem pólen do estigma para o estame nas eudicotiledôneas mencionadas.
d) por pequenos mamíferos que transportam o pólen da antera para o estigma nas monocotiledô-
neas mencionadas.

91
8 - (ENEM 2019) Durante sua evolução, as plantas apresentaram grande diversidade de características, as
quais permitiram sua sobrevivência em diferentes ambientes. Na imagem, cinco dessas características
estão indicadas por números.

A aquisição evolutiva que permitiu a conquista definitiva do ambiente terrestre pelas plantas está indi-
cada pelo número

a) 1. b) 2. c) 3. d) 4. e) 5.

9 - (PUCPR 2020) Leia o trecho a seguir.


Trigo, arroz, milho: toda a família dos cereais tem sementes muito resistentes. Podem perder 95% da
água em suas células e sobreviver. Cereais adultos, infelizmente, são bem menos durões. Não precisava
ser assim: o DNA de qualquer planta ainda guarda as propriedades que ela possuía como “recém-nas-
cida”, só que dormentes. Há exceções (precisamente 135 plantas) que nunca abandonam sua criança
interior. Elas resistem à desidratação intensa por meses – até anos. Ao menor sinal de água, revivem
em questão de horas. Por esse feito, recebem o nome de Ressuscitadoras. ”Elas usam exatamente os
mesmos genes que uma semente, só que nas folhas, no caule, em tudo”, diz Jill Farrant, da Universidade
da Cidade do Cabo. Ela acredita que essas plantas guardam a chave para religar os mesmos genes em
outras espécies e torná-las mais duronas. Mais dia, menos dia, pode ser que a inteligência das plantas
esteja salvando economias mundo afora. Afinal, 9 em cada 10 calorias que você come vêm do trigo, do
arroz e do milho.
Disponível em: <https://super.abril.com.br/ciencia/a-inteligencia-secreta-das-plantas/>. Acesso em: 07/08/2019.

Com base nas características apresentadas no texto, é possível afirmar que as plantas citadas são
a) angiospermas, pois apresentam sementes protegidas por fruto.
b) gimnospermas, pois apresentam raiz, caule, folha e semente.
c) pteridófitas, pois dependem da água para a reprodução.
d) briófitas, pois são plantas de tamanho reduzido que vivem em ambientes úmidos.
e) fanerógamas, pois apresentam a semente como novidade evolutiva.

92
10 - (UPF 2018)

(Foto: Daniel Castellano. Disponível em: http://www.gazetadopovo.com.br/vida-ecidadania/ritmo-de-regeneracao-das-araucarias-


e-preocupante-0216q6xu4dor9. Acesso em 3 set. 2017)

“A araucária (Araucaria mportância) é a mais bela e imponente árvore do Sul do Brasil. É uma planta
muito antiga que chegou a constituir mais de 40% das árvores existentes na Floresta Ombrófila Mista,
por isso mesmo conhecida como Mata de Araucárias. É uma espécie pioneira, cuja proteção permite
o crescimento das demais espécies ao seu redor. Os animais e mesmo parte da cultura brasileira de-
pendem da araucária. Um dos pratos típicos, o pinhão, também está no cardápio obrigatório da fauna
desse ecossistema.”
(Disponível em: http://vamossalvarnossoplaneta.blogspot.com.br/2008/08/mportância-daaraucria-e-tentativa-de.html. Adaptado.
Acesso em 3 set. 2017)

Em relação às características botânicas da araucária, assinale a alternativa correta.


a) Seus gametófitos são microscópicos e nutricionalmente dependentes do esporófito.
b) É uma gimnosperma monoica, pertencente ao grupo das coníferas.
c) As sementes, os pinhões, são produzidas nos frutos denominados pinhas.
d) É uma espécie caducifólia, com folhas simples e lanceoladas.
e) Apresenta xilema constituído por traqueídeos e elementos de vaso, que funcionam como células
condutoras de água e de sustentação.

REFERÊNCIAS:
AMABIS, JOSÉ MARIANO; MARTHO, GILBERTO RODRIGUES. Volume 2: Biologia dos organismos –
3. Ed. – São Paulo: Moderna, 2010.
CAMPBELL, N.A.; REECE, J.B.; URRY, L.A.; CAIN, M.L.; WASSERMAN, S.A.; MINORSKY, P.V. & Jack-
son, R.B. Biologia. 10 ed. Artmed, Porto Alegre, 2010.
FAVARETTO, José Arnaldo. BIOLOGIA: Unidade e Diversidade. v3. São Paulo: FTD, 2017.
LINHARES, Sérgio. GEWANDSZNAJDER, Fernando. PACCA, Helena. Biologia Hoje - volume 2. 3 ed.
São Paulo: Ática, 2017.
LOPES, Sônia.; ROSSO, Sérgio. BIO – Volume 2. 3 ed. São Paulo: Saraiva, 2016.

93
SECRETARIA DE ESTADO DE EDUCAÇÃO DE MINAS GERAIS

PLANO DE ESTUDO TUTORADO


COMPONENTE CURRICULAR: QUÍMICA
ANO DE ESCOLARIDADE: 2º ANO – EM
PET VOLUME: 04/2021
NOME DA ESCOLA:
ESTUDANTE:
TURMA: TURNO:
BIMESTRE: 4º TOTAL DE SEMANAS:
NÚMERO DE AULAS POR SEMANA: NÚMERO DE AULAS POR MÊS:

SEMANA 1

EIXO TEMÁTICO:
Materiais - Aprofundamento.

TEMA / TÓPICO(S):
Propriedades dos Materiais.

HABILIDADE(S):
16.1. Reconhecer a variação na velocidade das Transformações Químicas (TQ).
16.1.1. Reconhecer que as TQ podem ocorrer em diferentes escalas de tempo.

CONTEÚDOS RELACIONADOS:
Cinética; Rapidez ou Velocidade de Reação.

TEMA: Equilíbrio dinâmico


Querido(a) estudante, iniciamos o quarto bimestre e, nesta primeira semana, você vai reconhecer que
a maioria das reações químicas são reversíveis e compreender o conceito de equilíbrio dinâmico nas
reações químicas e nos processos físicos.

REAÇÕES REVERSÍVEIS E O EQUILÍBRIO QUÍMICO


Muitas reações químicas processam-se apenas enquanto houver reagentes. Assim que todo o reagente
se transformou em produto a reação se encerra. Porém, a maioria das reações químicas pode ser con-
siderada reversível, ou seja, elas ocorrem nos dois sentidos da reação, chamados direto e inverso.

94
Nas reações reversíveis, a velocidade das reações direta e inversa não começa igual. A reação direta
inicia-se com a velocidade máxima, com o passar do tempo os reagentes vão se transformando em pro-
duto. Com a diminuição da concentração de reagentes a velocidade da reação também vai diminuindo.
Por outro lado, no início da reação, a concentração dos produtos é zero, portanto a velocidade da rea-
ção inversa também se inicia no zero, porém com o passar do tempo a velocidade da reação inversa vai
aumentando com o aumento da concentração dos produtos.
Após certo instante, as reações reversíveis atingem um estado no qual a rapidez da reação é igual nos
dois sentidos. Nessa situação, dizemos que a reação alcançou um estado de equilíbrio químico. Obser-
ve a variação da rapidez das reações direta e inversa no gráfico a seguir.

No momento do equilíbrio, a velocidade da reação direta é igual à velocidade da reação inversa:


Vd = Vi

No instante do equilíbrio, a concentração dos reagentes e dos produtos permanece constante, como
podemos observar no gráfico a seguir. Nessa situação, o equilíbrio dinâmico é estabelecido, isso signi-
fica que a reação não chega ao fim, as reações direta e inversa continuam acontecendo, porém com a
mesma velocidade, as concentrações dos reagentes e produtos não alteram mais.

No momento do equilíbrio, a concentração dos reagentes e produtos não se altera mais, ou seja,
permanece constante.

95
EQUILÍBRIO DINÂMICO DE TRANSFORMAÇÕES FÍSICAS
Considere um sistema fechado contendo água no estado líquido, nesse sis-
tema um equilíbrio se estabelece entre a água no estado gasoso e água no
estado líquido. A todo o momento, essas duas transformações estão acon-
tecendo simultaneamente. Porém, como a velocidade é a mesma, aparen-
temente, não é possível verificar uma diminuição no volume do líquido.

ESTALACTITES E ESTALAGMITES
No interior das cavernas, é comum observar estruturas pontiagudas, crescendo do teto e do chão.
As estalactites e as estalagmites se formam com o gotejamento de água saturada em calcita, ao longo
de sua infiltração em rochas calcárias. A estalactite forma-se do teto para baixo, pela superposição de
anéis de calcita. E a estalagmite “cresce” do piso da caverna para cima, bem embaixo da estalactite,
com o gotejamento de água saturada em calcita que se precipita da estalactite. 

Figura 2: Estalactite. Disponível em:<https://www.infoescola.com/geologia/estalactites/>. Acesso em 06 ago 2021.

A formação desses espeleotemas se dá através do estabelecimento de um equilíbrio químico. As águas


subterrâneas contêm dióxido de carbono (CO2) e estão a elevadas pressões, o que facilita a dissolução
de carbonato de cálcio (CaCO3) quando elas passam por terrenos contendo calcário. Com isso, ocorre a
seguinte reação:
CaCO3(s) + CO2(g) + H2O(l) → Ca2+(aq) + 2 HCO-3(aq)
No teto das cavernas, essas águas começam a gotejar bem lentamente e, com o tempo, vão liberando
dióxido de carbono e água por evaporação, ocorrendo a formação do carbonato de cálcio, que vai se
depositando na forma de estalactites no teto e de estalagmites no solo:
Ca2+(aq) + 2 HCO-3(aq) → CaCO3(s) + CO2(g) + H2O(l)
Observe que uma reação é exatamente o inverso da outra, sendo que os reagentes foram regenerados.
Portanto, temos a seguinte reação reversível:
CaCO3(s) + CO2(g) + H2O(l) ↔ Ca2+(aq) + 2 HCO-3(aq)
PARA SABER MAIS:
Para aprofundar sobre a formação de estalactites e estalagmites, assista ao vídeo: “Como se formam
as estalactites e estalagmites” Disponível em: <https://www.youtube.com/watch?v=-DsrLvax5JA>.
Acesso em: 06 ago. 2021.
Leia mais sobre o Equilíbrio Químico e as Reações Reversíveis: Disponível em: <https://guiadoestudan-
te.abril.com.br/curso-enem-play/equilibrio-quimico-reacoes-reversiveis/>. Acesso em: 06 ago. 2021.

96
ATIVIDADES
1 - Uma reação química atinge o equilíbrio químico quando:
a) ocorre simultaneamente nos sentidos direto e inverso.
b) as velocidades das reações direta e inversa são iguais.
c) os reagentes são totalmente consumidos.
d) a temperatura do sistema é igual à do ambiente.
e) a razão entre as concentrações de reagentes e produtos é unitária.

2 - Baseado na charge e em seus conhecimentos sobre Química, responda ao que se pede.

No trecho “Humm, nenhuma mudança”, o cientista concluiu que para uma reação ocorrer é necessário
observar mudança em alguma propriedade característica do sistema (cor, pressão, concentração etc).
O mesmo cientista construiu o gráfico abaixo, estudando as variações nas concentrações de A e B para
a reação hipotética A → B , afirmando que ela somente ocorre no intervalo de 0 a 10 minutos. O cientista
está correto em sua afirmativa? Justifique.

97
3 - Considere a reação hipotética:

Considere, também, o gráfico velocidade x tempo dessa reação:

Com base nessas informações, todas as afirmativas estão corretas, EXCETO:


a) No instante inicial, a velocidade v1 é máxima.
b) No instante inicial, as concentrações de C e D são mínimas.
c) No instante x, as concentrações dos reagentes e produtos são as mesmas.
d) No instante x, a velocidade v2 é máxima.
e) No instante x, as concentrações de A e B são as mesmas que no instante y.

4 - (UFG-GO) Os seguintes gráficos representam variáveis de uma reação química:

Analise os gráficos e responda o que se pede.


a) Em qual instante o equilíbrio químico é atingido?
b) Qual das curvas representa a variação da concentração dos reagentes e qual representa a varia-
ção da concentração dos produtos?
c) Qual das curvas representa a variação da velocidade da reação direta? Qual representa a varia-
ção da velocidade da reação inversa? ___________________________________________________
____________________________

98
SEMANA 2

EIXO TEMÁTICO:
Materiais - aprofundamento.

TEMA/ TÓPICO(S):
Transformações dos Materiais. / Equilíbrio nas Transformações Químicas.

HABILIDADE(S):
17.1.2. Reconhecer o equilíbrio químico nas reações químicas e fazer previsões sobre sua mudança.

CONTEÚDOS RELACIONADOS:
Equilíbrio Químico.

TEMA: Equilíbrio Químico e Constante de Equilíbrio


Caro(a) estudante, na semana anterior reconhecemos que a maioria das reações químicas ocorre no
sentido direto e inverso, chamamos essas reações de reversíveis. Compreendemos que as reações re-
versíveis, depois de certo tempo, atingem o equilíbrio químico, que é um processo dinâmico. Nessa se-
mana vamos aprofundar um pouco nosso conhecimento sobre esse estado de equilíbrio de uma forma
quantitativa.

CONSTANTE DE EQUILÍBRIO
Existe uma relação entre a concentração das espécies reagentes e dos produtos, no momento do equi-
líbrio. Essa relação é conhecida como constante de equilíbrio, ou Kc, esse é o valor das constantes
de equilíbrio, em uma temperatura específica , em função da concentração das espécies em mol.L -1.
Em caso da ocorrência da reação em fase gasosa, a constante é comumente expressa Kp, em função
das pressões parciais das espécies presentes no equilíbrio.
Consideremos o equilíbrio representado pela equação geral:

Supondo que as reações nos dois sentidos sejam elementares, temos:


v1 = k1[A]a[B]b v2 = k2[C]c[D]d
No equilíbrio: v1 = v2. Logo:
k1[A]a[B]b = k2[C]c [D]d

K1/k2 = Kc

99
Na dedução da expressão do Kc que acabamos de fazer, admitimos que as reações nos dois sentidos
fossem elementares. Na maioria dos casos, porém, essas reações ocorrem em várias etapas. Esse fato
pode afetar a interpretação cinética da dedução da expressão do Kc, mas não afeta a expressão do Kc..
As substâncias presentes no equilíbrio podem estar em diferentes estados físicos (líquido, sólido e/ou
gasoso), formando um equilíbrio heterogêneo nesse caso a constante K não considera, as concentra-
ções dos líquidos e sólidos puros, uma vez que, nesse contexto a sua variação de concentração será
desprezível.

CONSTANTE DE EQUILÍBRIO EXPRESSA EM FUNÇÃO DAS PRESSÕES PARCIAIS


Nos equilíbrios gasosos, a constante de equilíbrio pode ser expressa em função das pressões parciais
dos gases participantes. Essa constante de equilíbrio é representada por Kp. Este fato pode ser gene-
ralizado da seguinte maneira:

GRAU DE EQUILÍBRIO
Grau de equilíbrio (α) é a relação entre o número de mols consumidos de um reagente e o número de
mols inicial desse reagente.

O grau de equilíbrio não tem unidade, e seu valor será sempre um número entre zero e um. É muito co-
mum exprimir o grau de equilíbrio em %. É importante não confundir grau de equilíbrio com constante
de equilíbrio.
O grau de equilíbrio varia com a temperatura e com as concentrações das substâncias participantes.
No caso do equilíbrio do qual participam gases, o grau de equilíbrio varia também com a pressão.

PARA SABER MAIS:


Para aprofundar sobre constantes de equilíbrio, assista ao vídeo: “Equilíbrio Químico: Kc e Kp”. Dispo-
nível em: <https://www.youtube.com/watch?v=QKKd1lB-zOg>. Acesso em: 06 ago. 2021.
Para sistematizar as ideias trabalhadas ate aqui, leia mais sobre o Equilíbrio Químico no artigo a seguir:
<https://www.stoodi.com.br/resumos/quimica/equilibrio-quimico/>. Acesso em: 06 ago. 2021.

100
ATIVIDADES
1 - Escreva as expressões de Kc para os equilíbrios apresentados abaixo:
a) N2(g) + 3H2(g) ⇆ 2NH3(g)

b) Fe2O3(s) + 3CO(g) ⇆ 2Fe + 3CO2(g)

c) Zn(s) + 2HCl(aq) ⇆ ZnCl2(aq) + H2(g)

2 - O gráfico a seguir representa a variação da quantidade de matéria em função do tempo dos


participantes em uma reação. De acordo com o gráfico, pede-se determinar:

a) A equação química balanceada correspondente à reação.


b) Calcule o valor de Kc para a reação.

3 - (UFOP) Assinale a afirmativa INCORRETA:


a) Quando uma reação atinge o equilíbrio químico, as velocidades em ambos os sentidos são iguais.
b) a velocidade das reações químicas depende da frequência dos choques entre as espécies rea-
gentes.
c) a velocidade das reações químicas depende da direção dos choques entre as espécies reagentes.
d) a energia de ativação de uma reação química no sentido direto é sempre igual à energia de ati-
vação no sentido inverso.
e) o efeito da temperatura na velocidade de uma reação é aumentar a energia e a frequência dos
choques entre as espécies.

101
4 - (UFLA) Um mol/L do reagente A, é misturado a um mol/L de reagente B produzindo C + D, de acordo
com a reação

Quando o equilíbrio é atingido, as concentrações de C e D foram de 0,8 mol/L. A constante de equilíbrio,


Keq, dessa reação é

a) 0,64. b) 0,80. c) 2,56. d) 4,00. e) 16,00.

102
SEMANA 3

EIXO TEMÁTICO:
Materiais - aprofundamento.

TEMA/ TÓPICO(S):
Transformações dos Materiais. / Equilíbrio Nas Transformações Químicas.

HABILIDADE(S):
17.1.3. Prever o sendo do deslocamento de um equilíbrio químico, aplicando o Princípio de Le Chatelier.
17.1.4. Identificar os fatores que afetam o estado de equilíbrio, a partir de equações que representam siste-
mas em equilíbrio.

CONTEÚDOS RELACIONADOS:
Equilíbrio Químico.

TEMA: Equilíbrio Químico e o Princípio De Le Chatelier


Caro(a) estudante, nas semanas anteriores compreendemos de forma conceitual e quantitativa o es-
tado de equilíbrio químico dinâmico. Nessa semana estudaremos sobre os fatores que afetam esse
estado de equilíbrio e o comportamento do mesmo frente a essas perturbações.

PRINCÍPIO DE LE CHATELIER
Quando um sistema está em equilíbrio, a velocidade da reação direta e da reação inversa é igual e as
concentrações de todos os participantes permanecem constantes. Na ausência de uma ação de um
agente externo, o sistema tende a permanecer nessa situação de equilíbrio indefinidamente. Porém, se
for exercida uma ação externa sobre esse equilíbrio, ele tende a reagir de maneira a minimizar os efeitos
dessa ação. Esse fenômeno foi observado e estudado pelo cientista Henri Louis Le Chatelier e ficou
conhecido como Princípio de Le Chatelier.

Princípio de Le Chatelier: “Quando ocorre perturbação externa a um sistema em equilíbrio, como a


alteração da concentração de uma espécie, há favorecimento de um dos sentidos do equilíbrio de
modo a compensar a modificação imposta”.

FATORES QUE ALTERAM O ESTADO DE EQUILÍBRIO


Os fatores que podem afetar a condição de equilíbrio de um sistema são concentração, pressão e tem-
peratura. A seguir vamos analisar a influência de cada um dos fatores que podem afetar o equilíbrio.
Efeito da concentração
A adição de uma substância (reagente ou produto) em um equilíbrio, de modo geral, favorece o sentido
de consumo dessa substância. Analogamente, a remoção de uma substância em um equilíbrio, de modo
geral, favorece o sentido de formação dessa substância.

103
Efeito da pressão
Uma modificação na pressão de um sistema em equilíbrio só altera seu estado quando há variação dos
coeficientes estequiométricos dos gases nos reagentes em relação aos produtos. De maneira geral.

Aumento da pressão favorece o sentido com menor quantidade de mol de gás (menor volume).
Diminuição da pressão favorece o sentido com maior quantidade em mol de gás. (maior volume).

Exemplo:

No equilíbrio representado acima, um aumento de pressão sobre o sistema provocará um deslocamen-


to do equilíbrio no sentido de formação de NH3 (sentido com menor quantidade de mol de gás). A dimi-
nuição de pressão provocará um deslocamento de equilíbrio no sentido de formação dos reagentes N2
e H2 (sentido da reação que ocorre com expansão de volume, ou seja, maior quantidade em mol de gás).

Efeito da temperatura
O aumento da temperatura de um sistema em equilíbrio favorece o sentido da reação endotérmica,
por outro lado a diminuição da temperatura de um sistema em equilíbrio favorece o sentido da reação
exotérmica.
Exemplo:

Um aumento de temperatura no equilíbrio acima desloca o equilíbrio no sentido da reação de decom-


posição da H2O, ou seja, no sentido inverso. Uma diminuição da temperatura desloca esse equilíbrio no
sentido da reação de formação da H2O, o sentido direto.

Efeito do catalisador
Os catalisadores aceleram igualmente rapidez das reações direta e inversa, de forma que nenhum dos
sentidos do equilíbrio é favorecido. A presença de um catalisador fará que o sistema atinja o equilíbrio
mais rapidamente.
PARA SABER MAIS:
Para aprofundar sobre o princípio de Le Chatelier, assista ao vídeo: “Deslocamento do Equilíbrio” Dis-
ponível em: <https://www.youtube.com/watch?v=7VmHRLIpVXA>. Acesso em: 06 ago. 2021.
Para sistematizar as ideias trabalhadas até aqui, leia essa apresentação sobre o princípio de Le Chate-
lier: Disponível em: <https://view.genial.ly/61072485131c280da27a7aba>. Acesso em: 06 ago. 2021.

104
ATIVIDADES
1 - Preveja o efeito produzido no equilíbrio abaixo por cada uma das seguintes perturbações:
N2(g) + 3H2(g) ⇆ 2NH3(g) ΔH = -22,0 kcal
a) um aumento da pressão.

b) uma diminuição da temperatura.

c) pela introdução de H2 e/ou N2 no sistema.

d) pela retirada de NH3 do sistema.

e) pela introdução de um catalisador adequado.

2 - (UFV) Dados os sistemas em equilíbrio:


(A) N2(g) + 3H2(g) + calor ⇆ 2NH3(g)
(B) N2O(g) ⇆ N2(g) + 1/2O2(g) + calor
Assinale a opção correta:
a) o aumento da pressão desloca o equilíbrio do sistema A no sentido de formação dos reagentes.
b) o aumento da temperatura desloca o equilíbrio do sistema A no sentido da formação do produto.
c) a diminuição da pressão desloca o equilíbrio do sistema B no sentido da formação do reagente.
d) a diminuição da temperatura desloca o equilíbrio do sistema B no sentido da formação do reagente.
e) o aumento da temperatura e da pressão apresenta efeitos contrários, tanto no sistema A quanto
no sistema B.

3 - Quando aquecido em um recipiente fechado, o I2(s) sublima e forma I2(g). Estabelece-se, então, o
equilíbrio representado nessa equação:
I2(s) ⇆ I2(g)
Sobre esse equilíbrio responda o que se pede:
a) Assinalando com um X a quadrícula correspondente, INDIQUE se a substância é endotérmica ou
exotérmica.
[  ] ENDOTÉRMICA. [  ] EXOTÉRMICA.
b) Assinalando com um X a quadrícula correspondente, INDIQUE se, com o aumento da temperatura,
a constante de equilíbrio da sublimação do I2 diminui, permanece constante ou aumenta.
[  ] DIMINUI. [  ] PERMANECE CONSTANTE. [  ] AUMENTA.
c) Assinalando com um X a quadrícula correspondente, INDIQUE o efeito do aumento da tempera-
tura nesse equilíbrio.
[  ] DESLOCA PARA A DIREITA. [  ] DESLOCA PARA A ESQUERDA. [  ] NÃO É PERTURBADO.

105
4 - O ozônio é formado quando o oxigênio é submetido a alguns tipos de radiação ultravioleta por meio
da seguinte reação endotérmica:
3 O2(g) ⇆ 2 O3(g)
Considerando o Princípio de Le Chatelier, diga qual(is) da(s) situação(ões) abaixo favorecem a formação
do ozônio:
a) Aumento da temperatura.
b) Diminuição da temperatura.
c) Aumento da pressão.
d) Diminuição da pressão.
e) Aumento da concentração de gás ozônio. 

106
SEMANA 4

EIXO TEMÁTICO:
Materiais - aprofundamento..

TEMA/ TÓPICO(S):
Transformações dos Materiais. / Equilíbrio Nas Transformações Químicas.

HABILIDADE(S):
20.1. Compreender que as soluções apresentam comportamento ácido, básico ou neutro.
20.1.2. Representar ou identificar, por meio de equações ou fórmulas químicas, sistemas que apresentem
caráter ácido, básico ou neutro.

CONTEÚDOS RELACIONADOS:
Funções Inorgânicas; Ácidos e Bases.

TEMA: Acidez e Basicidade


Caro(a) estudante, esta semana focaremos nossa atenção nos ácidos e bases, em como o comporta-
mento dessas substâncias pode ser representado por meio de equações ou fórmulas químicas e como
identificar se uma substância tem caráter ácido ou básico.

ÁCIDOS E BASES
De acordo com a Teoria de Arrhenius, quando as substâncias classificadas como ácidas são adiciona-
das à água, ocorre a ionização dessa substância, ou seja, a formação do cátion H+ e um ânion que varia
de acordo com o ácido em questão.
Exemplos:
HCl H+ + Cl -
H2SO4 2 H+ + SO4 - 2
H3PO4 3 H+ + PO4 - 3
De acordo com a mesma teoria, as substâncias classificadas como bases, ou hidróxidos, quando adi-
cionadas à água, dissociam-se, ou seja, ocorre a separação do seu cátion e o ânion hidroxila OH-.
Exemplos:
KOH K+ + OH-
Mg(OH)2 Mg2+ + OH-

107
INDICADORES ÁCIDO - BASE
Certas substâncias adquirem coloração diferente em solução ácida e em solução básica. São os indicado-
res ácido-base, utilizados para reconhecer o caráter de uma solução. Os exemplos mais importantes são:

Indicador Ácido Base


Tornassol Azul torna-se róseo Róseo torna-se azul
Fenolftaleína Incolor Vermelho
Alaranjado de Metila Vermelho Amarelo

REAÇÃO DE NEUTRALIZAÇÃO
Ácidos e bases reagem para formar sais, essa reação é denominada reação de neutralização. Existem
dois tipos de reações de neutralização, a total e a parcial. A neutralização total ocorre quando todos os
átomos de hidrogênio ionizáveis provenientes do ácido são neutralizados por todos os ânions (hidroxi-
la) provenientes da base. Nesse caso, a solução resultante será neutra. O outro tipo é a neutralização
parcial, quando nem todos os hidrogênios ionizáveis do ácido ou nem todos os ânions hidróxido da base
são neutralizados.

PARA SABER MAIS:


Para aprofundar sobre o princípio de Le Chatelier, assista ao vídeo: “Funções inorgânicas: ácidos, bases
e sais” Disponível em: <https://www.youtube.com/watch?v=0o9Qz7VLmPs>. Acesso em: 06 ago. 2021.
Para sistematizar as ideias trabalhadas ate aqui, leia mais sobre os ácidos e as bases: Disponível em:
<https://www.educamaisbrasil.com.br/enem/quimica/acidos-e-bases>. Acesso em: 06 ago. 2021.

ATIVIDADES
1 - Complete as equações de ionização e dissociação de ácidos e bases:
a) NH4OH

b) Ba(OH)2

c) H2SO4

d) HF

108
2 - Considere os seguintes materiais:
I) solução de soda cáustica.
II) produtos de limpeza do tipo Ajax, fúria, etc.
III) vinagre.
IV) água de bateria de automóvel.
V) leite de magnésia.
Quais deles tornam azul o papel róseo de tornassol?
a) todos.
b) nenhum.
c) somente I, II e V.
d) somente III e IV.
e) somente I e II.

3 - Grande quantidade dos maus odores do nosso dia a dia está relacionada a compostos alcalinos. Assim,
em vários desses casos, pode-se utilizar o vinagre, que contém entre 3,5% e 5% de ácido acético, para
diminuir ou eliminar o mau cheiro. Por exemplo, lavar as mãos com vinagre e depois enxaguá-las com
água elimina o odor de peixe, já que a molécula de piridina (C5H5N) é uma das substâncias responsáveis
pelo odor característico de peixe podre.
SILVA, V. A.;BENITE, A.M.C.; SOARES , M. H.F.B. Algo aqui não cheira bem... A química do mau cheiro. Química Nova na Escola , v. 33, n.1,
fev. 2011 (adaptado).

A eficiência do uso do vinagre nesse caso se explica pela


a) sobreposição de odor, propiciada pelo cheiro característico do vinagre.
b) solubilidade da piridina, de caráter ácido, na solução ácida empregada.
c) inibição da proliferação das bactérias presentes, devido à ação do ácido acético.
d) degradação enzimática da molécula de piridina, acelerada pela presença de ácido acético.
e) reação de neutralização entre o ácido acético e a piridina, que resulta em compostos sem mau odor.

4 - Num determinado experimento, três soluções aquosas, límpidas e transparentes estavam


armazenadas em três recipientes distintos: A, B, C. Para caracterizar estas substâncias, um cientista
utilizou apenas o indicador fenolftaleína e observou que nos recipientes A e C não houve nenhuma
alteração, porém no recipiente B observou o aparecimento de coloração avermelhada. Qual(quais)
afirmação(ções) é possível fazer sobre o caráter ácido ou básico das substâncias testadas.

109
SEMANA 5

EIXO TEMÁTICO:
Materiais - aprofundamento.

TEMA/ TÓPICO(S):
Transformações dos materiais / Equilíbrio nas transformações químicas.

HABILIDADE(S):
17.2.1. Identificar ácidos e bases fortes de ácidos e bases fracos, com base em constantes de equilíbrio.
17.2.2. Escrever a equação de dissociação de ácidos e bases e a correspondente expressão da constante de
equilíbrio.

CONTEÚDOS RELACIONADOS:
Equilíbrio Iônico.

TEMA: Equilíbrio químico em sistemas aquosos


Caro(a) estudante, na semana anterior, relembramos alguns conceitos essenciais sobre ácidos e bases
que serão importantes para dar continuidade ao nosso estudo sobre o equilíbrio químico. Nessa sema-
na, você irá compreender que a dissociação e a ionização de bases e ácidos são sistemas reversíveis e,
portanto, atingem o equilíbrio dinâmico. Esse tipo de equilíbrio é especial, devido a presença de subs-
tâncias iônicas.

CONSTANTE DE IONIZAÇÃO DE ÁCIDOS


As reações de ionização constituem equilíbrios aos quais estão associados às respectivas constantes
de ionização. Considerando a reação de ionização genérica abaixo:
HX ⇆ H+ + X-
Sendo HX um ácido que, na presença de água, se ioniza formando H+ e X-, sua constante de ionização,
denominada Ka (constante de acidez), pode ser escrita como:

Ka =
Exemplo:
HCl ⇆ H+ + Cl- Ki =
Os poliácidos (HnA) ionizam-se em etapas, e cada etapa constitui um equilíbrio, ao qual está associado
uma constante de ionização e um grau de ionização (α). Observe o exemplo seguinte:

H2S ⇆ H+ + HS- Etapa(1)

K1 =

HS- ⇆ H+ + S-2 Etapa(2)

K2 =

110
CONSTANTE DE DISSOCIAÇÃO DE BASES
Assim como os ácidos, as bases também têm suas constantes de dissociação (Kb) e seus graus de dis-
sociação. Considerando a reação de dissociação genérica abaixo:
XOH ⇆ X+ + OH-
Sendo XOH uma base que, na presença de água, se dissocia formando X+ e OH-. Sua constante de disso-
ciação, denominada Kb (constante de basicidade), pode ser escrita como:
𝑋𝑋 ! . 𝑂𝑂𝑂𝑂"
Kb = 𝑋𝑋𝑋𝑋𝑋𝑋

FORÇA DO ÁCIDO E DA BASE


É possível determinar a força de um ácido e uma base pelo valor de sua constante de ionização. Assim,
como regra geral, podemos dizer que:

Quanto maior o Ka → mais forte é o ácido.


Quanto maior o Kb → mais forte é a base.

PARA SABER MAIS:


Para aprofundar sobre a constante de acidez e de basicidade, assista ao vídeo: “Equilíbrio iônico – Ka
e Kb”. Disponível em: <https://www.youtube.com/watch?v=44qeVYpeY6g>. Acesso em: 06 ago. 2021.
Para sistematizar as ideias trabalhadas ate aqui, leia esse artigo sobre o equilíbrio iônico de ácidos e
bases. Disponível em: <https://pt.khanacademy.org/science/chemistry/acids-and-bases-topic/copy-
-of-acid-base-equilibria/a/weak-acid-base-equilibria>. Acesso em: 06 ago. 2021.

ATIVIDADES
1 - Escreva a expressão de Ka para os ácidos e a expressão de Kb para as bases relacionadas a seguir.
a) NH4OH

b) Ba(OH)2

c) H2SO4

d) HF

111
2 - (Enem 2016) Após seu desgaste completo, os pneus podem ser queimados para a geração de energia.
Dentre os gases gerados na combustão completa da borracha vulcanizada, alguns são poluentes e
provocam a chuva ácida. Para evitar que escapem para a atmosfera, esses gases podem ser borbulhados
em uma solução aquosa contendo uma substância adequada.

Dentre as substâncias listadas no quadro, aquela capaz de remover com maior eficiência os gases po-
luentes é o(a)
a) fenol.
b) piridina.
c) metilamina.
d) hidrogenofosfato de potássio.
e) hidrogenosulfato de potássio.

3 - (UFOP) Para obtenção do valor da constante de equilíbrio para o sistema, numa dada temperatura,
CH3COOH(aq) ⇆ CH3COO-(aq) + H+(aq)
Forneceram-se os seguintes dados:
I) Concentração molar inicial do ácido acético.
II) Concentração molar do ânion acetato.
III) Concentração molar do íon H+.
IV) Temperatura do sistema.
O levantamento do valor da constante de equilíbrio requer apenas os dados fornecidos por:
a) I e IV.
b) I e III.
c) II e IV.
d) I, II, III e IV.
e) I, III e IV.

4 - Calcule a concentração de íons H+ em uma solução aquosa 0,40 mol/L de H2S.


Dado: Constantes de ionização do H2S:
K1 = 1,0 . 10-7
K2 = 1,3 . 10-13.

112
SEMANA 6

EIXO TEMÁTICO:
Materiais - aprofundamento.

TEMA/ TÓPICO(S):
Transformações dos materiais. / Equilíbrio Nas Transformações Químicas.

HABILIDADE(S):
22.1. Conceituar pH e pOH.
22.1.1. Compreender os procedimentos utilizados para calcular valores de pH e pOH, partindo de concentra-
ções de H+ (H3O+) e OH, e vice-versa.
22.1.2. Identificar o caráter ácido ou básico de uma solução a partir de valores de pH.
22.1.3. Utilizar fórmulas para determinação de pH e pOH a partir da concentração de suas soluções.

CONTEÚDOS RELACIONADOS:
Equilíbrio de Ionização da água: pH e pOH.

TEMA: Equilíbrio de Ionização da água: pH e pOH


Querido(a) estudante, chegamos à última semana deste plano de estudos, ao final desse bimestre espe-
ramos que vocês tenham uma compreensão ampla sobre o equilíbrio químico, constantes de equilíbrio
e fatores que influenciam esse estado. Nessa semana vamos estudar outro tipo especial de equilíbrio:
a ionização da água. Com isso será possível compreender o conceito de pH e pOH e identificar o caráter
ácido ou básico de uma solução à partir desse parâmetro.
EQUILÍBRIO IÔNICO DA ÁGUA
A água apresenta um caráter anfótero, comportando-se ora como ácido, ora como base. A transferên-
cia de prótons entre moléculas de água é chamada autoionização da água, veja a representação esque-
mática e a equação desse processo abaixo.

Fonte: <https://mundoeducacao.uol.com.br/quimica/equilibrio-ionico-agua.htm>. Acesso em: 06 ago. 2021.

A constante de dissociação da água (Kw), também conhecido como produto iônico da água, pode ser
expressa por:
Kw = [H+] . [OH-] = 10-14 (a 25ºC)
Em meio Neutro:
[H+] = [OH-] = 10-7 (a 25ºC)
Em meio ácido
[H+] > 10-7 e [OH-] < 10-7 ( a 25ºC)
Em meio básico
[H+] < 10-7 e [OH-] >10-7 ( a 25ºC)

113
DEFINIÇÕES DE pH e pOH
Devido à dificuldade de trabalhar com potência de dez, optou-se por uma escala mais simples, que tem
como base o uso do logaritmo. Essa escala foi chamada de escala de pH.
O pH de uma solução aquosa pode ser calculado da seguinte forma:

pH = -log
De maneira análoga, foi definido o pOH de uma solução aquosa:

pOH = - log

RELAÇÃO ENTRE pH E pOH


É importante notar que esses dois conceitos indicam que em qualquer solução coexistem H+ e OH–. Por
mais ácida que seja a solução, sempre existirão íons OH–. Nas soluções básicas também estarão pre-
sentes os íons H+. As concentrações desses íons jamais se anulam. Experimentalmente foi possível de-
finir que, a 25 °C, a soma dos valores de pH e pOH para uma mesma solução aquosa é sempre igual a 14
pH + pOH = 14 (a 25 °C)
Fornecendo-se os valores de pH ou pOH podemos calcular as respectivas concentrações de H+ e OH-
pelas expressões:
[H+] = 10–pH mol/L
[OH-] = 10–pOH mol/L

ESCALA DE pH
A escala de pH apresenta normalmente valores que variam de zero a 14. O esquema a seguir mostra uma
relação entre os valores de pH e as concentrações dos íons H+ e OH- a 25 °C.

Fonte: https://wikiciencias.casadasciencias.org/wiki/index.php/pH>. Acesso em: 06 ago. 2021.

Com relação a escala de pH é importante observar que:


• Quanto menor o valor do pH, maior a concentração dos íons H+ e maior é a acidez da solução.
• Quanto maior a concentração dos íons H+ numa solução, menor é a concentração dos íons OH-
e vice-versa.
• A variação de uma unidade na escala de pH corresponde a uma variação de 10 unidades nas con-
centrações de H+ e OH–. Por exemplo, admitamos que um suco de limão tem um pH = 2 e que um
suco de tomate tenha pH = 4. Podemos concluir que o suco de limão é mais ácido, mas não deve-
mos pensar que ele é apenas 2 vezes mais ácido e, sim que sua concentração de H+ é 100 vezes
maior que a do suco de tomate. Logo, o suco de limão é 100 vezes mais ácido que o suco de tomate.

114
PARA SABER MAIS:
Para aprofundar sobre a autoionização da água, assista ao vídeo: “Ionização da água e pH” Disponível
em: <https://www.youtube.com/watch?v=GCXU1S3QXc8>. Acesso em: 06 ago. 2021.
Para sistematizar as ideias trabalhadas até aqui, leia mais sobre o pH: Disponível em: <https://vaiqui-
mica.com.br/o-que-e-ph/>. Acesso em: 06 ago. 2021.

ATIVIDADES
1 - Dados os pH dos materiais seguintes:

Sistema pH a) Quais desses materiais são ácidos?


Vinagre 3,0 b) Quais desses materiais são alcalinos (básicos)?
c) Qual o material mais próximo da neutralidade?
Suco de laranja 4,0
d) Qual dos materiais apresenta a maior concentração hidroge-
Suco de tomate 5,0
niônica [H+]?
Saliva 6,0
Leite 6,8
Sangue 7,4
Clara de ovo 8,0

2 - Um suco de tomate tem pH = 4. Isto significa que:


a) o suco apresenta propriedades alcalinas.
b) a concentração de íons H+ presentes no suco é 104 mol/L.
c) a concentração de íons H+ presentes no suco é 10– 4 mol/L.
d) a concentração de íons OH- presentes no suco é 104 mol/L.
e) a concentração de íons OH- presentes no suco é 10–4 mol/L.

3 - Complete as informações sobre pH, pOH, [H+] e [OH-]


a) [H+] = 0,001 mol/L
pH = _______________________
pOH = _____________________
[OH-] = _____________________
b) pH = 12
[H+] = ____________________
[OH-] = ___________________
pOH = ____________________
c) pH = 1
[H+] = ____________________
[OH-] = _____________________
pOH = _____________________

115
4 - A agricultura de frutas cítricas requer que o valor do pH do solo esteja na faixa ideal entre 5,8 e 6,0.
Em uma fazenda, o valor do pH do solo é 4,6. O agricultor resolveu testar três produtos de correção de
pH em diferentes áreas da fazenda. O primeiro produto possui íons sulfato e amônio, o segundo produto
possui íons carbonato e cálcio e o terceiro produto possui íons sulfato e sódio.
O íon que vai produzir o efeito desejado de correção no valor do pH é o
a) cálcio, porque sua hidrólise produz H+, que aumenta a acidez.
b) amônio, porque sua hidrólise produz H+, que aumenta a acidez.
c) sódio, porque sua hidrólise produz OH−, que aumenta a alcalinidade.
d) sulfato, porque sua hidrólise produz OH−, que aumenta a alcalinidade.
e) carbonato, porque sua hidrólise produz OH−, que aumenta a alcalinidade.

Chegamos ao fim de mais um volume do PET, parabéns pela sua caminhada até aqui, sabemos que
não foi fácil, toda grande jornada tem suas adversidades, mas esperamos que ao final dela você tenha
aprendido bastante e se preparado para novos desafios. Esse ano chega ao fim, mas com muito foco
e força de vontade, no próximo ano, maiores e melhores desafios virão e você continuará vencendo e
evoluindo a cada etapa.

REFERÊNCIAS:
FOGAÇA, Jennifer Rocha Vargas. Reações reversíveis. s.d. Disponível em: https://mundoeducacao.
uol.com.br/quimica/reacoes-reversiveis.htm. Acesso em 06 ago. 2021.
INFOESCOLA. Constante de equilíbrio. s.d. Disponível em: https://www.infoescola.com/quimica/
constante-de-equilibrio/. Acesso em 06 ago. 2021.
LISBOA. J. C. F; et al. Ser Protagonista: Química. v. 3, 3ª ed. editora SM, São Paulo, 2016
MINAS GERAIS, Secretaria do Estado de Educação. Conteúdo Básico Comum: CBC Química. Belo
Horizonte: SEE, 2007.
PERUZZO. F. M; CANTO. E. L. Química na abordagem do cotidiano. v 3, 3 ed. São Paulo: Moderna, 2003.
REIS. M, Química: Manual do Professor, v.3. São Paulo: Ática, 2013.

116
SECRETARIA DE ESTADO DE EDUCAÇÃO DE MINAS GERAIS

PLANO DE ESTUDO TUTORADO


COMPONENTE CURRICULAR: FÍSICA
ANO DE ESCOLARIDADE: 2º ANO – EM
PET VOLUME: 04/2021
NOME DA ESCOLA:
ESTUDANTE:
TURMA: TURNO:
BIMESTRE: 4º TOTAL DE SEMANAS:
NÚMERO DE AULAS POR SEMANA: NÚMERO DE AULAS POR MÊS:

SEMANA 1

EIXO TEMÁTICO:
II. Transferência, Transformação e Conservação da Energia.

TEMA/TÓPICO:
5. Calor e Movimento. / 14. Máquinas Térmicas.

HABILIDADE(S):
14.1.5. Compreender que o funcionamento de máquinas térmicas requer sempre troca de calor entre duas
fontes, uma quente e outra fria.
14.1.6. Compreender que, numa máquina térmica, só uma parte do calor fornecido é transformado em traba-
lho. BNCC: EM13CNT102, EM13CNT301, EM13CNT307.

CONTEÚDOS RELACIONADOS:
Trabalho, Calor, Conversão de energia em um sistema.

INTERDISCIPLINARIDADE:
História, Sociologia, Matemática.

TEMA: Calor e Movimento I


Querido(a) estudante, iniciamos o quarto bimestre e, nesta primeira semana, você vai entender como
o calor -energia térmica - pode ser transformado em trabalho mecânico, utilizado, por exemplo, para
movimentar turbinas em usinas termelétricas.
O CALOR COMO ENERGIA
O calor passou a ser entendido como forma de energia desde que, em 1798, Benjamim Thompson- Con-
de de Rumford- ao estudar a fabricação de canhões, percebeu que a perfuração das peças causava
aumento de temperatura no material perfurado e que esse aumento de temperatura só poderia vir da
energia mecânica das brocas. Mais tarde, por volta de 1840, Julius Robert Mayer e James Prescott Joule
estabeleceram a equivalência entre o calor e a energia mecânica. As experiências realizadas por Joule,
naquela época, acabaram estabelecendo, definitivamente, que o calor é uma forma de energia.

117
A MÁQUINA DE HERON
Ainda que somente no séc. XIX o calor tenha sido concebido definitivamente
como forma de energia, desde a Antiguidade ele já era utilizado para produzir o
vapor que seria usado na realização de trabalho mecânico.
No séc. I d. C, Heron de Alexandria construiu um dispositivo, como o represen-
tado na figura 1, que utilizava o vapor produzido no aquecimento da água que, ao
escapar pelos orifícios da esfera de metal, colocava essa esfera em rotação.
Esse dispositivo é constituído por uma esfera oca com tubos curvados por onde
o vapor é expelido, uma bacia com água e uma fonte de calor.
Figura 1:Modelo da primeira máquina térmica, projetada por Heron, no séc 1 d.C. Disponível em:
https://commons.wikimedia.org/wiki/File:Aeolipile_illustration.png. Acesso em: 20 jul. 2021.

FUNCIONAMENTO
A água é aquecida na bacia que está ligada à esfera giratória por um par de tubos que também servem
como eixo para a esfera. O vapor que sai por ambos os furos tem a mesma pressão, logo, o que a faz
mover é justamente a pressão do vapor que sai dos tubos.
Atualmente, sabemos que esse aparelho projetado por Heron é um modelo de máquina térmica, pois
esse dispositivo transforma calor em trabalho mecânico.

MÁQUINAS TÉRMICAS - EVOLUÇÃO, USOS E SOCIEDADE


A partir do séc XVIII, inicia-se a utilização das primeiras máquinas térmicas em escala industrial, mas
essas, ainda apresentavam rendimentos muito baixos , pois consumiam grande quantidade de combus-
tível para produzir pouco trabalho.
Por volta de 1770, o inventor escocês James Watt (1736-18190) apresentou um modelo mais eficiente de
máquina térmica que utilizava vapor proveniente de uma caldeira onde a água era aquecida. Esse mo-
delo apresentava vantagens e, por isso, substituiu as então existentes. O modelo de máquina de Watt foi
inicialmente empregado para movimentar moinhos e, posteriormente, em locomotivas e barcos a vapor.
Na Primeira Revolução Industrial que iniciou na Inglaterra no séc XVIII, a utilização da máquina a va-
por permitiu a automatização do modo de produção industrial que passou do método artesanal para o
método de produção por máquinas. Isso alterou completamente o modo de vida das pessoas. Foi uma
verdadeira transformação social, pois essas máquinas ocuparam o lugar da força de trabalho majorita-
riamente feminino e infantil nas fábricas têxteis inglesas .
Os modelos de máquinas térmicas a vapor são pouco utilizados atualmente. Mas, a energia térmica do
vapor continua sendo empregada nas usinas termelétricas para movimentar turbinas. Além disso, no
decorrer do séc. XX, outros tipos de máquinas térmicas foram desenvolvidos, como o motor a explosão,
que utiliza a gasolina como combustível e é parte fundamental para o funcionamento de automóveis.

RENDIMENTO DE UMA MÁQUINA TÉRMICA


Analisando o funcionamento das máquinas térmicas, verificamos que todas operam em ciclos. Isto é,
retornam periodicamente a condição inicial. A cada ciclo, a máquina retira calor de uma fonte, denomi-
nada fonte quente, utiliza parte desse calor para realizar um trabalho e rejeita uma quantidade de calor
para uma fonte fria.

118
A figura a seguir mostra o esquema de uma máquina térmica:

Figura 2: Representação esquemática de uma máquina térmica. Fonte: <https://www.infoescola.com/wp-content/uploads/2009/08/


maquina_termica1.jpg>. Acesso em: 20 jul. 2021.

Uma fonte quente na parte superior,(Q1); no centro, a máquina térmica; a parte inferior representa a
fonte fria (Q2). As setas indicam o sentido do fluxo de calor. A máquina térmica ideal seria aquela que
conseguisse converter totalmente o calor retirado da fonte quente em trabalho realizado. Uma máquina
assim teria 100% de rendimento e eficiência. No entanto, cientistas perceberam que é impossível cons-
truir uma máquina que apresente um rendimento de 100%.

CÁLCULO DO RENDIMENTO DE UMA MÁQUINA TÉRMICA


Denomina-se rendimento R de uma máquina térmica a relação entre o trabalho τ realizado em cada
ciclo e o calor Q1, absorvido da fonte quente, em cada ciclo. Isto é:

Logo, para determinada quantidade de calor absorvido, quanto maior for o trabalho realizado, maior
será o rendimento (a eficiência) dessa máquina térmica!

PARA SABER MAIS:


Para aprofundar no funcionamento de uma máquina térmica, sugestão de vídeos:
“Máquinas Térmicas, funcionamento e usos, parte I”. Disponível em : https://youtu.be/rxTov7BDlOk .
Acesso em: 22 jul. 2021.
“Como funciona a locomotiva”. Disponível em: https://youtu.be/9CaQfq05GLo . Acesso em: 22 jul. 2021.
Para aprofundar mais, sugestão de leitura: SILVA, J; PINTO, A. C; LEITE.C. Calor: o motor das revolu-
ções. São Paulo: Ed. do Brasil, 2000.

119
ATIVIDADES
Agora é a sua vez de exercitar. Releia o texto, procure outras fontes, mostre o que você aprendeu
essa semana.

1 - O rendimento de uma máquina térmica é medido em porcentagem e é definido pelo quociente entre
o trabalho realizado e a quantidade de calor absorvido da fonte quente. Assim, uma máquina que retira
600 cal da fonte quente e realiza um trabalho de 1600 J tem um rendimento de: Lembre-se: 1 cal equivale
a 4,2 J)

a) 82 %. b) 64 %. c) 60 %. d) 15 %. e) 45 %.

As questões 2 e 3 referem-se à figura a seguir:


A figura abaixo representa o esquema de uma usina termoelétrica:

Fonte imagem: ufmg-2019-ufmg-processo-seletivo-ebap-2019-prova.pdf/pag.36

2 - Identifique, a partir desta imagem, o que corresponde a cada uma dessas funções relacionadas
abaixo:
a) A fonte quente:___________________________________________________
b) A fonte fria:______________________________________________________
c) O trabalho realizado:_______________________________________________

3 - Analise o esquema da usina termelétrica apresentado e indique as transformações de energia que


ocorrem a cada ciclo do funcionamento da usina:

120
4 - Pesquise os motores elétricos de automóveis e compare-os em relação à eficiência e à autonomia,
com os motores a combustão interna. Quais são as vantagens e desvantagens desses tipos de motores?
(Retirada do livro: Matéria, Energia e Vida. Uma abordagem interdisciplinar - Materiais e energia: Transformações e Conservação
- Ed. Scipione, São Paulo 2020 - página 126)

121
SEMANA 2
EIXO TEMÁTICO:
II. Transferência, Transformação e Conservação da Energia.

TEMA/TÓPICO:
5. Calor e Movimento. / 14. Máquinas Térmicas.

HABILIDADE(S):
14.1. Aplicar o conceito de energia e suas propriedades para compreender situações envolvendo máquinas
térmicas.
14.1.3 Compreender que o sistema cilindro-gás poderá representar uma máquina térmica se o pistão voltar à
sua posição inicial para realizar a expansão novamente, em ciclos sucessivos.
BNCC: EM13CNT102, EM13NT301, EM13CNT302.

CONTEÚDOS RELACIONADOS:
Ciclo de Carnot e 2ª Lei da Termodinâmica.

INTERDISCIPLINARIDADE:
Matemática.

TEMA: Calor e Movimento II


Querido(a) estudante, na semana 2 você compreenderá mais sobre o rendimento de uma máquina tér-
mica e conhecerá a máquina de Carnot. a 1. Conhecerá a Segunda Lei da Termodinâmica aplicada às
máquinas térmicas. Leia os textos explicativos, pesquise as sugestões de leitura e vídeo de acordo com
a seção “Para saber mais” e resolva as atividades propostas. Tudo isso enriquecerá ainda mais seus es-
tudos, o que te proporcionará mais conhecimentos.

MÁQUINAS TÉRMICAS E A CONSERVAÇÃO DA ENERGIA


O Princípio da Conservação da Energia aplica-se ao funcionamento das máquinas térmicas e, devido a
esse Princípio, teremos que Q1 = τ + Q2. Ou τ = Q1 - Q2.
Então, podemos expressar o rendimento de uma máquina térmica da seguinte maneira:

LEMBRETE: τ = Trabalho Q1 = Calor absorvido da fonte quente Q2 = Calor cedido à fonte fria

COMO SE FAZ?
Vejamos o exemplo a seguir, em que calcularemos o rendimento de uma máquina térmica:
Um motor a diesel funciona realizando, em cada ciclo, um trabalho de 1000 J. A quantidade de calor,
em calorias, que o motor recebe da fonte quente equivale a 625 cal e a quantidade de calor rejeitada
equivale 375 cal, a cada ciclo.

122
Determine o rendimento desse motor:
RESOLUÇÃO:
R= 1 - 375 cal = 1 - 0,60 logo: R = 0,40 Portanto, o rendimento desse motor é de 40%
   625 cal

A SEGUNDA LEI DA TERMODINÂMICA


O processo pelo qual uma máquina térmica funciona constitui um processo cíclico. Isto é, inicia-se
um ciclo e, após passar por transformações, retorna-se ao seu estado inicial. Vimos que, a cada ciclo,
a máquina retira calor da fonte quente, transforma uma parte dele em trabalho e rejeita o restante desse
calor para uma fonte fria. Assim, uma máquina térmica teria rendimento de 100% se ela transformasse
todo o calor retirado da fonte quente em trabalho. Entretanto, existe na natureza uma Lei que não pode
ser violada. A essa lei, os cientistas deram o nome de Segunda Lei da Termodinâmica e ela nos mostra
que qualquer dispositivo existente na natureza, ao efetuar um ciclo, não conseguirá transformar total-
mente em trabalho o calor retirado da fonte quente. Para completar o ciclo, sempre será necessário
rejeitar calor para a fonte fria.
Assim, de acordo com a Segunda Lei da Termodinâmica, conclui-se que é impossível construir uma
máquina térmica que apresente 100% de rendimento. Na realidade, os rendimentos das máquinas tér-
micas mais utilizadas até a atualidade estão muito abaixo desse limite. As locomotivas a vapor, por
exemplo, conhecidas como “Maria Fumaça”, tem rendimento em torno de 10% somente. Já os motores
a diesel nunca ultrapassam os 40%.

MÁQUINA DE CARNOT
Muitos cientistas empenharam-se no estudo do desenvolvimento de máquinas térmicas, com o obje-
tivo de aprimorarem um modelo realmente eficiente. Dentre eles, destacou-se o engenheiro francês
Sadi Carnot, que foi quem descreveu e apresentou um modelo de máquina que operava com o máximo
rendimento possível. Esse modelo ficou conhecido como Máquina de Carnot .
A Figura 3, apresentada a seguir, mostra o Ciclo de Carnot para
um sistema cilindro- gás ideal. Esse ciclo corresponde ao ren-
dimento máximo que uma máquina pode obter operando entre
duas fontes térmicas. Ele consiste em duas transformações iso-
térmicas alternadas com duas transformações adiabáticas.
Na transformação isotérmica AB, o gás absorve calor da fonte
quente enquanto se expande. Em BC, o sistema não troca calor
com a vizinhança e a temperatura cai. Essa transformação é uma
expansão adiabática. Em CD, o gás sofre uma compressão iso-
térmica e cede calor para a fonte fria. E, finalmente, em DA, o gás
sofre uma compressão adiabática e sua temperatura aumenta.
Assim, completa-se o ciclo.
FIG. 3: Ciclo de Carnot. Fonte: https://upload.wikimedia.org/wikipedia/commons/thumb/4/4e/Carnot-cycle-p-V-diagram.svg/206px-
Carnot-cycle-p-V-diagram.svg.png. Acesso em: 22 jul. 2021.

123
Carnot mostrou que o rendimento máximo de uma máquina térmica é sempre menor que 1 e relaciona-se
com as temperaturas Tq ( da fonte quente) e Tf ( da fonte fria), de acordo com a seguinte expressão:

η = 1 - Tf Tf = temperatura da fonte fria ( medida em Kelvin).


   Tq Tq = temperatura da fonte quente (medida em Kelvin).
η = rendimento) da máquina térmica.
Nessa expressão, para o cálculo do rendimento de uma máquina de Carnot, as temperaturas devem ser
expressas na unidade Kelvin.

PARA SABER MAIS:


Sugestão de leituras: “Ciclos Termodinâmicos “. Mayara Cardoso. Disponível em: www.infoescola.com/
fisica/ciclos-termodinamicos/ . Acesso em: 22 jul. 2021.
“Segunda Lei da Termodinâmica”. Carla Evangelista. Disponível em: www.infoescola.com/fisica/se-
gunda-lei-da-termodinamica/-. Acesso em: 22 jul. 2021.
O Ciclo de Carnot. Thomas Carvalho. Disponível em: www.infoescola.com/fisica/ciclo-de-carnot/.
Acesso em 22 jul. 2021.

ATIVIDADES
Agora chegou a sua vez de aplicar os conhecimentos adquiridos nesta semana. Procure sempre pes-
quisar sobre os assuntos apresentados, busque a orientação do seu professor e tire suas dúvidas.
A resolução das atividades te ajudará a fixar todo o conteúdo estudado. Então, vamos lá!

1 - Uma máquina de Carnot opera entre uma fonte quente, cuja temperatura é 400 K e uma fonte fria a
280 k. Determine o rendimento dessa máquina:

a) 60%. b) 32%. c) 30%. d) 40%. e) 44%.

2 – Um motor térmico trabalha nas temperaturas de 2000 K e 700K apresentando um rendimento de 40%.
Esse rendimento que o motor apresenta é o máximo que ele pode alcançar? Justifique sua resposta:

3 - Uma máquina a vapor, em cada ciclo, retira 100 cal da fonte quente e realiza um trabalho de 90J. Com
base nessas informações, calcule: (1 cal = 4,18J)
a) O rendimento dessa máquina:

b) A quantidade de calor, em calorias, que ela rejeita para a fonte fria:

124
4 - Em uma feira científica, um participante afirmou ter construído uma máquina térmica que, a cada ciclo,
retirava a quantidade de 60 cal de calor da fonte quente e realizava um trabalho de 250 J. Com base no
que você aprendeu sobre a Segunda Lei da Termodinâmica, seria possível a máquina desse participante?
Justifique a sua resposta:

5 - (ENEM - 2014) As máquinas térmicas foram aprimoradas durante a primeira Revolução Industrial,
iniciada na Inglaterra no século XVIII. O trabalho do engenheiro francês Nicolas Léonard Sadi Carnot,
que notou a relação entre a eficiência da máquina a vapor e a diferença de temperatura entre o vapor e
o ambiente externo, foi fundamental para esse aprimoramento.
A solução desenvolvida por Carnot para aumentar a eficiência da máquina a vapor foi:
a) Reduzir o volume do recipiente sob pressão constante.
b) Aumentar o volume do recipiente e reduzir a pressão proporcionalmente.
c) Reduzir o volume do recipiente e a pressão proporcionalmente.
d) Reduzir a pressão dentro do recipiente e manter seu volume.
e) Aumentar a pressão dentro do recipiente e manter seu volume.

125
SEMANA 3

EIXO TEMÁTICO:
IV. Luz, som e calor.

TEMA/TÓPICO:
10. Ondas/ 24. Ondas.

HABILIDADE(S):
24.1. Compreender o comportamento das ondas.
24.1.1. Compreender como as ondas transferem energia sem transferir matéria.
24.1.2. Saber explicar o que significa a frequência, o período, o comprimento de ondas e a amplitude de
uma onda.
24.1.3. Conhecer e saber usar na solução de problemas simples a relação entre velocidade, frequência e com-
primento de onda.
BNCC: EM13CNT301, EM13CNT306, EM13CNT307.

CONTEÚDOS RELACIONADOS:
Mecânica, propriedades ondulatórias da matéria, Movimento Harmônico.

INTERDISCIPLINARIDADE:
Matemática.

TEMA: Ondas - parte 1


Caro (a) estudante, nesta semana você compreenderá melhor os fenômenos ondulatórios, conhecerá
os tipos de ondas e os conceitos básicos do movimento ondulatório. Assim como saber relacionar as
grandezas físicas envolvidas nesses fenômenos.

PRODUZINDO ONDAS
Para produzir algumas ondas, você pode, por exemplo, segurar uma das extremidades de uma corda
fixa e oscilar a sua mão para cima e para baixo. Você verá que alguns pulsos são produzidos e eles se
propagam ao longo dessa corda, como mostram as figuras abaixo:


Imagens: http://www.gta.ufrj.br/grad/; http://www.geocities.ws/luzdoser/ser/frequencia/ondas_na_corda.jpg. Acesso em: 23 jul. 2021.

126
Uma série de pulsos como esses constitui uma onda propagando-se na corda. Os pontos mais altos da
onda são denominados cristas e os mais baixos, são os vales.
Ao se propagar, a onda transporta energia, mas não transporta matéria. Podemos definir as ondas
como: oscilações que se propagam em um meio transportando energia, sem o transporte de matéria.
As ondas são classificadas de acordo com:

A) Natureza da propagação:
• Ondas mecânicas: Necessitam de um meio material para se propagarem. Ex: ondas em cordas,
ondas sonoras, o ultrassom, o infrassom, dentre outras.
• Ondas eletromagnéticas: São ondas geradas a partir das oscilações de campos elétricos e mag-
néticos. Não necessitam de meio material para se propagarem. Ex.: Luz, micro-ondas, raios-X,
ultravioleta, ondas de rádio, dentre outras.
• Ondas gravitacionais: São vibrações que se propagam no rígido tecido do espaço-tempo causadas
por explosões ou colisões de corpos altamente massivos tais como estrelas ou buracos negros.
B) Direção de propagação da vibração:
• Longitudinais: a vibração ocorre na mesma direção de propagação da onda.
• Transversais: a vibração se propaga perpendicularmente à direção de propagação da onda.
C) Direção de propagação no espaço:
• Unidimensionais: a vibração se propaga em uma dimensão. Exemplo: Ondas em uma corda.
• Bidimensionais: a vibração se propaga em duas dimensões.Exemplo: Ondas na superfície da água.
• Tridimensionais: a vibração se propaga em três dimensões. Exemplo: Ondas eletromagnéticas.

GRANDEZAS FÍSICAS ASSOCIADAS ÀS ONDAS


AMPLITUDE: É a máxima distância que cada ponto do meio material por onde a onda se propaga pode
apresentar em relação à sua posição de equilíbrio.
FREQUÊNCIA DA ONDA: Corresponde ao número de oscilações completas que uma onda executa por
unidade de tempo. As unidades de medida de frequência mais utilizadas são: Hertz (Hz) ou Rotações por
minuto (rpm).
PERÍODO DA ONDA: Tempo gasto para efetuar uma oscilação completa, geralmente medido em segundos.
COMPRIMENTO DE ONDA : Representada pela letra grega λ (lâmbda), o comprimento de onda equivale à
distância entre dois vales ou duas cristas consecutivos. A figura abaixo mostra o comprimento de onda:

Fonte: https://static.mundoeducacao.uol.com.br/mundoeducacao/conteudo_legenda/643047ea97fc86ad36abaa4594eb88ed.jpg.
Data de acesso: 27/07/2021.

127
VELOCIDADE: A velocidade da onda é definida pelo produto do seu comprimento de onda (λ) multipli-
cado pela sua frequência f. Assim, temos que:
v=λ.f
Com V medido em m/s ; λ medido em metros e f medida em Hertz .
As ondas estão muito presentes em nosso cotidiano. A luz visível, as ondas de rádio, que permitem o
funcionamento dos GPS, o som, as ondas sísmicas, o infravermelho nos leitores de códigos de barras,
as ondas do mar, são alguns exemplos. Por fazer parte dos fenômenos naturais, o ser humano teve a
curiosidade de investigar os comportamentos e as naturezas das ondas, o que possibilitou inúmeros
avanços científicos e tecnológicos.

ALGUNS FENÔMENOS ASSOCIADOS AO MOVIMENTO ONDULATÓRIO:


a) REFLEXÃO DE ONDA: Ocorre quando a onda encontra um obstáculo em seu caminho de pro-
pagação. A onda bate no obstáculo, muda de direção ou sentido de propagação, sem mudar de
meio. A velocidade, a frequência e o comprimento de onda não são alterados. Esse fenômeno
obedece às Leis da Reflexão.
b) REFRAÇÃO DE ONDA: Ocorre quando a onda muda de meio de propagação. Isso provoca a mu-
dança no valor da velocidade e do comprimento de onda, mas a frequência não se altera. Esse
fenômeno obedece às Leis da Refração.
c) INTERFERÊNCIA: Acontece quando há uma superposição de ondas. Dependendo de sua conse-
quência, ela pode ser de dois tipos: Interferência construtiva: as duas ondas se somam, aumentan-
do o tamanho. Interferência destrutiva: as duas ondas se anulam, diminuindo ou desaparecendo.

PARA SABER MAIS:


MARTINS, L.: Ondulatória. Disponível em: https://www.infoescola.com/fisica/ondulatoria-ondas.
Acesso em: 29 jul. 2021.
Simulador que possibilita criar ondas e pulsos e visualizar alguns dos fenômenos ondulatórios:
Disponível em: https:// phet.colorado.edu/pt_BR/simulation/wave-on-a-string. Acesso em: 29 jul. 2021.

ATIVIDADES
Para resolver as atividades propostas, releia o texto, pesquise outras fontes e siga as orientações de
seu professor.

1 - (ENEM 2021)Para se deslocar e obter alimentos, alguns mamíferos, como morcegos e golfinhos,
contam com a sofisticada capacidade biológica de detectar a posição de objetos e animais pela emissão
e recepção de ondas ultrassônicas.
O fenômeno ondulatório que permite o uso dessa capacidade biológica é a
a) Reflexão.
b) Difração.
c) Refração.
d) Dispersão.
e) Polarização.

128
2 - Sobre as ondas sonoras é correto afirmar que elas não podem se propagar:
a) Na água.
b) Nos materiais metálicos.
c) Na madeira.
d) No vácuo.
e) Nos gases.

3 - Um trem de ondas se propaga em uma corda com velocidade de 2,0 m/s. Sabendo que cada onda
tem comprimento de 0,8 m, determine:
a) A frequência das ondas em Hertz:

b) O período da onda em segundos:

4 - Considere uma onda cuja frequência é de 40 Hz. Podemos afirmar que sua frequência em rpm
equivale a:
a) 1200 rpm.
b) 360 rpm.
c) 2400 rpm.
d) 800 rpm.
e) 4800 rpm.

5 - Pesquise a diferença entre as ondas mecânicas e as ondas eletromagnéticas e cite alguns exemplos
de como esses dois tipos de ondas podem fazer parte do nosso dia a dia:

129
SEMANA 4

EIXO TEMÁTICO:
IV. Luz, som e calor.

TEMA/TÓPICO:
10. Ondas/ 24. Ondas.

HABILIDADE(S):
24.1. Compreender o comportamento das ondas.
24.1.1. Compreender como as ondas transferem energia sem transferir matéria.
24.1.2. Saber explicar o que significa a frequência, o período, o comprimento de ondas e a amplitude de
uma onda.
24.1.3. Conhecer e saber usar na solução de problemas simples a relação entre velocidade, frequência e com-
primento de onda.
BNCC: EM13CNT301, EM13CNT306, EM13CNT307

CONTEÚDOS RELACIONADOS:
Mecânica, propriedades ondulatórias da matéria, Movimento Harmônico.

INTERDISCIPLINARIDADE:
Matemática.

TEMA : Ondas - parte 2


Caro (a) estudante, nesta semana você vai conhecer alguns dos fenômenos ondulatórios que fazem par-
te do mundo à nossa volta e reconhecê-los em situações vivenciadas no cotidiano. Assim, poderá ob-
servá-los e analisá-los do ponto de vista científico.
FENÔMENOS ONDULATÓRIOS
Na semana anterior, foram apresentados alguns fenômenos ondulatórios. Agora, vamos analisar alguns
deles e procurar identificá-los em situações cotidianas.
Refração de ondas
Independente do tipo de onda, todas elas refratam-se ao mudar de meio de propagação.
A figura 1, a seguir, mostra a refração de uma onda. Nela, pode-se observar dois meios diferentes: meio
1 e meio 2. Esses meios podem ser, por exemplo, o ar ( meio 1) e água ( meio 2) ou ar e vidro ou, ainda,
vidro e água, ou quaisquer outros meios diferentes. Há uma superfície de separação entre eles e quan-
do a onda, que se propagava inicialmente no meio 1, encontra-se com essa superfície de separação,
a partir daí, passa a se propagar no meio 2. Esses dois meios de propagação, que podem ser materiais
ou não, apresentam características diferentes e, por isso, a onda apresenta comportamentos diferen-
tes em cada um deles.

130
Nessa figura, observa-se o vetor V1, que é a velocida-
de da onda no meio 1, e o comprimento dessa onda no
meio 1 é λ1 . Note que, ao se propagar no meio 2, a ve-
locidade e o comprimento de onda se alteram, passan-
do aos valores V2 e λ2 . Nesse caso, ambos diminuíram.
A direção da trajetória da onda também se altera por
meio de um desvio angular. Caso a onda incida de ma-
neira perpendicular, não haverá desvio na trajetória,
mas V e λ serão modificados.
Tudo isso caracteriza uma refração: a velocidade, o
comprimento de onda e a direção de propagação são
alterados.
Figura 1: Uma onda refrata-se ao mudar de meio Mas, o que pode-se dizer sobre a frequência das ondas
de propagação. Disponível em: <https://static.
mundoeducacao.uol.com.br/mundoeducacao/conteudo_ no caso de uma refração?
legenda/ce2668cd72ffbe6132603c1f032c4006.jpg.
A frequência das ondas não se altera na refração. Isto
Acesso em: 28 jul. 2021.
é, ela se mantém constante. Isso ocorre porque a fre-
quência só depende da fonte que produz as ondas. Assim, de acordo com a equação v = λ. f, a velocidade
e o comprimento de ondas são diretamente proporcionais.
Portanto, se em uma refração o comprimento de onda diminuir, sua velocidade também irá diminuir, na
mesma proporção. E, do contrário, se o comprimento de onda aumentar, a velocidade também aumen-
tará proporcionalmente.
Exemplo:
Na figura 1, suponha que a frequência e a velocidade da onda no meio 1 sejam, respectivamente, 20 Hz e
8,0 m/s. Sabendo que, ao se refratar, sua velocidade diminuiu para 4,0m/s, determine:
A frequência da onda no meio 2:
O comprimento de onda no meio 2:
Solução:
A frequência da onda no meio 2 é de 20 Hz, pois na refração a frequência permanece constante.
Pela expressão v = λ. f, concluímos que, no meio 1, o comprimento de onda tem valor igual a 0,4m.
Ao se refratar, a velocidade da onda diminuiu 0,5 vezes, passando de 8,0 m/s para 4,0 m/s. Portanto, seu
comprimento de onda também diminuirá 0,5 vezes e passará a valer 0,2 m.

REFRAÇÃO NO DIA A DIA


Se pegarmos um lápis inteiro e mergulharmos somente a sua metade dentro de um
copo com água, será possível observar um fato curioso, como o representado na
figura 2. Teremos a impressão de que o lápis está quebrado. Mas, sabemos que, na
verdade, não está. Isto ocorre porque a luz, que é uma onda eletromagnética, refra-
ta-se e sua velocidade de propagação dentro da água é menor que sua velocidade
de propagação no ar.
O arco íris é outro fenômeno que também está relacionado com a refração da luz.
Figura 2: Refração da luz. Imagem: https://cdn.pixabay.com/photo/2017/06/14/23/11/pencil-2403662_1280.jpg.
Acesso em: 27 jul. 2021 .

131
A refração ocorre, também, no caso de uma onda mecânica que se propaga em cordas de densidades
lineares diferentes, como observa-se na figura 2.
Um pulso, inicialmente na corda mais fina, encontra
uma corda mais grossa. Haverá, portanto, mudança de
meio de propagação, pois o pulso, que antes se propa-
gava na corda fina, agora passa a se propagar, também,
na corda mais grossa (mais densa). Note que parte do
pulso será refletida e outra parte será refratada.
Na refração em cordas, observa-se que a velocidade
da onda diminui, se essa passar de uma corda fina para
uma corda mais grossa. Do contrário, a velocidade au-
mentará quando a onda passar de uma corda grossa
para uma fina.
Figura 3: Refração em cordas de densidades diferentes. Imagem: https://s3.static.brasilescola.uol.com.br/img/2017/10/onda-corda.
jpg>. Acesso em: 27 jul. 2021 .

LEI DA REFRAÇÃO
A refração, assim como todos os fenômenos físicos, ocorre obedecendo leis. No caso da refração da
luz, a Lei de Snell- Descartes nos permite calcular o desvio na trajetória de um raio luminoso, ao ser
transmitido de um meio para outro. Essa Lei nos diz que:
Onde:
i - é o ângulo de incidência da onda incidente
r - é o ângulo de refração da onda refratada
n1 - é o índice de refração do meio 1
Imagem: https://
mundoeducacao.uol.com.br/ n2 - é o índice de refração do meio 2
upload/conteudo/images/ro1.jpg. v1 - é a velocidade de propagação da onda no meio 1
Acesso em: 27 jul. 2021 v2 - é a velocidade de propagação da onda no meio 2
O índice de refração (n) é uma característica do meio de propagação da luz. É um número adimensional
e está relacionado à velocidade da luz naquele meio.

DENSIDADE LINEAR DE CORDAS


A densidade linear de uma corda é a relação entre sua massa e seu comprimento. Essa relação é expres-
sa por μ = m/L , onde μ é a densidade linear da corda, m é a massa e L o seu comprimento. Ou seja, quanto
mais pesada for a corda, maior a sua densidade linear. Ao unir cordas de densidades lineares diferentes,
ocorrerá a refração, caso uma onda seja transmitida da corda mais fina para a mais grossa ou vice-versa.

PARA SABER MAIS:


Para se aprofundar um pouco mais, algumas sugestões: de leituras:
Ondas Mecânicas. Texto disponível em: https://www.infoescola.com/fisica/ondas-mecanicas/. Aces-
so em: 28 jul. 2021.
Interferência entre ondas. Texto disponível em: https://www.infoescola.com/fisica/interferencia-en-
tre-ondas/. Acesso em: 28 jul. 2021.
A Física e as ondas do mar. Texto disponível em: https://brasilescola.uol.com.br/fisica/a-fisica-as-on-
das-no-mar.htm. Acesso em: 28 jul. 2021.

132
ATIVIDADES
Agora, chegou sua vez de colocar em prática o que você aprendeu. Leia o texto, pesquise outras fontes
de seu interesse. Peça orientações ao seu professor. A prática de exercícios ajuda a fixar o conteúdo
estudado.

1 - (FGV) Verifica-se que, ao sofrer refração, um trem de ondas mecânicas apresenta um novo perfil
de oscilação, em que a distância entre duas cristas consecutivas de suas ondas tornou-se maior.
Comparativamente ao que possuía o trem de ondas antes da refração, a frequência se ______, a velocidade
de propagação se _______ e a amplitude se manteve, já que o novo meio é ________ refringente.
Assinale a alternativa que preenche corretamente as lacunas.
a) alterou … alterou … menos
b) alterou … manteve … mais
c) manteve … alterou … mais
d) manteve … alterou … menos
e) manteve … manteve … mais

2 - A figura 3 da leitura desta semana representa um pulso que se propaga inicialmente em uma corda
fina e, em seguida, esse pulso é transmitido para uma corda grossa. Trata-se, portanto, do fenômeno
da refração de ondas. Suponha que uma onda esteja se propagando na corda fina com velocidade v= 1,5
m/s e que o comprimento de onda vale λ = 30 cm.
a) Qual a frequência com que um ponto qualquer da corda fina está oscilando? __________________
b) Qual o tempo que a onda gasta para efetuar uma oscilação completa? _________________________
c) A onda continua se propagando e é transmitida para corda grossa. Qual a frequência da onda na
corda grossa? _____________________________________
d) Sendo 1,0 m/s a velocidade de propagação da onda na corda grossa, determine a distância entre
duas cristas consecutivas dessa onda na corda grossa. _______________________________

3 - Uma rolha de cortiça está flutuando na água contida em um tanque. Bate-se com uma régua, na
superfície da água, de 20s em 20 s, de modo a produzir uma onda de pulsos retos tais que a distância entre
duas cristas consecutivas dessa onda seja de 5,0 cm. (Retirada do livro: Física contextos & aplicações -
Volume 2- MÁXIMO, A. ALVARENGA, B. Editora Scipione)
a) Qual o período da onda? __________________________
b) Descreva o movimento da rolha enquanto a onda passa por ela. _____________________________
___________________________________________________________
c) Quantas oscilações por segundo a rolha efetua em seu movimento? ________________________
d) Qual é a velocidade de propagação da onda? _____________________________________________

133
SEMANA 5

EIXO TEMÁTICO:
IV. Luz, som e calor.

TEMA/TÓPICO:
10. Ondas. / 25. Som.

HABILIDADE(S):
25.1 Compreender as propriedades e efeitos das ondas sonoras.
25.1.1. Compreender como o som provoca a vibração do tímpano.
25.1.4. Explicar a relação entre a intensidade do som e a amplitude da vibração.
25.1.5. Conhecer a relação entre o som grave, agudo e a frequência.
BNCC: EM13CNT301, EM13CNT306, EM13CNT307.

CONTEÚDOS RELACIONADOS:
Movimento Ondulatório, Aparelho Auditivo.

INTERDISCIPLINARIDADE:
Biologia.

TEMA: Ondas sonoras


Caro (a) estudante, nesta semana você vai compreender o que são as ondas sonoras, como os materiais
oscilam e produzem o som e, finalmente, como os escutamos.

ONDAS SONORAS
As ondas sonoras são produzidas por oscilações de objetos materiais. De fato, o som é a propagação de
ondas mecânicas e, portanto, os fenômenos sonoros estão relacionados com as oscilações em meios
materiais (sólidos, líquidos ou gasosos).
Por exemplo, quando uma pessoa fala, o som emitido é produzido pelas oscilações das cordas vocais.
O som de um violão é produzido pela oscilação das cordas desse instrumento. Quando batemos em um
pedaço de madeira, ou de metal, esses objetos vibram e emitem som. É necessário ter matéria para
emitir ondas sonoras, sendo assim, não é possível produzir som no vácuo.

COMO OUVIMOS OS SONS?


Observe a figura abaixo que é uma representação da orelha humana subdividida em três partes: orelha
externa, orelha média e orelha interna:

134
Fonte: https://www.infoescola.com/wp-content/uploads/2010/08/anatomia-orelha-ouvido-780609469-1000x768.jpg.
Acesso em: 28 jul. 2021.

As ondas sonoras, ao atingir a orelha externa, são dirigidas para o interior do canal auditivo (meato
acústico externo) na extremidade do qual há a membrana timpânica. Essa membrana é tão delicada e
sensível que pequenas variações de pressão de ondas sonoras já podem colocá-la em vibração.
Essas vibrações são, por sua vez, comunicadas a um pequeno osso chamado de martelo. A partir daí,
outros ossos do interior da orelha média são acionados e, nesse processo, as vibrações são sucessiva-
mente amplificadas e isso faz com que nossa orelha perceba sons de intensidades muito baixas.
Finalmente, as oscilações amplificadas chegam até a orelha interna. No interior da cóclea existe um
líquido que facilita a propagação do som.

O QUE É O SOM?
O som é classificado como uma onda mecânica, longitudinal cuja frequência está compreendida, apro-
ximadamente, na faixa de 20 hertz a 20.000 hertz. Uma onda longitudinal propagando-se com frequên-
cias abaixo de 20 hertz é denominada INFRASSOM e aquelas cujas frequências estão acima de 20.000
hertz são denominadas ULTRASSOM.
O infrassom e o ultrassom não provocam sensação sonora ao atingirem a orelha humana. Entretanto, al-
guns animais são capazes de perceber o ultrassom, como por exemplo, cachorros, morcegos e golfinhos.
O morcego consegue voar no escuro sem colidir em algum objeto, porque emite um ultrassom que, após
ser refletido pelo obstáculo, volta a ele, possibilitando a localização desse objeto.

135
A VELOCIDADE DO SOM
Durante uma tempestade, o relâmpago e o trovão ocorrem no mesmo instante. É um fato curioso que
só conseguimos ouvir o trovão certo tempo após termos visto o relâmpago. Ou seja, primeiro vemos o
relâmpago e, só um tempo após, podemos ouvir o trovão. Isso é devido à velocidade da luz ser muito
maior que a velocidade do som. A percepção da luz do relâmpago é, praticamente, instantânea. O in-
tervalo entre a percepção do relâmpago e a do trovão é o tempo que a onda sonora leva para chegar até
nossas orelhas.
No séc. XVII cientistas realizaram experimentos para medir a velocidade do som e obtiveram, em 1685,
o valor aproximado de 450 m/s. Medidas mais recentes, realizadas com mais precisão, mostraram que
essa velocidade é de 340m/s, no ar, à temperatura de 20° C.

INTENSIDADE DO SOM
A intensidade é uma propriedade da onda sonora que está relacionada com a energia que a onda trans-
porta. Mas, para todos os fenômenos ondulatórios, quanto maior a energia transportada, maior a am-
plitude da onda. Assim, conclui-se que: A intensidade de um som é tanto maior quanto maior for a
amplitude da onda sonora.
Mas, na prática, o que é intensidade do som?
Uma música tocada no rádio, em seu volume máximo, possui maior intensidade que o som emitido pelo
tic-tac do relógio. No entanto, é preciso tomar certo cuidado para não confundir a intensidade do som
com a altura do som. São duas propriedades distintas.

A ALTURA DO SOM
A altura do som é uma propriedade da onda sonora que está relacionada com sua frequência. Podemos
distinguir os sons graves de sons agudos: o som grave, como o emitido pelos tambores, é de menor
frequência e, portanto, é um som baixo. Já o som emitido pelas cordas de um violino é um som agudo,
de maior frequência e, portanto, é um som alto. A frequência de um som caracteriza sua altura.

PARA SABER MAIS


Sugestão de leitura: Ondas Sonoras. Luiz Guilherme Rezende Rodrigues. Texto disponível em: https://
www.infoescola.com/fisica/ondas-sonoras. Acesso em: 28 jul. 2021.
Velocidade do som. Luiz Bruno Vianna. Texto disponível em: https://www.infoescola.com/fisica/velo-
cidade-do-som. Acesso em: 28 jul. 2021.

136
ATIVIDADES
1 - Após ler o texto apresentado na semana 5, analise as alternativas abaixo e marque aquela que você
julga ser a correta. Esse exercício te ajudará a fixar os conceitos abordados sobre as ondas sonoras.
a) O som é um tipo de onda mecânica, bidimensional e longitudinal. As ondas sonoras propagam-se
mais rápido em meios gasosos do que em meios sólidos.
b) O som é um tipo de onda mecânica, tridimensional e longitudinal. As ondas sonoras propagam-se
mais rápido em meios gasosos do que em meios sólidos.
c) O som é um tipo de onda mecânica, tridimensional e longitudinal. As ondas sonoras propagam-se
mais rápido em meios sólidos do que em meios líquidos.
d) O som é um tipo de onda mecânica, tridimensional e transversal. As ondas sonoras propagam-se
mais rápido em meios sólidos do que em meios gasosos.
e) O som é um tipo de onda mecânica, unidimensional e longitudinal. As ondas sonoras propagam-se
mais rápido em meios gasosos do que em meios sólidos.

2 - Para realizar a atividade a seguir, procure fazer uma pesquisa sobre a velocidade do som nos diferentes
meios onde ele pode se propagar: líquidos, gasosos e sólidos, identificando os fatores que podem
interferir nas velocidades de propagação em cada um. Peça também, a orientação de seu professor.
A seguir, são feitas algumas afirmações sobre as ondas sonoras. Marque V para as informações verda-
deiras e F para as falsas.
(  ) As ondas sonoras, assim como qualquer tipo de onda, podem sofrer polarização.
(  ) A velocidade do som nos sólidos é maior que a velocidade do som nos líquidos.
(  ) A velocidade máxima possível para o som no ar é de 340 m/s.
(  ) Quanto maior a temperatura de um gás, maior será a velocidade de propagação das ondas sonoras nele.
(  ) A velocidade do som na água fria é maior que a velocidade do som na água quente.

a) V V V V F. b) V F F V V. c) F V F F F. d) F V V V F. e) F V F V F.

3 - Uma fonte de ondas sonoras está imersa em água, emitindo ondas a uma frequência de 740 Hz. Sabendo
que o comprimento de onda das ondas produzidas pela fonte é de 200 cm. Determine a velocidade de
propagação das ondas sonoras na água.

a) 1400 m/s. b) 1480 m/s. c) 1500 m/s. d) 340 m/s. e) 7400 m/s.

4 - Sobre o ouvido humano, assinale a alternativa que completa a frase abaixo:


Os pequenos ossos encontrados na orelha média têm a função de transmitir as vibrações sonoras
a) do tímpano para o nervo auditivo.
b) da aurícula para o tímpano.
c) do canal auditivo para o nervo auditivo.
d) do tímpano para a janela oval.
e) da orelha média para o tímpano.

137
SEMANA 6

EIXO TEMÁTICO:
IV. Luz, som e calor.

TEMA/TÓPICO:
10. Ondas/ 25. Som.

HABILIDADE(S):
25.1. Conhecer as propriedades e efeitos das ondas sonoras.
25.1.4. Explicar a relação entre a intensidade do som e a amplitude da vibração.
25.1.5. Conhecer a relação entre som grave, agudo e a frequência.
BNCC: EM13CNT301, EM13CNT306, EM13CNT307.

CONTEÚDOS RELACIONADOS:
Movimento Ondulatório.

INTERDISCIPLINARIDADE:
Música.

TEMA: As propriedades do som


Caro (a) estudante, nesta semana você conhecerá ainda mais algumas das propriedades do som e sa-
berá relacioná-las com suas aplicações nos instrumentos musicais. Além disso, entenderá o fenômeno
denominado Efeito Doppler do som.
ALTURA DO SOM: NOTAS MUSICAIS
Altura do som é uma qualidade das ondas que nos permite distinguir um som grave de um som agudo.
Quanto mais agudo for o som, mais elevada será a sua frequência. Assim, na linguagem musical, um som
agudo é alto e um som grave é baixo. Note que essa é uma classificação diferente daquela utilizada para
a intensidade do som, que foi apresentada na semana anterior.
As notas musicais Dó, Ré, Mi, Fa, Sol, Lá, SI, são caracterizadas por suas frequências. Cada uma dessas
notas emite uma onda que vibra numa frequência específica. As notas de menores frequências são as
notas graves, e as de maiores frequências, são as agudas.
A tabela ao lado apresenta as notas musicais e os valores das
frequências de cada uma delas. A nota Dó, cuja frequência vale
264 Hertz é classificada como grave e a nota Si, de frequência
igual a 495 hertz é uma nota aguda.
O TIMBRE
Se emitirmos a nota Ré por um piano e essa mesma nota for
emitida, na mesma intensidade, por um violino, conseguire-
mos distinguir o som do piano do som do violino. Isso é possível
porque esses dois instrumentos possuem timbres diferentes.
Isso se aplica, também nos demais instrumentos musicais.
Imagem: https://mundoeducacao.uol.com.br/upload/conteudo/images/nota-e-frequencia(1).jpg. Data de acesso: 30 jul. 2021.

138
Cada um emite uma onda que apresenta uma forma própria, característica daquele instrumento.
No caso de seres humanos, cada pessoa apresenta um timbre de voz próprio. E é por esse motivo, que
reconhecemos uma pessoa por sua voz.
A imagem a seguir mostra as formas próprias de ondas sonoras emitidas, na mesma frequência, ou
seja, na mesma nota musical, por 3 instrumentos musicais diferentes: diapasão, flauta e violão, além da
voz humana. As ondas se diferem pelos formatos das suas cristas e dos seus vales. Portanto, cada uma
apresenta um timbre diferente.

Imagem disponível em: https://s2.static.brasilescola.uol.com.br/be/2020/02/timbre.jpg. Data de acesso: 30 jul. 2021.

O EFEITO DOPPLER
A imagem a seguir representa duas pessoas: Cristina e Pedro. Ambos observam uma ambulância que se
movimenta com sua sirene ligada. Essa ambulância se afasta de Cristina, enquanto se aproxima de Pedro .

Imagem: https://www.infoescola.com/wp-content/uploads/2007/11/efeito-doppler-658148101-600x400.jpg.
Data de acesso: 30 jul. 2021.

Na situação descrita acima, podemos considerar a ambulância como uma fonte sonora. Cristina e Pe-
dro são os observadores.
Como a fonte sonora está se afastando de Cristina, ela irá perceber o som da sirene com menor fre-
quência. Isto significa que o número de ondas sonoras( emitidas pela sirene) que chegam às suas ore-
lhas, por segundo, é menor que o número de ondas sonoras que chegam, por segundo, nas orelhas de
Pedro, pois, para ele, a fonte sonora está se aproximando.

139
Há, portanto, uma variação aparente da frequência do som, causada pelo movimento da fonte sonora.
Isso ocorreria, também, caso a fonte estivesse em repouso e os observadores em movimento. Esse
efeito é denominado Efeito Doppler e foi analisado no séc.XIX pelo físico austríaco Christian Doppler,
que concluiu: o som de uma fonte sonora parece mais agudo se ela se aproxima do observador e parece
estar mais grave, no caso em que a fonte se afasta.

EFEITO DOPPLER DA LUZ


O Efeito Doppler não é um fenômeno específico do som. Ele ocorre, também, com outras ondas. No caso
da luz, a faixa de onda de menor frequência é a do vermelho, enquanto que a faixa luminosa de maior
frequência é a do violeta, de acordo com o espectro da luz visível. Mas, o efeito doppler da luz seria mais
facilmente percebido se a fonte luminosa se movimentasse com uma velocidade tão elevada quanto a
da luz (3,0 x108 m/s). Percebe-se esse efeito no caso de estrelas e galáxias que se afastam da Terra.
Quando a fonte luminosa se aproxima, a frequência da luz percebida pelo observador é aparentemente
maior que a frequência emitida. Haverá, então, um desvio para o azul e este fenômeno é conhecido
como blue shift. Em caso contrário, quando a fonte se afasta do observador, há um desvio para o ver-
melho e, esse, é chamado de red shift.

PARA SABER MAIS:


Sugestão de leitura:
Texto Efeito Doppler. RODRIGUES. Luiz G.R. Disponível em: https://www.infoescola.com/fisica/efeito-
-doppler/. Acesso em : 28 jul 2021.
Texto: O espectro da luz visível - CAVALHEIRO; Carlos A. Disponível em: <https://www.infoescola.com/
fisica/espectro-visivel/>.Acesso em : 28 jul. 2021.

ATIVIDADES
Agora, é o momento de você testar o que aprendeu. Leia o texto, pesquise outras fontes e peça orienta-
ções ao seu professor. Resolvas as atividades propostas a seguir. Bons estudos!

1 - (UFSCar- 2018) Um homem adulto conversa com outro de modo amistoso e sem elevar o nível sonoro
de sua voz. Enquanto isso, duas crianças brincam emitindo gritos eufóricos, pois a brincadeira é um
jogo interessante para elas. O que distingue os sons emitidos pelo homem dos emitidos pelas crianças
a) É o timbre, apenas.
b) É a altura, apenas.
c) São a intensidade e o timbre, apenas.
d) São a altura e a intensidade, apenas.
e) São a altura, a intensidade e o timbre.

140
2 - (IFRS) O som é a propagação de uma onda mecânica longitudinal apenas em meios materiais. O som
possui qualidades diversas que o ouvido humano normal é capaz de distinguir. Associe corretamente
as qualidades fisiológicas do som apresentadas na coluna da esquerda com as situações apresentadas
na coluna da direita.

Qualidades fisiológicas Situações

(1) Intensidade (  ) Abaixar o volume do rádio ou da televisão.


(  ) Distinguir uma voz aguda de mulher de uma voz grave de homem.
(2) Timbre
(  ) Distinguir sons de mesma altura e intensidade produzidos por vozes de
(3) Frequência pessoas diferentes.
(  ) Distinguir a nota Dó emitida por um violino e por uma flauta.
(  ) Distinguir as notas musicais emitidas por um violão.
A sequência correta de preenchimento dos parênteses, de cima para baixo, é:

a) 1 – 2 – 3 – 3 – 2 b) 1 – 3 – 2 – 2 – 3 c) 2 – 3 – 2 – 2 – 1 d) 3 – 2 – 1 – 1 – 2 e) 3 – 2 – 2 – 1 – 1

3 -. (PUCCAMP-SP) Um professor lê o seu jornal sentado no banco de uma praça e, atento às ondas
sonoras, analisa três eventos:
I – O alarme de um carro dispara quando o proprietário abre a tampa do porta-malas.
II –Uma ambulância se aproxima da praça com a sirene ligada.
III – Um mau motorista, impaciente, após passar pela praça, afasta-se com a buzina permanentemente ligada.
O professor percebe o efeito Doppler apenas:
a) no evento I, com frequência sonora invariável.
b) nos eventos I e II, com diminuição da frequência.
c) nos eventos I e III, com aumento da frequência.
d) nos eventos II e III, com diminuição da frequência em II e aumento em III.
e) nos eventos II e III, com aumento da frequência em II e diminuição em III.

Caro(a) estudante, você chegou ao final de mais uma etapa importante de sua jornada na Educação.
Eu gostaria de continuar a te encorajar, pois, juntos, venceremos todos esses desafios que nos foram
impostos nesses últimos meses deste ano. Acredite, todo conhecimento que você adquire por meio de
seus estudos, por meio da escola, pode sim, te ajudar a transformar sua vida! Deixo aqui, meus votos
de felicidades! Até breve!

REFERÊNCIAS:
MÁXIMO, A., ALVARENGA B. Curso de Física. Volume 2, 1ª Ed. São Paulo: Scipione, 2010.
MAGIE, W. A Source Book in Physics, McGraw-Hill, New York, 1935; Pg 151
MORTIMER, E.; HORTA, A.;MATEUS, A.; PANZERA, A.; GARCIA, E.; PIMENTA, M.; MUNFORD, D.;
FRANCO, L.; MATOS, S. Matéria, Energia e Vida Uma Abordagem Interdisciplinar. 1ª Ed. São Paulo:
Scipione, 2020.

141
SECRETARIA DE ESTADO DE EDUCAÇÃO DE MINAS GERAIS

PLANO DE ESTUDO TUTORADO


COMPONENTE CURRICULAR: GEOGRAFIA
ANO DE ESCOLARIDADE: 2º ANO – EM
PET VOLUME: 04/2021
NOME DA ESCOLA:
ESTUDANTE:
TURMA: TURNO:
BIMESTRE: 4º TOTAL DE SEMANAS:
NÚMERO DE AULAS POR SEMANA: NÚMERO DE AULAS POR MÊS:

SEMANA 1

EIXO TEMÁTICO:
Os cenários da globalização e fragmentação.

TEMA/TÓPICO:
As novas fronteiras do capitalismo global: os territórios nas novas regionalizações/ Sociedade da informação.

HABILIDADE(S):
Avaliar a importância das redes mundiais de informação na produção do espaço mundial.

CONTEÚDOS RELACIONADOS:
Tecnologia, Redes Geográficas, Aldeia Global, Revolução Industrial, Migrações, Fronteira.

INTERDISCIPLINARIDADE:
Língua Portuguesa, Língua Estrangeira, Arte, Sociologia, História, Filosofia.

TEMA: Cenários de um mundo globalizado


Caro (a) estudante, nesta semana você vai compreender sobre os fluxos da informação e de pessoas e
os seus impactos econômicos e sociais no contexto da globalização.
FLUXOS DA SOCIEDADE GLOBAL
Os avanços promovidos pela Revolução Técnico-Científica Informacional acarretaram uma maior ex-
pansão do sistema capitalista pelo mundo, transcendendo todas as suas fronteiras e ampliando os seus
horizontes de ação. Assim, consolidou-se o processo de globalização – visto, por muitos, como uma
mundialização –, que permitiu a instauração da chamada “Aldeia Global”.
A globalização, sob vários aspectos (econômico, político, urbano, territorial etc.), atua por meio da con-
solidação de um sistema informacional, que se estrutura a partir da formação de redes geográficas, ou
seja, por um sistema interconectado de pontos e ligações entre eles. A partir disso, podemos entender
a relação de nós interconectados entre si ou a composição de fixos e fluxos que estruturam a economia
mundial. De toda forma, o processo de globalização seria inimaginável se não houvesse os fluxos inter-
nacionais que estruturam a sua existência.

142
Entende-se por fluxos da sociedade global a cadeia interconectada entre as diferentes partes do mun-
do que permite a circulação – nem sempre livre – de elementos econômicos, informações e pessoas.
Portanto, os fluxos podem ser considerados, em muitas abordagens, como a materialização da globali-
zação no espaço geográfico.

Disponível em: <https://cafecomsociologia.com/o-que-e-globalizacao-2/>. Acesso em: 14 ago. 2021.

Por outro lado, destaca-se também que essa rede de comunicação global não se consolidou de maneira
igualitária ou democrática no mundo, havendo aqueles países ou lugares em que a menor parte da po-
pulação possui acesso à internet ou, até mesmo, a diversas formas mais convencionais de comunica-
ção e obtenção de notícias. Os países desenvolvidos, como era de se esperar, possuem mais da metade
de seus habitantes integrados mundialmente, enquanto os dos países emergentes e subdesenvolvidos
caminham a passos lentos em direção a esse progresso.
Por extensão aos avanços tecnológicos provocados ao longo do século XX e início do século XXI, o fluxo
internacional de pessoas também vem se intensificando na era da globalização atual. A expansão desse
fluxo acontece de duas formas: o turismo e a migração.
O turismo, não por acaso, é a atividade do setor terciário que mais vem crescendo no planeta, com
milhões de pessoas se deslocando todos os anos sob os mais diferentes interesses. Com isso, as cida-
des receptoras e também os meios de transporte vão se adequando a essa realidade, o que resulta na
modernização de seus respectivos sistemas de recepção, deslocamento e hospedagem, gerando cifras
milionárias em termos de lucros e produção de riquezas.
As migrações internacionais também se intensificam no planeta e configuram-se sob muitos aspec-
tos. Muitas migram por razões humanitárias, sociais, econômicas e afetivas, muito embora existam
muitas barreiras estabelecidas pelos países para conter esse processo. É muito comum a migração de
pessoas de um país para outro (muitas vezes por meios ilegais) em busca de maior geração de renda
e oportunidades. 
Não resta dúvida quanto às migrações internacionais fazem parte da vida contemporânea, uma vez que
estão espalhados ao redor do globo, seja por aqueles que se vão ou por aqueles que chegam. De alguma
maneira em todos os cantos do planeta é possível observar este fenômeno.
Portanto, como podemos observar, os fluxos que estruturam a sociedade global e suas redes inter-
nacionais são compostos por interações econômicas, informacionais e demográficas. Estas, por sua
vez, permitem a expansão mundial de outros elementos, tais como os costumes culturais ou regionais,
religiões e as práticas socioespaciais de um modo geral.

143
ATIVIDADES
1 - Com relação ao espaço mundial, o processo de globalização provoca uma homogeneização da
produção e do consumo em nível global, porém esse processo não é uniforme em todo o planeta. Desse
modo, pode-se afirmar que a globalização resultou na:
a) melhoria da qualidade de vida das populações periféricas.
b) utilização de mão de obra com baixa qualificação profissional.
c) acentuação da desigualdade social entre diferentes regiões.
d) diminuição dos impactos ambientais gerados no globo.

2 - “Sozinho vai descobrindo o caminho/ O rádio fez assim com seu avô/ Rodovia, hidrovia, ferrovia/
E agora chegando a infovia/ Para alegria de todo o interior”.
GIL, G. Banda larga cordel. Disponível em: www.uol.vagalume.com.br. Acesso em: 17 agosto 2021 (fragmento).

O trecho da canção faz referência a uma das dinâmicas centrais da globalização, diretamente associada
ao processo de
a) evolução da tecnologia da informação.
b) expansão das empresas transnacionais.
c) ampliação dos protecionismos alfandegários.
d) expansão das áreas urbanas do interior.

3 - A figura a seguir foi difundida pelo geógrafo David Harvey:

Fonte: https://brainly.com.br/tarefa/39401870. Acesso em: 14 ago. 2021.

144
Responda: Interpretando a figura, o que ela representa? Pode-se concluir, corretamente, que o planeta
encolheu?

4 - Observe atentamente a charge e explique seu significado considerando o que você aprendeu sobre
Globalização.

Disponível em: <https://psicod.org/geografia-leituras-e-interaco-volume-3-v2.html?page=52>. Acesso em: 14 ago 2021.

145
SEMANA 2

EIXO TEMÁTICO:
Os cenários da globalização e fragmentação.

TEMA/TÓPICO:
As novas fronteiras do capitalismo global: os territórios nas novas regionalizações./ Desterritorialização e
redes de solidariedade.

HABILIDADE(S):
Avaliar as possibilidades de desterritorialização a partir de projetos de inclusão digital e de estratégias dos
migrantes.

CONTEÚDOS RELACIONADOS:
Fronteira, lugar, território, diáspora, migrações, refugiados.

INTERDISCIPLINARIDADE:
Língua Portuguesa, Arte, Sociologia, História, Filosofia.

TEMA: Globalização e fragmentação I


Caro (a) estudante, nesta semana você vai identificar as diferentes manifestações culturais de minorias
étnicas, compreendendo a multiplicidade no espaço geográfico.
ORIENTE MÉDIO: A QUESTÃO DA PALESTINA
A questão palestina é um termo utilizado em referência à luta dos povos palestinos após a perda de
seus territórios, o que ocorreu em função dos desdobramentos ligados à criação do Estado de Israel
em 1948. Atualmente, os territórios palestinos reduzem-se a restritas áreas na Cisjordânia e também na
Faixa de Gaza (ver mapa), onde são comuns conflitos entre judeus e árabes.
MAPA DOS TERRITÓRIOS PALESTINO NA ATUALIDADE

Disponível em: <https://s2.static.brasilescola.uol.com.br/img/2015/04/palestina-nos-dias-atuais.jpg>. Acesso em: 14 ago. 2021.

146
Os povos palestinos são constituídos por uma etnia do mediterrâneo composta por uma miscigena-
ção entre filisteus, árabes e cananeus; são maciçamente muçulmanos e utilizam o idioma árabe. Já a
Palestina (de Filistina – “terra dos Filisteus”) é uma região considerada histórica tanto pelos próprios
palestinos quanto pelos judeus. Esses últimos ocuparam essa região há mais de quatro mil anos, que é
considerada por eles como uma área sagrada: a Terra Prometida.
Um dos conflitos que mais geram tensões e preocupações em todo o mundo é o que envolve judeus e
muçulmanos no território de enclave entre Israel e Palestina. Ambos os lados reivindicam o seu próprio
espaço de soberania, embora atualmente esse direito seja exercido plenamente apenas pelos israelen-
ses. Com isso, guerras são travadas, grupos considerados terroristas erguem-se, vidas são perdidas, e
uma paz duradoura encontra-se cada vez mais distante.
Pode-se dizer que tudo começou com o surgimento do movimento sionista, no final do século XIX.
O desejo de autodeterminação do povo judeu e o retorno para a “terra santa” contribuíram para o pro-
cesso migratório para a região. Esse movimento político defendia o direto dos judeus de terem sua
pátria na região que a bíblia chamou de “Terra de Israel”.
Durante a 2ª Guerra Mundial (1939-1945), houve o extermínio de judeus pelos nazistas, o que causou a
migração de famílias judias para fora da Europa. Grande parte delas foi para a Palestina, onde seria cria-
do o Estado de Israel em 1948.
Enquanto os palestinos foram enfraquecendo-se e, consequentemente, dispersando parte de sua po-
pulação para outros territórios (como o Líbano, a Síria e o Egito), Israel intensificou a sua força e tornou-
-se militar e politicamente preponderante na região do Oriente Médio, principalmente após as vitórias
nas guerras do Suez (1956), dos Seis Dias (1967) e do Yom Kippur (1973). Essa configuração favoreceu a
criação, por parte dos palestinos, de vários grupos extremistas que passaram a lutar não só pela criação
de um Estado Palestino, mas também pela total destruição de Israel e expulsão dos judeus da região.
Nesse intuito, foi fundada a Organização para a Libertação Palestina (OLP) em 1964, liderada pelo gru-
po Al Fatah, que realizava atos extremistas desde 1959 e era comandado por Yasser Arafat. Mais tarde,
em 1987, foi fundado outro grupo extremista, o Hamas, que hoje é formado por três frentes: um partido
político, um braço armado e uma organização filantrópica pró-palestina. Esse grupo é considerado por
muitos países como uma organização terrorista.
Em 2002, iniciaram-se as construções do Muro de Israel ou Muro da Cisjordânia para a separação dos
territórios controlados pelos palestinos do restante do território de Israel. No entanto, essa construção
vem sendo bastante criticada, em razão das acusações de que Israel estaria ocupando, durante o ergui-
mento da muralha, áreas que deveriam ser de controle palestino.
Em 2006, para tornar o cenário ainda mais tenso politicamente, o Hamas venceu as eleições no terri-
tório palestino, derrotando pela primeira vez o Fatah, o que gerou uma recusa por parte de Israel e das
potências internacionais de reconhecerem a Palestina, isolando a Autoridade Palestina politicamente.
Como se vê, parece que o conflito ainda está longe de se resolver, mesmo com os avanços nas últimas
décadas e com a quase criação do Estado da Palestina. Mas ainda é cedo pra dizer que os palestinos go-
zam de uma soberania plena, já que integridade territorial da Palestina é recorrentemente questionada
pelas ações de Israel. De qualquer modo, enquanto o governo israelense seguir com políticas intransi-
gentes em relação à Palestina e a intolerância e o extremismo colocarem o diálogo e o bom senso em
segundo plano, dificilmente haverá paz nesta região.

147
ATIVIDADES
1 - Entende-se por sionismo:
a) a intenção proeminente dos povos árabes de tentar erradicar os judeus do Oriente Médio.
b) a crença religiosa de que judeus e muçulmanos são povos excludentes e que jamais entrarão em paz.
c) a busca dos judeus pela Terra Prometida, nos arredores de Jerusalém, com a consequente cria-
ção de seu Estado-Nação.
d) o movimento de resistência dos judeus frente às constantes ameaças árabes promovidas em
todo o mundo.

2 - Quais são as áreas que correspondem os territórios ocupados por palestinos?

3 - Qual foi o objetivo da construção do Muro de Israel?

4 - O que significa a sigla OLP?

148
5 - Observe atentamente a charge abaixo e explique o seu significado.

Disponível em: https://1.bp.blogspot.com/--Sr9tARERxM/X5jSpqvzboI/AAAAAAAAe9I/cHa8P6dxfWI9X0_


ocpQYHwvBeCI5zC9JgCLcBGAsYHQ/s519/CHARGE.PNG Acesso em 14 ago. 2021.

149
SEMANA 3

EIXO TEMÁTICO:
Os cenários da globalização e fragmentação.

TEMA/TÓPICO:
As novas fronteiras do capitalismo global: os territórios nas novas regionalizações./ Desterritorialização e
redes de solidariedade.

HABILIDADE(S):
Avaliar as possibilidades de desterritorialização a partir de projetos de inclusão digital e de estratégias dos
migrantes.

CONTEÚDOS RELACIONADOS:
Fronteira, lugar, território, diáspora, migrações, refugiados.

INTERDISCIPLINARIDADE:
Língua Portuguesa, Arte, Sociologia, História, Filosofia.

TEMA: Globalização e fragmentação II


Caro (a) estudante, nesta semana você vai conhecer um pouco do povo rohingya uma minoria mulçu-
mana apátria de Mianmar e sofre perseguições pelo exército daquele país ao sul do continente asiático.

DIÁSPORA DO POVO ROHINGYA


Minoria islâmica num país de maioria budista, há décadas que os rohingya são perseguidos e mortos
pelo exército de Mianmar, especialmente no estado de Rakhine, o único do país em que estão em maio-
ria. O país de onde são originários não lhes reconhece cidadania nem direitos. O resto da população
vê-os como um povo menor.
Estas divergências têm raízes históricas e remontam ao período colonização britânica daquele país,
a antiga Birmânia. Em 1948, quando Mianmar conseguiu a independência, todos os rohingya viram
a sua cidadania negada para aquele país. Desde então que não têm direitos, como acesso a saúde,
acesso a educação, permissão para viajarem entre estados, para serem proprietários e até para se
casarem. Em 2014, foram mesmo excluídos dos censos, recusando reconhecer como parte do país
os que não afirmassem ser bengalis. Não pertencem legalmente a Mianmar nem a lado nenhum -
e ninguém os quer.

150
MAPA – LOCALIZAÇÃO GEOGRÁFICA DE MIANMMAR

Disponível em: <https://static.dw.com/image/40521460_105.png>. Acesso em: 14 ago. 2021.

Cerca de um milhão de pessoas dessa minoria, a maior comunidade no mundo, vivem em Mianmar, país
predominantemente budista. A maioria mora de forma precária no Estado de Rakhine, palco dos epi-
sódios recentes de violência. Segundo a ONU, a repressão militar contra o povo rohingya possui ca-
racterísticas de uma verdadeira limpeza étnica, tendo como consequências um êxodo em massa e um
possível genocídio.
Dada a proximidade física e a possibilidade de passagem por terra, é o Bangladesh o país que mais os
tem recebido desde que começaram a fuga, tempo que já remonta aos anos 70.
A origem da etnia rohingya ainda é incerta e gera controvérsias. Eles se autodeclaram um povo tradi-
cional ou indígena e possuem um dialeto próprio. Outros apontam que, na verdade, são mulçumanos de
origem bengali que migraram para Mianmar durante a colonização britânica.
Durante a independência do país, em 1948, muitos rohingya foram considerados cidadãos birmane-
ses. A partir de 1962, a Junta Militar que governou o país começou a cortar os direitos da comunidade.
Em 1982, com a Lei de Cidadania, os rohingya tiveram a nacionalidade birmanesa negada e hoje cons-
tituem a maior população apátrida do mundo. Essa condição os coloca à margem de muitos direitos
de cidadãos como ter acesso a escolas, votar, usar hospitais e direitos no mercado de trabalho. Eles
também não têm o direito de possuir terra ou propriedades e são proibidos de viajar sem permissão.
Há décadas essa minoria é vítima de perseguição e desconfiança. Tanto que a população rohingya pos-
sui um histórico de imigração para países como Bangladesh, Arábia Saudita e Índia. Nos últimos anos,
o avanço do nacionalismo budista aumentou a hostilidade e a discriminação.
Um recente relatório da ONU concluiu que existe uma operação sistemática para expulsar definitiva-
mente a população da região. As conclusões da investigação da ONU apontam abusos destinados a
“incutir um medo profundo e maciço” entre a população rohingya. Os relatos de refugiados contam que
militares incendiaram vilarejos, espalharam minas terrestres na fronteira com Bangladesh, realizaram
assassinatos em massa de civis e estupros contra mulheres e meninas.

151
Antes da explosão da violência, quase um milhão de muçulmanos rohingyas viviam em Mianmar, muitos
deles há várias gerações.
Além de cruzarem a fronteira a pé, os refugiados também tentam fugir pelo rio Naf, que estabelece a
fronteira entre Bangladesh e o extremo sudeste de Mianmar. Prevê-se que a população total desses as-
sentamentos em breve superará um milhão. Segundo a Unicef, cerca de 60% dos novos refugiados são
crianças. Bangladesh mantém uma política de acolhimento aos refugiados.

ATIVIDADES
1 - Recentemente, em Mianmar, a minoria étnica Rohingya foi vítima de graves massacres e assassinatos
praticados por grupos extremistas da maioria étnico-religiosa do país. Entre 2017 e 2018, os atos de
violência provocaram deslocamentos forçados de aproximadamente 650 mil Rohingyas principalmente
para um dos países vizinhos. Qual é o principal país para onde se deslocam os Rohingyas?

2 - Os rohingya fogem 
a) das Filipinas e cruzam a fronteira em direção à Índia.
b) de Mianmar e dirigem-se, principalmente, para Bangladesh. 
c) do Nepal e migram em massa para acampamentos na China.
d) da Indonésia e deslocam-se, principalmente, para o Paquistão.

3 - Por que os rohingya são apátridas?

152
4 - A crise humanitária envolvendo o povo Rohingya, considerada por muitos como a de maior gravidade
hoje no planeta, recentemente foi denunciada pela ONU como uma “limpeza étnica”, tendo o Alto
Comissariado para os Direitos Humanos utilizado o termo “genocídio” para qualificá-la. Sobre essa crise,
é CORRETO afirmar:
a) O povo Rohingya, uma minoria católica dentro do Estado de Mianmar, vem sendo massacrado
pelo exército daquele país. Tal situação provocou, inclusive, um encontro do Papa Francisco
com a ativista birmanesa, a nobel da paz, Augn San Suu Kyi.
b) A presidente Augn San Suu Kyi, renomada ativista pela democracia em Mianmar, Nobel da paz
em 1991, vem sendo alvo de várias críticas lançadas inclusive por outros agraciados pelo prêmio
Nobel da paz, entre os quais o arcebispo sul-africano Desmond Tutu e a ativista paquistanesa
Malala Yousafzai, diante da inação do seu governo frente à “limpeza étnica e os crimes contra a
humanidade” sofridos pelos Rohingyas.
c) Desde o início da nova onda de repressão ao povo Rohingya, em meados de 2017, a ONU calcula
que ao menos 655 mil Rohingya tenham buscado refúgio no Laos, país vizinho.
d) Os Rohingyas, povo muçulmano, diferem da maioria budista em sua cultura e religião. Habitam
predominantemente o Estado de Rakhine, em Mianmar, antiga Birmânia. Porém, não são reco-
nhecidos pelo governo como cidadãos nem, tampouco, como um dos mais de cem grupos étni-
cos que compõem o país.

5 - “CAMPO DE KUTUPALONG, Bangladesh, 05 de setembro de 2017 – De pés descalços e correndo para


salvar sua vida, a rohingya Dilara, de 20 anos, conseguiu chegar à Bangladesh recentemente com seu filho
mais novo no colo. Sua família foi devastada devido ao conflito em Mianmar. (...) Cerca de 123 mil mulheres,
crianças e homens como Dilara e seu filho chegaram em Bangladesh após dias caminhando, forçados a
fugir da violência em Rakhine, ao norte de Mianmar. Muitos estão famintos, em condições físicas precárias
e precisam de ajuda para salvar suas vidas”.
(ACNUR, www. acnur.org, acesso em 15/08/2021)

A respeito dessa notícia e do povo rohingya, assinale a alternativa CORRETA:


a) O povo rohingya é originário da Índia e do Paquistão e migrou para Mianmar durante o século XX
devido às perseguições que sofriam por serem muçulmanos. Em Mianmar, foram aceitos pelos
budistas que não temem sua religião, mas, nos últimos tempos, passaram a exigir a autonomia
do território em que vivem, causando o conflito.
b) Em Mianmar, a minoria rohingya é considerada como “imigrantes ilegais de Bangladesh”, trazi-
dos pelos colonizadores britânicos que governaram Mianmar até 1948. Porém, os historiadores
apontam para uma presença de muçulmanos na região que remonta ao século XI. Rohingya sig-
nifica “habitante de Rohang”, o nome pelo qual era conhecida Rakhine.
c) Os rohingya são um grupo muçulmano que migrou do Paquistão para Mianmar nos últimos vinte
anos com o objetivo de impor sua religião aos budistas da região de Rakhine, ao norte de Mian-
mar, sendo sustentados por grupos fundamentalistas do Paquistão que pretendem ampliar sua
ação em território asiático.
d) O exército de Mianmar é acusado de promover uma limpeza étnica com a perseguição e morte de
centenas de rohingyas, grupo minoritário muçulmano originário do norte do país e que, inconfor-
mados com a forma como são tratados pela população e governo, exigem a autonomia da região
de Rakhine.

153
SEMANA 4

EIXO TEMÁTICO:
Os cenários da globalização e fragmentação.

TEMA/TÓPICO:
As novas fronteiras do capitalismo global: os territórios nas novas regionalizações./ Desterritorialização e
redes de solidariedade.

HABILIDADE(S):
Avaliar as possibilidades de desterritorialização a partir de projetos de inclusão digital e de estratégias dos
migrantes.

CONTEÚDOS RELACIONADOS:
Fronteira, lugar, território, migrações, região, povo.

INTERDISCIPLINARIDADE:
Língua Portuguesa, Arte, Sociologia, História, Filosofia.

TEMA: Globalização e fragmentação III


Caro (a) estudante, nesta semana você vai compreender as principais barreiras à constituição do Cur-
distão como Estado Soberano.

ORIENTE MÉDIO: A QUESTÃO CURDA


Trinta milhões. Essa é aproximadamente a quantidade populacional de um povo que luta a mais de dois
séculos por reconhecimento étnico e geográfico.
O povo curdo é um grupo étnico que se julga nativo de uma região do Oriente Médio chamada de Curdis-
tão, que abrange parte dos territórios do Irã, Iraque, Síria e Turquia.
O povo curdo aspira conquistar independência política e territorial do Irã, Iraque, Síria e Turquia. A luta
pela autonomia desse povo vem sendo combatida de maneira violenta, especialmente pelo Iraque e
Turquia. No Iraque, os curdos estão estabelecidos em uma região rica em petróleo, por isso, na década
de 80, o ditador Saddam Hussein usou armas químicas para matar 5.000 curdos.

154
Disponível em: <https://ichef.bbci.co.uk/news/640/cpsprodpb/D119/production/_109192535_kurdish_settlements_portuguese_640.png>.
Acesso em: 14 ago. 2021.

Os curdos são a 4º etnia do Oriente Médio após árabes, persas e turcos. Já eram citados desde a An-
tiguidade pelo historiador grego Xenefonte, depois descritos pelo viajante Marco Polo no séc. 13 e em
livros árabes da Idade Média. Um dos grandes líderes muçulmanos durante as Cruzadas, Saladino, per-
tencia à etnia curda.
A maioria dos curdos do Oriente Médio vive na Turquia, 14 milhões de pessoas; Irã, 7 milhões; e Iraque, com
6 milhões. Países como Síria, Azerbaijão e Rússia, têm comunidades nativas de curdos. Na Europa, desta-
ca-se a Alemanha que possui uma comunidade de 1 milhão de curdos, na sua maioria de cidadania turca.
No início do século 20, muitos curdos começaram a considerar a criação de um Estado, geralmente conhe-
cido como Curdistão. Depois da Primeira Guerra Mundial e da derrota do Império Otomano, os países oci-
dentais vitoriosos fizeram uma provisão para a criação de um Estado curdo em 1920 no Tratado de Sèvres.
Mas os planos foram frustrados três anos depois, quando o Tratado de Lausanne, que estabeleceu as
fronteiras da Turquia moderna, não tratou de um Estado curdo e os deixou com status minoritário em
seus respectivos países. Nas oito décadas seguintes, qualquer ação dos curdos para estabelecer um
Estado independente foi brutalmente anulada.
No período de 1980/1988, durante a guerra Irã X Iraque, os curdos se viram no meio do conflito, devido à
sua localização. Aproveitando a ocasião, a Turquia intensificou os ataques aos curdos, pois era a chan-
ce de eliminar um povo indesejado.
Em resposta às duras repressões, os curdos organizaram-se em diversos grupos armados ligados ao
PKK (Partido dos Trabalhadores Curdos), reconhecidos como terroristas, que atuam muitas vezes por
meio da ativação de carros-bombas e atentados públicos contra os governos, principalmente na Turquia
e, mais recentemente, na crise da Síria. Essa instabilidade e a dura repressão do governo turco em re-
lação os curdos são, inclusive, uns dos entraves que o país possui para conseguir sua entrada na União
Europeia, que não admite governos tidos como antidemocráticos e que violam os direitos humanos.
Na Turquia, que abriga mais de 14 milhões de curdos, é proibido o estudo da língua curda nas institui-
ções de ensino. As perseguições se intensificaram a partir do século XX, pois o principal grupo separa-
tista, o Partido dos Trabalhadores do Curdistão, passou a reagir às repressões do governo turco, dando
início a uma luta armada. Esse conflito resultou na morte de mais de 40 mil pessoas, sendo a maioria da
etnia curda.

155
Aparentemente, os curdos estão longe de constituir o seu próprio Estado. Primeiramente, a atuação
do grupo PKK, considerado terrorista por muitos países, dificulta o reconhecimento da luta desse povo
no contexto internacional. Em segundo lugar, cabe destacar que o território reivindicado, além de fa-
zer parte de seis países, contém uma série de recursos naturais e nascentes de águas, isso em uma
região onde esse recurso é extremamente escasso, o que faz com que os países apresentem uma forte
resistência. Por fim, a falta de influência política dos curdos e o desinteresse das potências ocidentais
colocam a luta curda em um plano secundário na Geopolítica do Oriente Médio.

ATIVIDADES
1 - Dentre os principais conflitos e impasses étnico-territoriais na região do Oriente Médio, um deles se
caracteriza pela luta para a criação de um Estado Nacional que abrigaria aquela que é a maior nação
sem pátria do mundo. O texto se refere:
a) aos Curdos, que lutam pela criação do Curdistão.
b) aos Turcos, que tiveram seu território perdido desde a dissolução do Império Turco-Otomano.
c) aos povos da Mesopotâmia, que lutam em uma guerra civil sem trégua para a criação de seu
território.
d) aos povos bascos, que liderados pelo grupo terrorista ETA buscam a independência de seu território.

2 - Os curdos:
a) ocupam extensa área do território sírio onde estão as maiores densidades do grupo no Oriente
Médio; no Irã e na Turquia apresentam menor peso demográfico.
b) formam comunidades autônomas na Turquia e no Irã, no Iraque e na Síria, porém, são fortemente
reprimidos em razão da possibilidade de formar um Estado curdo.
c) formam o maior grupo de apátridas do mundo; representam cerca de 30 milhões que ocupam
terras de 4 países do Oriente Médio, entre os quais a Turquia, que abriga metade desse grupo.
d) constituem um numeroso grupo que vive disperso por sete países do Oriente Médio e se desta-
cam por ter sua identidade baseada na religião cristã, predominantemente católica.

3 - Trata-se de um grupo étnico que se configura como a maior nação sem pátria no mundo, ou seja,
sem um Estado constituído. No total, eles formam uma população superior a 30 milhões de habitantes.
Estamos falando dos:

a) Xiitas. b) Sunitas. c) Wahabitas. d) Curdos. e) Palestinos.

4 - Por que os curdos ainda não têm um Estado próprio?

156
5 - A questão curda é um dos principais temas do Oriente Médio. Quais são os países que a população
curda se faz presente?

6 - Em qual país encontramos a maior presença de Curdos?

157
SEMANA 5

EIXO TEMÁTICO:
Os cenários da globalização e fragmentação.

TEMA/TÓPICO:
As novas fronteiras do capitalismo global: os territórios nas novas regionalizações./ Fronteiras.

HABILIDADE(S):
Analisar as causas e os efeitos da migração clandestina nos países centrais e periféricos.

CONTEÚDOS RELACIONADOS:
Fronteira, lugar, território, migrações, refugiados, região.

INTERDISCIPLINARIDADE:
Língua Portuguesa, Arte, Sociologia, História, Filosofia.

TEMA: A África no mundo global I


Caro (a) estudante, nesta semana você vai compreender os fatores que contribuíram para fomentar
hostilidades entre o norte e o sul sudanês que culminaram no processo de independência.
NASCE UM NOVO PAÍS: O SUDÃO DO SUL
Mais novo país, o Sudão do Sul herdou, com a independência, muitos problemas sociais, financeiros e
um intenso conflito armado com o norte.
Em fevereiro de 2011, a população no Sudão – país localizado na região norte da África – foi às urnas para
definir, em referendo, a separação e emancipação da região na porção meridional do país. Com a apro-
vação de esmagadores 98,8% dos votantes, surgiu o até então mais novo país: o Sudão do Sul, tendo
como capital a cidade de Juba.
MAPA – LOCALIZAÇÃO GEOGRÁFICA DO SUDÃO DO SUL

Disponível em: <https://inverta.org/jornal/agencia/internacional/africa/os-interesses-imperialistas-presentes-na-partilha-do-sudao>.


Acesso em 14 ago. 2021.

158
Apesar das intensas celebrações nas ruas, não tardou para que a população percebesse que havia
muito pouco para comemorar, pois o novo país nascia com graves convulsões sociais e pesados desa-
fios a enfrentar.
Isso porque, desde o período da colonização do Continente Africano, as divisões territoriais foram fei-
tas pelos colonizadores. A junção de povos de diferentes etnias, por exemplo, refletiu em conflitos e
guerras civis. No caso do Sudão, não foi diferente. Quando o Sudão do Sul se tornou independente,
as demarcações territoriais também não foram bem estabelecidas.
Um dos fatores que gerava conflitos entre a população de ambos os países era a questão religiosa.
Isso porque, o sul é majoritariamente composto por cristãos e animistas, enquanto o norte possui
maioria islâmica.
Dessa forma, apesar das diferenças étnico-culturais ainda causaram algum atrito, o principal moti-
vo dos conflitos é em relação ao petróleo. O recurso natural é fonte econômica tanto do Sudão como
do Sudão do Sul. Com a separação dos países, a parte sul ficou com a maior região onde o petróleo
está concentrado.
Os dados são escassos, porém pouco animadores. Mais de 70% da população é analfabeta. As taxas de
mortalidade infantil também são elevadas e o número de mães que morrem durante os partos é alto.
Estima-se que aproximadamente 45% da população não possua acesso a nenhuma fonte de água po-
tável. A população sofre com a falta de hospitais – que na maioria dos casos oferecem péssimas condi-
ções estruturais e de higiene.
Muitas comunidades vivem em regiões isoladas, onde o acesso da ajuda humanitária é difícil ou impos-
sível por terra, agravando a situação de insegurança alimentar. Os números demonstram a dimensão
do problema e a gravidade da fome. Das 12 milhões de pessoas no Sudão do Sul, 3 milhões sofrem de
insegurança alimentar severa.
Para agravar a situação, a guerra e os constantes bombardeios – principalmente nas regiões de fronteira
– intensificam o número de mortos e refugiados, além de fazer com que o governo do sul invista quase
50% das riquezas do país em armas em detrimento de investimentos em educação e saúde. Os sul-suda-
neses estão vivendo no limbo, e os recentes confrontos em algumas partes do país continuam a deslocar
milhares de pessoas – que não poderão colher as suas safras e dependerão de ajuda humanitária.
O deslocamento forçado é uma das principais consequências do conflito no Sudão do Sul. Quase dois
milhões de pessoas fugiram pelas fronteiras do Sudão do Sul e outros dois milhões são deslocadas in-
ternas. Refugiados e deslocados passam a viver em difíceis condições, em abrigos improvisados, sob
árvores, campos abertos sem refúgio ou lugares isolados de difícil acesso.
De acordo com a Agência da ONU para os Refugiados (Acnur) os refugiados sul-sudaneses estão em
Uganda (698 mil), Etiópia (342 mil), Sudão (305 mil), Quênia (89 mil), República Democrática do Congo
(68 mil) e República Centro-Africana (4.900). Desde dezembro de 2013, com a deterioração da crise no
Sudão do Sul, mais de 2 milhões de sul-sudaneses foram obrigados a se deslocar para países vizinhos,
enquanto as estimativas são de que outros dois milhões permaneçam deslocados internamente.
Apesar de acordos feitos, inclusive pela Organização das Nações Unidas (ONU), os conflitos internos
que abalam o Sudão do Sul estão longe de acabar. Enquanto isso, a população sofre com falta de estru-
tura social, política e econômica, agravando a situação dos refugiados no país.

159
ATIVIDADES
1 - O Sudão do Sul é um país localizado na parte nordeste do Continente Africano. Sua área territorial é
de 644.330 km2 e com população de 12.531.000 habitantes.
Responda: Quais são os países que possuem suas fronteiras terrestres com o Sudão do Sul?

2 - Considere a informação a seguir. O referendo sobre a independência do Sudão do Sul realizou- -se
em de 2011. A quase totalidade dos eleitores votou pela separação do sul do Sudão. Assinale a alternativa
que caracteriza CORRETAMENTE o novo Estado africano. 
a) Exportador de carne bovina, e a forma de governo é a Monarquia. 
b) Baixa mortalidade infantil, e a maioria da população vive nos centros urbanos. 
c) Carência de reservas hídricas, e economia concentrada na extração de diamantes. 
d) Rico em petróleo, mas a maioria da população está abaixo da linha da pobreza. 

3 - Quais os fatores responsáveis pela guerra civil no Sudão do Sul?

4 - As manchetes que atualmente são publicadas sobre a África, como as apresentadas abaixo,
expressam o trágico quadro socioeconômico desse continente.

Assinale a opção que NÃO inclui um aspecto desse quadro.


a) A baixa expectativa de vida de grande parte da população.
b) Os conflitos e guerras tribais envolvendo nações africanas.
c) As guerras civis estimuladas pelas potências imperialistas europeias.
d) O contingente de africanos fora de seus países de origem, em busca de trabalho.

160
5 - A tabela a seguir mostra o número total de refugiados no mundo em 2017, segundo relatório do Alto
Comissariado das Nações Unidas Para Refugiados (UNHCR ou ACNUR em português).

Sobre os refugiados e sua distribuição no mundo, é CORRETO afirmar:


a) Os provenientes do Sudão do Sul e da Somália são acolhidos na Turquia, onde encontram oferta
de empregos nas atividades comerciais, tradição econômica do país, desde o século XVII.
b) A maioria provém da África, devido aos processos de desertificação, e tem como destino o
Oriente Médio e a Europa.
c) São majoritariamente provenientes do Oriente Médio, África e Ásia, deslocam‐se, forçadamente,
devido a longas guerras, em grande parte para países e/ou regiões fronteiriços.
d) O Irã recebe majoritariamente refugiados de países da África Subsaariana, dentre os quais se
destacam o Sudão e o Sudão do Sul.

161
SEMANA 6

EIXO TEMÁTICO:
Os cenários da globalização e fragmentação.

TEMA/TÓPICO:
As novas fronteiras do capitalismo global: os territórios nas novas regionalizações./ Fronteiras.

HABILIDADE(S):
Analisar as causas e os efeitos da migração clandestina nos países centrais e periféricos.

CONTEÚDOS RELACIONADOS:
Fronteira, lugar, território, migrações, refugiados, região.

INTERDISCIPLINARIDADE:
Língua Portuguesa, Arte, Sociologia, História, Filosofia.

TEMA: A África no mundo global II


Caro (a) estudante, nesta semana você vai conhecer um pouco sobre o Apartheid e sua rigorosa legisla-
ção separatista implantada durante anos na África do Sul e que atingiu todos os aspectos da sociedade.

A VIOLÊNCIA RACIAL E O APHARTEID


O termo apartheid se refere a uma política racial implantada na África do Sul (mapas abaixo). De acordo
com esse regime, a minoria branca, os únicos com direito a voto, detinha todo poder político e eco-
nômico no país, enquanto à imensa maioria negra restava a obrigação de obedecer rigorosamente à
legislação separatista.


Disponível em: <http://historiaeculturandc.blogspot.com/2010/06/africa-do-sul-um-pequeno-pais-uma.html>.
Acesso em: 14 ago. 2021.

Apartheid significa “separação” em africâner, língua falada na África do Sul cujas origens remetem ao
idioma neerlandês, dos holandeses. O apartheid foi um sistema de segregação racial instituído na Áfri-
ca do Sul em 1948 pelas elites brancas que controlavam o país e sustentava-se no mito da superioridade
racial europeia. Baseando-se na crença de que os brancos europeus eram superiores aos negros e ou-
tras etnias, os brancos acreditavam que deveriam viver separados.

162
O apartheid ficou famoso no mundo todo pelas duras leis segregacionistas impostas à sociedade sul-
-africana. Durante os períodos de vigência desse sistema, os não brancos eram proibidos de frequentar
os mesmos lugares que os brancos, de ter a posse de terras, de circular livremente pelo território e é
claro, de participar das decisões políticas do país.
Entre os séculos XVII e XIX, o território sul-africano foi alvo de disputas entre Holanda e o Reino Unido,
interessados principalmente nas minas de ouro e diamantes existentes na região. O país foi dominado
primeiramente pelos holandeses e posteriormente pelos britânicos, que assumiram o poder na região
em 1902, após vencer a Guerra dos Bôeres. Foi em 1910 que Louis Botha, o então Primeiro-Ministro da
União da África do Sul, adotou as primeiras leis de segregação racial.
Entre as primeiras leis de segregação e a instituição do apartheid – que transformou o racismo em lei
– algumas normas foram adotadas e contribuíram para a construção do que viria a ser um dos ordena-
mentos jurídicos mais truculentos da humanidade.
Diversas outras leis foram criadas durante o regime do apartheid. Todas elas tinham o intuito de man-
ter a população não branca em condições precárias e posições sociais marginalizadas. Dessa forma,
garantia-se à elite branca seu poder econômico, político e militar e impedia-se a ascensão social das
raças consideradas “inferiores”.
Os negros precisavam andar com cadernetas por determinação da Lei dos nativos, conhecida como
Lei do passe, o documento deveria ser apresentado à polícia sempre que solicitado. Os locais e equipa-
mentos públicos também eram segregados, placas com a indicação “Somente europeus” eram coloca-
das para impedir o contato com negros.
A divisão entre raças pode ser observada já em 1913 com a Lei das Terras, que dá 90% das terras aos
brancos e apenas 10% aos negros. Anos mais tarde, durante o apartheid, são criadas pequenas pátrias,
os bantustões, para separar as raças. Os bantustões foram pseudo-estados de base tribal criados pelo
regime de forma a manter os negros fora dos bairros e terras brancas, mas suficientemente perto delas
para servirem de fontes de mão-de-obra barata.
Uma das justificativas do regime do apartheid é a concepção do desenvolvimento separado, que expli-
ca que cada raça deve se desenvolver sem a mistura com outras. Com isso, os negros recebiam servi-
ços públicos inferiores, inclusive a educação oferecida para os brancos tinha qualidade superior.
A oposição ao apartheid teve início de forma mais intensa na década de 1950, quando o Congresso
Nacional Africano (CNA), organização negra criada em 1912, lançou uma desobediência civil. Em 1960,
a polícia matou 67 negros que participavam de uma manifestação. O Massacre de Sharpeville, como
ficou conhecido, provocou protestos em diversas partes do mundo. Como consequência, a CNA foi de-
clarada ilegal e seu líder, Nelson Mandela, foi preso em 1962 e condenado à prisão perpétua.
O personagem mais icônico que lutou contra regime foi Nelson Mandela (1918-2013). Mandela militou no
Congresso Nacional Africano, liderando a ala jovem do partido, realizando atos de sabotagem e promo-
vendo greves.
Através das pressões por parte dos negros segregados e também o embargo econômico implantado
pelo governo norte-americano em 1986, o então presidente Frederik de Klerk alterou as leis que res-
tringia a participação negra e reformulou em sua totalidade sua essência racista, mas a extinção do
Apartheid teve fim somente em 1994, quando Nelson Mandela alcançou a presidência.
Mandela deixou bem claro que sua intenção não era criar outro regime segregacionista, separando a
minoria branca da maioria negra. A ideia era proporcionar uma sociedade igualitária, independente do
credo, religião, cor, etc.

163
Para tanto, estudos apontam que o problema da segregação racial é uma realidade na África do Sul.
Além dos negros é importante ressaltar que imigrantes chineses e indianos, sofreram também com o
regime do apartheid, e ainda hoje, lidam com a xenofobia no país.
No entanto, problemas como a miséria, a injustiça e a desigualdade entre brancos e negros ainda são
comuns no país.

ATIVIDADES
1 - Várias políticas ditatoriais foram implantadas no continente africano. O apartheid foi uma política
racial imposta pela minoria branca da população, únicos com direito a voto, restando à maioria da
população, composta por negros, obedecer rigorosamente às leis separatistas. Em qual país da África
foi implantado o apartheid?
a) Angola.
b) Nigéria.
c) Camarões.
d) África do Sul.

2 - Em que ano se iniciou o apartheid, e quais as reações da população local em relação a essa política
segregacionista racial?

3 - A repercussão negativa ao regime político racial instalado na África do Sul envolveu a maioria da
população local (negros) e organismos internacionais como, por exemplo, a Organização das Nações
Unidas (ONU). Várias manifestações foram realizadas contra o apartheid, sendo o principal líder
desses movimentos:
a) Mahatma Gandhi.
b) Frederick de Klerk.
c) Thabo Mbeki.
d) Nelson Mandela.

4 - Em 1912, dois anos após a formação da União Sul-africana, uma organização política formada por
bôeres (descendentes de holandeses) e subjugada à autoridade britânica, uma importante organização
de negros sul-africanos foi fundada.
Que nome era dado a essa organização política?

164
5 - O que eram os bantustões?

6 - Nelson Mandela, ex-presidente da República da África do Sul (1994-2000), ganhou o prêmio Nobel da
Paz, em conjunto com Frederik de Klerk, em 1993, e hoje é nome de rua em Paris, Rio de Janeiro, Dacar
e em Dar Es-Salam; é nome de praça em Salvador, Haia, Glasgow e em Valência; é nome de escola em
Bangalore, Berlim, Birmingham e em Baton Rouge.
Essa extraordinária popularidade de Nelson Mandela deve–se, sobretudo,
a) aos vinte anos que passou injustamente encarcerado pelo regime racista então vigente na África
do Sul.
b) à sua campanha incansável em favor dos milhões de africanos vitimados pela Aids e deixados
sem assistência.
c) ao fim, negociado e sem revanchismo, do regime do apartheid e ao seu desprendimento com
relação ao poder.
d) à sua luta contra o imperialismo e em favor da independência de todos os países do continente
africano.

Caro (a) estudante, espera-se que tenha tido sucesso na resolução do PET - 04 de Geografia. Saiba
que, ao longo do ano, você irá entender e compreender sobre a complexidade das paisagens, das
atividades humanas, das sociedades e do nível de desenvolvimento entre os países, as transforma-
ções tecnológicas, o modo como a sociedade se relaciona com a natureza e as formas de organização
espacial. A Geografia tem muito a contribuir para a compreensão do espaço mundial, cada vez mais
complexo, cujas transformações são cada vez mais surpreendentes.

REFERÊNCIAS:
SENE, Eustáquio de; MOREIRA, João Carlos. Geografia geral e do Brasil: espaço geográfico e glo-
balização. 2 ed. São Paulo: Scipione. 2014.
FREITAS, Eduardo de. “Povo curdo”; Brasil Escola. Disponível em: https://brasilescola.uol.com.br/
geografia/povo-curdo.htm. Acesso em 18 de agosto de 2021.
ADAS, M; ADAS, S. Expedições Geográficas: 8° ano. 3° Edição. São Paulo: Moderna, 2018.
PENA, Rodolfo F. Alves. “Questão curda”; Brasil Escola. Disponível em: https://brasilescola.uol.com.
br/geografia/questao-curda.htm. Acesso em 17 de agosto de 2021
FRANCISCO, Wagner de Cerqueira e. “Apartheid”; Brasil Escola. Disponível em: https://brasilescola.
uol.com.br/geografia/apartheid.htm. Acesso em 17 de agosto de 2021.

165
SECRETARIA DE ESTADO DE EDUCAÇÃO DE MINAS GERAIS

PLANO DE ESTUDO TUTORADO


COMPONENTE CURRICULAR: HISTÓRIA
ANO DE ESCOLARIDADE: 2º ANO – EM
PET VOLUME: 04/2021
NOME DA ESCOLA:
ESTUDANTE:
TURMA: TURNO:
BIMESTRE: 4º TOTAL DE SEMANAS:
NÚMERO DE AULAS POR SEMANA: NÚMERO DE AULAS POR MÊS:

SEMANA 1

EIXO TEMÁTICO:
II- Cultura e Política na Construção do Estado Nacional Brasileiro (1822-1930).

TEMA/TÓPICO:
Embates Políticos e Culturais no Processo de Construção e Afirmação do Estado Nacional. Fim da Monarquia
no Brasil e início da República.

HABILIDADE(S):
Analisar o conceito de liberalismo, suas apropriações no Império e suas reapropriações ao longo da história
brasileira.

CONTEÚDOS RELACIONADOS:
Liberalismo.

INTERDISCIPLINARIDADE:
Filosofia.

TEMA: Embates Políticos e Culturais no Processo de Construção e Afirmação do Estado Nacional


Caro(a) estudante, nesta semana você vai avaliar o conceito de liberalismo e suas influências na his-
tória do Brasil. Um Estado nacional contemporâneo tem como princípio realizar a soberania política e
militar dentro de um determinado território delimitado por fronteiras que definem quando termina um
território e inicia outro. O liberalismo é uma filosofia política e moral baseada em liberdade e igualdade,
defendendo a manutenção dos direitos individuais. Tais ideias surgiram sobretudo com a investida de
pensadores iluministas no século XVIII, como John Locke e Montesquieu[1] e foram pouco a pouco as-
similadas por setores específicos da sociedade no decorrer da história brasileira. Com a Proclamação
da República em 1889, o movimento liberal em sua organização tradicional desapareceu por um longo
período, muito pelo caráter distante das ideias liberais que foi esse momento de transição no Brasil.

166
APRESENTAÇÃO
A construção do Estado Brasileiro se iniciou com a vinda da Família Real para a América Portuguesa em
1808 e o estabelecimento do governo de D. João VI (o período Joanino) e se desenvolveu ao longo dos
anos seguintes, até a abdicação de D.Pedro I, em 1831. O principal motivo da vinda da Família Real para a
América Portuguesa, foi o Bloqueio continental napoleônico, que tinha o objetivo de dinamizar a indús-
tria francesa e acabar com a supremacia de sua principal rival, a Inglaterra. Quem desobedecesse seu
comando, seria invadido pelo exército francês. Essa transferência representou o aumento do domínio
inglês sobre Portugal e o início do processo de independência do Brasil. Já no Brasil, D. João VI realizou
várias medidas que causaram impacto na sociedade colonial portuguesa, como a abertura dos portos,
representando o fim do pacto colonial, além da permissão de manufaturas, que havia sido proibida por
D. Maria I, no alvará de 1785, e ainda a realização de reformas urbanas e culturais que ocorreram no Rio
de Janeiro para abrigar a Corte portuguesa, como a criação de instituições de ensino (Escola de Comér-
cio e a Academia Militar), Imprensa Régia e da Academia de Belas-Artes, marcada pela vinda de artistas
franceses que compuseram a “Missão Artística Francesa”.
A transferência dos principais órgãos da administração do Estado português para o Brasil (ministérios
do Reino, da Marinha e Ultramar), chamado de Transplante do Estado Português. Assim sendo, o centro
de decisões foi interiorizado no Brasil, deixando de ser em Portugal; e um dos mais importantes, o que
elevou o Brasil a Reino Unido a Portugal e Algarves(1815) devido às pressões do Congresso de Viena.
As principais revoluções ocorridas durante o Período Joanino foram a Revolução Pernambucana (1817),
movimento com influências iluministas, composto por senhores rurais e homens livres que se manifes-
tavam contra aos altos impostos gerados pelos vultuosos gastos da Corte no RJ e que desejavam maior
participação política; e a Revolução Liberal do Porto (1820), movimento organizado pela burguesia portu-
guesa, influenciado pela Revolução Francesa, que exigia o retorno imediato de D. João VI, a elaboração
de uma constituição liberal e um projeto de recolonização para o Brasil. Dessa forma, D. João VI foi para
Portugal deixando D. Pedro, seu filho, como príncipe regente no Brasil. Nesse contexto ocorreu uma ci-
são política no Brasil: Partido Português (funcionários públicos, militares e comerciantes portugueses)
que concordava com a política das Cortes de Portugal x Partido Brasileiro (grandes latifundiários) que
defendia a manutenção do liberalismo comercial. O Partido Brasileiro, temendo uma fragmentação polí-
tica, se aglutinou em torno de D. Pedro que declarou a Independência do Brasil.

PARA SABER MAIS:


Liberalismo no Brasil. Disponível em: https://pt.wikipedia.org/wiki/Liberalismo_no_Brasil. Acesso em:
09 ago. 2021.
Sugestão de Livro: História das cavernas ao terceiro milênio. Autores: Patrícia Ramos Braick e Myriam
Becho Mota.

ATIVIDADES
1 - (ENEM 2010) Leia o texto:
“Eu, o Príncipe Regente, faço saber aos que o presente Alvará virem: que desejando promover e adian-
tar a riqueza nacional, e sendo um dos mananciais dela as manufaturas e a indústria, sou servido abolir
e revogar toda e qualquer proibição que haja a este respeito no Estado do Brasil”.
(Alvará de liberdade para as indústrias (1º de Abril de 1808). In: Bonavides, P.; Amaral, R. Textos políticos da História do Brasil. Vol. 1.
Brasília: Senado Federal, 2002 (adaptado).

167
O projeto industrial de D. João, conforme expresso no alvará, não se concretizou. Que características
desse período explicam esse fato?
a) A ocupação de Portugal pelas tropas francesas e o fechamento das manufaturas portuguesas.
b) A dependência portuguesa da Inglaterra e o predomínio industrial inglês sobre suas redes de
comércio.
c) A desconfiança da burguesia industrial colonial diante da chegada da família real portuguesa.
d) O confronto entre a França e a Inglaterra e a posição dúbia assumida por Portugal no comércio
internacional.
e) O atraso industrial da colônia provocado pela perda de mercados para as indústrias portuguesas.

2 - (Fuvest) Qual o papel conferido ao Imperador pela Constituição de 1824?


a) Subordinação ao poder legislativo.
b) Instrumento da descentralização político-administrativa.
c) Chave de toda a organização política.
d) Articulador da extinção do Padroado.
e) Liderança do Partido Liberal.

3 - (Fuvest-SP) “... quando o príncipe regente português, D. João, chegou de malas e bagagens para
residir no Brasil, houve um grande alvoroço na cidade do Rio de Janeiro. Afinal era a própria encarnação
do rei (...) que aqui desembarcava. D. João não precisou, porém, caminhar muito para alojar-se. Logo
em frente ao cais estava localizado o Palácio dos Vice-Reis”. (Lilia Schwarcz. As Barbas do Imperador.)
O significado da chegada de D. João ao Rio de Janeiro pode ser resumido como:
a) decorrência da loucura da rainha Dona Maria I, que não conseguia se impor no contexto político
europeu.
b) fruto das derrotas militares sofridas pelos portugueses ante os exércitos britânicos e de Napo-
leão Bonaparte.
c) inversão da relação entre metrópole e colônia, já que a sede política do império passava do cen-
tro para a periferia.
d) alteração da relação política entre monarcas e vice-reis, pois estes passaram a controlar o man-
do a partir das colônias.
e) imposição do comércio britânico, que precisava do deslocamento do eixo político para conse-
guir isenções alfandegárias.

168
SEMANA 2

EIXO TEMÁTICO:
II- Cultura e Política na Construção do Estado Nacional Brasileiro (1822-1930).

TEMA/TÓPICO:
Confrontos: fim da monarquia no Brasil e início da República.

HABILIDADE(S):
Confrontar os conceitos de monarquia e república.

CONTEÚDOS RELACIONADOS:
Período Monárquico e Brasil República.

INTERDISCIPLINARIDADE:
Filosofia.

TEMA: Confrontos: fim da monarquia no Brasil e início da República


Caro(a) estudante, nesta semana você vai comparar os períodos da história brasileira final da monar-
quia e início da República, as alterações e novidades, trazidas por cada época.
O Brasil é o único país das Américas que teve um regime monárquico estável e reconhecido mundialmen-
te. Haiti e México também tiveram suas experiências monárquicas, mas estas duraram pouco, tendo ne-
nhuma relevância na história de ambos. O período monárquico brasileiro, que durou de 1822, ano da in-
dependência do país, até 1889, quando foi proclamada a República dos Estados Unidos do Brasil, marcou
de forma indelével a história nacional. Escolheu-se a denominação Império do Brasil para o novo país que
nascia pelo fato de que o monarca reinava sob uma gama variada de súditos, considerando-se as grandes
diferenças que caracterizavam e ainda caracterizam o país. Assim, o imperador tinha súditos brancos de
origem europeia, índios nativos estabelecidos nos confins da selva amazônica, africanos e descendentes
de africanos livres ou escravos, imigrantes alemães, recém-chegados ao país, e assim por diante.

APRESENTAÇÃO
A partir de 1870, a monarquia brasileira passou a ser questionada por diversos setores da sociedade
brasileira. Esse fato, aliado a outros acontecimentos, foi desgastando essa forma de governo. Integran-
tes da elite, da camada média e da população mais pobre passaram a defender o fim da monarquia e a
implantação da república. As ideias republicanas ganharam força a partir do final do século XIX, porém,
desde o século XVIII alguns movimentos rebeldes, como a Inconfidência Mineira, a Conjuração Baiana,
a Revolução Pernambucana, a Confederação do Equador e a Guerra dos Farrapos, já haviam defendido
a implantação da república no Brasil. Por que um regime de governo que vinha sendo adotado há tanto
tempo no Brasil já não servia mais? Vários foram os motivos, mas o principal foi a restrição à partici-
pação popular nas decisões políticas. O dia 15 de novembro de 1889 marcou o fim do império e o início
da república no Brasil. Houve uma transformação significativa. Em 7 de setembro de 1822, D. Pedro
proclamou a independência do Brasil e, ao contrário de outros países do continente, adotou a monar-
quia como forma de governo. O período em que D. Pedro I governou (1822-1831) foi chamado de Primeiro
Reinado. Em 1840 começou o Segundo Reinado, sendo D. Pedro II o imperador. Nesse período, o Brasil
passou por diversas modificações internas, começando o reaquecimento da economia por meio do cul-
tivo do café, que passou a ser o principal produto gerador de riqueza.

169
A estrutura econômica brasileira continuava a mesma do Período Colonial, ou seja, o latifúndio predo-
minava como propriedade rural, a mão de obra continuava sendo escrava negra, e mantinha-se a mono-
cultura voltada para a exportação.
As pessoas, descontentes com o regime monárquico, fundaram associações que defendiam maior li-
berdade para as províncias, eleições diretas e o direito de votar para todos os homens maiores de 18
anos e alfabetizados. Também defendiam que os senadores e outros políticos tivessem mandato com
tempo preestabelecido para não ocuparem os cargos a vida toda.
Ideias republicanas circulavam desde o Período Colonial. Faltava apenas uma conjuntura histórica pro-
pícia para derrubar a monarquia.
A criação do Partido Republicano Paulista (PRP), em 1873, marcou a adesão de importantes represen-
tantes da cafeicultura paulistana. O sucesso da proclamação da República deve-se também ao apoio
das camadas econômicas notáveis da sociedade brasileira.
O governo imperial procurou reverter o processo de desgaste da monarquia. A intenção era apresentar
propostas de caráter liberal que inibissem os ânimos republicanos.
As propostas não foram bem aceitas pelo Partido Conservador, maioria na Câmara dos deputados, e a
crise imperial, que se agravaram, além das questões da abolição da escravatura, militares e religiosas,
chegou ao extremo. O Exército foi influenciado pelo pensamento positivista, uma escola filosófica ins-
tituída pelo francês Auguste Comte, que defendia uma ditadura republicana, um governo representati-
vo, em que o ditador poderia afastar-se dos interesses da população pelo “bem da república”.
Um dos lemas positivistas, o amor por princípio, a ordem por base e o progresso por fim, está presente
na bandeira nacional, marcando os ideais positivistas dos republicanos — especialmente dos militares.
Estes lideraram o golpe contra a monarquia.

PARA SABER MAIS:


Embates políticos no processo de construção e afirmação do Estado Nacional. Disponível em: <https://
domingosjustinoribeiro.com.br/2020/07/06/embates-politicos-no-processo-de-construcao-e-afir-
macao-do-estado-nacional-historia-se>. Acesso em: 09 ago. 2021.
Sugestão de Livro: História das cavernas ao terceiro milênio. Autores: Patrícia Ramos Braick e Myriam
Becho Mota.

ATIVIDADES
1 - (FGV) Sobre a formação do absolutismo na França, é incorreto afirmar que:
a) seus antecedentes situam-se, também, nos reinados de Felipe Augusto, Luís IX e Felipe IV, entre
os séculos XII e XIV.
b) fez-se necessária nesse processo a centralização dos exércitos, dos impostos, da justiça e das
questões eclesiásticas.
c) a abolição da soberania dos nobres feudais não teve um importante papel nesse contexto.
d) a Guerra dos Cem Anos foi fundamental nesse processo.
e) durante esse processo que a aliança com a burguesia fez-se necessária para conter e controlar
a resistência de nobres feudais.

170
2 - (Mackenzie) Sobre a participação dos militares na Proclamação da República, é correto afirmar que:
a) o Partido Republicano foi influenciado pelos imigrantes anarquistas a desenvolver a consciência
política no seio do exército.
b) a proibição de debates políticos e militares pela imprensa, a influência das ideias de Augusto
Comte e o descaso do Imperador para com o exército favoreceram a derrubada do Império.
c) o descaso de membros do Partido Republicano, como Sena Madureira e Cunha Matos, em rela-
ção ao exército, expresso por meio da imprensa, levou os “casacas” a proclamarem a República.
d) O Gabinete do Visconde de Ouro Preto formalizou uma aliança pró-republicana com os militares
positivistas no Baile da Ilha Fiscal.
e) a aliança dos militares com a Igreja acirrou as divergências entre militares e republicanos, cul-
minando na Questão Militar.

3 - (VUNESP) Na Primeira República (1889-1930) houve a reprodução de muitos aspectos da estrutura


econômica e social constituída nos séculos anteriores. Noutros termos, no final do século XIX e início do
XX conviveram, simultaneamente, transformações e permanências históricas. (Francisco de Oliveira.
Herança econômica do Segundo Império, 1985.)
O texto sustenta que a Primeira República brasileira foi caracterizada por permanências e mudanças
históricas. De maneira geral, o período republicano, iniciado em 1889 e que se estendeu até 1930, foi
caracterizado:
a) pela predominância dos interesses dos industriais, com a exportação de bens duráveis e de capital.
b) por conflitos no campo, com o avanço do movimento de reforma agrária liderado pelos antigos
monarquistas.
c) pelo poder político da oligarquia rural e pela economia de exportação de produtos primários.
d) pela instituição de uma democracia socialista graças à pressão exercida pelos operários anarquistas.
e) pelo planejamento econômico feito pelo Estado, que protegia os preços dos produtos manufa-
turados.

171
SEMANA 3

EIXO TEMÁTICO:
II- Cultura e Política na Construção do Estado Nacional Brasileiro (1822-1930).

TEMA/TÓPICO:
Confrontos: fim da monarquia no Brasil e início da República.

HABILIDADE(S):
Comparar a Constituição do Império (1824) com a primeira Constituição Republicana (1891): o que se explicita,
o que se silencia, avanços e recuos dos direitos de cidadania.

CONTEÚDOS RELACIONADOS:
Direitos de cidadania.

INTERDISCIPLINARIDADE:
Filosofia.

TEMA: Principais diferenças entre as Constituições de 1891 e a Constituição Imperial de 1824


Caro(a) estudante, nesta semana você vai comparar a Constituição Imperial de 1824 e a Constituição de
1891, analisando suas características e diferenças.
A Constituição Brasileira de 1824 foi outorgada por Dom Pedro I em 25 de março de 1824. A primeira
Carta Magna brasileira garantia a unidade territorial, instituía a divisão do governo em quatro poderes
e estabelecia o voto censitário (voto ligado à renda do cidadão). Foi elaborada por um grupo reduzido
devido às desavenças entre o Imperador e a Assembleia Nacional Constituinte.
Após a proclamação da independência e da aclamação de Dom Pedro I como imperador do Brasil, o país
precisava organizar sua estrutura política e administrativa.
Já a Constituição de 1891 foi a primeira Constituição do Brasil da era republicana. Teve como caracte-
rísticas a instituição do regime republicano presidencialista e a separação entre o Estado e a Igreja.
O Brasil passava por um momento de transição do regime monárquico para o republicano. Desta manei-
ra, o governo precisava mudar a Carta Magna que regia o país desde 1824 e criar uma Constituição que
ajustasse à nova realidade.
Foi escolhida uma Assembleia legislativa que elaborou a nova Constituição em três meses. Na verdade,
grande parte da redação ficou a cargo dos juristas Rui Barbosa e Prudente de Morais.
A nova Constituição se inspirou, dentre outras, na Carta Magna dos Estados Unidos, tendo como eixo
a federalização dos Estados e a descentralização do poder. Inclusive o nome do novo país recebeu in-
fluência americana, pois foi denominado “Estados Unidos do Brasil”.
Em 24 de fevereiro de 1891 foi aprovada e promulgada a nova Constituição do Brasil. Esta seria alterada
em 1926 e revogada quatro anos mais tarde por causa da Revolução de 1930.
APRESENTAÇÃO
A maior diferença entre a Constituição de 1891 e a Constituição Imperial de 1824, era a Laicidade: Esta-
do e Igreja estavam separados oficialmente, e o Brasil passou a não ter religião oficial.
Também a ampliação do direito ao voto, apesar de ainda muito limitado, agora haveria maior participa-
ção política, porque a relação entre voto e renda havia sido rompida.

172
A Constituição outorgada em 25 de março de 1824, foi marcada por forte centralismo administrativo e
político, tendo em vista a figura do Poder Moderador.
A Constituição de 1891 estruturou o federalismo brasileiro segundo o modelo norte-americano. Ajustou
a um sistema jurídico-constitucional estrangeiro uma realidade completamente diversa. Daí resultou
que a Constituição escrita não pôde reproduzir, como não reproduziu, a Constituição real do país.
O Brasil passava por um momento de transição do regime monárquico para o republicano. Desta manei-
ra, o governo precisava mudar a Carta Magna que regia o país desde 1824 e criar uma Constituição que
ajustasse à nova realidade. Foi escolhida uma Assembleia legislativa que elaborou a nova Constituição
em três meses. Na verdade, grande parte da redação ficou a cargo dos juristas Rui Barbosa e Prudente
de Morais.
A nova Constituição se inspirou, dentre outras, na Carta Magna dos Estados Unidos, tendo como eixo
a federalização dos Estados e a descentralização do poder. Inclusive o nome do novo país recebeu in-
fluência americana, pois foi denominado “Estados Unidos do Brasil”.
Em 24 de fevereiro de 1891 foi aprovada e promulgada a nova Constituição do Brasil. Esta seria alterada
em 1926 e revogada quatro anos mais tarde por causa da Revolução de 1930.

PARA SABER MAIS:


Acesse o link: Estado Republicano. Disponível em:<https://www.todamateria.com.br/estado-republi-
cano/>. Acesso em: 09 ago. 2021.

ATIVIDADES
1 - Por meio da Constituição de 1824, foi instituído o Poder Moderador. Entre as características desse
poder, estava:
a) nomear apenas os membros do Poder Judiciário.
b) nomear e destituir os ministros do Poder Executivo.
c) não interferir na composição e na dissolução da Câmara dos Deputados.
d) garantir toda autonomia aos três poderes.
e) não interferir em nenhuma das esferas legislativas do poder.

2 - (FUVEST) O sistema eleitoral adotado no Império brasileiro estabelecia o voto censitário. Essa
afirmação significa que:
a) o sufrágio era indireto no que se referia às eleições gerais.
b) Para ser eleitor era necessário possuir determinada renda anual.
c) as eleições eram efetuadas em dois turnos sucessivos.
d) o voto não era extensivo aos analfabetos e às mulheres.
e) por ocasião das eleições, realizava-se o recenseamento geral da população.

173
3 - Em 15 de novembro de 1889, o Brasil elaborou uma nova constituição que foi promulgada no dia 24 de
fevereiro de 1891. Marque a alternativa correta sobre esse momento histórico do Brasil.
a) A Constituição de 1891 decretou a soberania da Igreja Católica no Brasil, ficando o Estado sub-
misso às ordens papais.
b) Essa Constituição decretou uma política reformista que contribuiu para o fim do coronelismo e
dos latifundiários no Brasil.
c) Essa nova constituição adotou a forma de governo presidencialista, garantindo às antigas pro-
víncias mais autonomia política.
d) A nova constituição adotou o presidencialismo como forma de governo, mas não possibilitou
mais autonomia política para os estados, que se afundaram em revoltas contra a República.

4 - Marque a alternativa que contém algumas das mudanças provocadas pela Constituição de 1891.
a) O Estado brasileiro passou a ter três poderes independentes: Executivo, Legislativo e Judiciá-
rio. Além disso, eliminou-se o voto censitário. Homens a partir de 21 anos poderiam votar, porém
analfabetos e mulheres continuaram sem o direito ao voto.
b) O Estado brasileiro passou a ter três poderes independentes: Executivo, Legislativo e Judiciário.
O voto censitário permaneceu para os eleitores e as mulheres passaram a ter o direito ao voto.
c) O regime político passou a ser o presidencialista. A maioria da população passou a ter acesso ao
voto livre e secreto, e o Estado adotou um regime dividido em três poderes: Executivo, Legisla-
tivo e Judiciário.
d) Excetuando as mulheres, o restante da população com idade acima de 21 anos poderia votar. A
Igreja passou a ter maior controle sobre os patrimônios públicos e sobre o Estado. O voto passou
a ser secreto.

174
SEMANA 4
EIXO TEMÁTICO:
II - Cultura e Política na Construção do Estado Nacional Brasileiro (1822-1930).
TEMA/TÓPICO:
Embates Políticos e Culturais no Processo de Construção e Afirmação do Estado Nacional.
HABILIDADE(S):
Analisar o movimento abolicionista e republicano, suas características e efeitos sobre a sociedade brasileira.
CONTEÚDOS RELACIONADOS:
Afirmação do Estado Nacional.
INTERDISCIPLINARIDADE:
Literatura.

TEMA: Movimento abolicionista e Movimento republicano sobre a sociedade brasileira.


Caro(a) estudante, nesta semana você vai discutir o significado histórico dos movimentos abolicionista
e republicano. O movimento abolicionista no Brasil agrupou pessoas de diferentes classes sociais que
agiram de diversas maneiras na luta pela abolição da escravatura. Incluída no movimento abolicionista
não está apenas a ação da parcela livre da sociedade mas também a resistência dos escravos.
O movimento republicano trouxe uma série de novos atores e novas ideias políticas, estruturados ofi-
cialmente a partir de 1870, quando foi lançado o Manifesto Republicano. Ao redor das ideias republica-
nas, formou-se um grupo consistente que organizou um golpe contra a monarquia em 1889.

APRESENTAÇÃO
A aprovação da Lei Áurea foi uma conquista popular, fruto do crescimento que o movimento abolicio-
nista teve na segunda metade do século XIX. O movimento abolicionista cresceu de maneira notável a
partir da década de 1870, e nesse período uma série de associações abolicionistas surgiu em diferentes
partes do país. A ação desses grupos era diversa, e o crescimento do debate abolicionista alcançou a
política, apesar da grande resistência existente contra a abolição. Entre as personalidades de destaque
que atuaram pelo fim da escravidão no Brasil estão: André Rebouças, Luís Gama, José do Patrocínio,
Joaquim Nabuco, Castro Alves, entre outras.
O crescimento da causa abolicionista fez a criação de associações desse cunho disparar entre 1878 e
1885, e assim surgiram 227 sociedades abolicionistas. Esses grupos pró-abolição organizavam confe-
rências, comícios na rua, distribuíram panfletos, publicaram artigos em jornais, e também ajudavam
na resistência organizando rotas de fugas para escravizados, abrigando-os, incentivando-os a fugi-
rem etc. Os escravizados que fugiam, muitas vezes, mudavam-se para as cidades e misturavam-se
à multidão de negros libertos e escravizados que já habitavam esses locais. Outros, por sua vez, pre-
feriam mudar-se para os quilombos — redutos que abrigavam escravizados fugidos. No final do sé-
culo XIX, surgiram quilombos em diferentes partes do Brasil, sobretudo ao redor de cidades como o
Rio de Janeiro.

175
Já o movimento republicano no Brasil, que ocorre por quatro anos, podendo ser renovado por mais quatro
desde que o administrador eleito seja novamente aprovado por voto popular, traz como Características:
A defesa do patrimônio público, a participação dos cidadãos na definição de novas políticas, a utilização
de funcionários governamentais, o regime de cobrança de impostos, a escolha dos representantes por
voto popular, e a descentralização do poder, dividido entre executivo, legislativo e judiciário.
O período republicano é dividido em cinco fases: República Velha (1889 - 1930), Era Vargas (1930 - 1945),
República Populista (1945 - 1964), Ditadura Militar (1964 - 1985) e Nova República (1985 – dias atuais). Foi
Marechal Deodoro da Fonseca (1827 - 1892) quem proclamou a República no Brasil em 1889 e em 1891 foi
promulgada a primeira Constituição da Era Republicana.
Até 1850, as terras no Brasil eram transferidas, por doações, do Estado aos proprietários e dos proprie-
tários aos parentes. A Lei de Terras estabelecia que as posses deveriam ser legalizadas e só poderiam
ser transferidas por meio de contratos de compra e venda (o que não era simples nem barato). Tais
exigências visavam dificultar o acesso à propriedade da terra, induzindo o trabalhador livre imigrante
a, necessariamente, empregar-se num latifúndio. Na prática, a lei impedia o surgimento de pequenos
proprietários e garantia oferta de mão de obra livre à aristocracia, reafirmando o poder da grande pro-
priedade, situação que permanece até hoje na nossa estrutura agrária. Com relação à imigração, em
1847, tentou-se implementar um novo sistema de trabalho, a parceria. Nesse esquema, tanto os custos
com o transporte do imigrante da Europa para o Brasil como as despesas com a fixação e a sobrevivên-
cia das famílias nas fazendas corriam por conta do fazendeiro e constituíam uma dívida dos imigrantes.
Segundo as leis da época, as famílias não podiam abandonar a fazenda enquanto não saldassem tais
dívidas. Como o ganho que os parceiros obtinham na exploração do café nunca era suficiente para pa-
gá-las, permaneciam cativos na fazenda, quase como escravos brancos. Esse sistema não atraiu mais
imigrantes e fracassou. No começo da década de 1880, com o aumento das pressões contra a escravi-
dão, criou-se um novo sistema, o colonato. No colonato, os gastos com transporte e as demais despe-
sas não constituíam dívida da família imigrante e o sistema de remuneração era misto, composto por
uma parte dos ganhos com a venda do café (como na parceria) e por um salário fixo anual. Além disso, as
famílias podiam produzir parte dos gêneros de subsistência que consumiam e vendiam o excedente em
mercados próximos. Esse sistema, um pouco mais vantajoso aos imigrantes, possibilitou a transição
para o trabalho livre na agricultura brasileira.
O período que vai de 1889 e 1930, também chamado de República das Oligarquias, é destacado pela al-
ternância de poder entre líderes do setor agrário na chamada “política do café com leite”.
A Revolução de 1930 marca o fim desse período e inaugura a Era Vargas, que vai até 1945.
Após 1850, o fim da escravidão tornou-se inevitável. A partir de então, a aristocracia latifundiária bra-
sileira, com mórbido pragmatismo, procurou prolongar ao máximo a vigência do sistema (para poder
explorar o trabalho dos escravos restantes até a morte), tratou de se antecipar aos riscos da adoção do
trabalho livre (aprovando a Lei de Terras, em 1850) e, finalmente, adotou medidas de apoio à imigração.

PARA SABER MAIS:


Abolicionismo no Brasil. Disponível em: Abolicionismo no Brasil. Disponível em: <https://pt.wikipedia.
org/wiki/Abolicionismo_no_Brasil>. Acesso em: 09 ago. 2021.

176
ATIVIDADES

1 - (Enem/2015) TEXTO I
Em todo o país, a lei de 13 de maio de 1888 libertou poucos negros em relação à população de cor.
A maioria já havia conquistado a alforria antes de 1888, por meio de estratégias possíveis. No entanto,
a importância histórica da lei de 1888 não pode ser mensurada apenas em termos numéricos. O impac-
to que a extinção da escravidão causou numa sociedade constituída a partir da legitimidade da proprie-
dade sobre a pessoa não cabe em cifras.
ALBUQUERQUE. W. O jogo da dissimulação: abolição e cidadania negra no Brasil. São Paulo: Cia. das Letras, 2009 (adaptado).

TEXTO II
Nos anos imediatamente anteriores à Abolição, a população livre do Rio de Janeiro tornou-se mais
numerosa e diversificada. Os escravos, bem menos numerosos que antes, e com os africanos mais
aculturados, certamente não se distinguiam muito facilmente dos libertos e dos pretos e pardos livres
habitantes da cidade. Também já não é razoável presumir que uma pessoa de cor seja provavelmente
cativa, pois os negros libertos e livres poderiam ser encontrados em toda parte.
CHALHOUB, S. Visões da liberdade: uma história das últimas décadas da escravidão na Corte. São Paulo: Cia. das Letras, 1990
(adaptado).

Sobre o fim da escravidão no Brasil, o elemento destacado no Texto I que complementa os argumentos
apresentados no Texto II é o(a)
a) variedade das estratégias de resistência dos cativos.
b) controle jurídico exercido pelos proprietários.
c) inovação social representada pela lei.
d) a ineficácia prática da libertação.
e) O significado político da Abolição.

2 - A resistência à escravidão no Brasil da década de 1880 ocorreu por diversas estratégias e espalhou-
se por diferentes classes do país. Entre as estratégias utilizadas na luta pela abolição da escravatura,
NÃO está correta:
a) a fuga de escravizados.
b) a formação de quilombos.
c) a atuação de advogados na defesa dos escravizados.
d) a panfletagem para divulgar a causa.
e) a busca por financiamento internacional.

3 - (Mackenzie) Sobre o contexto histórico responsável pela proclamação da República NÃO se inclui:
a) a insatisfação dos setores escravocratas com o governo monárquico após a Lei Áurea.
b) a ascensão do exército após a Guerra do Paraguai, passando a exigir um papel na vida política do país.
c) a perda de prestígio do governo imperial junto ao clero, após a questão religiosa.
d) a oposição de grupos médios urbanos e fazendeiros do oeste paulista, defensores de maior au-
tonomia administrativa.
e) o alto grau de consciência e participação das massas urbanas em todo o processo da proclama-
ção da República.

177
4 - (Cescea) A proclamação da República, representa basicamente:
a) uma profunda transformação na estrutura política e social do Brasil.
b) a introdução do sistema democrático no Brasil.
c) o fim do período colonial brasileiro.
d) uma modificação do regime político sem grande influência na estrutura econômica e social do País.
e) um movimento popular derrubado do chamado “Antigo Regime’’.

178
SEMANA 5
EIXO TEMÁTICO:
II - Cultura e Política na Construção do Estado Nacional Brasileiro (1822-1930).

TEMA/TÓPICO:
Confrontos: fim da monarquia no Brasil e início da República.

HABILIDADE(S):
Debater a inserção/exclusão das camadas populares no processo político.

CONTEÚDOS RELACIONADOS:
Revolução Francesa.

INTERDISCIPLINARIDADE:
Sociologia.

TEMA: As camadas populares no processo político brasileiro.


Caro(a) estudante, nesta semana você vai avaliar a participação das camadas populares no processo
político brasileiro. Pedro I proclamou a independência, o controle sobre o Brasil ainda permaneceu, de
certa forma, nas mãos da Coroa Portuguesa. Por isso, é fundamental salientar que esse movimento
foi exclusivamente elitista e da nobreza. A participação popular em todo o processo foi praticamente
nula. Os grupos sociais que não faziam parte da elite brasileira, sendo representados pela chamada
baixa da população, não teve nenhuma participação direta nesta revolução, sendo apenas expectado-
res do movimento.

APRESENTAÇÃO
Considerando-se que uma parcela significativa da população da colônia era composta de escravizados,
fica evidente que não havia, para esses, nenhuma possibilidade de exercício da cidadania, no menor
grau que fosse. Privados de direitos civis básicos, como integridade física, liberdade e, em alguns ca-
sos, até mesmo a vida, foram longos os caminhos até a inserção desses indivíduos como titulares de
algum direito perante a lei. Pelo contrário, até a abolição da escravidão, os escravizados eram equipa-
rados a animais e considerados como propriedade de seus senhores, que podiam os vender, trocar,
prender, espancar e matar, sem sofrer nenhuma consequência ou sanção. Os direitos políticos e sociais
nem se cogitam.
O desenvolvimento lento e pouco incentivado da cidadania no Brasil também pode ser explicado pela
total ausência de incentivo público à educação nesse período. No início, quem promovia a alfabetiza-
ção na colônia eram os jesuítas, mas após a expulsão dos religiosos, o Estado se encarregou de pro-
movê-la. Essa promoção pelo Estado se dava de maneira totalmente inadequada, não havendo sequer
registro dos níveis de alfabetização durante o período colonial. Da independência até o fim da Primei-
ra República, é possível se perceber, de forma geral, a inexistência de uma população politicamente
ativa e organizada. Existiram dois movimentos da época que alcançaram mobilização nacional, e de
alguma forma representava um despertar da conscientização política, o movimento abolicionista e o
tenentismo, em que o primeiro foi importante para difundir a ideia de liberdade e unificação nacional
e o segundo teve o seu mérito na luta contra o poder das oligarquias locais. Também é preciso se en-
tender que, mesmo não havendo um exercício de cidadania dentro de seus instrumentos formais de

179
efetivação, o povo se expressava de outras formas. Mesmo não tendo papel central nos processos de
independência e de formação da República, não foram poucas as revoltas populares que aconteceram
no período. Rapidamente, podemos citar as manifestações no Rio de Janeiro contra o regresso de
D. Pedro a Portugal, em 1822, as rebeliões populares do período regencial, a Revolta do Vintém em
1880 e a Revolta da Vacina em 1904. Portanto, é correto se dizer que ainda não tínhamos um exercício
organizado de cidadania capaz de impactar os rumos da política e da sociedade, mas é importante sa-
lientar sobre o um possível despertar do povo sobre o papel que o Estado deveria assumir na proteção
dos direitos da população.

PARA SABER MAIS:


A construção da cidadania no Brasil. Disponível em: <https://monografias.brasilescola.uol.com.br/direi-
to/a-construcao-da-cidadania-no-brasil-historico-desafios-e-caminhos.htm>. Acesso em: 09 ago. 2021.
Da Monarquia à república. Disponível em: <https://pt.wikipedia.org/wiki/Da_Monarquia_%C3%A0_
Rep%C3%BAblica_-_momentos_decisivos>. Acesso em: 09 ago. 2021.

ATIVIDADES
1 - (PUC – Rio, 2006) Durante a Primeira República (1889 – 1930), houve, na sociedade brasileira, revoltas
que, a despeito das diferenças, expressaram a insatisfação e a crítica de grupos populares quanto aos
mecanismos de exclusão social e política e às estratégias de expansão dos interesses oligárquicos,
então vigentes. Assinale a alternativa que identifica CORRETAMENTE revoltas dessa natureza:
a) Guerra de Canudos e Revolta da Vacina.
b) Revolta Federalista e Guerra do Contestado.
c) Revolta da Vacina e Revolta da Armada.
d) Revolta da Chibata e Revolta Federalista.
e) Guerra do Contestado e Revolta da Armada.

2 - ”Trata-se de uma estratégia muito usada em hábitos políticos coronelistas, em que, por exemplo,
os eleitores trocam seu voto por um favor, como um bem material (sapatos, roupas, chapéus etc.) ou
algum tipo de serviço (atendimento médico, remédios, verba para enterro, matrícula em escola, bolsa
de estudos etc.).”
Adaptado de JusBrasil. Consultado em 05.ago .2020.

O trecho acima descreve uma prática eleitoral da República Velha denominada:


a) Voto secreto.
b) Voto de cabresto.
c) Voto censitário.
d) Voto aberto.
e) Voto semi aberto.

180
3 - Qual das organizações abaixo não se configura como um movimento social?
a) Greves trabalhistas.
b) Coletivos feministas.
c) Movimentos estudantis.
d) Câmaras municipais.
e) Greves estudantis.

4 - Pode-se afirmar que são características estruturais dos novos movimentos sociais, exceto:
a) Decisão centralizada em uma figura de liderança.
b) Desenvolvimento de ações coletivas.
c) Desenvolvimento de ações locais, regionais e globais.
d) Utilização de ferramentas da internet para a divulgação de reivindicações.

181
SEMANA 6

EIXO TEMÁTICO:
II - Cultura e Política na Construção do Estado Nacional Brasileiro (1822-1930).

TEMA/TÓPICO:
Confrontos: fim da monarquia no Brasil e início da República.

HABILIDADE(S):
Relacionar as políticas de imigração com o processo de abolição da escravatura.

CONTEÚDOS RELACIONADOS:
Revolução Francesa; Conjuração Mineira.

INTERDISCIPLINARIDADE:
Filosofia.

Tema: Confrontos: fim da monarquia no Brasil e início da República.


Caro (a) estudante, nessa semana você vai analisar, a abolição da escravatura no Brasil, a resistência
realizada pelos próprios escravizados ao longo do século XIX e sua adesão de parte da nossa socie-
dade à causa por meio de associações abolicionistas. A história do abolicionismo no Brasil remonta à
primeira tentativa de abolição da escravidão indígena no Brasil, em 1611, à sua abolição definitiva pelo
Marquês de Pombal durante o reinado de D. José I, e aos movimentos emancipacionistas no período
colonial, particularmente a Conjuração Baiana de 1798, em cujos planos encontrava-se a erradicação
da escravidão. Após a Independência do Brasil (1822), as discussões a este respeito estenderam-se
por todo o período do Império, tendo adquirido relevância a partir de 1850 e caráter verdadeiramente
popular a partir de 1870, culminando com a assinatura da Lei Áurea de 13 de maio de 1888, que extinguiu
a escravidão no Brasil.

APRESENTAÇÃO
A abolição da escravatura aconteceu no Brasil, em 13 de maio de 1888, por meio da Lei Áurea, sendo re-
sultado da resistência dos escravos e da mobilização popular. A abolição da escravatura foi o resultado
de um processo de luta popular, que contou com a adesão de parcelas consideráveis da sociedade bra-
sileira, além de ter sido marcada pela resistência dos escravizados. O Brasil foi o último país das Améri-
cas a abolir com a escravidão. O fim do uso da mão de obra escrava em nosso país não foi resultado do
humanismo ou da benevolência da família real brasileira, conforme muitos acreditam, mas aconteceu
porque um grande número de pessoas de nossa sociedade mobilizou-se para forçar o Império a pôr fim
ao trabalho escravo.
A abolição da escravatura no Brasil aconteceu por meio da resistência realizada pelos próprios escra-
vizados ao longo do século XIX, adesão de parte da nossa sociedade à causa por meio de associações
abolicionistas e pela mobilização política dos defensores do abolicionismo. Além disso, havia a ques-
tão dos novos padrões civilizacionais que estavam surgindo e que condenavam a prática do trabalho
escravo. Isso colocava o Brasil numa posição vexatória internacionalmente. O fortalecimento do abo-
licionismo fez com que algumas leis fossem aprovadas no Parlamento brasileiro: a Lei do Ventre Livre

182
(determinava que todo filho de escravo nascido após 1871 seria considerado livre, cabendo ao dono do
escravizado dar sua liberdade com oito anos de idade, recebendo indenização, ou aos 21 anos de idade,
sem receber indenização), e também pela Lei dos Sexagenários (1885, concedia alforria aos escraviza-
dos que possuíssem idade superior a 60 anos. Os escravizados alforriados ficavam obrigados por lei a
prestar “serviços indenizatórios” durante três anos).
Ambas as leis atendiam demandas importantes para os escravocratas: a demanda por indenização dos
cativos (seja por meio de trabalho compulsório, seja por meio de um valor específico) que garantiam sua
liberdade e o enfraquecimento temporário dos movimentos abolicionistas. A Lei dos Sexagenários, por
exemplo, foi encarada pelos abolicionistas da época como uma verdadeira derrota para a causa.
A década de 1880 foi um momento de grande agitação política e a abolição do trabalho escravo foi a
pauta que agitou a sociedade brasileira. O movimento abolicionista tinha ganhado uma força muito
grande e a causa foi abraçada por inúmeros grupos de nossa sociedade. As associações abolicionistas
espalhavam-se pelo país e atuavam em diversas frentes: legais e ilegais (à luz da legislação da época).
Desde o começo do século XIX, o Brasil era pressionado pela Inglaterra para proibir o tráfico negreiro,
e essas pressões resultaram na Lei Eusébio de Queirós, em 1850. Com a proibição do tráfico, em 1850,
o processo até a abolição foi realizado de maneira lenta e gradual.

PARA SABER MAIS:


Abolição da escravatura. Disponível em: <https://mundoeducacao.uol.com.br/historiageral/abolicao-
-escravatura.htm>. Acesso em: 09 ago. 2021.
Imigração no Brasil. Disponível em: <https://pt.wikipedia.org/w/index.php?title=Especial:Pesqui-
sar&search=imigra%C3%A7%C3%A3o+na+C3%A9poca+da+aboli%C3%A7%C3%A3o+no+Brasil&go=-
Go&ns0=1>. Acesso em: 09 ago. 2021.

ATIVIDADES
1 - Em 1885 foi promulgada a Lei dos Sexagenários, também conhecida como Lei Saraiva-Cotegipe. Essa
lei determinava que os escravos com mais de 60 anos seriam libertos e que, como forma de indenização,
trabalhariam por mais três anos para seu antigo senhor. Essa lei é enxergada pelos historiadores como:
a) um ganho legítimo do movimento abolicionista.
b) uma medida reformista dos escravocratas com o intuito de retardar o avanço do abolicionismo
no Brasil.
c) um mecanismo para encarecer o preço do escravo, a fim de atender aos interesses dos trafican-
tes africanos.
d) o resultado de uma negociação realizada entre políticos e ex-escravos abrigados em um quilom-
bo nos arredores do Rio de Janeiro.
e) o resultado da pressão internacional sobre o Brasil.

183
2 - (TJ-SC) No Brasil as primeiras leis antiescravistas foram assinadas a partir de meados do século XIX.
A lei que extinguiu o tráfico negreiro no nosso país, decretada em 1850, foi:
a) Lei Visconde do Rio Branco.
b) Lei do tráfico de Bill Aberdeen.
c) Lei Eusébio de Queirós.
d) Lei Saraiva-Cotegipe.
e) Lei Áurea.

3 - Sobre a Lei de terras criada em 1850 no Brasil é correto afirmar que:


a) Foi uma política criada pelo governo de Deodoro da Fonseca com o objetivo de realizar a Reforma
Agrária no Brasil.
b) A lei de terras foi uma estratégia política da elite brasileira, em que a terra passou a ter valor de
compra e de venda para impedir o monopólio de muitas terras nas mãos dos ricos fazendeiros.
c) Foi uma lei estabelecida pela elite brasileira, em que as terras passaram a ter valor de compra e
de venda para impedir que trabalhadores assalariados tivessem acesso a elas e gerassem con-
corrência com os grandes fazendeiros.
d) Foi uma política reformista praticada pelo Imperador brasileiro D. Pedro I que decretou que as
terras seriam adquiridas através da compra e não da posse ou da prática das sesmarias, como
era feito durante o Brasil Colônia.
e) Foi uma lei estabelecida pelo governo da época, em que as terras passaram a ter valor de com-
pra e de venda para aumentar o valor ainda mais, gerando grande concorrência entre os grandes
fazendeiros.

184
SECRETARIA DE ESTADO DE EDUCAÇÃO DE MINAS GERAIS

PLANO DE ESTUDO TUTORADO


COMPONENTE CURRICULAR: FILOSOFIA
ANO DE ESCOLARIDADE: 2º ANO – EM
PET VOLUME: 04/2021
NOME DA ESCOLA:
ESTUDANTE:
TURMA: TURNO:
BIMESTRE: 4º TOTAL DE SEMANAS:
NÚMERO DE AULAS POR SEMANA: NÚMERO DE AULAS POR MÊS:

SEMANA 1

EIXO TEMÁTICO:
Agir e Poder.

TEMA/TÓPICO:
Os valores.

HABILIDADE(S):
Distinguir e circunscrever a esfera da moral como lugar das ações e escolhas humanas.
Distinguir fato e valor. Reconhecer que o agir humano é de natureza valorativa.

CONTEÚDOS RELACIONADOS:
Senso moral; Consciência moral; Juízo de fato e Juízo de valor; Ética e violência.

TEMA: Ética e moral - parte 1


Olá, estudante!
Ao longo dessa nossa jornada de aprendizado neste campo de conhecimento das Ciências Humanas,
que é a Filosofia, descobrimos que filosofar não é declarar verdades ou certezas, mas utilizar a razão
para investigar as coisas e o que pensamos sobre elas. Que tipo de coisas? Praticamente tudo. Por
exemplo, as angústias e os desejos que sentimos, a busca pela felicidade, a vontade de liberdade e o
prazer; mas também os problemas sociais e éticos que interferem na nossa vida, como a degradação
do meio ambiente, o consumismo, os direitos humanos, os preconceitos, as relações com o Estado,
o exercício do Poder, a intervenção da tecnologia no cotidiano; assim como a forma de pensarmos e
conhecermos as coisas, a ciência, a lógica e a argumentação. Tudo é passível de reflexão: até mesmo
nossa identidade, o tempo e a crença em uma fé. Se filosofar é pensar, e se fizermos isso habitualmen-
te, não só teremos mais clareza sobre este ou aquele aspecto investigado, mas também desenvolve-
remos habilidades e competências que favorecerão o conjunto de nosso entendimento ou da nossa
capacidade de compreensão.

185
Hoje daremos início ao nosso PET 4. Nesse material de estudo, dando continuidade aos diversos temas
já abordados, trataremos da importância da ética e da moral para nossa vida, já que somos seres so-
ciais, de convivência e de conflitos. A ética e a moral constituem os alicerces da conduta dos seres hu-
manos; desse modo, definem seu caráter e suas virtudes, e estabelecem como ele poderá (ou deverá)
se comportar em sociedade. Desde o nascimento aprendemos o que é “certo” e o que é “errado”; a partir
daí reproduzimos e refletimos sobre os valores da sociedade em que vivemos.
Vamos começar, pois, temos muita coisa para estudar!
“A virtude moral é uma consequência do hábito. Nós nos tornamos o que fazemos repetidamente.
Ou seja: nós nos tornamos justos ao praticarmos atos justos, controlados ao praticarmos atos de auto-
controle, corajosos ao praticarmos atos de bravura.”(Aristóteles. Ética à Nicômaco)
Às vezes somos tomados por emoções e sentimentos que se relacionam aos acontecimentos e situa-
ções do nosso contexto social, como a desigualdade social, a corrupção, a pobreza entre outros. Nestes
momentos, somos movidos pela solidariedade, que surge da maneira como avaliamos as situações que
nos dizem respeito, ou aos nossos semelhantes, mas como distinguimos o certo e errado, o justo e o
injusto? De onde se originam nossas ideias de bem e de mal, de vício e de virtude?

Senso moral
É o modo como mobilizamos, no senso comum, as ideias e valores que aprendemos desde o nascimen-
to. Veja: em muitas ocasiões, ficamos felizes, contentes e emocionados diante de uma pessoa honesta,
que age com justiça, e tem atitudes nobres; sentimos admiração por esta pessoa. Do mesmo modo,
somos tomados pelo horror diante da violência, e repudiamos quando uma pessoa se mostra corrupta,
injusta ou tem atitudes opressoras. Tais emoções e sentimentos exprimem nosso senso de moral, isto
é, a maneira como avaliamos a conduta e a ação de outras pessoas, em comparação à ideia que faze-
mos sobre a realidade. É o modo como criamos as referências, os critérios, para estabelecer a diferen-
ça entre a justiça e injustiça, o certo e o errado, o bem e o mal.
As coisas começam a ficar um pouco mais difíceis quando percebemos que nenhuma pessoa é 100%
justa, boa ou pacífica, e nem 100% injusta, errada ou violenta. Esta compreensão nos coloca em “xeque”
quanto às atitudes que devemos tomar, porque nós também somos incompletos, como todos os seres
humanos.

Consciência moral
Nossas dúvidas quanto à decisão a se tomar não manifestam nosso senso moral, mas põem à prova
nossa consciência moral, pois exige de nós, que justifiquemos para nós mesmos e para os outros, as
razões de nossas decisões e de todas as consequências que ela possa trazer.
O senso moral e consciência moral dizem respeito a valores, sentimentos, intenções, decisões e ações
referidos ao bem e ao mal, ao desejo de liberdade e ao exercício de liberdade. Dizem respeito às rela-
ções que mantemos com os outros e, portanto, nascem e existem como parte de nossa vida com outros.
O senso e a consciência moral são por isso constitutivos (formadores) de nossa existência intersubjeti-
va (reciprocidade com o outro), isto é, de nossas relações com as outras pessoas.

Juízo de fato e Juízo de valor


Os juízos de fato são aqueles que dizem o que as coisas são, como são e por que são. Quando dizemos
“está chovendo”, estamos enunciando um acontecimento constatado por nós. Não há dúvidas quanto à
validade da afirmação: ou de fato está chovendo (e a afirmação é verdadeira), ou de fato não está cho-
vendo (e a afirmação é falsa).

186
Já os juízos de valor avaliam as coisas, pessoas, ações, experiências, acontecimentos, sentimentos,
estados de espírito, intenções, e decisões. Classificam como bons ou maus, desejáveis ou indesejáveis,
e assim por diante. Não se contentam em dizer como algo é, mas se referem a como algo deve ser.
Dessa perspectiva, os juízos de valor são normativos, isto é, enunciam normas que dizem como devem
ser as coisas, tomando como referência as ideias que temos de bem, de mal, de liberdade, de felicidade
etc. Assim, podemos dizer que a diferença entre juízo de fato e de valor corresponde à diferença entre
natureza e cultura. A primeira é constituída por estruturas e processos necessários, que existem por si
mesmos. Por exemplo: a chuva, que é parte do ciclo hidrológico, e que se comporta de acordo com leis
universais, que podem ser enunciadas cientificamente. Por sua vez, a cultura nasce da maneira como
interpretamos o mundo. Damos significado simbólico e valores às coisas, pessoas, à natureza, e a tudo
e a todos. Exemplo: “Na noite lenta e morna, morta noite sem ruído, um menino chora....” ( Carlos Drum-
mond, poema, Menino Chorando na Noite)

Ética e violência
Evidentemente as várias culturas e sociedades existentes não definem a violência da mesma maneira.
Quando uma cultura e uma sociedade definem o que entendem por mal, crime e vício, definem aquilo
que julgam violência contra um indivíduo ou contra o grupo. Simultaneamente, elegem valores que con-
sideram positivos, como o bem, o mérito e a virtude, como barreiras éticas contra violência.
É preciso pontuar que ética e moral são termos distintos e que essa distinção veremos nas semanas
seguintes.

O agente moral
É preciso que exista o agente consciente, isto é, aquele que conhece a diferença entre bem e mal, certo
e errado, virtude e vício. A consciência e a responsabilidade são condições indispensáveis da vida éti-
ca. O campo ético é, assim, constituído pelos valores e pelas obrigações, que constituem as condutas
morais, isto é, a virtude.
O ser humano só pode existir se for:
• Consciente de si mesmo e dos outros, sendo capaz de refletir e de reconhecer a existência dos
outros como sujeitos éticos iguais a si;
• Dotado de vontade, isto é, de capacidade para controlar e orientar seus desejos, impulsos, ten-
dências e sentimentos;
• Responsável, isto é, que consiga assumir a responsabilidade sobre suas ações;
• Livre, isto é, capaz de reconhecer-se como causador de seus próprios sentimentos, atitudes
e ações.
O ser humano não deve se permitir a passividade, pois assim será controlado pelos seus impulsos (e não
exercerá controle sobre eles). Ao contrário, deve ser ativo e virtuoso, para controlar suas inclinações e
suas paixões. Isso o torna um ser autônomo.

REFERÊNCIAS:
CHAUÍ, Marilena. Iniciação à Filosofia. São Paulo: Ática, 2014.

187
ATIVIDADES
1 - O que é um juízo de fato e o que é um juízo de valor? Defina-os e dê novos exemplos diferentes do que
aparece no texto.

2 - (ENEM 2011) O brasileiro tem noção clara dos comportamentos éticos e morais adequados, mas
vive sob o espectro da corrupção, revela pesquisa. Se o país fosse resultado dos padrões morais que
as pessoas dizem aprovar, pareceria mais com a Escandinávia do que com Bruzundanga (corrompida
nação fictícia de Lima Barreto).
O distanciamento entre “reconhecer” e “cumprir” efetivamente o que é moral constitui uma ambiguidade
inerente ao humano, porque as normas morais são:
a) decorrentes da vontade divina e, por esse motivo, utópicas.
b) parâmetros idealizados, cujo cumprimento é destituído de obrigação.
c) amplas e vão além da capacidade de o indivíduo conseguir cumpri-las integralmente.
d) criadas pelo homem, que concede a si mesmo a lei à qual deve se submeter.
e) cumpridas por aqueles que se dedicam inteiramente a observar as normas jurídicas.

3 - Apesar das diferenças culturais, o que todas as culturas consideram que seja virtude?

188
SEMANA 2

EIXO TEMÁTICO:
Agir e poder.

TEMA/TÓPICO:
Ser e dever ser.

HABILIDADE(S):
Conhecer algumas entre as diversas posições filosóficas a respeito do bem e do mal.
Analisar e compreender os fundamentos da ética em diferentes culturas.

CONTEÚDOS RELACIONADOS:
Definição de ética e moral, ética clássica e ética religiosa.

TEMA: Ética e moral - parte 2


Olá, estudante!
Daremos continuidade nesta semana, aprofundando um pouco sobre a origem da ética e da moral para
compreendermos como se dá a constituição de nossa identidade como ser individual, cultural e social.
Toda cultura e sociedade institui uma moral, que é válida para todos os seus membros. Culturas e so-
ciedades fortemente hierarquizadas ou de classe podem até mesmo ter morais distintas para cada uma
delas. No entanto, a simples existência da moral não significa a presença explícita de uma ética. Esta
pressupõe uma disciplina filosófica, uma reflexão que discute, problematiza e interpreta o significado
dos valores morais, do caráter pessoal e dos costumes que neles se baseiam.
Embora sejam usados em conjunto e muitas vezes como sinônimos, os conceitos de ética e moral são
diferentes entre si, tanto no que diz respeito ao significado quanto à origem etimológica das palavras.
“O céu estrelado por sobre mim e a lei moral dentro de mim.” Immanuel Kant
Ética vem do grego ethos , e literalmente significa morada, refúgio. Em termos filosóficos está ligada
ao modo de ser, caráter, natureza e índole. A palavra moral tem origem no termo latino mores, e seu
significado está ligado aos costumes e tradições, aos “combinados” sociais.
Portanto, a partir da análise etimológica já é possível definir que a ética está mais ligada ao indivíduo,
enquanto a moral tem relação mais próxima com a sociedade. Além disso, costuma-se admitir que a
ética é um processo de reflexão sobre a moral.

Ética
Se refere ao modo de ser de um indivíduo, a natureza, o caráter e a postura adotados diante de uma
dada situação. Ações éticas são individuais, são uma escolha de cada ser humano, e partem de seu pro-
cesso de reflexão sobre o mundo e a vida. Evidentemente, como o indivíduo sempre está inserido em
um contexto social, histórico, político etc, toda a reflexão ética se baseia numa moral, ou seja, recebe
influência direta do meio no qual existe este indivíduo.

189
Moral
O conceito de moral pode ser definido como um conjunto de regras, valores e normas, estabelecidas
num determinado contexto histórico e social, que definem e norteiam o comportamento e o julgamento
dos indivíduos. A moral está ligada aos padrões culturais vigentes, costumes, valores e tradições, ele-
mentos necessários para o convívio dos indivíduos.

Uso da ética e da moral


A lei que define a prioridade no atendimento de idosos e gestantes, bem como assentos reservados
para os mesmos, expressa a moral de uma sociedade. Vejamos agora o seguinte exemplo: Maria não
cedeu o assento à mulher grávida. Percebemos aqui que, mesmo Maria tendo conhecimento das regras,
não a cumpriu. A ética é resultado da decisão de cada um.
Agora um exemplo para entendermos a moral como uma construção histórica. No passado, não era mo-
ralmente aceito que mulheres usassem calças. Atualmente a peça é usada por homens e mulheres. Os
momentos históricos e os contextos sociais influenciam na criação das regras morais, que sofrem alte-
rações ao longo do tempo. O músico brasileiro Belchior escreveu numa canção: “No presente, a mente,
o corpo é diferente. E o passado é uma roupa que não nos serve mais”.
Quando nós falamos em ética, nós não falamos apenas de teorias, práticas, filosofia, teologia, mas tam-
bém falamos da própria vida humana. A ética é sempre aplicada no nosso dia a dia, no trabalho, na esco-
la, na família, etc. Ela está ligada diretamente à construção do nosso caráter, e ao exercício da liberdade
humana. A ética é um estudo (teoria) no que se refere à conduta humana, do bem e mal, do certo e erra-
do de acordo com cada costume, comportamento e cultura de cada região.

Ética Grega Antiga


“ A reflexão grega neste campo surgiu como uma pesquisa sobre a natureza do bem moral, na busca de
um princípio absoluto da conduta.” (VALLS,1994, p.24)
Mas em que consiste este bem moral? Qual é o maior dos bens? Seria a felicidade, o dinheiro, as honras,
o prazer? E o que é este “Bem Supremo” de que tanto falam os filósofos gregos? Para Platão, Sócrates e
Aristóteles este Bem Supremo só pode ser alcançado mediante a prática de uma vida virtuosa, tendo a
razão como guia de nossas ações.
Para Sócrates, uma vida virtuosa depende do conhecimento de si mesmo. Ele afirma que o mais im-
portante está dentro de cada um de nós; que quanto melhor nos conhecermos, maiores serão nossas
virtudes; e que em nosso interior está a pureza de viver pelas nossas próprias escolhas. Assim, Sócra-
tes inaugura o chamado sujeito moral, ou agente moral, aquele que saber o que faz, conhece as causas
e os fins de suas ações. Este saber conduz o indivíduo à reflexão e à conscientização de suas atitudes
e de seu caráter.
Se devemos a Sócrates o início da filosofia moral, devemos também a Aristóteles a distinção entre sa-
ber teórico ou contemplativo e saber prático. O saber teórico é o conhecimento de seres e fatos que
existem e agem independentemente de nós e sem nossa interferência, isto é, de seres e fatos naturais
e/ou divinos. Já o saber prático é o conhecimento daquilo que existe como consequência de nossa ação
e, portanto, depende diretamente de nós, de nossas decisões. A ética e a política, por exemplo, são sa-
beres práticos. O saber prático pode ser de dois tipos: práxis ou técnica. Na práxis, o agente, a ação e a
finalidade do agir são inseparáveis. Exemplo: Somos aquilo que fazemos e o que fazemos é a finalidade
boa ou virtuosa. Já na técnica tem como finalidade a fabricação de alguma coisa diferente do agente.
Exemplo: Um carpinteiro, ao fazer uma mesa, realiza uma ação técnica, mas ele próprio não é essa ação
nem a mesa produzida por ele.

190
Também devemos a Aristóteles a definição de um destes saberes práticos, que é o campo das ações
éticas, ou seja, relativas às escolhas que fazemos. Trata-se do campo de saber onde tudo o que deve
ser e/ou acontecer depende de nossa vontade e de nossa ação.
“O valor fundamental da vida depende da percepção e do poder de contemplação ao invés da mera so-
brevivência”. Aristóteles
Princípios da vida moral:
Se examinarmos o pensamento filosófico dos antigos gregos, veremos que nele a ética afirma princí-
pios da vida moral como:
1. Por natureza, os humanos aspiram ao bem e à felicidade, que só são alcançáveis pela conduta
virtuosa.
2. A virtude é uma excelência, alcançada pelo caráter.
3. A conduta ética é aquela na qual o agente sabe o que está e o que não está em seu poder de rea-
lizar.
O ser humano não se submete aos acasos da sorte, nem à vontade e aos desejos de outro, nem à tirania
das paixões; mas obedece apenas à sua consciência e à sua vontade racional. A busca do bem e da fe-
licidade é a essência da vida ética.

Ética e religião
A religião está de tal forma presente na vida das pessoas que uma boa parte da sociedade pauta suas
ações (ou pelo menos orienta suas ações) em torno de mandamentos divinos que são, em sua essên-
cia, regras de conduta moral. Diferente das outras religiões da antiguidade, o Cristianismo nasce como
religião de indivíduos que não se definem por seu pertencimento a uma nação ou a um Estado, mas por
sua fé num único Deus. Assim, o Cristianismo introduz duas diferenças primordiais em relação à antiga
concepção ética:
1. A ideia de que a virtude se define por nossa relação com Deus, e não com a cidade (polis), nem com
as outras pessoas;
2. A afirmação de que somos seres dotados de livre-arbítrio e que, em decorrência do “pecado origi-
nal”, o impulso espontâneo de nossa liberdade se dirige para o mal, para o pecado. Assim, confor-
me a interpretação do Cristianismo, se somos pecadores, pressupõe-se que somos incapazes de
fazer o bem e praticar virtudes apenas por nossa vontade.
Assim, a concepção cristã introduz uma nova ideia na discussão ética: a ideia do dever, isto é, de que
a virtude é a obrigação de cumprir o que é ordenado pela lei divina. O Cristianismo introduz essa ideia
de dever para oferecer um caminho seguro para nossa vontade, que, sendo livre, mas fraca, sente-se
dividida entre o bem e o mal.
A partir do dever, a ação humana pode ser entendida como intencional. O dever não se refere apenas
às ações visíveis, mas também às intenções invisíveis, que podem ser julgadas eticamente olhando-se
para as atitudes do sujeito.

PARA SABER MAIS:


Ética e Moral: Como diferenciar? Disponível em: <https://www.youtube.com/watch?v=FEASxRw2Gb0>.
Acesso em: 26 ago. 2021.
Ética a Nicômaco: Aristóteles. Disponível em: <https://www.youtube.com/watch?v=xdqQKaacdVc>.
Acesso em: 26 ago. 2021.

191
REFERÊNCIAS:
CHAUÍ, Marilena. Iniciação à Filosofia. São Paulo: Ática, 2014.

ATIVIDADES
1 - Como podemos diferenciar “moral” e “ética”?
a) Não podemos diferenciar, são palavras sinônimas.
b) Moral é um conjunto de valores, e Ética é a reflexão sobre esses valores.
c) Moral é a prática da Ética no nosso dia a dia.
d) Moral é sinônimo de “ética aplicada”.
e) Nenhuma das alternativas acima.

2 - “Vemos que toda cidade é uma espécie de comunidade, e toda comunidade se forma com vistas a
algum bem, pois todas as ações de todos os homens são praticadas com vistas ao que lhe parece um
bem; todas as comunidades visam algum bem, é evidente que a mais importante de todas elas e que
inclui todas as outras têm mais que todas este objetivo e visa ao mais importante de todos os bens”
(Aristóteles Política.Brasília: UnB 1988)
No fragmento, Aristóteles promove uma reflexão que associa dois elementos essenciais à discussão
sobre a vida em comunidade, a saber:
a) Ética e política, pois conduzem à eudaimonia (que significa felicidade).
b) Retórica e linguagem, pois cuidam dos discursos na ágora.
c) Democracia e sociedade pois se referem a relações sociais.
d) Geração e corrupção, pois abarcam o campo da physis.
e) Nenhuma das alternativas acima.

3 - Quando e de que forma nasce a filosofia moral ou a disciplina filosófica denominada ética?

192
SEMANA 3

EIXO TEMÁTICO:
Ser e dever ser.

TEMA/TÓPICO:
Compreender os elementos culturais que constituem as identidades.

HABILIDADE(S):
Compreender e analisar os fundamentos da ética em diferentes culturas, identificando processos que con-
tribuem para a formação de sujeitos éticos que valorizem a liberdade, a autonomia e o poder de decisão
(vontade).

CONTEÚDOS RELACIONADOS:
Período Helenístico.

TEMA: Ética e moral - parte 3


Olá, estudante!A abordagem que estamos fazendo sobre as formulações da ética e da moral, e sua
construção ao longo da história, mostram o quanto são complexas as diversas teorias e visões sobre
o tema. Que faz parte da existência humana em sociedade e de sua conduta para com os outros. Que
mesmo estando submetidos a regras que existem para o equilíbrio da convivência, nos pegamos cons-
tantemente em meio a conflitos, agressões, julgamentos, divergências e violência. Isso porque somos
seres de desejo e vontade, e muitas vezes queremos ter o domínio, o poder e o controle das situações.
Por isso, a comunicação tem papel fundamental na construção da moralidade e da eticidade, que vão se
modificando, evoluindo e transformando ao longo da história, juntamente com a cultura, colaborando
diretamente uma com a outra neste processo.

Período Helenístico
A palavra “helenística” deriva de helenismo, termo que corresponde ao período que vai de Alexandre
Magno, o macedônico, até o da dominação romana (338 a.C. até sua anexação pelo romanos em 146
a.C.) Alexandre foi o grande responsável por estender a influência grega desde o Egito até a Índia. Esse
período perdurou por cerca de trezentos anos.
A filosofia helenística correspondeu a um desenvolvimento intelectual com forte influência dos temas
pré-socráticos; porém, sobretudo ela é profundamente marcada pelo espírito socrático. A experiência
com outros povos também lhe permitiu desempenhar certo papel no desenvolvimento da noção de cos-
mopolitismo, isto é, da ideia de homem como cidadão do mundo.
As escolas helenísticas têm em comum a atividade filosófica, como amor e investigação da sabedoria,
sendo esta um modo de vida. Elas não se diferenciavam muito na escolha da forma de sabedoria. Todas
elas definiam a sabedoria como um estado de perfeita tranquilidade da alma. Nesse sentido, a filosofia
é uma abordagem dos cuidados, das angústias e da miséria humana, miséria resultante das conven-
ções e obrigações sociais.
Todas as escolas helenísticas trazem certa herança socrática ao admitir que os homens estão submer-
sos na miséria, na angústia e no mal, porque estão na ignorância; o mal não está nas coisas, mas no
juízo de valor que os homens atribuem a elas. Disso decorre uma exigência: que os homens cuidem de

193
mudar radicalmente seus juízos de valor e seu modo de pensar e ser. E isso só é possível mediante a paz
interior e a tranquilidade da alma.
Este período do Helenismo é marcado basicamente pelo paganismo e pela crença da capacidade huma-
na de explicar os problemas humanos a partir das causas humanas. A seguir, vamos apresentar rapida-
mente as principais escolas do helenismo.

Cinismo
Quem nunca ouviu ou já falou: “fulano ou beltrano é cínico!”. Você sabe o que significa ser uma pessoa
cínica? Bom, vamos lá, eu explico: atualmente quando falamos ou ouvimos uma pessoa falar sobre ci-
nismo é normal atribuirmos o significado a alguém imprudente, insensível, que não liga para o sofri-
mento de outras pessoas ou mesmo nem se preocupa com algo que nós mesmo achamos de suma
importância. Pois bem, saiba que nem sempre foi assim.
O cinismo nasceu por volta de 400 a.C., fundado por um aluno de Sócrates chamado Antístenes. Basi-
camente uma pessoa adepta da filosofia cínica defende que um ser humano não precisa se apegar aos
bens materiais ou as vaidades criadas pela sociedade para ser feliz. As vaidades da sociedade dizem
respeito às posições sociais que adquirimos com o nosso trabalho ou à qual pertencemos desde o nos-
so nascimento.
Diógenes e Alexandre Magno
O grande expoente da filosofia cínica foi Diógenes de Sinope. Diógenes morava em um barril, vestia
apenas um pedaço de pano, e uma de suas únicas posses era um cajado; animais viviam e circulavam
ao seu redor. Figura simples, conta a história que certa vez este pensador recebeu a visita de Alexandre
Magno. Alexandre, parado na frente de Diógenes (este sentado a boca de sua morada) perguntou qual
seria o desejo dele e que bastaria ele dizer a Alexandre Magno que seu desejo seria realizado. Diógenes
rapidamente respondeu: “Sai da frente do meu sol”.

Pintura Alexandre e Diógenes, de Nicolas-André Monsiau (1818). Disponível em: <https://www.meisterdrucke.pt/kunstwerke/400w/


Nicolas%20Andre%20Monsiau%20-%20Alexander%20and%20Diogenes%201818%20%20-%20(MeisterDrucke-191862).jpg>.
Acesso em 26 ago 2021.

Assim nasceu a filosofia cínica, que contesta os apegos materiais e as pompas da sociedade, ou seja,
as ambições pela riqueza, os luxos e a ostentação. O homem é um animal e deve viver como tal. Ainda
frisa que todos os conhecimentos intelectuais que formam as bases da sociedade grega como: mate-
mática, física, música etc, são inúteis ao homem, isto porque são invenções do homem. Assim, para os
cínicos a felicidade significa o desapego aos bens materiais e aos títulos sociais. Este desapego conduz
o homem à liberdade.

194
Estoicismo
O Estoicismo surgiu por volta de 300 a.C. Seu fundador chamava-se Zenão de Citio. Esta corrente filo-
sófica partia da ideia do direito natural. Zenão acreditava que todas as pessoas são um tipo de “micro-
cosmo” e que todos fazem parte de um cosmos maior ainda, o universo ou “macrocosmo”. Pensar assim
significa retirar de todos os seres humanos as suas diferenças, ou seja, somos todos, em natureza,
iguais. Brancos, negros, amarelos, cristãos, não cristãos, escravos ou não.
Há aqui uma grande diferença com os filósofos cínicos. Os estóicos interessavam-se por política, e de-
claravam que a ética e a moral são exigências para uma vida correta e austera. Grandes homens deste
período foram estoicos, entre eles: o imperador romano Marco Aurélio (121-180); e o filósofo, político e
orador Cícero (106-43 a.C). Este segundo lançou as bases daquilo que ficou conhecido como humanis-
mo grego, fundamentado no antropocentrismo, ou seja, a ideia de que o homem ocupa o centro simbó-
lico da realidade. Os estóicos ainda acreditavam que tudo o que acontece conosco é algo que de algum
modo já estava estabelecido pela própria natureza. Assim, não devemos nos preocupar, nem sofrer
com os revezes que nos afligem, pois não podemos controlar o universo.

Epicurismo
Em 300 a.C. Epicuro (341-270 a.C) desenvolveu um modelo de pensamento que ficou conhecido como
epicurismo ou “filosofia do Jardim”. Este pensador defendia que a felicidade estava relacionada aos
prazeres humanos. Na sua ótica devemos preferir aqueles prazeres que podem ser considerados dura-
douros, e rejeitar aqueles que são desnecessários, ou que apenas aparentam ser prazeres.
É interessante perceber que Epicuro afirmava que a distinção entre essas categorias de prazeres, que
nos levam à felicidade, separa os prazeres extraídos dos cinco sentidos, daqueles que alcançamos por
meio da nossa razão. Por isso devemos tratar e aperfeiçoar o nosso intelecto. Para os epicuristas deve-
mos tomar quatro remédios para atingirmos a felicidade. São eles: 1) os deuses não devem ser temidos,
isso porque os deuses não estariam interessados em nós; 2) é considerado um absurdo temer a morte,
visto que ainda não morremos; 3) o prazer está a disposição de todos, e pode ser alcançado através da
virtude; 4) o mal dura pouco sendo fácil de suportá-lo: para isso devemos lembrar de coisas boas e feli-
zes ou criar boas expectativas para o futuro.

Ceticismo
Existem dois nomes quanto à criação e ao desenvolvimento da filosofia cética. O fundador foi Pirro de
Élis (360-270 a.C), e o seu desenvolvedor, no século II, foi Sexto Empírico. Basicamente, após a morte
de Alexandre, o Grande, a política na Grécia Antiga ficou estremecida. A democracia não mais respondia
aos anseios do povo grego, e assim governos autoritários tomaram conta das Cidades-Estado. Esta ins-
tabilidade contribuiu para o surgimento da filosofia cética, uma vez que muitas dúvidas surgiram sobre
a capacidade de governo dos sistemas políticos. A filosofia cética parte de três questões básicas, que
são: qual a natureza real das coisas? Quais as disposições que se convém adotar a seu respeito? E quais
as consequências que resultarão desta atitude?
A resposta a estes questionamentos, em geral, nos leva a compreender o seguinte: as coisas são di-
fíceis de se diferenciar uma das outras; nós não sabemos diferenciar as essências das coisas porque
não conseguimos percebê-las pelos nossos sentidos, e nossa razão também pode nos deixar “na mão”
às vezes. Por isso não devemos confiar no que a realidade nos apresenta. Isso nos leva à dúvida. Quan-
do isso acontece, a solução que temos é “suspender o juízo” (afasia; recusa em se pronunciar) sobre
as coisas. A postura cética se transforma em atitude contrária o dogmatismo (já que essa corrente
afirma que há uma verdade absoluta, e que ela pode ser conhecida e declarada). Enquanto o dogmá-

195
tico aceita uma determinada postura que por ele é considerada correta, o cético questiona a validade
desta forma de pensar.
Por este motivo é importante você saber que os céticos duvidam, mas não de tudo. Duvidam apenas de
coisas que não podem ser explicadas de forma direta, com um simples olhar.

Neoplatonismo
O Neoplatonismo, que teve inspiração em Platão, foi a grande corrente filosófica que concorreu direta-
mente com o Cristianismo, apesar de ter alguma proximidade em certos pontos. Plotino (205-270 d.C)
foi o principal representante desta corrente. Este pensador acreditava na existência de um Uno (Deus),
que seria a Luz. Assim, identificava a existência de um todo onde conviveriam a luz e as sombras (Tre-
vas). Na luz, Deus (Uno); nas trevas, há a ausência de luz e só isso. O ponto de encontro com o Uno está
dentro de nossa alma, quando nos encontramos com o próprio Deus e não nos diferenciamos dele, nos
tornando uma única coisa. A este tipo de experiência chamamos de experiência mística.

Misticismo
Para os místicos, pessoas que vivem a experiência mística ou que buscam uma experiência desta na-
tureza, viver significa exercer a simplicidade. Para isso, o místico busca a purificação e a iluminação de
sua alma. Não haveria de algum modo a separação entre a alma, Deus e o Universo. Neste sentido, tudo
estaria ligado e o caminho da purificação aproximaria tudo, sendo a experiência mística o auge da pu-
rificação humana. No Ocidente (ou seja, para o Islamismo, o Cristianismo e o Judaísmo) a busca é para
se chegar à presença de um Deus pessoal; no Oriente, busca-se uma experiência na qual o místico quer
de algum modo ligar-se ao Cosmos, ou seja, ao Universo.
“De todas essas coisas, a prudência é o princípio e o supremo bem, razão pela qual ela é mais preciosa
do que a própria filosofia; é dela que se originaram todas as demais virtudes; é ela que nos ensina que
não existe vida feliz sem prudência, beleza e justiça, e que não existe prudência, beleza e justiça sem
felicidade.“ (Epicuro)
PARA SABER MAIS:
Assista aos vídeos a seguir.
Grécia Antiga: Período Helenístico. Disponível em: <https://www.youtube.com/watch?v=dNfTXfR-
B32k>. Acesso em 26 ago. 2021.
Filosofia Helenista. Disponível em: <https://www.youtube.com/watch?v=cV4uIR-Spd0>. Acesso em 26
ago. 2021.

REFERÊNCIAS:
CHAUÍ, Marilena. Iniciação à Filosofia. Ed.São Paulo: Ática, 2014

196
ATIVIDADES
1 - (UCS) Os povos da Antiguidade Clássica foram responsáveis por um legado que permanece vivo nas
sociedades contemporâneas, em especial nas ocidentais. Sobre esse legado é correto afirmar que:
a) a religião monoteísta, praticada por gregos e romanos, foi a base para o surgimento do Cristia-
nismo, que, ainda hoje, congrega a maior comunidade de fiéis do Planeta.
b) a medicina praticada, tanto pelos gregos quanto pelos romanos, realizou progressos extraordiná-
rios; graças a eles foi possível conhecer a circulação do sangue e as infecções dos olhos e dentes.
c) a filosofia e a ciência praticada pelos gregos constituem-se a base do pensamento científico e
filosófico das sociedades ocidentais.
d) os idiomas dos países ocidentais, na sua totalidade, derivam do latim ou da fusão dele com os
dialetos bárbaros.
e) o sistema de numeração empregado atualmente é uma contribuição direta dos gregos e roma-
nos, especialmente o conhecimento do zero.

2 - (Fepese – 2012 –SC) Entre as muitas heranças do mundo da Antiguidade, herdamos as chamadas
filosofias pagãs do:
a) Estoicismo e niilismo.
b) Escolástica e marxismo.
c) Epicurismo e marxismo.
d) Estoicismo e epicurismo.
e) Escolástica e tomismo.

3 - (FUNCAB 2014) – O epicurismo e o estoicismo foram as duas filosofias éticas predominantes no


período helenístico. O estoicismo, em contraposição à ética epicurista, relaciona o bem:
a) à atividade da alma segundo a razão.
b) ao prazer por coisas materiais.
c) à indiferença diante da dor e do sofrimento.
d) aos costumes ou convenções sociais.
e) ao dever de cumprir o que é ordenado pela lei divina.

4 - (UFRN) Felipe II, rei da Macedônia, conquistou a Grécia. Seu filho, Alexandre, o Grande, consolidou
as conquistas do pai e expandiu o império em direção à Ásia, chegando até a Índia. Na perspectiva
histórica, a obra de Alexandre e de seus sucessores imediatos foi importante porque:
a) substituiu a visão mística do mundo, presente nos povos orientais, pelo reconhecimento intelec-
tual proveniente da razão e do raciocínio lógico.
b) favoreceu a difusão do modelo político das cidades-Estados da Grécia pelas regiões conquista-
das no Oriente, estimulando um governo fundamentado na liberdade e na democracia.
c) suplantou o poder despótico predominante nos grandes impérios orientais, os quais atribuíam
aos governantes uma origem divina.
d) possibilitou o intercâmbio de culturas, difundindo as tradições gregas nas terras do Oriente, en-
quanto as mesopotâmicas, egípcias, hebraicas e persas expandiram-se para o Ocidente.

197
SEMANA 4

EIXO TEMÁTICO:
Ser e dever ser.

TEMA/TÓPICO:
Ética categórica e utilitarismo.

HABILIDADE(S):
Utilizar os conhecimentos históricos para compreender e valorizar os fundamentos da cidadania e da demo-
cracia, favorecendo uma atuação consciente do indivíduo na sociedade.
Avaliar criticamente conflitos culturais, sociais, políticos, econômicos ou ambientais ao longo da história.

CONTEÚDOS RELACIONADOS:
Ética Kantiana; Utilitarismo.

TEMA: Ética e moral - parte 4


Olá estudante, vamos nesta semana fazer uma retomada sobre o pensamento de Kant, que reafirma o
papel importante que a razão exerce em nossas vidas e na esfera da ética e da moral. Vamos conhecer
também um pouco sobre o utilitarismo, que é uma doutrina que avalia a moral e, sobretudo, as con-
sequências dos atos humanos. Com isso, poderemos perceber que as escolhas individuais precisam
trazer o bem-estar de todos, do coletivo. Prepare-se, pois temos muita reflexão pela frente!
“O homem não é nada além daquilo que a educação faz dele.” Immanuel Kant

Ética Kantiana
Immanuel Kant (1724-1804), buscou criar um modelo ético que fosse independente de qualquer tipo
de justificação moral religiosa e se baseasse apenas na capacidade de julgar, que é própria do ser hu-
mano. (Teoria Deontológica – Que significa teoria do dever. Defendendo que o valor moral de uma ação
reside em si mesma.) para ele não existe bondade natural. O homem é por natureza egoísta, ambicioso,
destrutivo, agressivo, cruel, em busca de prazeres insaciáveis, pelos quais ele mata, rouba e mente.
É justamente por causa dessas atitudes que precisamos do dever – proporcionado pela capacidade de
razão, que é inata em nós - para nos tornar seres morais.
Para isso, Kant elaborou um imperativo, uma ordem, de forma que o indivíduo pudesse utilizá-la como
uma bússola moral: o imperativo categórico. Esse imperativo é uma lei moral interior ao indivíduo, ba-
seada apenas na razão humana e não possui nenhuma ligação com causas sobrenaturais, supersticio-
sas ou relacionadas a uma autoridade do Estado ou religiosa (vamos lembrar que Kant é fortemente
influenciado pelos ideais do Iluminismo). O filósofo buscou fazer com a filosofia o que Nicolau Copérni-
co fez com as ciências. A revolução copernicana transformou toda a forma de compreensão do mundo.
A ética kantiana está desenvolvida, sobretudo, no livro Fundamentação da Metafísica dos Costumes, de
1785. Nele, o autor busca estabelecer esse embasamento racional para o dever.
Kant reforça essa ideia ao afirmar que somente o pensamento autônomo poderia conduzir os indivíduos
ao esclarecimento e a maioridade. A maioridade em Kant não está relacionada com a idade, ou maio-
ridade civil, ela é a independência dos indivíduos fundamentada na sua capacidade racional de decidir
por si mesmo o que é o dever. A moral kantiana se opõe à moral cristã, na qual o dever é entendido como

198
uma heteronomia (significa ausência de autonomia. Sendo que autonomia em Kant é a capacidade da
vontade humana de se autodeterminar segundo uma legislação moral), uma norma vinda de fora para
dentro, a partir das Escrituras ou dos ensinamentos religiosos.

Imperativo Categórico
Busca estabelecer uma fórmula moral para a resolução das questões relativas à ação. O imperativo
categórico, ao longo das obras de Kant, aparece formulado de três maneiras diferentes. Nela, as ações
devem ser orientadas pela razão, sempre saindo do particular, da ação individual, para o universal, da
lei moral:
• Age como se a máxima de tua ação devesse ser erguida por tua vontade em lei universal da Na-
tureza. As leis da Natureza são universais e necessárias, todos os seres a cumprem, não há al-
ternativa. Como a lei da gravidade, os ciclos de vida e outras leis que submetem todos os seres
e é inquestionável.
• Age de tal maneira que trate a humanidade, tanto na tua pessoa como na pessoa de outrem,
sempre como um fim e nunca como um meio. Deste modo, um ser humano jamais pode ser en-
tendido como um instrumento para se alcançar qualquer tipo de objetivo. A humanidade é o fim
das ações e nunca um meio. Kant, nesse momento contraria, por exemplo, a ideia de que “os fins
justificam os meios” ou qualquer visão utilitária da ética.
• Age como se a máxima de tua ação devesse servir de lei universal para todos os seres racionais.
O ser humano deve tomar como princípio a ideia de que sua ação possa servir como lei para todas
as pessoas. Ou seja, com base na razão, a boa ação é a que está em conformidade com o dever.
O dever é como algo que nasce em nosso interior, proposto pela razão ou coração, não é algo imposto
por uma vontade estranha à nossa.
“Somos todos iguais perante o dever moral.” Immanuel Kant
Utilitarismo
A origem dessa corrente filosófica, consequencialista (Que significa que o valor moral de um ato é de-
terminado exclusivamente por suas consequências) ocorreu na Inglaterra com a formulação de Jeremy
Bentham (1748-1832), chamada de “o maior princípio da felicidade”, e a sua continuidade foi estimulada
por Jonh Stuart Mill (1806-1873), que publicou a obra “Utilitarismo” em 1861. O utilitarismo, caracteriza-
-se pela ideia de que as condutas adotadas devem promover a felicidade ou prazer do coletivo, evitando
assim as ações que levam ao sofrimento e à dor. Foi criado no século XVIII, e defendia que a única condi-
ção moral deve ser a busca da felicidade para o maior número de pessoas e que as ações são definidas
como certas ou erradas a depender dos seus efeitos. Embora tenha tido muitos opositores, essa linha
de pensamento causou grande impacto nos trabalhos voltados para o estudo da moral e nas estruturas
política, social e econômica da época. Com o ideal de coletividade, ela foi considerada uma das primei-
ras correntes a abordar a questão da igualdade, uma vez que rejeita a ideia de bem-estar apenas para
uma classe social. Os utilitaristas ainda consideram que o principal objetivo da moral é melhorar o mun-
do, potencializando a felicidade e diminuindo os danos como a dor e o sofrimento.
O utilitarismo parte do princípio de otimização, ou seja, exige a maximização do bem-estar geral, o que
não se apresenta como algo facultativo, mas sim como um dever. Também propõe o princípio da impar-
cialidade e do universalismo. Isso quer dizer que os prazeres e sofrimentos são considerados da mesma
importância, quaisquer que sejam os indivíduos afetados.
 “Aquele princípio que aprova ou desaprova qualquer ação, segundo a tendência que tem a aumentar ou
a diminuir a felicidade da pessoa cujo interesse está em jogo”.

199
PARA SABER MAIS
Kant - Ética do dever. Disponível em: <https://www.youtube.com/watch?v=dtJKzLO_0OE>. Acesso em:
26 ago. 2021.
Kant e o imperativo categórico. Disponível em: <https://www.youtube.com/watch?v=DzUaP56WpoU>.
Acesso em: 26 ago. 2021.
Utilitarismo. Disponível em: <https://www.youtube.com/watch?v=gW2XJgUFEps>. Acesso em: 26 ago.
2021. Utilitarismo: o certo é o que gera mais felicidade. Disponível em: <https://www.youtube.com/
watch?v=Xg34a-aZmVw>. Acesso em: 26 ago. 2021.

REFERÊNCIAS:
CHAUÍ, Marilena. Iniciação à Filosofia. São Paulo: Ática, 2014.

ATIVIDADES
1 - Segundo a fórmula universal do imperativo categórico, uma ação está correta quando:
a) Está de acordo com o princípio de autoconservação.
b) Quando gera a maior felicidade e bem –estar para o maior número de pessoas.
c) Está de acordo com os dez mandamentos.
d) É uma ação correta.
e) Pode ser praticada por todas as pessoas.

2 - (Ufsm 2015) A necessidade de conviver em grupo fez o homem desenvolver estratégias adaptativas
diversas. Darwin, num estudo sobre a evolução e as emoções, mostrou que o reconhecimento de
emoções primárias, como raiva e medo, teve um papel central na sobrevivência. Estudos antigos e
recentes mostram que a moralidade ou comportamento moral está associado a outros tipos de emoções,
como a vergonha, a culpa, a compaixão e a empatia. Há, no entanto, teorias éticas que afirmam que
as ações boas devem ser motivadas exclusivamente pelo dever e não por impulsos ou emoções. Essa
teoria é a ética:
a) deontológica ou kantiana.
b) das virtudes.
c) utilitarista.
d) contratualista.
e) teológica.

3 - Qual das alternativas abaixo define melhor o princípio básico do utilitarismo?


a) Uma ação moralmente correta é aquela que está de acordo com a regra “não faça aos outros
aquilo que não gostaria que fizessem para ti”.
b) Uma ação moralmente correta é aquela que produz o maior saldo positivo de prazer ou bem-es-
tar para todos os afetados pela ação, considerados imparcialmente.
c) Uma ação moralmente correta é aquela está de acordo com o imperativo categórico.
d) Uma ação moralmente correta é aquela que gera mais felicidade para o agente.
e) Uma ação moralmente correta é aquela em que a pessoa age com honestidade.

200
4 - Assinale a alternativa que contenha apenas nomes de filósofos utilitaristas:
a) Platão e Stuart Mill.
b) Rousseau e Aristóteles.
c) Kant e Stuart Mill.
d) Stuart Mill e Bentham.
e) Kant e Bentham.

5 - Explique o que é ética deontológica.

201
SEMANA 5

EIXO TEMÁTICO:
Ser e dever ser.

TEMA/TÓPICO:
Ética Contemporânea.

HABILIDADE(S):
Analisar a importância dos valores éticos na estruturação política das sociedades.
Utilizar os conhecimentos históricos para compreender e valorizar os fundamentos da cidadania e da demo-
cracia, favorecendo uma atuação consciente do indivíduo na sociedade.

CONTEÚDOS RELACIONADOS:
Ética Contemporânea x Bioética.

TEMA: Ética e moral - parte 5


Olá estudante, o tema desse PET 4, ética e moral, é bem extenso. Todos estamos fazendo o melhor pos-
sível, não é mesmo? Então, para incentivar você a prosseguir na busca pelo entendimento mais amplo
das questões relacionadas à moral e à ética na filosofia, vou deixar aqui pra você os nomes de alguns
dos autores mais importantes neste assunto. Suas obras, assim como a contextualização de seu pen-
samento, são bem fáceis de encontrar. São eles: São Tomás de Aquino (1225–1274), Santo Agostinho
(354–430), David Hume (1711-1776), Ludwig Wittgenstein (1889–1951), Frederich Nietzsche (1844-1900),
Jean Jacques Rousseau (1712-1778), Nicolau Maquiavel (1469–1527), Thomas Hobbes (1588-1679), Frie-
drich Hegel (1770-1831), Bertrand Russell (1972–1970), Martin Heidegger (1889 – 1976). Há inúmeros ou-
tros, mas com estes já é possível uma excelente introdução. Vamos lá, você consegue!

Ética Contemporânea
Da ética aristotélica para a ética atual, é preciso um estudo reflexivo profundo das transformações na
conduta humana no campo ético, seja no âmbito empresarial, profissional ou do senso comum. Lem-
brando que o comportamento ético é o ato de agir, de decidir, de fazer o bem de fato, por uma conduta
virtuosa na comunidade em que estamos inseridos, deste modo conduzimos uma vida mais harmonio-
sa em sociedade, na busca da felicidade, da liberdade e da justiça. E na filosofia moderna, a conduta
virtuosa designa uma disposição moral para o bem individual e comum.
Ela está fortemente marcada por motivações institucionais e legais, muito mais que morais. De certa
maneira, a técnica, a ciência, representam um avanço e propiciam uma ligeira sensação de que basta
estar disponível para justificar um uso. A conhecida “virada” técnico científico da modernidade. A valo-
rização da autonomia do sujeito moral leva à busca de valores subjetivos e ao reconhecimento do valor
das paixões, o que acarreta o individualismo exacerbado e a anarquia dos valores. Resulta ainda na
descoberta de várias situações particulares com suas respectivas morais. Mas que devemos reprimir
certos desejos e reforçar outros se queremos atingir a felicidade e o equilíbrio.
É preciso aqui citar para conhecimento e para que você estudante possa buscar mais informações, as
principais correntes de pensamento da ética contemporânea foram: O Existencialismo, o Pragmatis-
mo, a Psicanálise, o Marxismo, o Neopositivismo e a Filosofia Analítica. Pois, não conseguiremos aqui,
falar de todas.

202
Baruch de Espinosa (1632–1677)
Baruch Spinoza, foi um filósofo racionalista holandês, um dos mais importantes da filosofia moderna.
Nascido em Amsterdã, na Holanda e era descendente de judeus de origem portuguesa. Aprofundou
seus estudos nas áreas da teologia, línguas, filosofia e política. No entanto, suas ideias eram conside-
radas ateístas, o que levou a ser excomungado pela comunidade judaica.
Para Espinosa, somos seres naturalmente afetivos, isto é, nosso corpo é ininterruptamente afetado
por outros (que podem conservá-lo e regenerá-lo ou enfraquecê-lo e destruí-lo) e afeta outros corpos
(também podendo conservá-los, regenerá-los, enfraquecê-los ou destruí-los). Essas afecções (afetos)
corporais se exprimem em nossa alma ou sentimento. O afeto e o sentimento é, portanto, constitutivo
de nosso corpo e de nossa alma.
Nossos afetos são naturalmente paixões, pois somos naturalmente passionais quando sofremos a ação
de causas exteriores a nós. As paixões não são nem boas e nem más, simplesmente são naturais. Não
são vícios da natureza humana, e sim, a maneira como existimos, recebendo e sofrendo a ação das
causas externas. Uma paixão indica a força ou a fraqueza de nosso ser para existir e pensar. Três são as
paixões originais ou primitivas do homem: alegria, tristeza e desejo.
O que é vício? Não é ter paixões, e sim, a fraqueza para existir, agir e pensar. Deixando-se conduzir pelas
causas externas.
O que é virtude? Não é cumprir deveres e obrigações, mas ter força interior para passar da passividade
à atividade, ou seja, de afetos passivos (as paixões) a afetos ativos (as ações éticas). A virtude é ação, é
a realização e concretização de nossa capacidade cognitiva ou racional.
Na ética de Espinosa, jamais se fala em pecado e em dever; afastando a ideia de virtude e vício como
afirmava Aristóteles e a moral cristã, ao contrário, fala em fraqueza e em força para ser, pensar e agir.

Karl Marx (1818–1883)


Foi um filósofo, sociólogo, economista, jornalista e teórico político alemão. Junto a Friedrich Engels,
elaborou uma teoria política que embasou o chamado socialismo científico. Sendo considerado um dos
fundadores do materialismo histórico.
O pensamento marxista está fundamentado no reconhecimento de um sistema de exploração da classe
operária pela burguesia. Fundado por Karl Marx e Friedrich Engels, o marxismo, fundou-se com base
em um pensamento socialista já existente na Europa industrial, a fim de criar uma doutrina amparada
pela socialização dos meios de produção (indústrias) e pela tomada de poder da classe operária, tendo
em vista sua libertação do sistema explorador. O marxismo, na sua forma pura, defende que deve haver
uma revolução pela qual a classe operária toma para si os meios de produção e o governo, suprimindo a
burguesia e os seus meios de hegemonia e manutenção do poder, que constituem os conjuntos chama-
dos infraestrutura e superestrutura. Com base nisso, deveria ser criado um Estado forte, com um go-
verno chamado de socialista, que acabaria com a propriedade privada e controlaria toda a propriedade
em nome da população, formando uma ditadura do proletariado.
Os pilares da ética marxista se fundamenta na concepção de homem como ser concreto, social e históri-
co, em uma ontologia imanentista (investigação da condição humana), na visão materialista da história,
nas determinações da existência social dos homens sob relações sociais e econômicas determinadas,
em uma sociedade dividida em classes.
A principal contribuição da teoria marxista para a filosofia reside no conceito de materialismo histórico
dialético. Essa concepção filosófica opera uma reviravolta no que se entendia por dialética até então,
de Platão a Hegel. O conceito de dialética ganha um escopo estritamente prático e material, dispen-
sando o idealismo alemão em alta no cenário intelectual filosófico do século XIX. Muitos intelectuais

203
contemporâneos filiaram-se ao pensamento marxista, contribuindo para sua disseminação. Alguns
mais revolucionários, como o filósofo italiano Antônio Gramsci, partiram para a defesa direta de uma
revolução. Já os teóricos da Escola de Frankfurt fizeram uma releitura do marxismo para uma aplicação
que se encaixasse melhor no panorama econômico e político do século XX.

Jürgen Habermas (1929-)


É um dos maiores intelectuais dos séculos XX e XXl. Filósofo e sociólogo, com formação ligada à Escola
de Frankfurt, ele tem dedicado sua vida a estudar a democracia e apresentou um modelo de ação co-
municativa para que sejam desenvolvidos consensos no espaço público sem coerção física, mas pela
coerção do melhor argumento. A democracia pode ser, assim, aperfeiçoada pelos processos coletivos
de deliberação, que propiciam a coexistência dos diferentes e a participação de todos na produção de
normas sociais.

Teoria da ação comunicativa


É embasada por duas perspectivas diferentes, o materialismo histórico de Marx e o funcionalismo de
Max Weber, além da filosofia da linguagem e da teoria crítica da Escola de Frankfurt. A ação comuni-
cativa é uma complexa teoria da interpretação do mundo e da socialização. A socialização é complexa,
pois é um resultado dos processos individuais que se colocam em conjunto.
Assim, começa-se um processo de fundamentação da ética que resulta das ações individuais e do con-
vencimento das pessoas com base na comunicação. A comunicação é o mais fundamental processo
humano na ótica de Habermas, pois é ela que permite a interação e a instauração de processos éticos
e de socialização. A ação comunicativa é um processo de comunicação livre e racional, de extrema im-
portância para a consolidação da democracia.

Razão comunicativa
É a razão, ou a racionalidade, por trás da ação comunicativa. Ela surge como uma proposta de eman-
cipação do ser humano (influência da Escola de Frankfurt) em oposição à razão instrumental, descrita
por Adorno e Horkheimer (filósofos da Escola de Frankfurt) como parte da lógica capitalista brutal que
apenas utiliza a racionalidade como meio para algo e não reflete sobre si mesma. Essa razão instrumen-
tal foi o tipo de processo racional que desencadeou episódios como o holocausto judeu, e é descrita
também como uma espécie de lógica da barbárie pelos filósofos frankfurtianos.
Habermas é hoje um dos maiores pensadores das ciências humanas. Na educação, sua noção de ação
comunicativa é utilizada para defender, na escola e na universidade, a interdisciplinaridade, contra a ideia
positivista de separar as disciplinas. O diálogo e a busca de estruturas racionais são atitudes que podem
enriquecer todos os campos do conhecimento. Na política sua teoria é alternativa à democracia repre-
sentativa. A defesa de outras formas de participação política, como comitês e audiências públicas, resga-
ta ainda que parcialmente, aspectos da democracia direta e dos plebiscitos comum na antiguidade.

Bioética
Um dos conceitos que definem a Bioética (“ética da vida”) é que esta é a ciência “que tem como objetivo
indicar os limites e as finalidades da intervenção do homem sobre a vida, identificar os valores de refe-
rência racionalmente proponíveis, denunciar os riscos das possíveis aplicações” (LEONE; PRIVITERA;
CUNHA, 2001). A Bioética trata das questões referentes à vida humana e as melhorias na qualidade de
vida do homem. É composta por estudos multidisciplinares na área da Biologia, da Medicina e da Fi-

204
losofia. Com o notável avanço da medicina no campo da genética surgiram grandes preocupações no
campo da ética. A clonagem humana e a fecundação artificial são novas práticas genéticas que vêm
alterar conceitos e realidades da sociedade de hoje. Por exemplo, com as descobertas da biociência,
passou-se a questionar os pilares sobre os quais a família moderna está baseada.
É um ramo de estudos interdisciplinar que investiga as implicações éticas em relação à utilização e
manipulação da vida.

PARA SABER MAIS


Bioética. Disponível em: <https://www.youtube.com/watch?v=eCUKQU58SQI>. Acesso em: 26 ago 2021.
Karl Marx. Disponível em: <https://www.youtube.com/watch?v=HGwBJ-GY2rU>. Acesso em: 26 ago 2021.
Três lições de Espinosa. Disponível em: <https://www.youtube.com/watch?v=frqGWpueCLs>. Acesso
em: 26 ago 2021.

REFERÊNCIAS:
CHAUÍ, Marilena. Iniciação à Filosofia. São Paulo: Ática, 2014.

ATIVIDADES
1 - Explique a diferença entre vontade e desejo em Espinosa.

2 - (UEM 2011) Jürgen Habermas (1929) pertenceu inicialmente à escola de Frankfurt, também conhecida
como Teoria Crítica, antes de fazer seu próprio caminho de investigação filosófica.
Sobre o pensamento de Jürgen Habermas, assinale o que for correto.
a) Ao afastar-se da Escola de Frankfurt, Jürgen Habermas abandona, ao mesmo tempo, a teoria
crítica da sociedade e a crítica da razão instrumental.
b) Ao contrário de Max Horkheimer, Theodor W. Adorno e Walter Benjamin, Jürgen Habermas con-
tinua fiel ao materialismo histórico, ou seja, à ortodoxia marxista.
c) A relação posta pela Filosofia positivista entre o objeto da investigação científica e o sujeito que
investiga é, para Jürgen Habermas, o caminho a ser adotado por uma racionalidade que deseja
a emancipação humana.
d) A racionalidade comunicativa, contida na Teoria da ação comunicativa de Jürgen Habermas,
elabora-se na interação intersubjetiva, mediatizada pela linguagem de sujeitos que desejam al-
cançar, por meio do entendimento, um consenso autêntico.
e) Nenhuma das Alternativas anteriores.

205
3 - Explique o que você entende por Bioética.

206
SEMANA 6

EIXO TEMÁTICO:
Ser e dever ser.

TEMA/TÓPICO:
Ética da Alteridade.

HABILIDADE(S):
Utilizar os conhecimentos históricos para compreender e valorizar os fundamentos da cidadania e da demo-
cracia, favorecendo uma atuação consciente do indivíduo na sociedade.
Analisar a importância dos valores éticos na estruturação política das sociedades.

CONTEÚDOS RELACIONADOS:
Ética da Alteridade.

TEMA: Ética e moral - parte 6


Olá, estudante! Estamos finalizando mais um Plano de Estudos Tutorados, mas os nossos estudos e a
nossa busca pelo conhecimento de nós mesmos, pelos sinais da verdade nas coisas e de tudo que en-
volve nossa existência em sociedade será uma constante sempre. Neste momento, faremos um breve
apanhado sobre a ética da alteridade. Será que você se sente preparada ou preparado para ver as ou-
tras pessoas como destinatários da sua ação ética? Bons estudos!
“Querer ser livre é também querer livres os outros.” Simone de Beauvoir
“O homem é livre; mas ele encontra a lei na sua própria liberdade.” Simone de Beauvoir

Emmanuel Lévinas (1906 – 1995)


Diante da complexidade das relações humanas no mundo atual, por conta das expressivas tiranias e
violências do paradigma liberal, bem como da tendência da sociedade contemporânea em “banalizar o
mal”, o pensamento de Lévinas nos coloca para refletir.
Ele foi impactado pela experiência pessoal de ter vivenciado as duas grandes guerras mundiais. Re-
cebeu a influência das obras de Franz Rosenzweig, Husserl, Heidegger bem como da bíblia hebraica.
Assim, Lévinas extrai tanto da fenomenologia quanto da religião os elementos formadores de seu pen-
samento. Por tais motivos, sua obra é caracterizada pela crítica aos fundamentos da tradição filosófica
do ocidente, que no entendimento do filósofo, proporciona uma racionalidade conivente com a vio-
lência humana. Para Lévinas a redefinição pelo homem moderno de conceitos como sujeito e pessoa,
produziu resultado contrário ao que desejava, pois se de um lado houve um avanço para a filosofia em
relação ao entendimento do indivíduo, por outro, gerou atitudes humanas inaceitáveis como as guerras
e outras formas de violência que evidenciam o distanciamento entre o homem e seus valores éticos.
Toda a base de construção do pensamento levinasiano é o Outro. Da relação Eu-Outro emerge uma
nova perspectiva de ponderação, a de pensar a si mesmo e à sociedade a partir e com o Outro. A ética,
enquanto produto da interpelação do rosto do Outro. Entender a “ética da alteridade” para o pensamen-
to de Emmanuel Lévinas é entender que para o mesmo a ética se apresenta como a primeira forma de
filosofia, sendo as demais apenas ramos desta. Para Lévinas, a ética se apresenta diretamente quando
eu estou face-a-face com o outro indivíduo (COSTA, 2000, p. 25. Lévinas: Uma introdução.) Para o filó-

207
sofo a vista do rosto do outro conduz a uma atitude ética, pois convoca o Eu a ser por ele responsável,
Lévinas (1997, p. 144) declara que:
É sempre a partir do rosto, da responsabilidade por outrem, que aparece a justiça, que comporta
julgamento e comparação daquilo que, em principio, é incomparável, pois cada ser é único; todo
outrem é único. Nesta necessidade de se ocupar com a justiça aparece a ideia de equidade, sobre a
qual está fundada a ideia de objetividade. Há, em certo momento, necessidade de uma pesagem, de
comparação, de pensamento, e a filosofia seria, nesse sentido, a aparição da sabedoria a partir do
âmago desta caridade inicial; ela seria - e não brinco com as palavras – a sabedoria desta caridade,
sabedoria do amor. (LÉVINAS. Entre nós. Ensaios sobre a alteridade, 1997)
Toda a filosofia levinasiana reverbera da perspectiva do Outro que tem em seu Rosto seu elemento
mais original. Por meio da alteridade Lévinas chama a atenção para a afirmação do indivíduo em um
momento em que a negação epistemológica do ser transcendente prevalecia. Dessa forma, para que o
Outro, infinito, não fique relegado ao plano do pensamento, Lévinas preconiza o rosto como o concreto
que impele as relações humanas com responsabilidade ética. A radicalidade ética de Lévinas, enquanto
resgate à alteridade mediante a expressão do rosto, se apresenta como negação à violência numa ten-
tativa de subversão da ordem estabelecida pela denominada totalidade.
O modo como o Outro se apresenta, ultrapassando a ideia de Outro em mim, chamando-o, de fato, ros-
to. Esta maneira não consiste em figurar como tema sob o meu olhar, em expor-se como um conjunto
de qualidades que formam uma imagem. O rosto de Outrem destrói em cada instante e ultrapassa a
imagem plástica que ele me deixa à minha medida e à medida do seu ideatum (LÉVINAS 1998, p.37-38).
Ao afastar do Rosto a experiência regulada pelo conhecimento, Lévinas se afasta da fenomenologia
e instaura uma nova valoração na qual a ética é sua filosofia primeira. Por fim, a nudez do Rosto, que
Lévinas explica como sendo o modo como o Outro aborda o eu livre e intacto, sem verniz, evidencia a
fragilidade humana. O filósofo esclarece que:
“[...] O rosto de Outrem está nu; é o pobre por quem posso tudo e a quem tudo devo. E eu, que sou eu,
mas que enquanto ‘primeira pessoa’ sou aquele que encontra processos p o ‘primeira pessoa’ sou aque-
le que encontra processos para responder ao apelo” (LÉVINAS, 1988, p. 37-38 e p. 61-62)
O homem, se faz livre ao ser responsável pelo outro, no acolher o outro na sua mais absoluta alteridade.
a partir do homem que encontra sua verdadeira raiz na ética, que o conduz a compreender a responsa-
bilidade do Eu para com o Outro.
A Alteridade, é muito mais que um conceito, é uma prática. Ela consiste, basicamente, em colocar-se
no lugar do outro, entender as angústias do outro e tentar pensar no sofrimento do outro. Alteridade
também é reconhecer que existem culturas diferentes e que elas merecem respeito em sua integridade.
“É a hora em que o indivíduo justo não encontra nenhum recurso exterior, em que nenhuma instituição
o protege, em que a consolação da presença divina no sentimento religioso infantil se nega também,
em que o indivíduo apenas pode triunfar em sua consciência, ou seja, necessariamente no sofri-
mento. Sentido especificamente judeu do sofrimento que não toma em nenhum momento o valor de
uma expiação mística pelos pecados do mundo. A posição de vítimas em um mundo em desordem…”
Emmanuel Lévinas
“As nossas análises são dirigidas por uma estrutura formal: a ideia do Infinito em nós. Para se ter a
ideia do Infinito, é preciso existir como separado. Essa separação não pode produzir-se como fazendo
eco apenas à transcendência do Infinito. Senão, a separação manter-se-ia numa correlação que res-
tauraria a totalidade e tornaria ilusória a transcendência. Ora, a ideia do Infinito é a transcendência,
o transbordamento de uma ideia adequada”. Emmanuel Levinas

208
PARA SABER MAIS
Alteridade. Disponível em: <https://www.youtube.com/watch?v=0GhVn4BtSmg>. Acesso em: 26 ago. 2021.
Cultura e Alteridade. Disponível em: <https://www.youtube.com/watch?v=e6yvczKRx4k>. Acesso em:
26 ago. 2021.

REFERÊNCIAS:
CHAUÍ, Marilena. Iniciação à Filosofia. São Paulo: Ática, 2014.

ATIVIDADES
1 - Emmanuel Lévinas foi um dos filósofos que desenvolveu uma ética marcada pelo compromisso
e responsabilidade do EU em relação ao outro. Sobre a ética da alteridade, proposta por Lévinas, é
CORRETO afirmar que:
I. A ética elaborada por Lévinas se configura a partir de um fundamento ontológico, o Bem demonstra-
do racionalmente.
II. A alteridade, no sentido levinasiano, é a possibilidade de relação do ser separado e finito com o ou-
trem, a partir do desejo metafísico.
III. A ética levinasiana é construída a partir da concepção heideggeriana, que concebe a solidão no in-
terior de relação.
IV. A relação ético-metafísica, apresentada por Lévinas, instaura no tempo a possibilidade de uma re-
lação para além da essência.
a) Somente I e II são verdadeiras.
b) Somente III e IV são verdadeiras.
c) Somente II e III são verdadeiras.
d) Somente I e III são verdadeiras.
a) Somente II e IV são verdadeiras.

2 - Defina Alteridade, com base na sua compreensão sobre o pensamento de Lévinas.

209
3 - O que é alteridade? É ser capaz de apreender o outro na plenitude de sua dignidade, dos seus direitos
e, sobretudo, da sua diferença. Quanto menos alteridade existe nas relações pessoais e sociais, mais
conflitos ocorrem.
Considerando as atitudes que Frei Betto defende para construir uma comunidade de alteridade, assina-
le com V as afirmativas verdadeiras e com F as falsas.

(  ) Manter a nossa tendência de colonizar o outro, ou partir do princípio de que eu sei e ensino para ele.
(  ) Refletir que os professores sabem algumas coisas e aqueles que não foram à escola sabem outras
tantas.
(  ) Defender que o equilíbrio emocional para lidar com as relações de alteridade só se aprende na escola.
(  ) Utilizar o diálogo e a capacidade de entender o outro a partir de sua experiência de vida e da sua
interioridade.
Assinale a alternativa que apresenta a sequência de letras CORRETA.
a) (V) (V) (F) (F).
b) (V) (F) (F) (V).
c) (F) (F) (V) (V).
d) (F) (V) (F) (V).
e) (V) (V) (V) (V).

210
SECRETARIA DE ESTADO DE EDUCAÇÃO DE MINAS GERAIS

PLANO DE ESTUDO TUTORADO


COMPONENTE CURRICULAR: LÍNGUA INGLESA
ANO DE ESCOLARIDADE: 2º ANO – EM
PET VOLUME: 04/2021
NOME DA ESCOLA:
ESTUDANTE:
TURMA: TURNO:
BIMESTRE: 4º TOTAL DE SEMANAS:
NÚMERO DE AULAS POR SEMANA: NÚMERO DE AULAS POR MÊS:

SEMANA 1

EIXO TEMÁTICO:
Recepção e Produção de Textos Orais e Escritos de Gêneros Textuais variados em Língua Estrangeira.

TEMA/TÓPICO:
Tema 10: Aspectos léxico-sistêmicos / Tópico: 56. Usos sociocomunicativos dos vários tipos de presente
(simples, contínuo, perfeito).

HABILIDADE(S):
56.1. Fazer uso adequado dos diversos tipos do tempo presente no processo de recepção /produção do texto
oral e escrito de vários gêneros textuais.

CONTEÚDOS RELACIONADOS:
Talking about past habits; Giving instructions.

INTERDISCIPLINARIDADE:
Vida em sociedade.

TEMA: CONNECTING OLD AND YOUNG

Hello! Nesta semana, você irá conhecer um projeto que promove uma conexão colaborativa entre
gerações ou idades diferentes. Já viu algum projeto desses na sua cidade?

APRESENTAÇÃO
Compreender como estabelecer vínculos entre gerações mais jovens e entre os mais idosos é algo que
tem o potencial de trazer grandes benefícios a nível individual e social. No material desta semana, co-
nheceremos o TOY e observaremos quais são as estratégias utilizadas para alcançar um objetivo tão
nobre - o estabelecimento de laços entre grupos distintos. Tenha um ótimo estudo!

211
PARA SABER MAIS:
Conheça o trabalho desenvolvido por esta instituição com vistas a desenvolver crianças e jovens
emocionalmente fortes, resilientes e socialmente competentes para a vida e estudos. Disponível em:
<https://www.connecting-generations.org/>. Acesso em: 17 jul. 2021.

ATIVIDADES
1 - You are going to read excerpts from a program called TOY. Look at the website menu and answer:
what does TOY stand for? What kind of actions does it promote? Write your guesses.

Disponível em: <http://www.toyproject.net/>. Acesso em 17 jul. 2021.

2 - Now, read two excerpts taken from the website and check your guesses. Then answer:
a) Where do you need to click on the website menu to find the first excerpt?

b) What is the second excerpt about? Where can you find this information easily?

212
Excerpt 1: A global movement to promote young children and older adults learning together
The purpose of the TOY Programme is to promote intergenerational learning and create new possibil-
ities for older adults and young children to learn together and benefit from each other’s company. We
believe that this is more important than ever before in all continents of the world.
People are living longer but older adults and young children are having less and less contact with each
other. Parents and grandchildren are migrating to cities and countries far away from grandparents.
For many grandparents it is also sometimes difficult to keep in touch with grandchildren. Other rea-
sons for the lack of contact between old and young is that in many.
countries, older adults are living in old peoples’ homes where they rarely see children and many young
children are spending their days with their own age group in day care centres, pre-schools and schools.
[...]
The Together Old and Young (TOY) approach to IGL brings young children (0-8) and older adults togeth-
er to share experiences, have fun, learn from each other and develop meaningful relationships. Inter-
generational Learning activities in TOY are friendly and informal social encounters, where children
and adults can equally partake as the learner and the teacher.

Excerpt 2: Old and Young share stories to keep history


alive!
November 21, 2017
On Wednesday, 8 November the new ‘Heritage Holders’
2017 were honored in the Amsterdam Museum. [...] Both
the children as the elderly enjoyed this impressive and
authentic morning in the museum. […]

The intergenerational education project ‘In my Neighbor-


hood’ believes in keeping the history of a neighborhood
alive by bringing different generations of local residents
together, share stories and create new, young ‘Heritage
Holders.’ [...]

Almost 3 000 students have been interviewing the older residents. During these interviews empathy is
born, prejudices are broken and the past becomes a shared history of the neighborhood where they all
live. What happened during the war in the house next to the supermarket where you do your shopping
every day? And how did the migrant feel in the 60s celebrating his first Christmas together with typical
Dutch neighbor Henk?

Yearly a selection of young official ‘Heritage Holders of Amsterdam’ (10-14 years old) are honored, and
the youngsters pledge to retell the personal, historical stories they heard from elderly from their own
neighborhood. The new Heritage Holders retell the stories during official moments in media, at com-
memorations and big events.

The honoring in the Amsterdam Museum this November was an impressive, touching and authentic
moment, in which the elderly officially gave their story to the youngster and gave their own heritage
holder a medal.
Available at: <http://www.toyproject.net/news/old-young-share-stories-to-keep-history-alive/>. (Adapted.)
Accessed on: Jan.15, 2020.

213
GLOSSARY
Dutch: holandês; Elderly: idosos; Heritage Holder: guardião da herança; IGL: intergenerational
learning (aprendizagem intergeracional); Is born: nasce; Lack of: falta de; Purpose: finalidade

3 - Find the following expressions in the excerpts.


a) Analyse their context and match them to their equivalent in Portuguese.

a) prejudice I. sustentar
b) keep in touch II. emocionante, comovente
c) support III. manter o contato
d) pledge IV. preconceito
e) touching V. compromisso, juramento

b) Use the word “prejudice” to write a sentence about the relationship between old and young people.

c) Use the expression “keep in touch” to write a sentence talking about people you want to keep in
touch with.

4 - Now read the excerpts more carefully and find the paragraph(s) that:
a) gives a description of the students activities done in the project “In my Neighborhood.”
____________________
b) presents explanations for the interactions between old and young not being constant in present
times. ____________________
c) states an opinion about the relevance of the program from an international perspective.
___________________
d) describes the feeling, the atmosphere of the commemorative event in the museum.
____________________
e) informs when and where the event happened. ____________________

5 - Read the excerpts again and answer the questions.


a) What are the characteristics of the intergenerational learning activities in TOY?

214
b) What is the task of a “Heritage Holder?”

c) Do you know any other similar initiatives in your city/region or in Brazil?

d) Can you think of other activities in which elders could collaborate in your community? In your
school? How?

REFERÊNCIAS:

WEIGEL, Adriana; RESCHKE, Tatiana. English and more! São Paulo: Richmond, 2020.

215
SEMANA 2
EIXO TEMÁTICO:
Recepção e Produção de Textos Orais e Escritos de Gêneros Textuais variados em Língua Estrangeira.

TEMA/TÓPICO:
Tema 10: Aspectos léxico-sistêmicos / Tópico: 57. Usos sociocomunicativos dos vários tipos de passado
(simples, contínuo, perfeito).

HABILIDADE(S):
57.1. Fazer uso adequado dos diversos tipos do tempo passado no processo de recepção /produção do texto
oral e escrito de vários gêneros textuais.

CONTEÚDOS RELACIONADOS:
Giving opinion; Descriptions.

INTERDISCIPLINARIDADE:
Arte.

TEMA: PAINTINGS THAT CHANGED THE WORLD

Hello! Nesta semana, você vai identificar obras que imortalizaram grandes artistas,
compreendendo algumas das razões por trás do impacto social promovido por elas.

APRESENTAÇÃO
Você já imaginou que a arte, para além de ser uma experiência estética, pode também promover um
impacto ao seu redor, levando a reflexões que podem até gerar atitudes concretas com vistas a mu-
danças? Nesta semana, você verá alguns trabalhos artísticos que são considerados atemporais e será
levado a ler essas obras de uma forma distinta da apreciação estética. Bons estudos!

PARA SABER MAIS:


What is art? Listening practice. Disponível em: <https://www.eslvideo.com/quiz.php?id=24999>. Aces-
so em: 17 jul. 2021.

216
ATIVIDADES
1 - You are going to read an excerpt from an article about art. First, read its title and subtitle, and
hypothesize:

TEN PAINTINGS THAT CHANGED THE WORLD


by Mary Strickson
Art extends beyond aesthetics and technique; it can affect and alter the world we know into something
new and extraordinary.
Available at: <https://www.voicemag.uk/feature/885/ten-paintings-that-changed-the-world>. Accessed on: Jan. 27, 2020.

a) Can art change the world? How?

b) Which artists and/or paintings do you expect to be mentioned in the article? Why?

2 - Before reading the article, do the activities below.


a) According to the European Academies of Fine Art, there are five categories of paintings. Can you
think of different paintings to suit each category?

1. History Painting: Religious, historical or allegorical work, with a moral message.


2. Portrait Art: Includes individual, group or self-portraits.
3. Genre Painting: Scenes of everyday life.
4. Landscape Painting: Paintings whose principal content is a scenic view.
5. Still Life Painting: An arrangement of domestic objects or everyday items.

217
b) Explore now the paintings below. How would you classify them according to the previous catego-
ries? Some may fit more than one category.
a) ________________________________ b) ________________________________

c) ________________________________ d) ________________________________

e) ________________________________ f) ________________________________

c) Consider the title of the article and imagine: what kind of change do the paintings represent?

GLOSSARY
be reclaimed: ser recuperado; For the better/worse: para melhor/pior; In turn: por sua vez; Raised
questions: levantou perguntas, questões; Rather than: em vez de; Their own world: seu próprio
mundo; Throwaway and fickle society: sociedade descartável e inconstante

218
3 - Now, read the article excerpt and do the activities. Match the paintings to the descriptions. Note that
there are six paintings and five descriptions.

TEN PAINTINGS THAT CHANGED THE WORLD


by Mary Strickson
Art extends beyond aesthetics and technique, it can affect and alter the world we know into something
new and extraordinary.
These artists challenged what already existed, created something completely innovative. Some of
these artists used art to comment on their own world, to implement change, and to change the wider
world for the better. They used art as a powerful tool to express themselves and in turn change the
world for the good. [...]
Leonardo Da Vinci, The Mona Lisa, 1503 __________________________________________
There’s a reason why this painting is the most famous in the world. Leonardo Da Vinci challenged how
we create portraiture and changed the use of composition within art. It also raised questions about
women’s role in art and changed art practice as it had been.
Andy Warhol, Campbell’s Soup Cans, 1962 __________________________________________
Andy Warhol is known for leading the pop art movement. He introduced the idea of art not having to be
about the big and powerful, but to be about ordinary mundane objects. His focus on consumables chan-
ged the world in making us question our throwaway and fickle society. His methods have been endlessly
explored, questioned and mimicked and have changed how we think about art and paintings. [...]
Pablo Picasso, Guernica, 1937 __________________________________________
Picasso’s Guernica reflects a world that has been changed for the worse, an innocence that can never
be reclaimed. War permanently altered the world and mankind which this painting shows. The hor-
rors to humanity highlighted a constant threat upon the world and civilian tragedies that changed the
world. [...]
Van Gogh, Starry Night, 1889 __________________________________________
The dawn of impressionism changed the way that artists created paintings. Rather than simply captu-
ring what they saw, they added feelings and sentiment. Starry night in particular captures emotions,
how Gogh was feeling at the time. He didn’t simply draw what he could see but used his imagination to
embellish it, letting the viewer enter Gogh’s inner dreams and nightmares. [...]
Salvador Dalí, Persistence of Memory, 1931 __________________________________________
Dalí’s surrealist paintings welcomed those looking at the picture to enter a world of dreams, confusion
and imagination. His famous iconic melting clocks, distorted figures and bent branches confound the
viewer into entering the illusion and reading new meanings into paintings.
Which paintings do you feel most changed your world? Let us know!
Available at: <https://www.voicemag.uk/feature/885/ten-paintings-that-changed-the-world>. Accessed on: Jan. 27, 2020.

4 - From the paintings listed before, which one would you use to illustrate the current days? Justify your
opinion.

219
5 - Read the article quickly.
5.1 Identify the artist who...
a) depicts daily objects in a very different form. ______________________
b) criticizes consumerism. ______________________
c) shows the horrors of war. ______________________
d) brings a new way to paint portraits. ______________________
5.2 Now, choose the appropriate options ( ✔). According to the text, we can conclude that…
a) the ability of artists to change the world makes their work special and great.
b) the relevance of a work of art is related to how the public receives it.
c) works of art are exotic.
d) creating techniques is something all artists do.
e) there is no piece of art from the 21st century described there.
f) the artists and artworks represent different art movements around the world.

6 - What’s your opinion? Think about the following questions and then share your ideas with a classmate
or a friend.
a) What feelings do you think Van Gogh expresses in Starry Night?

b) What does Salvador Dalí’s melting clocks represent?

c) Which questions about women’s role in art do you think the Mona Lisa raised?

REFERÊNCIAS:

WEIGEL, Adriana; RESCHKE, Tatiana. English and more! São Paulo: Richmond, 2020.

220
SEMANA 3

EIXO TEMÁTICO:
Recepção e Produção de Textos Orais e Escritos de Gêneros Textuais variados em Língua Estrangeira.

TEMA/TÓPICO:
Tema 10: Aspectos léxico-sistêmicos / Tópico: 58. Usos sociocomunicativos dos vários tipos de futuro (sim-
ples, contínuo, perfeito).

HABILIDADE(S):
58.1. Fazer uso adequado dos diversos tipos do tempo futuro no processo de recepção /produção do texto
oral e escrito de vários gêneros textuais.

CONTEÚDOS RELACIONADOS:
Nouns; Adjectives; Adverbs; Prepositions; Cohesion.

INTERDISCIPLINARIDADE:
Arte; Literatura.

TEMA: NOUN PHRASES AND CONCRETE POETRY

Hello! Nesta semana, você irá entender o uso de grupos nominais, reconhecer a necessidade do
uso de preposições para promover a coesão e analisar poesia concreta em Língua Inglesa.

BREVE APRESENTAÇÃO
Nesta semana, veremos a importância da utilização de preposições ao conectar as partes da sentença,
aproveitando para se colocar no idioma comunicando os seus planos para o dia - uma forma significa-
tiva de utilizar a língua estrangeira. Outro assunto de impacto na Língua Inglesa é acerca do uso de fra-
ses nominais: afinal, como posso ter um grupo de substantivos que se referem a um outro substantivo
principal funcionando como “especificadores”, assim como faz um adjetivo? É exatamente assim que
funcionam os chamados “noun groups”: the kitchen table possui o substantivo principal “table”, que por
sua vez, é especificado por “kitchen”, mas veremos tudo isso com maiores detalhes adiante! Para fina-
lizar a semana, falaremos também de poesia concreta. Ótimos estudos!

221
ATIVIDADES
1 - Read the cartoon and answer:
a) Why does the teenager decide to come back to talk to his parents?

Disponível em: <https://1.bp.blogspot.com/-qlEJLK-BDa0/XsWopOdq8wI/


AAAAAAAAAqo/8LOWG2AGqC4KXHHQcr3cxDjMjUWDsw2AgCLcBGAsYHQ/s640/1.png>. Acesso em: 17 jul. 2021.

b) How about you, what are you going to do today?

NOUN PHRASES

2 - Analyze the groups of words and answer the questions.

GLOSSARY
backdrop: pano de fundo; fabric: tecido; foreground: primeiro plano; interlocking: entrelaçado(a);
stitched together: costurado(a) junto; woven with a needle: tecido(a) com uma agulha

“their own world” - “pop art movement” - “ordinary mundane objects” - “His famous iconic melting clocks”
a) What is the most important word, in terms of meaning, in each group?

b) What kind of words are they?

222
c) What’s their position in the group?

d) What is the purpose of the other words in the group?

e) What kind of words are they?

3 - Use your answers above to complete the explanation below.

A noun phrase is a group of words usually based on a ____________. The other items of the noun group
can be ___________ , ___________ , articles, adverbs, ___________, etc., and are used to give more
____________ about the _________________ .

4 - Based on your conclusions, read the description of Ñandutí III, by Olga Blinder, 1961 and select the
appropriate word order.

The Ñandutí of the title is a type of lace made in circular form/


form circular like a spider’s web, the centre filled with intri-
cate and infinitely designs varied/varied designs woven with a
needle in and around the supporting spokes. These circles are
then stitched together to form larger cloths such as the one
that forms the backdrop to the main scene of women working.
[...] The three women concentrating on creating their circle
laces/lace circles are all dressed in garments of different de-
signs/designs different. The figure in the foreground wears a
fabric decorated with leaves and organic forms [...]. Another
has a motif of a four-headed beast/beast four-headed that
alternates between white on black and black on white, while
the garment of the third (upper left) has an interlocking ser-
pent double-headed/double-headed serpent motif reminis-
cent of designs on baskets made by the Mbyá Guaraní peoples
Available at: <https://www.vads.ac.uk/digital/
collection/UECLAA/id/664/>. Accessed on: of southern Paraguay.
Apr. 11, 2021.

5 - Part of the work of artists is to experiment, to create and recreate. Study the images on the next
page – they are examples of concrete poems produced by Ian Hamilton Finlay.
5.1 Read them and comment:
a) What do they have in common?

223
5.2 Contrast the poems in images I and II and discuss:
a) How different/similar are they?

b) Do these differences create or suggest different meanings?

Cultural Tip
Ian Hamilton Finlay (28 October 1925-27 March 2006) was a Scottish poet, writer, artist and gardener.
He is famous for his “poem-objects:” poems inscribed into stone as sculptures to be incorporated in
the natural environment.

I.FINLAY, Ian Hamilton. – you – me – us, 1968. Silkscreen, 48 × 48 cm. II. FINLAY, Ian Hamilton. The Blue and Brown Poems 1 of 12, 1968.
Openings Press. Litograph, 50,7 × 38,4 cm. Susan Boutwell Gallery, München, Germany.

6 - Choose a piece of art or even a song or book you would like to recreate as a visual, concrete poem.
The idea is to make it portray an aspect of youth culture, of your own life, or of our time/society. Follow
these steps:
a) Brainstorm your ideas: write a list of scenarios you would like to recreate.

b) Learn more about visual poetry – characteristics, techniques, etc. Talk to your Art teacher for
more suggestions. (write here what you have learned)

224
c) Experiment working with the words, with different fonts, colors, spacing, layout, etc. If you deci-
de to have a sculptural poem, think about each material and the way you use it. (write here what
you have decided to use)

d) Make the draft of your concrete poem here.

After, photograph and write captions for your art pieces. You can exhibit your art in contrast with the
original one on a digital canvas (for example: <https://padlet.com/>. Accessed on: Mar. 24, 2020).

REFERÊNCIAS:

WEIGEL, Adriana; RESCHKE, Tatiana. English and more! São Paulo: Richmond, 2020.

PARA SABER MAIS:


• To learn about visual poetry techniques and experiment creating poems online, visit: <http://www.
languageisavirus.com/visual-poetry/index.php#.Xj2vB2hKiM9>. Accessed on: Feb. 11, 2020.
• To see more examples of concrete poetry from the Heide Museum of Art, Melbourne, Australia,
go to: <https://vimeo.com/68509574>.Accessed on: Feb. 11, 2020.

225
SEMANA 4

EIXO TEMÁTICO:
Recepção e Produção de Textos Orais e Escritos de Gêneros Textuais variados em Língua Estrangeira.

TEMA/TÓPICO:
Tema 10: Aspectos léxico-sistêmicos / Tópico: 56. Usos sociocomunicativos dos vários tipos de passado
(simples, contínuo, perfeito).

HABILIDADE(S):
56.1. Fazer uso adequado dos diversos tipos do tempo passado no processo de recepção /produção do texto
oral e escrito de vários gêneros textuais.

CONTEÚDOS RELACIONADOS:
Simple Past; Giving opinion.

INTERDISCIPLINARIDADE:
História.

TEMA: NOSSAS RAÍZES AFRICANAS: CONHECER, ENTENDER PARA, ENTÃO, SER.

Hello! Nesta semana, você irá conhecer e desenvolver vínculos com a sua própria história.
Tenha uma ótima jornada!

APRESENTAÇÃO
Nas semanas anteriores, trabalhamos com a relação de respeito colaborativo entre gerações distintas
como forma de tanto idoso quanto jovem irem além de onde estão situados. Pensamos também sobre
arte que impactou a história do mundo, estabelecendo até mesmo um paralelo entre arte e a ilustração
do contexto atual. Dentro dessa conversa entre o antigo e o atual, o histórico e o contemporâneo, não
se pode ausentar um diálogo com nossas próprias raízes. Aliás, quando você pensa nas raízes da sua fa-
mília, o que vem à sua mente? E ao considerar seus avós, bisavós, você sabe um pouco da história deles
e de suas origens? Tudo isso é muito importante no processo do autoconhecimento, de responder à tão
confusa pergunta do “quem sou eu”. Para te ajudar em seu processo de conhecer a si mesmo, traremos
à roda de conversa o historiador Laurentino Gomes. Leia um excerto de sua entrevista a seguir:
Autor das premiadas obras “1808”, “1822” e “1889”, que resgatam detalhes da história brasileira, o escritor
paranaense Laurentino Gomes se deu conta de que o assunto mais importante do passado nacional não
eram os ciclos econômicos, as revoluções, o império ou a monarquia. Era a escravidão. “Tudo o que nós já
fomos no passado, o que somos hoje e o que seremos no futuro tem a ver com nossas raízes africanas
e o modo como nos relacionamos com elas”, afirma.
Disponível em: https://revistagalileu.globo.com/Sociedade/Historia/noticia/2019/12/laurentino-gomes-um-pais-que-nao-estuda-
historia-e-incapaz-de-entender-si-mesmo.html Acesso em: 29 de ago. 2021.

E então, o que você conhece sobre suas raízes africanas? Como você se relaciona com elas?

226
ATIVIDADES
A seguir, você encontrará uma linha do tempo com alguns pontos principais dos Primeiros Registros da
História da África sob o título de “Africa’s History Timeline: Part I”
Disponível em: https://www.blackpast.org/global-african-history-timeline/ Acesso em: 19 jul. 2021.

1 - Be an active reader! Insert an emoji, an icon or symbols of “like”,


“dislike” and “share” on the circle of each card of the timeline to show
your reaction to the information or to call attention to the subject.
Imagem disponível em: https://cdn.pixabay.com/photo/2017/09/18/17/32/
emoji-2762568_960_720.png Acesso em: 17 ago. 2021.

Observe a timeline, seus cards, títulos e datas e responda:

2 - O que você espera aprender ao ler os títulos de i. “Africa


Migration” e também ii. “West Africa Civilizations”?
Escreva suas expectativas em Português.

3 - What happened in the West African Civilizations


report? Use the Simple Past.

4 - What do you know about the West African


Empires? Research about it and write down
four facts that you did not know.

227
5 - Read about Roman and European Slavery in the timeline. Where did slaves from Rome and Europe
come from?

6 - O que mais chamou sua atenção na linha do tempo ao lado? Como isso pode te ajudar a compreender
os dias atuais?

Africa’s History Timeline. Disponível em: https://www.blackpast.org/global-african-history-timeline/ Acesso em: 19 jul. 2021.

PARA SABER MAIS: conheça mais da história da África de uma forma interativa: https://mappinghis-
tory.uoregon.edu/english/AF/AF01-00.html

228
SEMANA 5
EIXO TEMÁTICO:
Recepção e Produção de Textos Orais e Escritos de Gêneros Textuais variados em Língua Estrangeira.
TEMA/TÓPICO:
Tema 10: Aspectos léxico-sistêmicos / Tópico: 56. Usos sociocomunicativos dos vários tipos de passado
(simples, contínuo, perfeito).
HABILIDADE(S):
56.1. Fazer uso adequado dos diversos tipos do tempo passado no processo de recepção /produção do texto
oral e escrito de vários gêneros textuais.
CONTEÚDOS RELACIONADOS:
Simple Present; Simple Past; Present Perfect.
INTERDISCIPLINARIDADE:
História; Sociologia.

TEMA: DE ONDE VEM A ESCRAVIDÃO?

Hello! Nesta semana, você irá compreender como se deu a ocorrência da escravidão desde
os primórdios da humanidade.

BREVE APRESENTAÇÃO
“A história é uma ferramenta de construção de identidade, olhando o passado sabemos quem somos
hoje” (Laurentino Gomes. Disponível em: https://www.bbc.com/portuguese/brasil-57575496 Acesso
em: 29 de ago. 2021.) É por isso que nesta semana vamos estudar sobre as origens da escravidão. Ao
final de completar as tarefas, não deixe de parar, refletir e discutir com colegas sobre a pergunta “de
onde vem a escravidão?”. Ótimo estudo!

229
ATIVIDADES
Read the text about The History of Slavery and answer the following questions.

History of Slavery Timeline


c. 3000 BC Slavery arrives as part of the package of civlization, along with armies,
public works and social hierarchies.
c. 1720 BC The Code of Hammurabi is the first surviving document to record the
law relating to slaves.
c. 1700 BC The biblical account suggests that around this period the Hebrews are
a captive tribe in Egypt.
c. 350 Frumentius, brought to Ethiopia as a slave, becomes the kingdom’s first
Christian bishop.
c. 700 The African slave trade through the Sahara is so extensive that a new town,
Zawila, is established as a trading station.
c. 850 The caliphs in Baghdad begin to employ Turkish slaves, or Mamelukes,
in their armies.
c. 950 So many Slavs are captured and sold, in the movement eastwards of the
Germans, that their name becomes the European word for a slave.
1446 Portugal claims ownership of the region of Guinea, subsequently the centre
of their slave trade on the west African coast.
1547 John Knox is captured in St Andrews and is sent to serve in the French fleet
as a galley slave.
Disponível em: http://www.historyworld.net/timesearch/default.
asp?keywords=%22slave%22&date=&sort2=&bottomsort=&topsort=&direction=&timelineid=&getyear=&viewtext=extended&
conid=timeline&event_number=20 shorturl.at/iGN67 Acesso em: 10 jul. 2021.

1 - According to the timeline, when and how did slavery start in the world?

2 - Why was the town of Zawila established?

230
3 - According to the timeline, what is the origin of the word slave? Explain it.

4 - Você é parte da equipe de cinema nigeriano, Nollywood, que hoje está em segundo lugar na produção
mundial de filmes. Como parte da popularização do trabalho, você precisa criar um podcast narrando a
sinopse da próxima grande produção de Nollywood, “Slavery in the world: an untold story”. Crie a seguir
o roteiro do podcast utilizando as informações vistas em History of Slavery Timeline.
• Escreva em Inglês.
• Use o Simple Past.
• Use marcadores de tempo, como data.
• Para cada trecho do seu roteiro, insira a música ou tipo de fundo musical que será colocado no
podcast durante a narração da frase a seguir.
Veja o exemplo:

[música típica na introdução do podcast] Hello everybody, we are getting back with one more epi-
sode on our season of “Slavery in the world: an untold story” [inserir música de suspense]. It all star-
ted a long time ago, back in 3000 BC [inserir som de páginas de livro passando seguidas de ventania
e sons de …] CONTINUE THE STORY...
E a história continua com a sua criatividade! Essa introdução é apenas um exemplo de como você pode
fazer no início do roteiro, mas dê asas à sua imaginação e inclua todas as ideias que passarem à sua
mente; posteriormente, você poderá ir polindo seu material até deixar no formato que quiser.
Lembre-se: o processo de criação é feito em etapas. Inicie com um draft (rascunho), depois crie sua
version 1 e, por fim, crie sua final version.

PARA SABER MAIS:


Get to know about the African American History through an interactive timeline. Disponível em: <https://
400years.berkeley.edu/timeline>. Acesso em: 29 ago. 2021.

REFERÊNCIAS:

WEIGEL, Adriana; RESCHKE, Tatiana. English and more! São Paulo: Richmond, 2020.

231
SEMANA 6

EIXO TEMÁTICO:
Recepção e Produção de Textos Orais e Escritos de Gêneros Textuais variados em Língua Estrangeira.

TEMA/TÓPICO:
Tema 3: Compreensão Oral (escuta) / Tópico: 14. Condições de produção do texto oral de gêneros textuais
diferentes.

HABILIDADE(S):
14.4. Reconhecer o local onde se passa o evento comunicativo. 14.5. Identificar a autoria do texto.

CONTEÚDOS RELACIONADOS:
Present Continuous; Narrative and Descriptive Genres; Simple Past.

INTERDISCIPLINARIDADE:
História; Literatura.

TEMA: SLAM POETRY AS A TOOL TO REFLECT ABOUT YOUR IDENTITY

Hello! Nesta semana, você conhecerá como se deu a instauração do sistema escravo que fez parte
da formação do povo brasileiro. Então, você reconhecerá o uso social da poesia slam por adoles-
centes ao redor do mundo para, através dela, revelar a história que tem profunda influência sobre a
sociedade brasileira, mas que é tão pouco contada.

BREVE APRESENTAÇÃO
Iniciamos com a África Antiga. Agora, veremos um pouco sobre a África caminhando para a Idade Média
e o início da Idade Moderna, quando há a sistematização de tráfico humano dos mais diversos reinos
africanos.Ao final, você terá a oportunidade de compartilhar tudo o que aprendeu através de um tipo de
poesia que está bem popularizada entre os adolescentes ao redor do mundo - a slam poetry. I wish you
a great study time!

232
ATIVIDADES
Ao lado está apresentada uma linha do tempo com alguns pontos principais dos Primeiros Registros da
História da África – Parte II.

1 - Read the timeline “Africa, Modern Age” and


answer:
1.1 True or False? If false, write the correct sen-
tence.
a) According to evidence, the Europeans were
the first travelers to come to the Americas. (  )
b) The only kingdom in Africa was Egypt. (  )
c) Back in 420 we could hear people defending
and fighting for the equality of all human
beings. (  )
d) There is no register of slavery in Europe before
1500. (  )
e) A great number of empires emerged along the
History of the African Continent. (  )
f) The slaves in Italy in 1400 was composed of
people from many African kingdoms. (  )
g) Between 1414 and 1423, 10.000 Nigerian slaves
are sold in Venice. (  )
h) The beginning of the direct involvement of Eu-
rope with the African slave trade dates back to
2700 BCE. (  )

Now, correct the sentences that were false:

233
1.2 Is there any surprising fact to you in this timeline? Explain.

Disponível em: https://www.blackpast.org/global-african-history-timeline/ Acesso em: 19 jul. 2021.

2 - Have you ever heard of spoken poetry? Can you think of a kind of spoken poetic battle typical of the
Northeastern region of the country? What do you know about it?

3 - Read the story about the Slam Poetry and answer the following questions.

Disponível em: https://www.mudandinkteaching.org/news/2018/4/7/slam-poetry-101-an-introduction Acesso em: 29 de ago. 2021.


Imagem disponível em: https://pixabay.com/pt/vectors/grilo-microfone-desenho-animado-309790/ Acesso em: 30 de ago. 2021.

a) How did slam poetry start? Research the definition of “blue collar” and include it in your explana-
tion.

b) What is Louder than a Bomb? Who joins the event and what happens on it?

234
4 - Read the definition of a teacher explaining what slam poetry is and answer: What is slam poetry?

Slam poetry is a competitive form of spoken word poetry. Students can perform poetry all they want,
but for it to be a SLAM poem, that means that it’s ready for battle! The idea of poetry being a “com-
petition” is ludicrous, as admitted by founder Marc Smith. As the saying goes, “the point is not the
points, the point is the POETRY”. The only reason slam poetry is a competition is to get the audience
to fill seats. Before slam, poetry readings were characteristically boring and not really an event any-
one was interested in attending. By adding the competition element to poetry, slam helps to build a
new, energetic, excited community around poetry. Please remind your students that, essentially, the
competition is a complete and total joke.
Disponível em: https://www.mudandinkteaching.org/news/2018/4/7/slam-poetry-101-an-introduction Acesso em: 29 de ago. 2021.

Read one example of slam poetry with the title “What teachers make”, by Taylor Mali, available
on https://taylormali.com/poems/what-teachers-make/,video on https://www.youtube.com/
watch?v=RxsOVK4syxU

He says the problem with teachers is You want to know what I make? I make parents tremble in fear when I
call home:
What’s a kid going to learn I make kids work harder than they ever
thought they could. Hi. This is Mr. Mali. I hope I haven’t
from someone who decided his best
called at a bad time,
option in life was to become a teacher? I can make a C+ feel like a Congressio-
nal Medal of Honor I just wanted to talk to you about some-
He reminds the other dinner guests
thing your son said today.
that it’s true what they say about and an A-­‐ feel like a slap in the face.
teachers: To the biggest bully in the grade, he
How dare you waste my time
said, “Leave the kid alone. I still cry
Those who can, do; those who can’t,
with anything less than your very best. sometimes, don’t you? It’s no big deal.”
teach.
I make kids sit through 40 minutes of And that was the noblest act of cour-
I decide to bite my tongue instead of his
study hall in absolute silence. No, you age I have ever seen.
and resist the temptation may not work in groups.
I make parents see their children for
Because we’re eating, after all, and No, you may not ask a question. who they are and what they can be.
this is polite conversation.
Why won’t I let you go to the bathroom? You want to know what I make? I make
I mean, you’re a teacher, Taylor. kids wonder,
Because you’re bored.
Be honest. What do you make? I make them question.
And you don’t really have to go to the
And I wish he hadn’t done that— asked bathroom, do you? I make them criticize.
me to be honest—
I make them apologize and mean it.
because, you see, I have this policy
I make them write. [...]
about honesty
if you ask for it, then I have to let you
have it.

235
5 - Now it’s your turn to make a slam poem! Using the topic of “Africa: its hidden story”, put together all
you have studied along these weeks about your own History. It’s time to tell this story with the structure,
intention and energy of a slam poem. Use the lines below.

PARA SABER MAIS:


Como se tornar um slam poet? Veja exemplos de slam poems. Disponível em: <https://www.youtube.
com/watch?v=9f8VcV8v2LE>. Acesso em: 29 ago. 2021.

REFERÊNCIAS:

WEIGEL, Adriana; RESCHKE, Tatiana. English and more! São Paulo: Richmond, 2020.

DESPEDIDA:
Chegamos ao final de mais um bimestre. Assim é a vida, precisamos fechar ciclos, cumprir etapas e não
pulá-las, porque é cumprindo cada uma delas que conseguimos ter a autoconfiança, coragem e compe-
tência de bater em novas portas para, então, adentrar naquilo que nos é novo e tão desejado para, mais
uma vez, completar novas belas etapas. See you!

236
SECRETARIA DE ESTADO DE EDUCAÇÃO DE MINAS GERAIS

PLANO DE ESTUDO TUTORADO


COMPONENTE CURRICULAR: ARTE
ANO DE ESCOLARIDADE: 2º ANO – EM
PET VOLUME: 04/2021
NOME DA ESCOLA:
ESTUDANTE:
TURMA: TURNO:
BIMESTRE: 4º TOTAL DE SEMANAS:
NÚMERO DE AULAS POR SEMANA: NÚMERO DE AULAS POR MÊS:

SEMANA 1

EIXO TEMÁTICO:
Conhecimento e Expressão em teatro.

TEMA/TÓPICO:
Percepção Dramática e Sensibilidade Estética: Análise de Produções de Teatro na Atualidade.

HABILIDADE(S):
Apreciação e análise de teatro contemporâneo.

CONTEÚDOS RELACIONADOS:
Teatro.

INTERDISCIPLINARIDADE:
Sociologia.

TEMA: Teatro e identidade - parte 1


Caro (a) estudante, nesta semana você identificará elementos que irão te ajudar a estabelecer relações
entre o Teatro contemporâneo, contextualização e identidade pessoal.

237
APRESENTAÇÃO

Cena do filme “Vista a minha pele” Disponível em: <https://www.ebc.com.br/sobre-a-ebc/noticias/2016/10/nacao-apresenta-edicoes-


que-debatem-o-preconceito-com-a-crianca-negra>. Acesso em:03 ago. 2021.

Uma das funções da Arte é estabelecer relações entre o ser humano e o mundo, buscando espaços para
que as experiências das pessoas possam se manifestar. Muitas das apresentações teatrais contempo-
râneas trabalham com a formação e a afirmação da identidade de um povo, carregando características
peculiares de um determinado lugar.
Pensar em identidade é lembrar dos antepassados, é participar de crenças e costumes, é possuir tra-
ços físicos e, também, e se relacionar com o território onde vive ou viveu. Muitos desses quesitos, se-
não todos, são elementos observados pelos produtores de teatro contemporâneo. Quando uma peça
teatral fala de identidade, as narrativas empregadas levam a plateia a se enxergar em cada movimento,
em cada figurino e em cada palavra falada. Esse fato pode contribuir na reafirmação de modelos co-
muns a um determinado povo.
A imagem mostrada acima, retrata uma cena do filme Vista a minha pele. O curta metragem problema-
tiza a vida de uma adolescente branca que vive em uma sociedade tomada pelo ideal negro. É uma troca
de realidade, a ideia é, através das cenas, mostrar como, na realidade, o negro é obrigado a encarar
inúmeros desafios por viver em uma sociedade caracterizada pelos valores da pessoa branca.
Essa produção nos desafia a pensar no outro, nas outras culturas e nos mostra como o teatro, e suas
variações, podem ser uma porta de entrada para debates de assuntos importantes para a sociedade.

PARA SABER MAIS:


Assista ao vídeo do filme “Vista Minha Pele” que trata da inversão de valores sociais. Disponível em:
<https://www.youtube.com/watch?v=FRq4fkkm5Iw>. Acesso em: 03 ago. 2021.

238
ATIVIDADES
1 - A partir do que você leu no texto acima, você considera que o filme “Vista minha pele” retrata a
realidade da sociedade brasileira? Justifique.

2 - A história do teatro nos conta que as narrativas sempre se alimentavam dos acontecimentos
mundiais para releitura das peças teatrais. Sendo assim, marque a opção ERRADA.
a) Os assuntos ligados às religiões gregas e romanas, também eram motivos de criação do teatro.
b) As peças teatrais renascentistas abriam espaço para o profano (fora da religião).
c) O teatro sempre foi a única forma de expressão de um indivíduo.
d) As narrativas teatrais, na história, são criadas e recriadas levando em consideração fatos verda-
deiros e fictícios.

3 - Tratando de identidade, construa um autorretrato (você desenhando você) buscando suas


características físicas. Discuta com alguém da sua família sobre a imagem que você criou.

239
SEMANA 2

EIXO TEMÁTICO:
Conhecimento e Expressão em teatro.

TEMA/TÓPICO:
Percepção Dramática e Sensibilidade Estética: Análise de Produções de Teatro na Atualidade.

HABILIDADE(S):
Apreciação e análise de teatro contemporâneo.

CONTEÚDOS RELACIONADOS:
Teatro.

INTERDISCIPLINARIDADE:
Sociologia.

TEMA: Teatro e identidade - parte 2


Caro (a) estudante, nesta semana você vai identificar elementos que irão te ajudar a estabelecer rela-
ções entre o teatro contemporâneo, contextualização e identidade pessoal. Vamos conhecer um pouco
as produções de teatro por povos segredados, indígenas e negros. Consideramos povos segregados,
aqueles excluídos por suas características físicas, culturais e ideológicas.
Você já parou para pensar que existem múltiplas produções artísticas feitas por povos que, não co-
mumente, aparecem nas mídias ou nos locais de espetáculos? Anualmente, construímos teatros nas
escolas e em outros lugares que afirmam pré-conceitos, em relação a estes povos, que ao invés de
fazer conhecidas suas histórias, ajudam a inferiorizar as culturas dos indígenas e dos negros, por
exemplo. A ideia aqui é saber que também os indígenas e os negros produzem arte, representada aqui
pelo teatro.
Podemos dizer que, no Brasil, as primeiras pessoas que tiveram contato direto com o teatro, foram os
indígenas. Eles receberam ensinamentos através das peças teatrais produzidas pelos jesuítas. Apesar
de não ser a maior manifestação de arte indígena, hoje é possível apreciar peças teatrais feitas pelos
próprios indígenas e que falam sobre suas origens, seus costumes, crenças e história. Algumas com-
panhias de teatro têm ministrado oficinas em tribos agregando conhecimentos das técnicas teatrais
para os indígenas. A exemplo disso, temos a CIA Art’Sacra que ofereceu oficinas para indígenas da tribo
Xerente no Tocantins em 2018.
O teatro africano é marcado pelo uso exagerado de indumentárias (objetos) que caracterizam suas cul-
turas. O uso de muitas cores que expressam a sensibilidade. A maioria de suas peças exalta a figura
humana representando os valores étnicos.

PARA SABER MAIS:


Leia o artigo no link abaixo e leia o texto que trata da criação do teatro regional e seu papel na socie-
dade. Disponível em: <https://ohoje.com/noticia/cultura/n/170238/t/teatro-regional-discute-assedio-
-pobreza-e-outros-temas-contemporaneos/>. Acesso em: 03 ago. 2021.

240
ATIVIDADES
1 - Faça uma pesquisa a respeito do teatro indígena e africano e descreva dois pontos em comum e dois
pontos divergentes, entre eles.

2 - Você acha interessante que outros povos, como os indígenas, tenham oficinas de teatro de pessoas
não indígenas? Justifique.

3 - Marque a opção correta.


a) Os indígenas e os negros precisam de aprender mais com os brancos sobre o teatro pois, eles
ainda estão atrás nessa prática.
b) O teatro africano é marcado pelo uso exagerado de indumentárias (objetos) que caracterizam
suas culturas. O uso de muitas cores que expressam a sensibilidade.
c) Os primeiros habitantes brasileiros a terem contato com o teatro foram os portugueses.
d) O teatro africano é marcado pelo uso exagerado de indumentárias (objetos) que caracterizam as
culturas europeias. O uso de muitas cores que expressam a sensibilidade.

241
SEMANA 3

EIXO TEMÁTICO:
Conhecimento e Expressão em teatro.

TEMA/TÓPICO:
Movimentos Artísticos em Teatro em Diferentes Épocas e Diferentes Culturas: Contextualização do Teatro na
História da Humanidade: Expressão Cênica e Teatral.

HABILIDADE(S):
Abrangência do teatro em diferentes períodos na história.

CONTEÚDOS RELACIONADOS:
Teatro.

INTERDISCIPLINARIDADE:
História.

TEMA: Teatro na história


Caro (a) estudante, nesta semana você vai contextualizar as produções de peças teatrais, discutindo a
importância de cada período na história. Entenderemos como o teatro sempre foi importante no desen-
volvimento da vida humana desde a sua origem.

APRESENTAÇÃO

Comédia ateniense - Neste relevo, o autor ateniense Menandro, do séc. IV a.C., segura uma máscara teatral na presença de uma musa.
Cópia romana de um original grego. Museus do Vaticano, Roma. Disponível em: <https://nationalgeographic.pt/133-vida-sapo/1286-ed-
especial-quotidiano-antiguidade-grecia-mar2017>. Acesso em: 03 ago. 2021.

Em toda a história, as pessoas sempre tiveram a necessidade de se comunicar usando o corpo, promo-
vendo gestos, criando danças, encenando algo. Nesse contexto, o teatro surge como um facilitador na
comunicação entre os assuntos políticos e as pessoas.

242
O teatro teve origem na Grécia, aproximadamente, no ano IV a.c. inicialmente, as peças teatrais acon-
teciam anualmente nos cultos ao deus Dionísio, deus do vinho. Nessa época, surgiram vários prédios
chamados de teatro e anfiteatro. Com o passar do tempo, o teatro grego começou a tratar a vida po-
lítica com humor e conhecimento. Isso também aconteceu no império romano, onde os atores eram
perseguidos por satirizar algumas figuras públicas.
A história nos mostra que as peças teatrais traduziam com clareza os movimentos de cada época.
Na cultura medial, por exemplo, quando a igreja católica imperava, o teatro era de caráter religioso
levando os fiéis à devoção. Outro período marcante na história do teatro foi o renascentista, século XV
e XVI, quando as apresentações eram baseadas em temas populares e cômicos, explorando as novas
invenções da época.
Podemos dizer também que, além da característica comunicativa, o Teatro tem o “poder” de dialogar
com as novas ideias, promovendo conhecimento e discutindo conceitos relevantes para a formação
das pessoas. Esse fato fica evidente quando, no século XX, as peças teatrais invadem o cinema proble-
matizando eventos inéditos para a humanidade. A crescente industrialização mundial foi retratada, em
1936, no filme Tempos Modernos, do ator Charles Chaplin.
Na atualidade, as manifestações teatrais têm ocupado não somente os palcos de grandes teatros, mas
também espaços públicos, fazendo comédias e discutindo variadas questões que envolvem nosso con-
texto. Violência, política, multiculturalismo, identidade e história, são algumas dessas questões.

PARA SABER MAIS:


Acesse o link e leia a matéria sobre a história do teatro. Disponível em: <https://www.todamateria.com.
br/historia-do-teatro/>. Acesso em: 03 ago. 2021.

ATIVIDADES
1 - Onde você vê o teatro no dia a dia? Você tem o costume de sair para assistir aos espetáculos teatrais?

2 - Como o teatro se apresenta na atualidade?


a) Em apresentações festivas nas comunidades populares.
b) Nos palcos de grandes salões em metrópoles pelo mundo.
c) Com caráter romântico, assim como no século XIX.
d) Em diversos lugares.

3 - “Podemos dizer também que, além da característica comunicativa, o Teatro tem o “poder” de dialogar
com as novas ideias, promovendo conhecimento e discutindo conceitos relevantes para a formação
das pessoas”. Explique, com suas palavras, o que você entendeu dessa frase.

243
SEMANA 4

EIXO TEMÁTICO:
Conhecimento e Expressão em Artes Audiovisuais.

TEMA/TÓPICO:
Elementos Formais da Obra de Artes Audiovisuais.

HABILIDADE(S):
Estrutura da obra audiovisual.

CONTEÚDOS RELACIONADOS:
Audiovisual.

TEMA: Prática audiovisual


Caro (a) estudante, nesta semana você vai reconhecer os elementos estruturais das obras audiovisuais.

APRESENTAÇÃO
Ao pesquisar a prática de produção audiovisual, saberemos que são inúmeros os elementos formais
que caracterizam essa produção. Tais elementos irão depender de qual obra está sendo construída
(filme, instalação, animação, etc, documentário, etc.). Nesta semana, buscaremos analisar de forma
técnica a produção de uma obra de arte audiovisual a partir da construção de um filme.
Sinopse ou storyline: história contada em uma frase, um parágrafo, serve como ponto de partida para
o autor e como cartão de visita do projeto (filme ou vídeo), no processo inicial de captação de recursos.
Argumento ou pré-roteiro: Texto literário, com a história completa com todos seus elementos dramáti-
cos. É a parte mais criativa do processo. Semelhante a um conto, porém mais objetivo, sem “literatices”
e normalmente sem diálogos, apenas referência a estes.
Exemplo: De longe e saltitante vem Ana Maria. Radiante em seus incompletos 16 anos, os cachos loiros
pulando como molas enquanto ela anda, não vê a hora de colocar seu novo biquíni. No caminho encontra
Seu Pedro, sorveteiro de praia. Eles se cumprimentam.
Roteiro: Texto técnico detalhado e descritivo, serve para levantamento das necessidades de cada cena
e como guia de filmagem. Por convenção os diálogos são escritos com travessão.
Story Board: história em quadrinhos que descreve com desenhos cada um dos planos de um filme se-
gundo o roteiro. Não é obrigatório, mas ajuda muito.
Decupagem ou Análise Técnica: Com o roteiro pronto, começa o processo de levantamento de neces-
sidades cena a cena. Este processo é conhecido como decupagem. É aqui que se decide, baseado no
custo e na opção estética, qual será o meio usado para o projeto.
Claquete: uma pequena lousa onde está marcado o número da cena, do plano e do take além do nome
do vídeo e um espaço para observações. Ela deve ser preenchida e filmada por cerca de 10 segundos
antes de cada take. Se o som estiver sendo captado por um gravador externo (não pela câmera) ela
deve ser ditada e ter capacidade de produzir um ruído seco que corresponda a um movimento rápido e
preciso na imagem (normalmente uma haste de madeira presa à claquete é usada para isto). Este som
e movimento vão permitir que o vídeo e o áudio sejam rapidamente sincronizados na edição.

244
Ficha de Filmagem:  Durante a filmagem, alguém da equipe, geralmente o assistente de direção, vai
preenchendo uma ficha de filmagem, onde se anota de forma rápida o que aconteceu em cada take e
quais foram os melhores.
Decupagem do material filmado: Depois de terminada a filmagem, usando a ficha de filmagem como
guia, são escolhidos os takes que serão utilizados na edição. E nada mais é que uma lista dos takes uti-
lizáveis, escritas no formato cena/plano/take.
Digitalização: passagem do material escolhido para o computador onde será feita a edição. Hoje em
dia os equipamentos de edição (dos mais baratos aos mais sofisticados) são baseados em computa-
dor e utilizam de um método chamado “Edição não linear” onde após a digitalização se tem um grande
controle sobre o material. O método mais antigo, edição linear, embora não perca tempo digitalizando
o material, é muito menos flexível e mais demorado.
Edição e Finalização: é o processo onde o vídeo ganha sua forma final, com todas as transições, letrei-
ros, sons e efeitos.
A construção da obra audiovisual levará em conta os recursos a serem usados, isso quer dizer que al-
guns dos processos citados acima podem sofrer alteração ou até mesmo não existir. Outro quesito
importante é o desenvolvimento da história usada, ela pode ser inventada ou verídica.

PARA SABER MAIS:


Assista ao vídeo que trata sobre a criação de várias linguagens das artes audiovisuais. Disponível em:
<https://www.youtube.com/watch?v=MkjCW86_ieI>. Acesso em: 03 ago 2021.

ATIVIDADES
1 - Indique um filme de que você mais gosta. Analise e escreva o que mais lhe atrai nesse filme, por que
você gosta tanto dele.

2 - A produção de filmes só foi possível com a invenção da fotografia anos antes. Explique por que isso
é verdade.

245
3 - Coloque V para verdadeiro e F para falso.
(  ) A arte audiovisual está ligada à produção de pinturas e desenhos somente.
(  ) É possível construir um filme caseiro usando o celular e massinhas de modelar.
(  ) Os story boards são produções em quadrinhos.
(  ) Podemos dizer que a sinopse é o resumo do filme.
a) V V F F.
b) V F V F.
c) F V V V.
d) F V V F.

246
SEMANA 5

EIXO TEMÁTICO:
Conhecimento e Expressão em Artes Audiovisuais.

TEMA/TÓPICO:
Relações entre as Artes Audiovisuais e a Arte Contemporânea.

HABILIDADE(S):
Estudo de diferentes produtos audiovisuais.

CONTEÚDOS RELACIONADOS:
Audiovisual.

TEMA: O áudio e o visual


Caro (a) estudante, nesta semana você vai entender sobre as relações de alguns processos audiovisuais.

APRESENTAÇÃO
A arte audiovisual é uma vertente que nos envolve pela sua capacidade de conectar várias linguagens,
como: a fotografia, a música, o desenho, a computação gráfica, a moda, a modelagem, entre outros
elementos artísticos que, unidos, transformam o imaginário das pessoas. Falar de processos audiovi-
suais é conhecer sobre a animação, o cinema, as instalações de painéis eletrônicos, a criação de jogos,
os documentários, etc. são inúmeras as possibilidades de estudo e criação dos produtos audiovisuais,
entretanto, vamos tentar analisar alguns.
Animação. Você já assistiu algum filme no qual aparecem pessoas de verdade e personagens ilustrados
ou, desenhados, dialogando como se estivessem frente a frente? Já se perguntou como isso acontece?
Atualmente, existem muitos programas de computador (programas gráficos) que possibilitam animar
personagens e colocá-los na mesma cena que um humano verdadeiro. Uma técnica bastante usada na
animação de tais personagens é o Stop Motion. Neste processo, o artista cria animações com o uso de
fotografias retiradas de cada movimento feito com os personagens criados. Com as fotos já prontas,
é hora de inseri-las em um programa gráfico que dará velocidade nas passagens das fotos criando uma
sensação de movimento. Esse tipo de animação pode ser produzido em casa usando uma câmera do
celular, papeis coloridos e massinhas de modelar. A fotografia abaixo ilustra bem o processo.

Disponível em: <https://www.turismosantos.com.br/?q=pt-br/node/16294>. Acesso em: 03 ago. 2021.

247
Instalação. Um dos processos mais interessantes no campo das artes audiovisuais, é a instalação.
Muitos modelos de instalações são encontrados em galerias de arte ou nas ruas, em forma de painéis
eletrônicos.

“No vídeo wall da entrada, Peter Coffin apresenta várias imagens de animais presos e soltos, em 30 moni-
tores, na tentativa de associar a uma jaula. Já na produção audiovisual ‘I See a Woman Crying’, da holan-
desa Rineke Dijsktra, crianças estão fazendo observações sobre uma obra de Picasso, que o público não
consegue ver e por isso vai aguçando a imaginação de cada visitante. ”
Retirado de: <https://www.soubh.com.br/noticias/variedades/novas-obras-inhotim-2018>. Acesso em: 04 mar. 2021.

Cinema. A produção de cinema é o campo da arte audiovisual que mais cresceu no mundo. Muitos mo-
tivos, que não serão tratados neste material, fizeram com que isso acontecesse. A produção cinemato-
gráfica conta com vários profissionais desenvolvendo variadas funções na produção de um filme. Entre
eles, podemos citar: o produtor que planeja, executa e acompanha o projeto até o final, o diretor de
fotografia que passa todo o roteiro para as imagens da tela, o editor que executa todo o final do projeto,
o ator encarregado de representar os papeis, o roteirista que cria as histórias, o colorista que trabalha
com a trilha sonora e o uso das imagens e o diretor que orienta a produção de tudo.

PARA SABER MAIS:


Assista ao vídeo que trata sobre a produção específica das artes audiovisuais. Disponível em: <https://
www.youtube.com/watch?v=MjHQ6-LgnXs>. Acesso em: 03 ago. 2021.

ATIVIDADES
1 - A arte audiovisual é uma expressão que (EXCETO):
a) Se utiliza do som e do vídeo como elementos fundamentais.
b) Cria trabalhos essencialmente estáticos.
c) Se utiliza da fotografia nas produções.
d) Mescla ideias visuais e auditivas para construir sensações.

248
2 - Comente sobre o filme que você citou na semana 4, dizendo se a imagem e o áudio são elementos
bem construídos.

3 - Indique a alternativa que não é um tipo de arte audiovisual.


a) Colagens.
b) Cinema.
c) Desenho animado.
d) Cine documentário.

249
SEMANA 6

EIXO TEMÁTICO:
Conhecimento e Expressão em Artes Audiovisuais.

TEMA/TÓPICO:
Relações entre as Artes Audiovisuais e a Arte Contemporânea.

HABILIDADE(S):
Estudo de diferentes produtos audiovisuais.

CONTEÚDOS RELACIONADOS:
Artes audiovisuais.

INTERDISCIPLINARIDADE:
Física.

TEMA: Tecnologia e Produção Audiovisual na Arte Contemporânea.


Caro (a) estudante, nesta semana você analisará o uso das tecnologias nas produções audiovisuais con-
temporâneas.

APRESENTAÇÃO
As artes audiovisuais são trabalhos expressados por meios visuais e sonoros simultaneamente, produ-
zindo mostras de que a imagem humana é comunicada não ao vivo frente a frente com o público, mas
por máquinas. Eles incluem artes sonoras e visuais, vídeo-dança, cinema e televisão, por exemplo, sen-
do obras desta arte, um filme, uma história de ficção televisiva, um videoclipe ou um comercial publi-
citário. Existem também obras de arte mais abstratas, como videoarte, escultura sonora e instalações
sonoras, em geral.
A produção audiovisual agrega valores de tempo e espaço, nos fazendo “viajar” pelas sensações causadas.
Na arte contemporânea, as obras audiovisuais têm usufruído de muitos recursos em sua construção,
possibilitando resultados surpreendentes. O avanço tecnológico associado à capacidade humana, pos-
sibilita a criação e manipulação de cores através do uso da luz, a produção de sons abstratos que simu-
lam vários ambientes, a criação de imagens a partir de elementos naturais e eletrônicos, são caracte-
rísticas das obras.
Um grande exemplo que podemos citar do uso prático das tecnologias na produção de obras audiovi-
suais é a criação de story board (construção gráfica que revela o conteúdo de alguma produção audio-
visual). Normalmente é feito como esboço do trabalho a partir de programas gráficos que auxiliam na
montagem dos quadros e nos desenhos.
Voltamos à imagem da semana cinco. O museu de arte contemporânea, Inhotim, localizado na cidade
de Brumadinho em Minas Gerais, propôs uma mostra de artes audiovisuais em 2018, com a finalidade de
oferecer ao público experiências diversas através dos sons e das imagens.

250
Vejamos:

 Foto: Camila Saraiva

“No vídeo wall da entrada, Peter Coffin apresenta várias imagens de animais presos e soltos, em 30 moni-
tores, na tentativa de associar a uma jaula. Já na produção audiovisual ‘I See a Woman Crying’, da holan-
desa Rineke Dijsktra, crianças estão fazendo observações sobre uma obra de Picasso, que o público não
consegue ver e por isso vai aguçando a imaginação de cada visitante.”
Retirado de: <https://www.soubh.com.br/noticias/variedades/novas-obras-inhotim-2018>. Acesso em: 04 mar. 2021.

Na mesma exposição, existiu uma obra que mostrava reações de crianças observando algum trabalho
do artista, Pablo Picasso. A grande ideia era que, quem visitava o trabalho não poderia ver as imagens
as quais as crianças estavam apreciando, somente suas reações eram expostas ao público. Isso nos
faz imaginar coisas que talvez não condizem com a realidade. Qual seria a imagem observada pelas
crianças? Por que elas emitiam tais sons? Suas expressões eram verdadeiras? Estas eram perguntas
possíveis de serem feitas no momento da visitação.
A arte contemporânea é diversificada, ela possui a capacidade de encantar com milhares de possibili-
dades existentes. Precisamos conhecer e pesquisar um pouco mais sobre cada uma delas.

PARA SABER MAIS:


Visite os vídeos indicados nos links. Cada um deles mostra mais sobre as possibilidades de uso das
novas tecnologias nas produções de arte audiovisual. Disponível em: <https://www.youtube.com/wat-
ch?v=dTXrnOWIl7g> e <https://www.youtube.com/watch?v=_e4t_mRJyBI>. Acesso em: 03 ago. 2021.

251
ATIVIDADES
1 - Nesta semana faremos uma atividade maior. A proposta é a construção de um pequeno filme usando
a prática do stop motion (para quem conseguir) ou o flip book.
• Na criação do filme stop motion, use papéis coloridos para construir cenários, recipientes vazios
(potes de iogurtes, caixinhas de fósforo, etc.) na criação de personagens e cenários. Use massi-
nhas de modelar para criar personagens que irão se mover. Coloque a câmera do celular, de ma-
neira que ela não se movimente, de frente para o cenário e comece a tirar fotos, movimentando
os personagens e objetos que preferir. Crie uma história interessante. Se divirta com as cenas.

Disponível em: <https://www.olharconceito.com.br/noticias/exibir.asp?id=5562&noticia=stop-motion-entenda-a-tecnica-de-


animacao-e-inscreva-se-no-curso-do-sesc>. Acesso em: 10 ago. 2021.

• Quem não conseguir fazer a animação acima, poderá construir um flip book. Consiste em pro-
duzir desenhos nas páginas de um bloquinho de anotações. Em cada página, é preciso criar de-
senhos se movimentando, para que no final seja observado uma ideia de movimento ao folear as
páginas. Use cores para que o trabalho fique ainda mais interessante.

Disponível em: <https://mariaeusebio12av1.wordpress.com/historia/brinquedos-opticos/flipbook/>. Acesso em: 10 de ago. 2021.

Querido (a), estudante. Finalizamos o PET número 4 do ano de 2021, espero que tenha gostado dos
assuntos tratados e aprendido um pouco mais sobre a arte em nossas vidas. Desejo muita força para
continuarmos e paz em tudo que for realizar. Não esqueça de se cuidar e de cuidar das pessoas! Gran-
de abraço do professor de Arte, André Mázio.

252
SECRETARIA DE ESTADO DE EDUCAÇÃO DE MINAS GERAIS

PLANO DE ESTUDO TUTORADO


COMPONENTE CURRICULAR: EDUCAÇÃO FÍSICA
ANO DE ESCOLARIDADE: 2º ANO – EM
PET VOLUME: 04/2021
NOME DA ESCOLA:
ESTUDANTE:
TURMA: TURNO:
BIMESTRE: 4º TOTAL DE SEMANAS:
NÚMERO DE AULAS POR SEMANA: NÚMERO DE AULAS POR MÊS:

SEMANA 1

EIXO TEMÁTICO:
Esportes.

TEMA/TÓPICO:
Futebol – Aprimoramento técnico-tático e regras.

HABILIDADE(S):
Analisar os elementos técnicos de cada modalidade, analisar táticas das modalidades em situações de jogo,
analisar regras dos diferentes esportes, alterar regras de acordo com o interesse do grupo, espaços e materiais.

CONTEÚDOS RELACIONADOS:
História, regras, técnicas e táticas.

INTERDISCIPLINARIDADE:
Língua portuguesa.

TEMA: Futebol
Prezado (a) estudante, iniciaremos nosso PET 4 abordando o esporte preferido da maioria dos brasi-
leiros, o Futebol. Nesta semana de estudos aprenderemos um pouco sobre a origem, regras e algumas
curiosidades desse esporte que encanta o mundo todo e que para nosso país em época de copa do
mundo. Bons estudos!

ORIGEM DO FUTEBOL
Não se sabe ao certo como o esporte se iniciou, mas encontra-se na história antiga, vários tipos de
esportes praticados com o uso da bola e muito semelhantes ao Futebol, embora não possuísse a orga-
nização e regras atuais. O mais parecido com o Futebol atual é um jogo encontrado na Grécia, por volta
do século I a.C., esse jogo era praticado por soldados que se dividiam em duas equipes de nove pessoas
e jogavam com uma bola cheia de areia.

253
Durante a Idade Média, um jogo muito parecido com o futebol se manifestava, porém este fazia o uso
frequente da violência entre os jogadores. Era composto de dois times com 27 jogadores cada e era
permitido qualquer tipo de agressão física para conseguir a vitória.
Foi na Inglaterra, porém, que as regras atualmente conhecidas por nós foram implementadas. Lá o jogo
foi sistematizado, o tamanho do campo foi determinado e passou-se a usar a bola de couro cheia de
ar. Aos poucos o jogo foi se popularizando por toda a Europa e em 1848 foi publicado o primeiro livro de
regras para que o jogo pudesse ser praticado da mesma maneira em qualquer lugar do mundo.
O Futebol foi trazido ao Brasil por Charles Miller um Inglês que ao se estabelecer no Brasil em 1894,
trouxe consigo uma bola de futebol e um livro de regras. O Futebol logo se tornou uma paixão nacional,
sendo o esporte mais praticado e com maior número de espectadores em nosso país.
Adaptado de ARAUJO, Ana Paula. Futebol. Disponível em: <https://www.infoescola.com/esportes/futebol/>. Acesso em: 05 de Julho de 2021.

REGRAS DO FUTEBOL
As regras do Futebol sofreram muitas mudanças desde a publicação do primeiro livro de regras em
1848 até os dias de hoje. Recentemente tivemos uma grande mudança que dividiu opinião, a presença
do VAR (Vídeo assistant referee) que traduzindo nada mais é do que a inclusão dos árbitros de vídeo, um
grupo de árbitros que revisam as jogadas para checar se houve alguma infração em algum lance capital
do jogo como por exemplo gols e pênaltis.
O Futebol possui 17 regras e seu livro oficial pode ser acessado na integra e gratuitamente através do se-
guinte link: <https://conteudo.cbf.com.br/cdn/202008/20200818145813_835.pdf>. Acesso em: 05 jul. 2021.
Abaixo segue o resumo das principais regras que você encontrará no livro:
• O campo de jogo do Futebol deve ter entre 120m a 90m de comprimento e 90m a 45m de largura.
• A trave de jogo mede 7,32m de comprimento e 2,44m de altura.
• Cada equipe é formada por 11 jogadores sendo um goleiro.
• Os jogadores são obrigados a usar camisas numeradas e de mesma cor, exceto o goleiro que por po-
der tocar a bola com as mãos, necessita atuar com camisa de cor diferente dos demais jogadores.
• O arbitro é a autoridade máxima do jogo, entre outras funções compete a ela assinar todas as
infrações ocorridas durante a partida.
• O arbitro assistente tem como prioridade assinalar o impedimento quando este acontece, mas
pode e deve auxiliar o arbitro na marcação de outras infrações durante o jogo.
• Cada partida tem duração total de 2 tempos de 45 minutos, com intervalo máximo de 15 minutos.
• Segurar, puxar, empurrar, calçar, obstruir e tocar a bola com as mãos (exceto o goleiro em sua
área) são exemplos de faltas que podem ser cometidas durante o jogo e devem ser punidas com
um tiro livre direto.
• Caso uma das ações acima seja cometida dentro da área, um pênalti deverá ser assinalado.
• Cartão amarelo é uma fora de advertir um jogador que comete uma falta dura. O cartão vermelho
é usado para expulsar o jogador por fazer uma segunda falta mais dura que mereça um segundo
cartão amarelo ou até mesmo de forma direta quando o jogador comete uma falta violenta.
• A regra de impedimento diz que: “estará em posição de impedimento o jogador que no momento
do passe, se encontrar em seu campo de ataque, mais próximo da linha de fundo que o penúltimo
defensor ou a bola”. A regra do impedimento é um pouco complexa, por isso disponibilizamos um
link no campo “PARA SABER MAIS” que explica de forma muito didática essa regra tão polêmica.

PARA SABER MAIS:


O impedimento é sem dúvida a regra mais complexa do Futebol. Clique no link a seguir para entender de
uma vez por todas como essa regra é aplicada. Disponível em: <https://www.youtube.com/watch?v=J-
NA2f3l9oaE>. Acesso em: 05 jul. 2021.

254
CURIOSIDADES DO FUTEBOL
1. “FOR THE GAME, FOR THE WORLD”, esse é o lema da FIFA que traduzindo diz que o Futebol é o
“jogo do mundo”. De fato o Futebol arrasta admiradores ao redor do mundo. A última final da copa
do mundo de 2018 entre França e Croácia por exemplo teve 3,572 bilhões de espectadores em
todo mundo, ou seja, mais da metade da população estava assistindo ao duelo entre Franceses e
Croatas. Vejamos agora mais algumas curiosidades do Futebol.
2. CELESTE OLÍMPICA E PRIMEIRA COPA DO MUNDO: O Uruguai era a potência do Futebol nas dé-
cadas de 20 e 30. Venceu as duas edições das olimpíadas de 1924 e 1928, período que ainda não
existia a copa do mundo. Por essa razão a primeira copa do mundo foi realizada em solo uruguaio
em 1930 e vencida pelos donos da casa. Eles voltaram a vencer a copa de 1950 contra a seleção
brasileira no que ficou conhecido como “Maracanaço” em alusão ao fracasso da seleção brasileira
jogando em casa, em pleno estádio Maracanã.
3. BRASIL – ÚNICO PAÍS A DISPUTAR TODAS AS COPAS: Mesmo com o vexame do “Maracanaço” na
copa de 1950 e o 7x1 para a Alemanha em 2014, o Brasil é o único país que disputou todas as edições
da copa do mundo, sendo também o país que mais levantou a taça com cinco conquistas até agora.
4. BOLA DE COURO E BEXIGA DE BOI: Até os anos 50 era comum usar bexiga de boi como câmera de
ar e couro de boi curtido como revestimento da bola. A partir de 1958 a bexiga começou a dar lugar
a câmera de ar e o couro curtido aos materiais sintéticos. Com isso as bolas foram ficando cada
vez mais modernas e fáceis de jogar.
5. A TAÇA ROUBADA: A taça Jules Rimet, primeira taça da copa do mundo, foi roubada DUAS vezes.
A primeira foi na Inglaterra em 1966 enquanto estava exposta para visitação. A taça foi achada
7 dias depois graças a ajuda e ao faro de um cachorro de nome Pickles. Em 1970 a taça ficou de
vez com o Brasil após ter vencido a copa do mundo por três vezes antes que as demais seleções.
Porém o objeto que seria orgulho para todos os Brasileiros, foi roubado da sede da CBF em 1983. A
polícia chegou a prender os envolvidos, mas a taça nunca foi recuperada. Acredita-se que a taça
de ouro maciça tenha sido derretida e vendida clandestinamente.

Foto da seleção Brasileira de 1970 que conquistou a copa do mundo pela terceira vez garantindo a posse definitiva da taça Jules Rimet
que seria roubada em 1983. Em pé: Carlos Alberto, Brito, Piazza, Felix, Clodoaldo, Everaldo. Agachados: Jairzinho, Rivelino, Tostão, Pelé,
Paulo Cézar Caju. Disponível em: https://commons.wikimedia.org/wiki/File:Brazil_national_team_1970.jpg. Acesso em 08/07/2021

255
6. NÚMEROS NA CAMISA: Os números na camisa não existiam até o ano de 1933 quando dois times
ingleses resolveram numerar as camisas dos jogadores a fim de ajudar na identificação do jo-
gador pelos árbitros e locutores. A numeração das camisas só se tornou obrigatória na copa do
mundo de 1950.
7. USO DE CARTÕES: Até a copa de 1970 não existiam cartões vermelhos e amarelos como forma
de punição aos jogadores. As diferentes línguas faladas em vários países dificultavam a vida dos
árbitros na condução do jogo. Foi aí que surgiu a ideia de se usar cartões inspirados nas cores dos
semáforos (vermelho e amarelo) como linguagem universal para advertir e expulsar jogadores por
suas condutas em campo.
8. MAIOR JOGADOR DE TODOS OS TEMPO: O brasileiro Edson Arantes do Nascimento, o Pelé, é con-
siderado por vários órgãos ligados ao Futebol como o maior jogador de todos os tempos. Nascido
na cidade de Três Corações em Minas Gerais, Pelé é até hoje o jogador mais jovem a vencer uma
copa do mundo. Ele tinha apenas 17 anos quando ajudou o Brasil a vencer sua primeira copa em
solo Sueco no ano de 1958.

PARA SABER MAIS:


Como visto anteriormente, Pelé foi o maior nome do Futebol de todos os tempos. Porém sua trajetória
não foi fácil. Pelé e outros diversos atletas negros do Futebol e de outros esportes, quase sempre en-
frentaram muito preconceito durante a carreira. Clique no link a seguir para assistir ao documentário
intitulado “O negro no Futebol Brasileiro”. Disponível em: <https://www.youtube.com/watch?v=ETdO-
P7pajJ0>. Acesso em: 05 jul. 2021.

ATIVIDADES
1 - Descreva a origem do Futebol no Brasil. Como e quando o esporte chegou, como e por que se tornou
o esporte mais popular e mais praticado em nosso país.

2 - Assinale (V) para sentenças verdadeiras e (F) para sentenças falsas.


(  ) O Futebol foi desenvolvido pelos Ingleses em parceria com os Alemães.
(  ) Cartões vermelhos e amarelos são usados desde a primeira copa do mundo para expulsar e advertir
jogadores.
(  ) As equipes são formadas por 11 jogadores mais um goleiro.
(  ) Um campo de Futebol pode ter largura máxima de 120m.
(  ) Ao arbitro assistente (bandeirinha), cabe somente a marcação do impedimento quando este acontece.
(  ) Os números nas camisas são opcionais e ajudam muito os árbitros e locutores a identificaram os
jogadores.

256
3 - “A regra é clara!” – Este é um bordão muito antigo, usado por um ex comentarista de arbitragem em
jogos de Futebol. Você acha mesmo a regra do Futebol clara? Assinale a alternativa correta em relação
às afirmações das regras do esporte.
a) Um campo de Futebol pode ser feito num quadrado gramado de 90m x 90m.
b) O cartão vermelho só pode ser aplicado se um jogador já tiver recebido cartão amarelo antes.
c) Um jogo de Futebol que começou as 21h, vai terminar exatamente as 22h e 30 minutos.
d) Os jogadores não são obrigados a usar camisas com numerações diferentes.
e) Toda falta dentro da área deve ser assinalado um pênalti.

257
SEMANA 2

EIXO TEMÁTICO:
Esportes.

TEMA/TÓPICO:
Futsal – Aprimoramento técnico-tático e regras.

HABILIDADE(S):
Analisar os elementos técnicos de cada modalidade, analisar táticas das modalidades em situações de jogo,
analisar regras dos diferentes esportes, alterar regras de acordo com o interesse do grupo, espaços e materiais.

CONTEÚDOS RELACIONADOS:
História, regras e técnicas.

INTERDISCIPLINARIDADE:
Língua portuguesa.

TEMA: Futsal
Prezado (a) estudante, nesta semana estudaremos o Futsal, um esporte derivado do Futebol que surgiu
no início dos anos 30 como uma alternativa à falta de espaços para campos de Futebol e que hoje é o
esporte mais praticado nas escolas brasileiras. Bons estudos!

ORIGEM DO FUTSAL
O futsal teve origem em Montevidéu, no Uruguai, em 1934 com o nome de “Indoor Football”. O esporte
surgiu da necessidade de criar uma alternativa para a falta de campos de futebol na capital Uruguaia.
Vale lembrar que na década de 1930 o Uruguai é que era o país do Futebol, vencedor das olimpíadas
de 1924 e 1928 além da copa do mundo de 1930, o país vizinho respirava futebol e muitas vezes faltava
campos para o número de pessoas que queriam jogar. Assim o professor, Juan Carlos Ceriani Gravier
idealizou o jogo e adaptou sua prática a locais fechados. Dai o nome “Indoor Football”.
A modalidade chegou ao Brasil no ano seguinte, em 1935, através dos professores João Lotufo e As-
drubal Monteiro que a batizaram de “Futebol de salão” devido a mesma ser jogada em local fechado.
Durante a década de 1950 surgiram várias federações estaduais de Futebol de salão por todo o Brasil.
O primeiro livro de regras foi publicado em 1957 e em 1971 surgiu a FIFUSA (federação internacional de
futebol de salão).
A FIFUSA organizou as três primeiras edições da copa do mundo de Futebol de salão, em 1982, 1985 e
1988. A FIFA que até então administrava apenas o Futebol de campo, resolveu organizar sua primeira
copa do mundo 1989. Como os direitos ao nome “Futebol de salão” pertenciam a FIFUSA, a FIFA resolveu
rabatizar o esporte como FUTSAL, nome que prevalece até hoje. Somados os mundiais organizados
pela FIFUSA e pela FIFA, a seleção brasileira é a maior vencedora do torneio com sete conquistas no
total, sendo duas pela FIFUSA e cinco pela FIFA.
Adaptado de CUISCA, Alana. Futsal. Disponível em: https://www.educamaisbrasil.com.br/enem/educacao-fisica/futsal.
Acesso em: 05 de Julho de 2021.

258
REGRAS DO FUTSAL
As regras do Futsal são muito semelhantes às do Futebol, haja visto que um esporte teve inspiração
no outro. Basicamente o livro de regras do Futsal é composto pelas mesmas regras do Futebol com
exceção do impedimento. Conheça abaixo as principais diferenças que o Futsal apresenta em relação
ao Futebol.
• A quadra de jogo deverá ter entre 38m e 42m de comprimento e 18m e 25m de largura.
• Cada equipe é formada por 5 jogadores, sendo 4 na linha mais um goleiro.
• A partida é dirigida por 4 árbitros. Dois atuando em quadra, sendo um o árbitro principal e outro
o árbitro auxiliar e outros dois atuando fora de quadra, sendo um o “anotador” que é responsável
por fazer as anotações do jogo (gols, cartões, etc.) e o outro o “cronometrista” responsável por
marcar o tempo de jogo.
• Uma partida oficial tem duração de dois tempos de 20 minutos “parados”. Diferentemente do
Futebol, toda vez que o jogo de Futsal é parado por uma falta ou por saída da bola de quadra, o
relógio é parado pelo cronometrista e só volta a ser acionado quando a bola volta a ser disputada
em jogo.
• Limite de faltas: No Futsal cada equipe pode cometer no máximo 5 faltas durante cada tempo
de jogo. Se por acaso cometer uma 6ª falta, essa e todas as demais serão cobradas a 10m do gol
sem barreira.
• Arremesso de meta: Diferente do “tiro de meta” do Futebol que é cobrado com os pés, no Futsal
essa cobrança é feita pelo goleiro e com as mãos.
• Lateral: No Futebol o “arremesso lateral” é cobrado com as mãos. No Futsal essa cobrança é feita
com os pés.
• Limite de recuos de bola para o goleiro: No Futebol o time pode recuar a bola para o goleiro
quantas vezes quiser, desde que este jogue com os pés. No Futsal é permitido recuar a bola para
o goleiro, que também deve jogar com os pés, porém é válido apenas um recuo a cada posse de
bola do time.
• Cartão vermelho: No Futebol se um jogador recebe cartão vermelho ele está expulso e seu time
ficará com um jogador a menos até o fim do jogo. No Futsal se o jogador receber cartão verme-
lho, estará expulso, porém seu time ficará com um jogador a menos somente durante 2 minutos
ou se seu time levar um gol. Após acontecer um ou outro, o time pode colocar um jogador reser-
va no lugar do jogador expulso.

FUNDAMENTOS DO FUTSAL
O Futsal assim como o Futebol classificado como esporte de invasão territorial são aqueles em que uma
equipe deve invadir o território do adversário para acertar a bola num alvo (gol, cesta, etc) e ao mesmo
tempo proteger o seu alvo, evitando que o adversário o conquiste.
Todos os esportes de invasão territorial possuem fundamentos, que são os movimentos básicos ne-
cessários para invadir o território do adversário e defender o próprio território. Todos os fundamentos
possuem uma técnica, que nada mais é do que a maneira mais eficaz que este fundamento é executado.
Vamos conhecer agora os principais fundamentos do Futsal.
• PASSE: É o ato de passar a bola para um companheiro de equipe. Geralmente o passe é feito com
a parte interna do pé, entretanto é possível usar qualquer parte do pé ou do corpo para passar
a bola, como o calcanhar, joelho, coxa, peito e cabeça. O importante é a bola chegar ao jogador
melhor colocado.
• RECEPÇÃO: Também conhecida como domínio, é o ato de receber e dominar a bola que foi pas-
sada por um companheiro. Geralmente é feita com a sola dos pés, pisando e dominando a bola.
Entretanto pode ser feita com qualquer parte do corpo como peito, cabeça, coxa. O importante
é ter o domínio da bola.

259
• CONDUÇÃO: Consiste em deslocar com a bola pela quadra, dando pequenos toques na mesma,
sempre mantendo o domínio e controle. Geralmente é feita com a parte de dentro, de fora ou até
sola dos pés. No Futsal os toques costumam ser curtos, já que o espaço é bem menor que no
Futebol de campo.
• CHUTE: Também conhecido como finalização, é a forma mais eficaz de marcar gols. Geralmente
é feito com o dorso (peito) do pé, mas pode ser feito também com a parte interna, externa ou até
de “bico”.
• CABECEIO: É usado para dominar, passar ou até finalizar a jogada. Também é muito usado para
defender e afastar a bola da meta defendida. Para se ter um bom cabeceio é necessário estar de
olhos abertos e fazer o movimento correto, sempre tocando com a testa na bola.
• DRIBLE: É o ato de passar pelo adversário, na direção da meta atacada, conduzindo a bola. Ao
adversário cabe tentar uma roubada de bola ou uma antecipação para evitar o drible. Existem
desde dribles simples como somente passar pelo adversário conduzindo a bola, como os dribles
plásticos como caneta e chapéu.
• FINTA: Semelhante ao drible, porém consiste em passar ou se livrar do adversário sem a posse
de bola. Por exemplo fingindo que vai para a esquerda e mudando de direção para a direita, a fim
que o adversário se confunda, deixando o jogador livre para receber um passe.


O brasileiro Alessandro Rosa Vieira, mais conhecido como Falcão executava os fundamentos do Futsal com maestria, feito que lhe
rendeu o título de melhor jogador do mundo da FIFA em 4 oportunidades. Ao lado dele, a rainha do Futsal Brasileiro, Amanda Lyssa de
Oliveira, também conhecida como “Amandinha” que também foi eleita melhor jogadora do mundo por 7 vezes consecutivas. Disponivel
em: <https://pt.wikipedia.org/wiki/Amandinha_(jogadora_de_futsal)> e <https://pt.wikipedia.org/wiki/Falc%C3%A3o_(jogador_de_
futsal)> Acesso em: 08 jul. 2021.

PARA SABER MAIS:


Clique no link a seguir para conhecer um pouco mais sobre os fundamentos do Futsal. Disponível em:
<https://www.youtube.com/watch?v=7iCYuoABks0>. Acesso em: 08 jul. 2021.

ATIVIDADES
1 - Cite e explique pelo menos 3 regras do Futsal que são diferentes do Futebol de campo.

260
2 – Numere a segunda coluna de acordo com o fundamento do Futsal retratado na primeira.

1. Passe 2. Chute 3. Finta 4. Condução 5. Drible

(  ) Forma de tocar a bola para um companheiro mais bem posicionado.


(  ) Forma de “carregar” a bola por toda extensão da quadra.
(  ) Forma de se finalizar a jogada tentando marcar um gol.
(  ) Forma de se livrar do adversário sem a posse de bola.
(  ) Forma de passar pelo adversário com a posse de bola.

3 - Sobre a história do Futsal podemos afirmar que:


a) Assim como o Futebol, surgiu na Inglaterra como uma variação da modalidade praticado no campo
b) A modalidade foi criada em clubes brasileiros como forma de contornar a falta de espaço para
construção de campos de Futebol.
c) O primeiro livro de regras foi publicado ainda na década de 1930, tão logo que a modalidade surgiu.
d) O nome original da modalidade no Uruguai era indoor football, no Brasil ficou conhecido como
Futebol de salão e posteriormente teve o nome alterado pela FIFA para Futsal.
e) O Brasil tem 7 títulos mundiais de Futsal da FIFA, sem contar o títulos de mundiais organizados
pela FIFUSA.

261
SEMANA 3

EIXO TEMÁTICO:
Esportes.
TEMA/TÓPICO:
Hóquei – Aprimoramento técnico-tático e regras.
HABILIDADE(S):
Analisar os elementos técnicos de cada modalidade, analisar táticas das modalidades em situações de jogo,
analisar regras dos diferentes esportes, alterar regras de acordo com o interesse do grupo, espaços e materiais.
CONTEÚDOS RELACIONADOS:
História, regras, técnicas e táticas.
INTERDISCIPLINARIDADE:
Língua portuguesa.

TEMA: Hóquei
Prezado (a) estudante, nesta semana estudaremos acerca do Hóquei, uma modalidade muito seme-
lhante ao Futebol que também teve sua origem na Inglaterra e que posteriormente ganhou uma versão
mais popular em pistas de gelo. Bons estudos!

CONHECENDO O HÓQUEI
O hóquei é um esporte praticado em pistas de gelo, ou na grama, no qual os jogadores carregam um
taco, com o qual conduzem um disco ou uma bola na direção do gol. Existem dois tipos principais de
hóquei: o hóquei no gelo e o hóquei no campo.
HÓQUEI NO GELO
O hóquei no gelo é praticado em pistas de gelo com temperatura de menos 10°C ou com gelo especial.
Nessa modalidade cada time tem 6 jogadores, e o objetivo é levar o disco até o gol. Para levar o disco
(puck) até o gol utiliza-se um taco em forma de L, com o segurador e a lâmina (que são feitos de madeira).
Cada partida tem 3 tempos de 20 minutos, com intervalos de 15 minutos. Porém, o cronômetro para
quando o disco não está em jogo, nos casos de cobrança de faltas, nas discussões, comemorações
após cada gol ou outro imprevisto qualquer. O hóquei no gelo teve início no Canadá, e atualmente é o
esporte oficial do país. É praticado na Finlândia, Suíça, Rússia e também nos Estados Unidos.


Imagens de dois jogos de Hóquei de gelo e grama respectivamente, disponível em: <https://commons.wikimedia.org/wiki/File:14T9529_
(10388391404).jpg> e <https://commons.wikimedia.org/wiki/File:HOCKEY_ARGENTINA_PAKISTAN.jpg>. Acesso em 12 jul. 2021.

262
HÓQUEI DE CAMPO
É um esporte praticado por dois times de 11 jogadores. Uma partida é dividida em dois tempos de 35
minutos, com intervalo de 10 minutos entre elas.
O jogo é praticado em um campo de grama. Conduzindo a bola com o taco, o objetivo do hóquei no cam-
po é tentar marcar o maior número de gols possíveis. A bola utilizada é feita de plástico (PVC) e cortiça,
pesa 170 gramas e tem 7 centímetros de diâmetro.
O hóquei de campo tem origem na Inglaterra, sendo que é um esporte olímpico desde 1908. Algumas na-
ções são destaque nesse estilo de hóquei: Austrália, Argentina, Países Baixos, China, Alemanha, Índia
e Paquistão.
Adaptado de PACIEVITCH, Thais. HÓQUEI. Disponível em: <https://www.infoescola.com/esportes/hoquei/>. Acesso em: 12 jul. 2021.

OUTRAS MODALIDADES DO HÓQUEI


Há diversas outras modalidades do Hóquei além dessas duas principais. Vamos conhecer algumas delas.
• HÓQUEI INDOOR OU DE “SALÃO” – Muito semelhante a relação FUTEBOL/FUTSAL. Trata-se de
um Hóquei praticado sem patins, numa quadra reduzida e com menor número de jogadores em
relação a versão de grama.
• HÓQUEI SOBRE PATINS – Versão na qual os jogadores usam um patins quad, com duas rodas no
eixo da frente e duas rodas no eixo de trás. Nessa modalidade usa-se uma bola de borracha e um
stick é semelhante ao do Hóquei de grama, em formato de “J”. Porém os gols são pequenos, do
mesmo tamanho da versão gelo.
• HÓQUEI IN LINE – Versão praticada também sobre patins roller (rodas em linha). Essa modalidade
se aproxima mais da versão no gelo, pois os sticks usados têm de “L”, além de se usar um puck
(disco) no lugar da bola.
• HÓQUEI SUBAQUÁTICO – Nessa modalidade o jogo se desenvolve de forma submersa em uma
piscina. O objetivo é tocar a bola de chumbo com um pequeno stick levando a mesma ao gol do
adversário. Os times fazem trocas regulares entre titulares e reservas para que os atletas pos-
sam respirar.
• HÓQUEI DE RUA – Modalidade semelhante à relação FUTEBOL/FUTEBOL DE RUA. Trata-se de
uma versão recreativa que pode ser jogada em qualquer lugar com regras improvisadas como no
Futebol de rua.

PARA SABER MAIS:


Clique nos links a seguir para conhecer um pouco mais das regras e curiosidades das principais mo-
dalidades de Hóquei (gelo e grama) que estão presentes nas olimpíadas. Disponível em: <https://www.
youtube.com/watch?v=Et78 ED_LnCo> e <https://www.youtube.com/watch?v=vxuou5_bZZo>. Acesso
em: 12 jul. 2021.

263
ATIVIDADES

1 - O Hóquei na grama está presente nos jogos olímpicos desde 1908. O Hóquei no gelo fez sua estreia
nas olimpíadas de 1920, despertando nos organizadores a criar uma versão de jogos olímpicos para
esportes de gelo e neve. Assim as olimpíadas foram divididas a partir de 1924 em olimpíadas de verão e
olimpíadas de inverno. Cite e explique as principais semelhanças e diferenças entre essas duas versões
principais do Hóquei.

2 - Associe a modalidade de Hóquei com sua respectiva característica ou equipamento

1. Hóquei no gelo 2. Hóquei de grama 3. Hóquei indoor


4. Hóquei subaquático 5. Hóquei in-line 6.Hóquei sobre patins

(  ) Bola de chumbo, troca constante entre jogadores titulares e reservas.


(  ) Uso de patins quad, gols pequenos, taco formato “J” e bola de borracha.
(  ) Uso de patins com laminas, puck (disco) e tacos em formato de “L”.
(  ) Ausência de patins, gols grandes e campo de jogo amplo.
(  ) Ausência de patins, gols grandes e campo de jogo reduzido.
(  ) Uso de patins “roller”, puck (disco) e taco com formato em “L”.

3 - “O jogo é disputado em dois tempos de 35 minutos por duas equipes com 11 jogadores cada. Vence
quem conseguir fazer o maior número de gols possíveis. Para marcar gol é necessário que o toque final
para o gol seja feito dentro da área de ataque”. Ao ler o trecho podemos concluir que a modalidade
descrita é:
a) Futebol.
b) Rugby.
c) Hóquei no gelo.
d) Hóquei na grama.
e) Futebol Americano.

264
SEMANA 4

EIXO TEMÁTICO:
Dança e expressões rítmicas.

TEMA/TÓPICO:
A diversidade cultural das danças brasileiras.

HABILIDADE(S):
Reconhecer a pluralidade das manifestações culturais na dança em nosso país. Vivenciar diferentes manifes-
tações culturais da dança.

CONTEÚDOS RELACIONADOS:
Dança criativa, improvisação, pantomina.

INTERDISCIPLINARIDADE:
Língua portuguesa, História.

TEMA: Dança Folclórica


Prezado (a) estudante! Nesta semana abordaremos a riqueza cultural de algumas danças folclóricas e
regionais espalhadas pelos quatro cantos deste nosso belíssimo país. Bons estudos.

DANÇAS FOLCLÓRICAS
As danças folclóricas representam um conjunto de danças sociais, peculiares de cada estado brasilei-
ro, oriundas de antigos rituais mágicos e religiosos. As danças folclóricas possuem diversas funções
como a comemoração de datas religiosas, homenagens, agradecimentos, saudações às forças espiri-
tuais, etc.
No Brasil, o folclore possui muitas danças que representam as tradições e as culturas de determina-
da região.
No país, as danças folclóricas surgiram da fusão das culturas europeia, indígena e africana. Elas são
celebradas em festas populares caracterizadas por músicas, figurinos e cenários representativos.
Confira abaixo algumas das principais danças folclóricas brasileiras:
Bumba meu boi
Esta dança folclórica, conhecida em outras regiões brasileiras como o boi-bumbá, é típica do norte e
do nordeste. O Bumba meu boi possui uma origem diversificada, pois apresenta traços das culturas
espanhola, portuguesa, africana e indígena. Trata-se de uma dança na qual a representação teatral é
um fator marcante. Assim, a história da vida e da morte do boi é declamada enquanto os personagens
realizam suas danças.
Samba de roda
O samba de roda surgiu no estado da Bahia no século XIX e representa uma dança associada à capoeira
e ao culto dos orixás. Surgiu como forma de preservação da cultura dos escravos africanos. O samba de
roda é uma variante do samba, que embora tenha se disseminado por várias partes do Brasil, é tradicio-
nal da região do Recôncavo Baiano.

265
Frevo
O frevo é uma dança típica do carnaval pernambucano surgida no século XIX. Diferente de outras mar-
chinhas carnavalescas, ele é caracterizado pela ausência de letras onde os dançarinos seguram pe-
quenos guarda-chuvas coloridos como elemento coreográfico. A palavra “frevo” é originária do verbo
“ferver”, representando, desta maneira, particularidades desta dança demasiadamente frenética.

Maracatu
O maracatu, termo africano que significa “dança” ou “batuque”, é uma dança típica da região nordeste
com grande destaque para a região de Pernambuco. Esse ritmo e dança apresentam fortes caracterís-
ticas religiosas, composto por uma mistura de elementos indígenas, europeus e afro-brasileiros.

Imagens de danças folclóricas típicas do Brasil. Respectivamente; Bumba meu boi, frevo e quadrilha, Disponível em: <https://commons.
wikimedia.org/wiki/ File:Bumba-meu-boi_em_Santo_Amaro_-_Bahia.jpg> , <https://commons.wikimedia.org/wiki/File:No_compasso_
do_frevo.jpg> e <https://commons.wikimedia.org /wiki/File:Quadrilha_junina_na_Bahia.jpg>. Acesso em: 15 jul. 2021.

Baião
O baião é uma dança e canto típicos do nordeste brasileiro que recebeu, em suas origens, influências
das danças indígenas e da música caipira. Com movimentos que se aproximam do forró, o baião é dan-
çado em pares e sua temática é baseada no cotidiano e nas dificuldades da vida dos nordestinos.

Quadrilha
A quadrilha foi popularizada no Brasil a partir do Século XIX mediante influência da Corte Portuguesa.
É uma dança típica das festas juninas, bailada em duplas de casais caracterizados com vestimenta tipi-
camente caipira. Atualmente, a quadrilha abrange todas as regiões do Brasil.

Catira
A catira ou cateretê é uma dança folclórica presente em vários estados brasileiros. Há controvérsias
em relação à sua origem, entretanto, acredita-se que ela contém influências indígena, africana, espa-
nhola e portuguesa. A catira apresenta muitos elementos ligados à cultura caipira caracterizada pelo
figurino dos dançarinos acompanhados ao som das violas.

O Brasil é um país riquíssimo em diversidade de danças folclóricas. Abaixo temos uma lista com outras
danças um pouco menos populares. Circule as danças que você conhece ou já ouviu falar.

Dança da Fita Bate Coxa Congada


Pastoris Carimbó Batuque
Reisado Marujada Ticumbi
Fandango Xaxado Chula
Adaptado de DIANA, Daniela. Danças folclóricas. Disponível em: <https://www.todamateria.com.br/dancas-folcloricas/>.
Acesso em: 12 de Julho de 2021.

266
PARA SABER MAIS:
Clique no link a seguir para conhecer um pouco mais sobre as diversas danças folclóricas brasileiras.
Disponível em: <https://www.youtube.com/watch?v=FJVj1kT0G_E>. Acesso em: 12 jul. 2021.

ATIVIDADES

1 - O Frevo e o Samba são danças e ritmos tradicionais e presentes na tradicional festa de carnaval
brasileira. Descreva as principais semelhanças e diferenças entre esses dois ritmos do carnaval.

2 - Associe a dança típica às suas respectivas características.

1. Bumba meu boi 2. Quadrilha 3. Samba 4. Baião 5. Frevo


(  ) Representação teatral de morte e renascimento de um animal.
(  ) Roupas coloridas, ritmo intenso uso de uma sombrinha como acessório.
(  ) Uso de pandeiros, tambores e outros instrumentos de percussão.
(  ) Vestimentas com remendos, dança típica das festas juninas.
(  ) Dança típica do nordeste brasileiro que inspirou a criação do forró.

3 - ENEM 2013: Própria dos festejos juninos, a quadrilha nasceu como dança aristocrática. oriunda
dos salões franceses, depois difundida por toda a Europa. No Brasil, foi introduzida como dança de
salão e, por sua vez, apropriada e adaptada pelo gosto popular. Para sua ocorrência, é importante a
presença de um mestre “marcante” ou “marcador”, pois é quem determina as figurações diversas que
os dançadores desenvolvem. Observa-se a constância das seguintes marcações: “Tour”, “En avant”,
“Chez des dames”, “Chez des cheveliê”, “Cestinha de flor”, “Balancê”, “Caminho da roça”, “Olha a chuva”,
“Garranchê”, “Passeio”, “Coroa de flores”, “Coroa de espinhos” etc.
No Rio de Janeiro, em contexto urbano, apresenta transformações: surgem novas figurações, o francês
aportuguesado inexiste, o uso de gravações substitui a música ao vivo, além do aspecto de competição,
que sustenta os festivais de quadrilha, promovidos por órgãos de turismo.
CASCUDO. L.C. Dicionário do folclore brasileiro. Rio de Janeiro: Melhoramentos. 1976.

As diversas formas de dança são demonstrações da diversidade cultural do nosso país. Entre elas,
a quadrilha é considerada uma dança folclórica por
a) possuir como característica principal os atributos divinos e religiosos e, por isso, identificar uma
região.
b) abordar as tradições e costumes de determinados povos ou regiões distintas de uma mesma nação.
c) apresentar cunho artístico e técnicas apuradas, sendo também, considerada dança-espetáculo.
d) necessitar de vestuário específico para a sua prática, o qual define seu país de origem.
e) acontecer em salões e festas e ser influenciada por diversos gêneros musicais.

267
SEMANA 5

EIXO TEMÁTICO:
Esporte.

TEMA/TÓPICO:
Esportes lazer e sociedade.

HABILIDADE(S):
Analisar limites e possibilidades para a prática esportiva de lazer. Compreender o esporte como direito social.
Analisar o esporte na perspectiva da inclusão /exclusão de sujeitos.

CONTEÚDOS RELACIONADOS:
Esportes adaptados e paralímpicos.

INTERDISCIPLINARIDADE:
Língua portuguesa.

TEMA: Esportes Adaptados


Prezado (a) estudante, nesta semana de estudos abordaremos os esportes adaptados e os jogos pa-
ralímpicos que buscam incluir e levar a prática esportiva também para as pessoas com deficiência.
Bons estudos!
OS JOGOS PARALÍMPICOS
Os Jogos Paraolímpicos consistem em um evento desportivo, que é constituído por competições entre
atletas de alto nível, desde portadores de algum tipo de deficiência, sejam elas sensoriais ou físicas.
Em poucas palavras, trata-se dos Jogos Olímpicos tradicionais, cuja disputa é feita apenas com espor-
tes adaptados. A semelhança com as Olimpíadas não terminam por aí: Desde 1988 os Jogos ocorrem no
mesmo local que as Olimpíadas tradicionais, porém são iniciados logo após o seu encerramento.
Historicamente, acredita-se que a origem de competições esportivas entre pessoas portadoras de
deficiências tenha ocorrido nos Estados Unidos e na Inglaterra, devido ao grande número de homens
que lutaram na Segunda Guerra Mundial e perderam algum membro ou sofreram algum tipo de trauma
grave. Essas competições tinham como objetivo principal reabilitar os ex-combatentes, além de esti-
mulá-los fisicamente e emocionalmente. Os primeiros jogos dessa natureza foram organizados num
hospital londrino, no ano de 1948, voltado à recuperação de pessoas com lesões na medula óssea. Esse
é considerado um marco do desporto adaptado. Após esse período, outros jogos foram organizados no
mesmo local, porém, começava a reunir atletas de outros lugares para também participarem.
Foi apenas em 1960 que ocorreu a primeira edição dos Jogos Paraolímpicos, na cidade italiana de Roma.
Esse evento chegou a reunir 400 atletas, de 23 países diferentes, para disputar as competições. Desde
então, o esporte adaptado vem ganhando a profissionalização: ele deixou de ser um esporte amador e
de reabilitação para atingir o alto nível. Além disso, o número de atletas que esses jogos vêm agregando
aumentou significativamente: os Jogos de Atenas, em 2004, contemplaram 143 países e mais de 4000
atletas participantes.
Ao iniciar a sua condição de atleta profissional, o interessado deve passar por uma avaliação condicio-
nal que o classificará em alguma das categorias estipuladas pelo Movimento Paraolímpico Internacio-
nal. São elas: amputados, paralisados cerebrais, deficientes visuais, lesionados na medula espinhal,
deficientes mentais e categoria constituída por indivíduos com falta de mobilidade que não se encai-
xam nas categorias anteriormente citadas.

268
   
Exemplos de esportes paralímpicos. Temos, respectivamente, basquete em cadeira de rodas, goalball e atletismo para amputados de
pernas. Disponível em: https://commons.wikimedia.org/wiki/File:Rio_2016-_basquete_feminino_em_cadeira_de_rodas_(28921117184).
jpg, https://commons.wikimedia.org/wiki/File: Brasil_vence_Canad%C3%A1_no_Goalball_paral%C3%ADmpico._(29568716835).jpg
e https://commons.wikimedia.org/wiki/File:070912_-_Michelle_Errichiello_-_3b_-_2012_Summer_Paralympics_(01).jpg.
Acesso em: 15 jul. 2021.

Conheça os principais esportes que fazem parte do programa dos jogos paralímpicos.
• Atletismo: composto por provas de corridas, saltos, lançamentos e arremessos, cujos atletas
participantes possuem deficiências físicas ou visuais.
• Basquetebol em cadeira de rodas: esporte disputado nas categorias feminino e masculino.
É disputado desde a primeira Paraolimpíada. A cadeira de rodas é adaptada, a altura da cesta é a
mesma do basquete normal.
• Bocha: É disputado por pessoas com paralisia cerebral severa, dependentes de cadeira de ro-
das. As competições ocorrem segmentadas entre os sexos, e nas categorias individual e dupla.
• Ciclismo: Disputado por pessoas de ambos os sexos com paralisia cerebral, deficiência visual, am-
putados e lesionados medulares. As bicicletas são adaptadas em triciclos e com “pedal manual”.
• Esgrima: Esporte restrito apenas às pessoas com dificuldade de locomoção com ou sem cadeira
de rodas. As regras de pontuação são praticamente as mesmas da esgrima condicional.
• Futebol de 5: Disputado por homens deficientes visuais. A bola tem um “guizo” que faz barulho e
orientar os jogadores. Um “chamador” bate uma moeda na trave a se fazer o gol para orientar a
direção do chute.
• Futebol de 7: Restrito a homens paralisados cerebrais. As regras são bem semelhantes ao do
Futebol convencional.
• Goalball: Esporte desenvolvido especialmente para pessoas com deficiência visual. Consiste em
fazer um arremesso de uma bola com “guizo” para que a mesma entre no gol do adversário.
• Halterofilismo: Mulheres e homens amputados, paralisados cerebrais e lesionados de medula
espinhal competem nessa modalidade. O levantamento geralmente é feito com um supino.
• Hipismo: É praticado por pessoas de ambos os sexos e abarca diversos tipos de deficiência.
As regras são bem semelhantes a do esporte convencional.
• Judô: Com disputas masculinas e femininas, esse esporte também é restrito aos deficientes
visuais. As regras são praticamente as mesmas do judô convencional.
• Natação: Os nadadores que podem participar das provas são aqueles com limitações motoras e
físicas, com deficiência visual e deficiência mental.
• Remo: Contempla diferentes tipos de deficiência. As embarcações são adaptadas segundo a
necessidade de cada atleta.
• Rugby de cadeira de rodas: Modalidade praticada por equipes masculinas e femininas. O objetivo
é chegar ao fundo da quadra do adversário com a posse de bola na cadeira de rodas.
• Tênis de Mesa: As disputas são feitas de modo individual e por equipe. Participam atletas cadei-
rantes, andantes e portadores de deficiência mental.
• Vôlei sentado: Disputado por equipes com 6 jogadores cada. A regra é praticamente a mesma do
vôlei convencional. As adaptações mais evidentes são as reduções da altura da rede e do tama-
nho da quadra.
Adaptado de RONDINELLI, Paula. “Jogos Paraolímpicos”; Brasil Escola. Disponível em: https://brasilescola.uol.com.br/educacao-fisica/
jogos-paraolimpicos.htm. Acesso em: 13 de julho de 2021.

269
PARA SABER MAIS:
Clique no link a seguir para conhecer um pouco melhor a história dos jogos paralímpicos. Disponível em:
<https://www.youtube.com/watch?v=d0P1OTHyFIw>. Acesso em: 15 jul. 2021.
Neste outro link você poderá ver os atletas paralímpicos em ação nas principais provas. Disponível em:
<https://www.youtube.com/watch?v=x-wG3MJ_WTs>. Acesso em: 15 jul. 2021.

ATIVIDADES
1 - Descreva como aconteceu o processo de adaptação de esportes para pessoas com deficiência.
Qual era o contexto histórico e como os jogos evoluíram até chegar a este mega evento dos “jogos
paralímpicos”.

2 - Assinale (V) para afirmativas verdadeiras e (F) para afirmativas falsas.


(  ) O Basquete em cadeira de rodas é um esporte exclusivo para cadeirantes e deficientes visuais.
(  ) O Futebol de 5 é uma adaptação para deficientes visuais, enquanto o Futebol de 7 é adaptado para
pessoas com paralisia cerebral.
(  ) No Vôlei sentado as medidas da bola, dimensões da quadra e altura da rede são as mesmas do Vôlei
convencional.
(  ) O Atletismo e a natação paralímpica incluem praticamente todas as deficiências; físicas, visuais,
auditivas e até cognitivas.
(  ) O levantamento de peso paralímpico é feito com um supino de academia, no qual o atleta deve ten-
tar levantar a barra tirando a mesma do apoio, esticar os braços e colocá-la de volta sobre o apoio.

3 - Assinale a modalidade esportiva paralímpica em que não temos um esporte convencional adaptado
para pessoas com deficiência e sim um esporte criado especificamente para atender uma determinada
deficiência.
a) Bocha.
b) Goalball.
c) Remo.
d) Esgrima.
e) Rugby em cadeira de rodas.

270
SEMANA 6

EIXO TEMÁTICO:
Ginástica.
TEMA/TÓPICO:
Ginástica, consumo e mídia.
HABILIDADE(S):
Analisar os padrões de corpo impostos pela cultura.
Analisar as implicações do consumismo nas práticas das modalidades da ginástica.
CONTEÚDOS RELACIONADOS:
Os perigos da imposição dos padrões estéticos.
INTERDISCIPLINARIDADE:
Língua portuguesa, Biologia, Sociologia.

TEMA: Padrões Estéticos


Prezado (a) estudante! Nesta semana abordaremos a questão dos padrões de estética e beleza, asso-
ciadas ao exercício físico. Neste estudo descobriremos que exercício físico e saúde nem sempre está
associado a exercício físico e estética. Bons estudos!

MÍDIA E O CULTO À BELEZA DO CORPO


Há nas sociedades contemporâneas uma intensificação do culto ao corpo, onde os indivíduos experi-
mentam uma crescente preocupação com a imagem e a estética. Entendida como consumo cultural,
a prática do culto ao corpo coloca-se hoje como preocupação geral, que perpassa todas as classes so-
ciais e faixas etárias, apoiada num discurso que ora lança mão da questão estética, ora da preocupação
com a saúde.
Segundo Pierre Bourdieu, sociólogo francês, a linguagem corporal é marcada pela distinção social, que
coloca o consumo alimentar, cultural e forma de apresentação – como o vestuário, higiene, cuidados
com a beleza etc. – como os mais importantes modos de se distinguir dos demais indivíduos.
Nas sociedades modernas há uma crescente preocupação com o corpo, com a dieta alimentar e o con-
sumo excessivo de cosméticos, impulsionados basicamente pelo processo de massificação das mídias
a partir dos anos 1980, onde o corpo ganha mais espaço, principalmente nos meios midiáticos. Não é
por acaso que foi nesse período que surgiram as duas maiores revistas brasileiras voltadas para o tema:
“Boa Forma” (1984) e “Corpo a Corpo” (1987).
Contudo, foi o cinema de Hollywood que ajudou a criar novos padrões de aparência e beleza, difundin-
do novos valores da cultura de consumo e projetando imagens de estilos de vida glamorosos para o
mundo inteiro.
Da mesma forma, podemos pensar em relação à televisão, que veicula imagens de corpos perfeitos
através dos mais variados formatos de programas, peças publicitárias, novelas, filmes etc. Isso nos
leva a pensar que a imagem da “eterna” juventude, associada ao corpo perfeito e ideal, atravessa todas
as faixas etárias e classes sociais, compondo de maneiras diferentes diversos estilos de vida. Nesse
sentido, as fábricas de imagens como o cinema, televisão, publicidade, revistas etc., têm contribuído
para isso.

271
Os programas de televisão, revistas e jornais têm dedicado espaços em suas programações cada vez
maiores para apresentar novidades em setores de cosméticos, de alimentação e vestuário. Propagan-
das veiculadas nessas mídias estão o tempo todo tentando vender o que não está disponível nas prate-
leiras: sucesso e felicidade.
O consumismo desenfreado gerado pela mídia em geral foca principalmente adolescentes como alvos
principais para as vendas, desenvolvendo modelos de roupas estereotipados, a indústria de cosméti-
cos lançando a cada dia novos cremes e géis redutores para eliminar as “formas indesejáveis” do corpo
e a indústria farmacêutica faturando alto com medicamentos que inibem o apetite.
Preocupados com a busca desenfreada da “beleza perfeita” e pela vaidade excessiva, sob influência dos
mais variados meios de comunicação, a Sociedade Brasileira de Cirurgia Plástica apresenta uma estima-
tiva de que cerca de 130 mil crianças e adolescentes submeteram-se a operações plásticas todos os anos.
Evidentemente a existência de cuidados com o corpo não é exclusividade das sociedades contempo-
râneas e que devemos ter uma especial atenção para uma boa saúde. No entanto, os cuidados com o
corpo não devem ser de forma tão intensa e ditatorial como se tem apresentado nas últimas décadas.
Devemos sempre respeitar os limites do nosso corpo e a nós mesmos.
CAMARGO, Orson. “Mídia e o culto à beleza do corpo”; Brasil Escola. Disponível em: https://brasilescola.uol.com.br/sociologia/a-
influencia-midia-sobre-os-padroes-beleza.htm. Acesso em 13 de julho de 2021.

EXERCÍCIO FÍSICO: SAÚDE X ESTÉTICA


É comum associar a prática regular de exercícios físicos a busca ou manutenção do “corpo perfeito”.
Há quem condene esta relação dizendo que ela é prejudicial à saúde, mas o fato é que ela existe, prin-
cipalmente entre grandes celebridades que usam o exercício físico como forma de manter o corpo per-
feito para divulgá-lo nas mídias sociais. Não há problema em usar o exercício para “modelar” o corpo,
mas, contudo, é importante lembrar que a função predominante do exercício físico é a manutenção da
saúde. Portanto saiba o limite! A partir do momento que o exercício associado a outras práticas estiver
prejudicando a sua saúde ao invés de melhorá-la, é sinal de que essa busca incessante pelo “corpo per-
feito” não vale tal sacrifício. Fiquem atentos! Conheça agora as principais atitudes prejudiciais na busca
pelo “corpo ideal”.
• Uso de anabolizantes e esteróides: O uso de anabolizantes e esteroides prometem resultados
rápidos, porém podem causar sobrecarga e até sérios danos irreversíveis ao fígado, rins e outros
órgãos vitais.
• Uso de suplementos milagrosos: Não existe nenhum suplemento milagroso que faz, por exem-
plo, emagrecer ou ganhar massa muscular e mesmo que houvesse ele seria muito caro. Mante-
nha o treino regular e a boa alimentação.
• Dietas “mirabolantes”: “Alimentar-se apenas de suco de limão”, “tomar 4 litros de café”, “Comer
apenas ovos e queijo” são algumas das diversas dietas mirabolantes que prometem resultados
rápidos. Nenhuma delas funciona. Lembre-se de que você demorou a chegar ao peso que está
e certamente o resultado que você quer também será demorado. Continue com treino regular,
alimentação balanceada e muita disciplina.
• Treinos de “blogueiros fitness”: Muito cuidado com o mundo das redes sociais. Durante a pan-
demia vimos surgir diversos “digitais influencer” com treinos milagrosos em redes sociais. Lem-
bre-se de que o treino é algo pessoal que deve respeitar os limites e individualidades biológicas
de cada um. Não faça treino de blogueiros, procure sempre a orientação de um profissional de
educação física com formação acadêmica adequada. Caso não tenha acesso à orientação pro-
fissional, dê preferência a exercícios de pouca intensidade como caminhadas.

272
Exemplos de exercícios físicos que podem ajudar a melhorar e manter a saúde, e consequentemente a estética corporal. Na primeira
imagem temos exemplos de exercícios que podem ser praticados ao ar livre (corrida, caminhada, ciclismo) A vantagem destes exercícios
é que podem ser praticados de forma gratuita em espaços públicos. Na segunda imagem temos uma academia de ginástica que tem
como desvantagem o fato de não ser gratuita, mas em contrapartida oferece muito mais opções de exercícios físicos além a orientação
e acompanhamento de um profissional de Educação Física. Disponível em: https://pxhere.com/pt/photo/1054702 e https://pxhere.
com/pt/photo/715253. Acesso em: 15 jul. 2021.

PARA SABER MAIS


Clique no link a seguir para conhecer a linha do tempo da ginástica de academia no Brasil, bem como
quais são as modalidades existentes. Quem sabe você não descobre o tipo de exercício físico ideal para
você. Disponível em: <https://www.youtube.com/watch?v=w2WomTOT0is>. Acesso em: 15 jul. 2021.

ATIVIDADES
1 - Descreva como a mídia influencia o padrão de beleza e estética. Essa influência ajuda a forma hábitos
saudáveis ou simplesmente influencia na busca a qualquer custo pelo corpo perfeito sem a mínima
preocupação com a saúde?

2 - Assinale (S) para atitudes que tem predominância da SAÚDE e (E) para práticas que têm a
predominância da ESTÉTICA.
(  ) Cirurgia plástica.
(  ) Alimentação orgânica.
(  ) Corrida / caminhada.
(  ) Suplementação alimentar.
(  ) Drenagem linfática.
(  ) Meditação.

273
3 - ENEM 2011: Na modernidade, o corpo foi descoberto, despido e modelado pelos exercícios físicos da
moda. Novos espaços e práticas esportivas e de ginástica passaram a convocar as pessoas a modelarem
seus corpos. Multiplicaram-se as academias de ginástica, as salas de musculação e o número de
pessoas correndo pelas ruas.
SECRETARIA DA EDUCAÇÃO. Caderno do professor: educação física. São Paulo, 2008.

Diante do exposto, é possível perceber que houve um aumento da procura por:


a) exercícios físicos aquáticos (natação/hidroginástica), que são exercícios de baixo impacto, evi-
tando o atrito (não prejudicando as articulações), e que previnem o envelhecimento precoce e
melhoram a qualidade de vida.
b) mecanismos que permitem combinar alimentação e exercício físico, que permitem a aquisição
e manutenção de níveis adequados de saúde, sem a preocupação com padrões de beleza insti-
tuídos socialmente.
c) Programas saudáveis de emagrecimento, que evitam os prejuízos causados na regulação meta-
bólica, função imunológica, integridade óssea e manutenção da capacidade funcional ao longo
do envelhecimento.
d) exercícios de relaxamento, reeducação postural e alongamentos, que permitem um melhor fun-
cionamento do organismo como um todo, bem como uma dieta alimentar e hábitos saudáveis
com base em produtos naturais.
e) dietas que preconizam a ingestão excessiva ou restrita de um ou mais macronutrientes (car-
boidratos, gorduras ou proteínas), bem como exercícios que permitem um aumento de massa
muscular e/ou modelar o corpo.

Prezado(a) estudante! Chegamos ao final deste PET 4! Caso tenha ficado alguma dúvida, entre em
contato com seu professor pelo CONEXÃO ESCOLA ou mande sua pergunta para TIRA DÚVIDAS AO
VIVO na Rede Minas e Youtube. Continue firme nos estudos e com as práticas regulares de exercícios
físicos, sempre respeitando os protocolos de segurança. Mantenha uma alimentação saudável e uma
hidratação regular. Em breve estaremos vacinados e juntos em nossas quadras. Um forte abraço da
equipe de Educação Física!

274

Você também pode gostar